You are on page 1of 612

Math 376 Prealgebra Textbook

Fall 2010 Version Department of Mathematics College of the Redwoods June 23, 2010

ii
Copyright All parts of this prealgebra textbook are copyrighted c 2009 in the name of the Department of Mathematics, College of the Redwoods. They are not in the public domain. However, they are being made available free for use in educational institutions. This oer does not extend to any application that is made for prot. Users who have such applications in mind should contact David Arnold at davidarnold@ redwoods.edu or Bruce Wagner at bruce-wagner@redwoods.edu. This work is licensed under a Creative Commons AttributionNonCommercial-ShareAlike 3.0 Unported License, and is copyrighted c 2009, Department of Mathematics, College of the Redwoods. To view a copy of this license, visit http://creativecommons.org/licenses/by-nc-sa/3.0/ or send a letter to Creative Commons, 543 Howard Street, 5th Floor, San Francisco, California, 94105, USA.

Version: Fall 2010

Contents

1 The Whole Numbers 1.1 An Introduction to the Whole Numbers . . . . Graphing numbers on the number line . . . . Ordering the whole numbers . . . . . . . . . Expanded notation . . . . . . . . . . . . . . . Rounding whole numbers . . . . . . . . . . . Tables and graphs . . . . . . . . . . . . . . . Exercises . . . . . . . . . . . . . . . . . . . . Answers . . . . . . . . . . . . . . . . . . . . . 1.2 Adding and Subtracting Whole Numbers . . . The Commutative Property of Addition . . . Grouping Symbols . . . . . . . . . . . . . . . The Associative Property of Addition . . . . The Additive Identity . . . . . . . . . . . . . Adding Larger Whole Numbers . . . . . . . . Subtraction of Whole Numbers . . . . . . . . Subtracting Larger Whole Numbers . . . . . Order of Operations . . . . . . . . . . . . . . Applications Geometry . . . . . . . . . . . Application Alternative Fuels . . . . . . . Exercises . . . . . . . . . . . . . . . . . . . . Answers . . . . . . . . . . . . . . . . . . . . . 1.3 Multiplication and Division of Whole Numbers The Multiplicative Identity . . . . . . . . . . Multiplication by Zero . . . . . . . . . . . . . The Associative Property of Multiplication . Multiplying Larger Whole Numbers . . . . . Division of Whole Numbers . . . . . . . . . . Division is not Commutative . . . . . . . . . Division is not Associative . . . . . . . . . . Division by Zero is Undened . . . . . . . . . iii

. . . . . . . . . . . . . . . . . . . . . . . . . . . . . .

. . . . . . . . . . . . . . . . . . . . . . . . . . . . . .

. . . . . . . . . . . . . . . . . . . . . . . . . . . . . .

. . . . . . . . . . . . . . . . . . . . . . . . . . . . . .

. . . . . . . . . . . . . . . . . . . . . . . . . . . . . .

. . . . . . . . . . . . . . . . . . . . . . . . . . . . . .

. . . . . . . . . . . . . . . . . . . . . . . . . . . . . .

. . . . . . . . . . . . . . . . . . . . . . . . . . . . . .

. . . . . . . . . . . . . . . . . . . . . . . . . . . . . .

1 2 2 3 4 6 8 9 13 15 15 16 17 17 18 19 20 20 21 23 25 30 33 34 34 35 35 37 39 39 40

iv Dividing Larger Whole Numbers . . . . . . . . Application Counting Rectangular Arrays . Application Area . . . . . . . . . . . . . . . Exercises . . . . . . . . . . . . . . . . . . . . . Answers . . . . . . . . . . . . . . . . . . . . . . Prime Factorization . . . . . . . . . . . . . . . . Divisibility . . . . . . . . . . . . . . . . . . . . Divisibility Tests . . . . . . . . . . . . . . . . . Prime Numbers . . . . . . . . . . . . . . . . . Factor Trees . . . . . . . . . . . . . . . . . . . Exponents . . . . . . . . . . . . . . . . . . . . Application . . . . . . . . . . . . . . . . . . . . Exercises . . . . . . . . . . . . . . . . . . . . . Answers . . . . . . . . . . . . . . . . . . . . . . Order of Operations . . . . . . . . . . . . . . . . Fraction Bars . . . . . . . . . . . . . . . . . . . The Distributive Property . . . . . . . . . . . . Exercises . . . . . . . . . . . . . . . . . . . . . Answers . . . . . . . . . . . . . . . . . . . . . . Solving Equations by Addition and Subtraction . Equivalent Equations . . . . . . . . . . . . . . Operations that Produce Equivalent Equations Wrap and Unwrap . . . . . . . . . . . . . . . . Word Problems . . . . . . . . . . . . . . . . . . Exercises . . . . . . . . . . . . . . . . . . . . . Answers . . . . . . . . . . . . . . . . . . . . . . Solving Equations by Multiplication and Division Word Problems . . . . . . . . . . . . . . . . . . Exercises . . . . . . . . . . . . . . . . . . . . . Answers . . . . . . . . . . . . . . . . . . . . . . . . . . . . . . . . . . . . . . . . . . . . . . . . . . . . . . . . . . . . . . . . . . . . . . . . . . . . . . . . . . . . . . . . . . . . . . . . . . . . . . . . . . . . . . . .

CONTENTS . . . . . . . . . . . . . . . . . . . . . . . . . . . . . . . . . . . . . . . . . . . . . . . . . . . . . . . . . . . . . . . . . . . . . . . . . . . . . . . . . . . . . . . . . . . . . . . . . . . . . . . . . . . . . . . . . . . . . . . . . . . . . . . . . . . . . . . . . . . . . . . . . . . . . . 40 41 42 44 49 51 51 52 53 54 55 57 58 62 64 67 68 71 73 75 76 77 79 81 85 87 88 90 94 96 97 98 99 99 101 102 102 104 106 110 113 113 114 115 117

1.4

1.5

1.6

1.7

2 The Integers 2.1 An Introduction to the Integers . . . The Integers . . . . . . . . . . . . Ordering the Integers . . . . . . . Opposites of Opposites . . . . . . Positive and Negative . . . . . . . Absolute Value . . . . . . . . . . . Applications . . . . . . . . . . . . Exercises . . . . . . . . . . . . . . Answers . . . . . . . . . . . . . . . 2.2 Adding Integers . . . . . . . . . . . . Vectors on the Number Line . . . Magnitude and Absolute Value . . Adding Integers with Like Signs . Adding Integers with Unlike Signs Version: Fall 2010

. . . . . . . . . . . . . .

. . . . . . . . . . . . . .

. . . . . . . . . . . . . .

. . . . . . . . . . . . . .

. . . . . . . . . . . . . .

. . . . . . . . . . . . . .

. . . . . . . . . . . . . .

. . . . . . . . . . . . . .

. . . . . . . . . . . . . .

. . . . . . . . . . . . . .

. . . . . . . . . . . . . .

. . . . . . . . . . . . . .

. . . . . . . . . . . . . .

. . . . . . . . . . . . . .

. . . . . . . . . . . . . .

CONTENTS Properties of Addition of Integers . . . . . . . . . . . . . Grouping for Eciency . . . . . . . . . . . . . . . . . . . Exercises . . . . . . . . . . . . . . . . . . . . . . . . . . . Answers . . . . . . . . . . . . . . . . . . . . . . . . . . . . Subtracting Integers . . . . . . . . . . . . . . . . . . . . . . Order of Operations . . . . . . . . . . . . . . . . . . . . . Change as a Dierence . . . . . . . . . . . . . . . . . . . Exercises . . . . . . . . . . . . . . . . . . . . . . . . . . . Answers . . . . . . . . . . . . . . . . . . . . . . . . . . . . Multiplication and Division of Integers . . . . . . . . . . . . The Product of a Positive Integer and a Negative Integer The Distributive Property . . . . . . . . . . . . . . . . . . The Multiplicative Property of Zero . . . . . . . . . . . . Multiplying by Minus One . . . . . . . . . . . . . . . . . The Product of Two Negative Integers . . . . . . . . . . . Memory Device . . . . . . . . . . . . . . . . . . . . . . . . Division of Integers . . . . . . . . . . . . . . . . . . . . . Exercises . . . . . . . . . . . . . . . . . . . . . . . . . . . Answers . . . . . . . . . . . . . . . . . . . . . . . . . . . . Order of Operations with Integers . . . . . . . . . . . . . . Evaluating Fractions . . . . . . . . . . . . . . . . . . . . . Absolute Value . . . . . . . . . . . . . . . . . . . . . . . . Exercises . . . . . . . . . . . . . . . . . . . . . . . . . . . Answers . . . . . . . . . . . . . . . . . . . . . . . . . . . . Solving Equations Involving Integers . . . . . . . . . . . . . Adding or Subtracting the Same Amount . . . . . . . . . Multiplying or Dividing by the Same Amount . . . . . . . Combining Operations . . . . . . . . . . . . . . . . . . . . Applications . . . . . . . . . . . . . . . . . . . . . . . . . Exercises . . . . . . . . . . . . . . . . . . . . . . . . . . . Answers . . . . . . . . . . . . . . . . . . . . . . . . . . . . . . . . . . . . . . . . . . . . . . . . . . . . . . . . . . . . . . . . . . . . . . . . . . . . . . . . . . . . . . . . . .

v 119 121 123 126 128 129 130 133 135 137 138 139 139 140 140 141 143 145 147 148 150 151 152 154 156 156 158 160 163 167 169 171 172 172 175 177 178 179 183 185 187 188 189 191 192

2.3

2.4

2.5

2.6

3 The Fundamentals of Algebra 3.1 Mathematical Expressions . . . . . . . . . . . . . . . Translating Words into Mathematical Expressions Combinations . . . . . . . . . . . . . . . . . . . . . Exercises . . . . . . . . . . . . . . . . . . . . . . . Answers . . . . . . . . . . . . . . . . . . . . . . . . 3.2 Evaluating Algebraic Expressions . . . . . . . . . . . Exercises . . . . . . . . . . . . . . . . . . . . . . . Answers . . . . . . . . . . . . . . . . . . . . . . . . 3.3 Simplifying Algebraic Expressions . . . . . . . . . . Speeding Things Up . . . . . . . . . . . . . . . . . The Distributive Property . . . . . . . . . . . . . . Moving a Bit Quicker . . . . . . . . . . . . . . . . Extending the Distributive Property . . . . . . . .

. . . . . . . . . . . . .

. . . . . . . . . . . . .

. . . . . . . . . . . . .

. . . . . . . . . . . . .

. . . . . . . . . . . . .

. . . . . . . . . . . . .

Version: Fall 2010

vi Distributing a Negative . . . . . . . Exercises . . . . . . . . . . . . . . . Answers . . . . . . . . . . . . . . . . Combining Like Terms . . . . . . . . . Like Terms . . . . . . . . . . . . . . Combining Like Terms . . . . . . . . Speeding Things Up a Bit . . . . . . Simplify . . . . . . . . . . . . . . . . Applications . . . . . . . . . . . . . Exercises . . . . . . . . . . . . . . . Answers . . . . . . . . . . . . . . . . Solving Equations Involving Integers II Variables on Both Sides . . . . . . . Exercises . . . . . . . . . . . . . . . Answers . . . . . . . . . . . . . . . . Applications . . . . . . . . . . . . . . . Consecutive Integers . . . . . . . . . Consecutive Odd Integers . . . . . . Tables . . . . . . . . . . . . . . . . . Exercises . . . . . . . . . . . . . . . Answers . . . . . . . . . . . . . . . . . . . . . . . . . . . . . . . . . . . . . . . . . . . . . . . . . . . . . . . . . . . . . . . . . . . . . . . . . . . . . . . . . . . . . . . . . . . . . . . . . . . . . . . . . . . . . . . . . . . . . . . . . . . . . . . . . . . . . . . . . . . . . . . . . . . . . . . . . . . . . . . . . . . . . . . . . . . . . . . . . . . . . . . . . . . . . . . . . . . . . . . . . . . . .

CONTENTS . . . . . . . . . . . . . . . . . . . . . . . . . . . . . . . . . . . . . . . . . . . . . . . . . . . . . . . . . . . . . . . . . . . . . . . . . . . . . . . . . . . . . . . . . . . . . . . . . . . . . . . . . 193 195 196 197 198 199 200 201 203 205 206 208 209 213 214 216 216 218 219 224 227 229 231 234 235 237 238 239 240 243 246 249 250 252 253 255 257 260 263 265 265 267 270 273 275

3.4

3.5

3.6

4 Fractions 4.1 Equivalent Fractions . . . . . . . . . . . . . . . The Greatest Common Divisor . . . . . . . . Reducing a Fraction to Lowest Terms . . . . Reducing Fractions with Variables . . . . . . A Word on Mathematical Notation. . . . . . Equivalent Fractions in Higher Terms . . . . Negative Fractions . . . . . . . . . . . . . . . Exercises . . . . . . . . . . . . . . . . . . . . Answers . . . . . . . . . . . . . . . . . . . . . 4.2 Multiplying Fractions . . . . . . . . . . . . . . Multiplication Rule . . . . . . . . . . . . . . Multiply and Reduce . . . . . . . . . . . . . Multiply and Cancel or Cancel and Multiply Parallelograms . . . . . . . . . . . . . . . . . Triangles . . . . . . . . . . . . . . . . . . . . Exercises . . . . . . . . . . . . . . . . . . . . Answers . . . . . . . . . . . . . . . . . . . . . 4.3 Dividing Fractions . . . . . . . . . . . . . . . . Reciprocals . . . . . . . . . . . . . . . . . . . Division . . . . . . . . . . . . . . . . . . . . . Exercises . . . . . . . . . . . . . . . . . . . . Answers . . . . . . . . . . . . . . . . . . . . . 4.4 Adding and Subtracting Fractions . . . . . . . Version: Fall 2010

. . . . . . . . . . . . . . . . . . . . . . .

. . . . . . . . . . . . . . . . . . . . . . .

. . . . . . . . . . . . . . . . . . . . . . .

. . . . . . . . . . . . . . . . . . . . . . .

. . . . . . . . . . . . . . . . . . . . . . .

. . . . . . . . . . . . . . . . . . . . . . .

. . . . . . . . . . . . . . . . . . . . . . .

. . . . . . . . . . . . . . . . . . . . . . .

. . . . . . . . . . . . . . . . . . . . . . .

CONTENTS Adding Fractions with Dierent Denominators . . . Least Common Multiple . . . . . . . . . . . . . . . . Least Common Multiple Using Prime Factorization Comparing Fractions . . . . . . . . . . . . . . . . . . Exercises . . . . . . . . . . . . . . . . . . . . . . . . Answers . . . . . . . . . . . . . . . . . . . . . . . . . Multiplying and Dividing Mixed Fractions . . . . . . . Changing Mixed Fractions to Improper Fractions . . Changing Improper Fractions to Mixed Fractions . . Multiplying and Dividing Mixed Fractions . . . . . Exercises . . . . . . . . . . . . . . . . . . . . . . . . Answers . . . . . . . . . . . . . . . . . . . . . . . . . Adding and Subtracting Mixed Fractions . . . . . . . Adding Mixed Fractions . . . . . . . . . . . . . . . . Working in Vertical Format . . . . . . . . . . . . . . Subtracting Mixed Fractions . . . . . . . . . . . . . Working in Vertical Format . . . . . . . . . . . . . . Borrowing in Vertical Format . . . . . . . . . . . . . Exercises . . . . . . . . . . . . . . . . . . . . . . . . Answers . . . . . . . . . . . . . . . . . . . . . . . . . Order of Operations with Fractions . . . . . . . . . . . Order of Operations . . . . . . . . . . . . . . . . . . Complex Fractions . . . . . . . . . . . . . . . . . . . Application Trapezoid . . . . . . . . . . . . . . . Exercises . . . . . . . . . . . . . . . . . . . . . . . . Answers . . . . . . . . . . . . . . . . . . . . . . . . . Solving Equations with Fractions . . . . . . . . . . . . Undoing Subtraction . . . . . . . . . . . . . . . . . . Undoing Addition . . . . . . . . . . . . . . . . . . . Undoing Multiplication . . . . . . . . . . . . . . . . Clearing Fractions from the Equation . . . . . . . . Applications . . . . . . . . . . . . . . . . . . . . . . Exercises . . . . . . . . . . . . . . . . . . . . . . . . Answers . . . . . . . . . . . . . . . . . . . . . . . . . . . . . . . . . . . . . . . . . . . . . . . . . . . . . . . . . . . . . . . . . . . . . . . . . . . . . . . . . . . . . . . . . . . . . . . . . . . . . . . . . . . . . . . . . . . . . . . . . . . . . . . . . . . . . . . . . . . . . . . . . . . . . . . . . . . . . . . . . . . . . . . . . . . . . . . . . . . . . . . . . . . . . . . . . . .

vii 277 280 281 284 285 289 291 291 293 295 297 299 301 301 303 304 306 306 308 309 311 312 315 318 321 324 326 326 326 327 329 332 336 339 341 342 342 344 346 348 350 353 356 359 359

4.5

4.6

4.7

4.8

5 Decimals 5.1 Introduction to Decimals . . . . . . Decimal Notation . . . . . . . . Pronouncing Decimal Numbers . Decimals to Fractions . . . . . . Rounding . . . . . . . . . . . . . Comparing Decimals . . . . . . . Exercises . . . . . . . . . . . . . Answers . . . . . . . . . . . . . . 5.2 Adding and Subtracting Decimals . Adding Decimals . . . . . . . . .

. . . . . . . . . .

. . . . . . . . . .

. . . . . . . . . .

. . . . . . . . . .

. . . . . . . . . .

. . . . . . . . . .

. . . . . . . . . .

. . . . . . . . . .

. . . . . . . . . .

. . . . . . . . . .

. . . . . . . . . .

. . . . . . . . . .

. . . . . . . . . .

. . . . . . . . . .

. . . . . . . . . .

. . . . . . . . . .

Version: Fall 2010

viii

CONTENTS Subtracting Decimals . . . . . . . . . . . . . . . . . . Adding and Subtracting Signed Decimal Numbers . . Exercises . . . . . . . . . . . . . . . . . . . . . . . . . Answers . . . . . . . . . . . . . . . . . . . . . . . . . . Multiplying Decimals . . . . . . . . . . . . . . . . . . . . Multiplying Signed Decimal Numbers . . . . . . . . . Order of Operations . . . . . . . . . . . . . . . . . . . Powers of Ten . . . . . . . . . . . . . . . . . . . . . . Multiplying Decimal Numbers by Powers of Ten . . . The Circle . . . . . . . . . . . . . . . . . . . . . . . . Exercises . . . . . . . . . . . . . . . . . . . . . . . . . Answers . . . . . . . . . . . . . . . . . . . . . . . . . . Dividing Decimals . . . . . . . . . . . . . . . . . . . . . Decimal Divisors . . . . . . . . . . . . . . . . . . . . . Dividing Signed Decimal Numbers . . . . . . . . . . . Rounding . . . . . . . . . . . . . . . . . . . . . . . . . Dividing by Powers of Ten . . . . . . . . . . . . . . . Order of Operations . . . . . . . . . . . . . . . . . . . Exercises . . . . . . . . . . . . . . . . . . . . . . . . . Answers . . . . . . . . . . . . . . . . . . . . . . . . . . Fractions and Decimals . . . . . . . . . . . . . . . . . . Terminating Decimals . . . . . . . . . . . . . . . . . . Repeating Decimals . . . . . . . . . . . . . . . . . . . Expressions Containing Both Decimals and Fractions Exercises . . . . . . . . . . . . . . . . . . . . . . . . . Answers . . . . . . . . . . . . . . . . . . . . . . . . . . Equations With Decimals . . . . . . . . . . . . . . . . . Combining Operations . . . . . . . . . . . . . . . . . . Combining Like Terms . . . . . . . . . . . . . . . . . . Using the Distributive Property . . . . . . . . . . . . Rounding Solutions . . . . . . . . . . . . . . . . . . . Applications . . . . . . . . . . . . . . . . . . . . . . . Exercises . . . . . . . . . . . . . . . . . . . . . . . . . Answers . . . . . . . . . . . . . . . . . . . . . . . . . . Introduction to Square Roots . . . . . . . . . . . . . . . Square Roots . . . . . . . . . . . . . . . . . . . . . . . Radical Notation . . . . . . . . . . . . . . . . . . . . . Order of Operations . . . . . . . . . . . . . . . . . . . Fractions and Decimals . . . . . . . . . . . . . . . . . Estimating Square Roots . . . . . . . . . . . . . . . . Exercises . . . . . . . . . . . . . . . . . . . . . . . . . Answers . . . . . . . . . . . . . . . . . . . . . . . . . . The Pythagorean Theorem . . . . . . . . . . . . . . . . Exercises . . . . . . . . . . . . . . . . . . . . . . . . . Answers . . . . . . . . . . . . . . . . . . . . . . . . . . . . . . . . . . . . . . . . . . . . . . . . . . . . . . . . . . . . . . . . . . . . . . . . . . . . . . . . . . . . . . . . . . . . . . . . . . . . . . . . . . . . . . . . . . . . . . . . . . . . . . . . . . . . . . . . . . . . . . . . . . . . . . . . . . . . . . . . . . . . . . . . . . . . . . . . . . . . . . . . . . . . . . . . . . . . . . . . . . . . . . 361 362 366 368 370 373 374 375 376 376 381 384 386 388 390 391 392 393 395 399 401 401 402 405 408 410 411 412 412 413 414 415 421 424 425 425 426 428 430 430 433 435 437 443 445

5.3

5.4

5.5

5.6

5.7

5.8

Version: Fall 2010

CONTENTS 6 Ratio and Proportion 6.1 Introduction to Ratios and Rates . . . . . . . . Rates . . . . . . . . . . . . . . . . . . . . . . Unit Rates . . . . . . . . . . . . . . . . . . . Exercises . . . . . . . . . . . . . . . . . . . . Answers . . . . . . . . . . . . . . . . . . . . . 6.2 Introduction to Proportion . . . . . . . . . . . Solving Proportions . . . . . . . . . . . . . . Applications . . . . . . . . . . . . . . . . . . Exercises . . . . . . . . . . . . . . . . . . . . Answers . . . . . . . . . . . . . . . . . . . . . 6.3 Unit Conversion: American System . . . . . . . Units of Length . . . . . . . . . . . . . . . . Units of Weight . . . . . . . . . . . . . . . . Units of Volume . . . . . . . . . . . . . . . . Units of Time . . . . . . . . . . . . . . . . . Converting Units of Speed . . . . . . . . . . Exercises . . . . . . . . . . . . . . . . . . . . Answers . . . . . . . . . . . . . . . . . . . . . 6.4 Unit Conversion: Metric System . . . . . . . . Units of Length . . . . . . . . . . . . . . . . Units of Mass . . . . . . . . . . . . . . . . . . Units of Volume . . . . . . . . . . . . . . . . Exercises . . . . . . . . . . . . . . . . . . . . Answers . . . . . . . . . . . . . . . . . . . . . 6.5 American Units to Metric Units and Vice-Versa Converting Units of Length . . . . . . . . . . Converting Units of Weight and Mass . . . . Converting Units of Volume . . . . . . . . . . Converting Units of Speed . . . . . . . . . . Exercises . . . . . . . . . . . . . . . . . . . . Answers . . . . . . . . . . . . . . . . . . . . . 7 Percent 7.1 Percent, Decimals, Fractions . . . . . . . . . Changing a Percent to a Fraction . . . . Changing a Percent to a Decimal . . . . . Changing a Decimal to a Percent . . . . . Changing a Fraction to a Percent . . . . Exercises . . . . . . . . . . . . . . . . . . Answers . . . . . . . . . . . . . . . . . . . 7.2 Solving Basic Percent Problems . . . . . . . Find a Given Percent of a Given Number Find a Percent Given Two Numbers . . . Find a Number that is a Given Percent of Exercises . . . . . . . . . . . . . . . . . .

ix 447 449 451 452 454 455 456 458 459 463 465 467 467 470 471 472 473 475 477 479 479 482 484 486 488 490 490 491 493 494 495 499 501 502 502 504 505 506 508 510 512 512 514 516 518

. . . . . . . . . . . . . . . . . . . . . . . . . . . . . . .

. . . . . . . . . . . . . . . . . . . . . . . . . . . . . . .

. . . . . . . . . . . . . . . . . . . . . . . . . . . . . . .

. . . . . . . . . . . . . . . . . . . . . . . . . . . . . . .

. . . . . . . . . . . . . . . . . . . . . . . . . . . . . . .

. . . . . . . . . . . . . . . . . . . . . . . . . . . . . . .

. . . . . . . . . . . . . . . . . . . . . . . . . . . . . . .

. . . . . . . . . . . . . . . . . . . . . . . . . . . . . . .

. . . . . . . . . . . . . . . . . . . . . . . . . . . . . . .

. . . . . . . . . . . . . . . . . . . . . . . . . . . . . . . . . . . . . . . . . . . . . . . . . . Another . . . . .

. . . . . . . . . . . . . . . . . . . . . . . . . . . . . . . . . . . . . . . . . . . . . . . . . . Number . . . . .

. . . . . . . . . . . .

Version: Fall 2010

x Answers . . . . . . . . . . . . . . . . . . General Applications of Percent . . . . . . Exercises . . . . . . . . . . . . . . . . . Answers . . . . . . . . . . . . . . . . . . Percent Increase or Decrease . . . . . . . Percent Increase . . . . . . . . . . . . . Percent Decrease . . . . . . . . . . . . . Discount . . . . . . . . . . . . . . . . . Exercises . . . . . . . . . . . . . . . . . Answers . . . . . . . . . . . . . . . . . . Interest . . . . . . . . . . . . . . . . . . . Extending the Simple Interest Formula Exercises . . . . . . . . . . . . . . . . . Answers . . . . . . . . . . . . . . . . . . Pie Charts . . . . . . . . . . . . . . . . . . Pie Charts . . . . . . . . . . . . . . . . Exercises . . . . . . . . . . . . . . . . . Answers . . . . . . . . . . . . . . . . . . . . . . . . . . . . . . . . . . . . . . . . . . . . . . . . . . . . . . . . . . . . . . . . . . . . . . . . . . . . . . . . . . . . . . . . . . . . . . . . . . . . . . . . . . . . . . . . . . . . . . . . . . . . . . . . . . . . . . . . . . . . . . . . . . . . . . . . . . . . . . . . . . . . . . . . . . . . . . . . . . . . . . . . . . . . . . . . . . . . . . . . . . . . . . . . . . . . . . . . . . . . . . . . . . . . . . . . . . . . . . . . . . .

CONTENTS . . . . . . . . . . . . . . . . . . . . . . . . . . . . . . . . . . . . . . . . . . . . . . . . . . . . . . . . . . . . . . . . . . . . . . . . . . . . . . . . . . . . . . . . . . . . . . . . . . . . . . . . . . . . . . . . . . . . . . . . . . . . . . . . . . . . . . . 519 521 525 528 529 529 531 534 538 541 542 545 548 550 552 553 560 566 569 570 572 576 580 581 582 585 589 591 595

7.3

7.4

7.5

7.6

8 Graphing 8.1 The Cartesian Coordinate System Allowing for Negative Numbers . Exercises . . . . . . . . . . . . . Answers . . . . . . . . . . . . . . 8.2 Graphing Linear Equations . . . . The Graph of an Equation . . . Linear Equations . . . . . . . . . Exercises . . . . . . . . . . . . . Answers . . . . . . . . . . . . . . Index

Version: Fall 2010

Chapter

The Whole Numbers


Welcome to the study of prealgebra. In this rst chapter of study, we will introduce the set of natural numbers, then follow with the set of whole numbers. We will then follow with a quick review of addition, subtraction, multiplication, and division skills involving whole numbers that are prerequisite for success in the study of prealgebra. Along the way we will introduce a number of properties of the whole numbers and show how that can be used to evaluate expressions involving whole number operations. We will also dene what is meant by prime and composite numbers, discuss a number of divisibility tests, then show how any composite number can be written uniquely as a product of prime numbers. This will lay the foundation for requisite skills with fractional numbers in later chapters. Finally, we will introduce the concept of a variable, then introduce equations and technique required for their solution. We will use equations to model and solve a number of real-world applications along the way. Lets begin the journey.

CHAPTER 1. THE WHOLE NUMBERS

1.1

An Introduction to the Whole Numbers

A set is a collection of objects. If the set is nite, we can describe the set completely by simply listing all the objects in the set and enclosing the list in curly braces. For example, the set S = {dog, cat, parakeet} is the set whose members are dog, cat, and parakeet. If the set is innite, then we need to be more clever with our description. For example, the set of natural numbers (or counting numbers) is the set N = {1, 2, 3, 4, 5, . . .}. Because this set is innite (there are an innite number of natural numbers), we cant list all of them. Instead, we list the rst few then follow with three dots, which essentially mean etcetera. The implication is that the reader sees the intended pattern and can then intuit the remaining numbers in the set. Can you see that the next few numbers are 6, 7, 8, 9, etc.? If we add the number zero to the set of natural numbers, then we have a set of numbers that are called the whole numbers. The Whole Numbers. The set W = {0, 1, 2, 3, 4, 5, . . .} is called the set of whole numbers. The whole numbers will be our focus in the remainder of this chapter.

Graphing numbers on the number line


It is a simple matter to set up a correspondence between the whole numbers and points on a number line. First, draw a number line, then set a tick mark at zero.

0 The next step is to declare a unit length.

The remainder of the whole numbers now fall easily in place on the number line. Version: Fall 2010

1.1. AN INTRODUCTION TO THE WHOLE NUMBERS

...

When asked to graph a whole number on a number line, shade in a solid dot at the position on the number line that corresponds to the given whole number. You Try It! EXAMPLE 1. Graph the whole numbers 1, 3, and 5 on the number line. Solution: Shade the numbers 1, 3, and 5 on the number line as solid dots. Graph the whole numbers 3, 4, and 6 on the number line.

...

Ordering the whole numbers


Now that we have a correspondence between the whole numbers and points on the number line, we can order the whole numbers in a natural way. Note that as you move to the left along the number line, the numbers get smaller; as you move to the right, the numbers get bigger. This inspires the following denition. Ordering the Whole Numbers. Suppose that a and b are whole numbers located on the number line so that the point representing the whole number a lies to the left of the point representing the whole number b.

Then the whole number a is less than the whole number b and write a < b. Alternatively, we can also say that the whole number b isgreater than the whole number a and write b > a.

Version: Fall 2010

CHAPTER 1. THE WHOLE NUMBERS

Comparison Property: When comparing two whole numbers a and b, only one of three possibilities is true: a<b or a=b or a > b.

You Try It! Compare the whole numbers 18 and 12. EXAMPLE 2. Compare the whole numbers 25 and 31. Solution: On the number line, 25 is located to the left of 31.

25

31

Answer: 18 > 12

Therefore, 25 is less than 31 and write 25 < 31. Alternatively, we could also note that 31 is located to the right of 25. Therefore, 31 is greater than 25 and write 31 > 25.

Expanded notation
The whole numbers D = {0, 1, 2, 3, 4, 5, 6, 7, 8, 9} are called digits and are used to construct larger whole numbers. For example, consider the whole number 222 (pronounced two hundred twenty two). It is made up of three twos, but the position of each two describes a dierent meaning or value. 2 hundreds 2 2

The rst two is in the hundreds position and represents two hundreds or 200. The second two is in the tens position and represents two tens or 20. The third two is in the ones position and represents two ones or 2. Consider the larger number 123,456,789. The following table shows the place value of each digit. Version: Fall 2010

ones

tens

1.1. AN INTRODUCTION TO THE WHOLE NUMBERS 1 hundred millions 2 3 4 hundred thousands 5 6 7 8 9

ten thousands

ten millions

thousands

hundreds

millions

millions

thousands

ones

In expanded notation, we would write 1 hundred million + 2 ten millions + 3 millions + 4 hundred thousands + 5 ten thousands + 6 thousands + 7 hundreds + 8 tens + 9 ones. We read the numeral 123,456,789 as one hundred twenty three million, four hundred fty six thousand, seven hundred eighty nine. Lets look at another example. You Try It! EXAMPLE 3. Write the number 23,712 in expanded notation, then pronounce the result. Solution: In expanded notation, 23,712 becomes 2 ten thousands + 3 thousands + 7 hundreds + 1 ten + 2 ones. This is pronounced twenty three thousand, seven hundred twelve. Write the number 54,615 in expanded notation. Pronounce the result.

ones

tens

You Try It! EXAMPLE 4. Write the number 203,405 in expanded notation, then pronounce the result. Solution: In expanded notation, 203,405 becomes 2 hundred thousands + 0 ten thousands + 3 thousands + 4 hundreds + 0 tens + 5 ones. Since 0 ten thousands is zero and 0 tens is also zero, this can also be written 2 hundred thousands + 3 thousands + 4 hundreds + 5 ones. This is pronounced two hundred three thousand, four hundred ve. Write the number 430, 705 in expanded notation. Pronounce the result.

Version: Fall 2010

CHAPTER 1. THE WHOLE NUMBERS

Rounding whole numbers


When less precision is needed, we round numbers to a particular place. For example, suppose a store owner needs approximately 87 boxes of ten-penny nails, but they can only be ordered in cartons containing ten boxes. Round up 80 81 82 83 84 85 86 87 88 89 90

Note that 87 is located closer to 9 tens (or 90) than it is to 8 tens (or 80). Thus, rounded to the nearest ten, 87 90 (87 approximately equals 90). The store owner decides that 90 boxes is probably a better t for his needs. On the other hand, the same store owner estimates that he will need 230 bags of peatmoss for his garden section. Round down 200 210 220 230 240 250 260 270 280 290 300

Note that 230 is closer to 2 hundreds (or 200) than it is to 3 hundreds (or 300). The store owner worries that might have overestimated his need, so he rounds down to the nearest hundred, 230 200 (230 approximately equals 200). There is a simple set of rules to follow when rounding. Rules for Rounding. To round a number to a particular place, follow these steps: 1. Mark the place you wish to round to. This is called the rounding digit . 2. Check the next digit to the right of your digit marked in step 1. This is called the test digit . a) If the test digit is greater than or equal to 5, add 1 to the rounding digit and replace all digits to the right of the rounding digit with zeros. b) If the test digit is less than 5, keep the rounding digit the same and replace all digits to the right of rounding digit with zeros.

Lets try these rules with an example or two. You Try It! Round the number 9,443 to the nearest ten. EXAMPLE 5. Round the number 8,769 to the nearest ten. Solution: Mark the rounding and test digits. Version: Fall 2010

1.1. AN INTRODUCTION TO THE WHOLE NUMBERS Test digit 8,7 6 9 Rounding digit

The test digit is greater than 5. The Rules for Rounding require that we add 1 to the rounding digit, then make all digits to the right of the rounding digit zeros. Thus, rounded to the nearest ten, 8, 769 8, 770. That is, 8,769 is approximately equal to 8,770. Answer: 9,440

Mathematical Notation. The symbol means approximately equal.

You Try It! EXAMPLE 6. Round the number 4,734 to the nearest hundred. Solution: Mark the rounding and test digits. Test digit 4, 7 3 4 Rounding digit The test digit is less than 5. The Rules for Rounding require that we keep the rounding digit the same, then make all digits to the right of the rounding digit zeros. Thus, rounded to the nearest hundred, 4, 734 4, 700. Answer: 6, 700 Round the number 6,656 to the nearest hundred.

Version: Fall 2010

8 Year Atmospheric CO2

CHAPTER 1. THE WHOLE NUMBERS 1965 319 1975 330 1985 344 1995 359 2005 378

Table 1.1: Atmospheric CO2 values (ppmv) derived from in situ air samples collected at Mauna Loa, Hawaii, USA.

Tables and graphs


Reading data in graphical form is an important skill. The data in Table 1.1 provides measures of the carbon dioxide content (CO2 ) in the atmosphere, gathered in the month of January at the observatory atop Mauna Loa in Hawaii. In Figure 1.1(a), a bar graph is used to display the carbon dioxide measurements. The year the measurement was taken is placed on the horizontal axis, and the height of each bar equals the amount of carbon dioxide in the atmosphere during that year. 500 Atmospheric CO2 (ppmv) 400 300 200 100 0 Atmospheric CO2 (ppmv) 1965 1975 1985 1995 2005 Year
(a) Bar graph.

500 400 300 200 100 0 1955 1965 1975 1985 1995 2005 2015 Year
(b) Line graph.

Figure 1.1: Using graphs to examine carbon dioxide data. In Figure 1.1(b), a line graph is used to display the carbon dioxide measurements. Again, the dates of measurement are placed on the horizontal axis, and the amount of carbon dioxide in the atmosphere is placed on the vertical axis. Instead of using the height of a bar to represent the carbon dioxide measurement, we place a dot at a height that represents the carbon monoxide content. Once each data point is plotted, we connect consecutive data points with line segments.

Version: Fall 2010

1.1. AN INTRODUCTION TO THE WHOLE NUMBERS

l l l

Exercises

l l l

In Exercises 1-12, sketch the given whole numbers on a number line, then arrange them in order, from smallest to largest. 1. 2, 8, and 4 2. 2, 7, and 4 3. 1, 8, and 2 4. 0, 4, and 3 5. 0, 4, and 1 6. 3, 6, and 5 7. 4, 9, and 6 8. 2, 4, and 3 9. 0, 7, and 4 10. 2, 8, and 6 11. 1, 6, and 5 12. 0, 9, and 5

In Exercises 13-24, create a number line diagram to determine which of the two given statements is true. 13. 3 < 8 or 3 > 8 19. 1 < 81 or 1 > 81 20. 65 < 83 or 65 > 83 21. 43 < 1 22. 62 < 2 or 43 > 1 or 62 > 2

14. 44 < 80 or 44 > 80 15. 59 < 24 or 59 > 24 16. 15 < 11 or 15 > 11 17. 0 < 74 or 0 > 74 18. 11 < 18 or 11 > 18

23. 43 < 28 or 43 > 28 24. 73 < 21 or 73 > 21

25. Which digit is in the thousands column of the number 2,054,867,372? 26. Which digit is in the hundreds column of the number 2,318,999,087? 27. Which digit is in the hundred thousands column of the number 8,311,900,272? 28. Which digit is in the tens column of the number 1,143,676,212? 29. Which digit is in the hundred millions column of the number 9,482,616,000? 30. Which digit is in the hundreds column of the number 375,518,067?

31. Which digit is in the ten millions column of the number 5,840,596,473? 32. Which digit is in the hundred thousands column of the number 6,125,412,255? 33. Which digit is in the hundred millions column of the number 5,577,422,501? 34. Which digit is in the thousands column of the number 8,884,966,835? 35. Which digit is in the tens column of the number 2,461,717,362? 36. Which digit is in the ten millions column of the number 9,672,482,548? Version: Fall 2010

10

CHAPTER 1. THE WHOLE NUMBERS

37. Round the number 93, 857 to the nearest thousand. 38. Round the number 56, 872 to the nearest thousand. 39. Round the number 9, 725 to the nearest ten. 40. Round the number 6, 815 to the nearest ten. 41. Round the number 58, 739 to the nearest hundred. 42. Round the number 93, 146 to the nearest hundred. 43. Round the number 2, 358 to the nearest ten. 44. Round the number 8, 957 to the nearest ten.

45. Round the number 39, 756 to the nearest thousand. 46. Round the number 24, 965 to the nearest thousand. 47. Round the number 5, 894 to the nearest ten. 48. Round the number 3, 281 to the nearest ten. 49. Round the number 56, 123 to the nearest hundred. 50. Round the number 49, 635 to the nearest hundred. 51. Round the number 5, 483 to the nearest ten. 52. Round the number 9, 862 to the nearest ten.

53. According to the U.S. Census Bureau, the estimated population of the US is 304,059,724 as of July 2008. Round to the nearest hundred thousand. 54. According to the U.S. Census Bureau, the estimated population of California is 36,756,666 as of July 2008. Round to the nearest hundred thousand.

55. According to the U.S. Census Bureau, the estimated population of Humboldt County is 129,000 as of July 2008. Round to the nearest ten thousand. 56. According to the U.S. Census Bureau, the estimated population of the state of Alasks was 686,293 as of July 2008. Round to the nearest ten thousand.

PPG Regular Gasoline

57. The following bar chart shows the average price (in cents) of one gallon of regular gasoline in the United States over ve consecutive weeks in 2009, running from May 18 (5/18) through June 22 (6/22). What was the price (in cents) of one gallon of regular gasoline on June 1, 2009?

280 270 260 250 240 230 220 5/18 5/25 6/1 6/15 6/22 Year

Version: Fall 2010

1.1. AN INTRODUCTION TO THE WHOLE NUMBERS 58. The following bar chart shows the average weekly NASDAQ index for ve consecutive weeks in 2009, beginning with week starting February 1 (2/1) and ending with the week starting March 1 (3/1). What was the average NASDAQ index for the week starting February 8, 2009? 1650 NASDAQ Weekly Index 1600 1550 1500 1450 1400 1350 1300 1250 2/1 2/8

11

2/15 2/22 Week

3/1

Number of pirate attacks

59. The population of Humboldt County is broken into age brackets in the following table. Source: WolframAlpha. Age in years Number under 5 7,322 26,672 5-18 18-65 78,142 over 65 16,194 Create a bar chart for this data set with one bar for each age category.

61. The following bar chart tracks pirate attacks o the coast of Somalia. 120 110 100 90 80 70 60 50 40 30 20 10 0 2003 2004 2005 2006 2007 2008 Year

60. The ve cities with the largest number of reported violent crimes in the year 2007 are reported in the following table. Source: Wikipedia. Violent Crimes City Detroit 2,289 St. Louis 2,196 1,951 Memphis Oakland 1,918 1,631 Baltimore Create a bar chart for this data set with one bar for each city.

Source: ICC International Maritime Bureau, AP Times-Standard, 4/15/2009 a) How many pirate attacks were there in 2003? b) How many pirate attacks were there in 2008? Version: Fall 2010

12

CHAPTER 1. THE WHOLE NUMBERS 63. A team of students separated a small bowl of M and Ms into ve piles by color. The following line plot indicates the number of M and Ms of each color. 10 9 8 7 6 5 4 3 2 1 0

62. A team of students separated a small bowl of M and Ms into ve piles by color. The following line plot indicates the number of M and Ms of each color. 10 9 8 7 6 5 4 3 2 1 0

Number of M and Ms.

Brown

Number of M and Ms.

Yellow

How many red M and Ms were in the bowl?

How many red M and Ms were in the bowl?

64. A team of students separated a small bowl of M and Ms into ve piles by color. The following table indicates the number of M and Ms of each color. Color Number Red 5 9 Green Blue 7 Yellow 2 3 Brown Create a lineplot for the M and M data. On the horizontal axis, arrange the colors in the same order as presented in the table above.

65. A team of students separated a small bowl of M and Ms into ve piles by color. The following table indicates the number of M and Ms of each color. Color Number Red 3 7 Green Blue 2 Yellow 4 9 Brown Create a lineplot for the M and M data. On the horizontal axis, arrange the colors in the same order as presented in the table above.

Version: Fall 2010

Yellow

Brown

Green

Blue

Green

Blue

Red

Red

1.1. AN INTRODUCTION TO THE WHOLE NUMBERS 66. Salmon count. The table shows the number of adult coho salmon returning to the Shasta River over the past four years. Round the salmon count for each year to the nearest ten. Times-Standard Shasta River coho rescue underway. Year 2007 2008 2009 2010 Salmon count 300 31 9 4

13

l l l
1. Smallest to largest: 2, 4, and 8. 0 1 2 3 4 5 6 7 8 9 3. Smallest to largest: 1, 2, and 8.

Answers
13.

l l l

3 Therefore, 3 < 8. 15.

0 1 2 3 4 5 6 7 8 9 5. Smallest to largest: 0, 1, and 4. 0 1 2 3 4 5 6 7 8 9 7. Smallest to largest: 4, 6, and 9. 0 1 2 3 4 5 6 7 8 9 9. Smallest to largest: 0, 4, and 7. 0 1 2 3 4 5 6 7 8 9 11. Smallest to largest: 1, 5, and 6. 0 1 2 3 4 5 6 7 8 9 21. 17.

24 Therefore, 59 > 24.

59

0 Therefore, 0 < 74. 19. 1 Therefore, 1 < 81.

74

81

1 Therefore, 43 > 1.

43

Version: Fall 2010

14 23. 28 Therefore, 43 > 28. 25. 7 27. 9 29. 4 31. 4 33. 5 35. 6 37. 94000 43

CHAPTER 1. THE WHOLE NUMBERS 59.

80000 70000 Humboldt Population 60000 50000 40000 30000 20000 10000 18-65 Yellow Blue 5-18 under 5 over 65 Brown 0

61. 39. 9730 41. 58700 43. 2360 45. 40000 47. 5890 49. 56100 51. 5480 53. 304,100,000 55. 130,000 57. Approximately 252 cents Number of M and Ms. 63. 9 65.

a) Approximately 21 b) Approximately 111

Version: Fall 2010

Green

Red

10 9 8 7 6 5 4 3 2 1 0

1.2. ADDING AND SUBTRACTING WHOLE NUMBERS

15

1.2

Adding and Subtracting Whole Numbers

In the expression 3 + 4, which shows the sum of two whole numbers, the whole numbers 3 and 4 are called addends or terms. We can use a visual approach to nd the sum of 3 and 4. First, construct a number line as shown in Figure 1.2. Start 3 0 1 2 3 4 4 5 6 7 8 End

Figure 1.2: Adding whole numbers on a number line. To add 3 and 4, proceed as follows. 1. Start at the number 0, then draw an arrow 3 units to the right, as shown in Figure 1.2. This arrow has magnitude (length) three and represents the whole number 3. 2. Draw a second arrow of length four, starting at the end of the rst arrow representing the number 3. This arrow has magnitude (length) four and represents the whole number 4. 3. The sum of 3 and 4 could be represented by an arrow that starts at the number 0 and ends at the number 7. However, we prefer to mark this sum on the number line as a solid dot at the whole number 7. This number represents the sum of the whole numbers 3 and 4.

The Commutative Property of Addition


Lets change the order in which we add the whole numbers 3 and 4. That is, lets nd the sum 4 + 3 instead. Start 4 0 1 2 3 4 5 3 6 7 8 End

Figure 1.3: Addition is commutative; i.e., order doesnt matter. As you can see in Figure 1.3, we start at zero then draw an arrow of length four, followed by an arrow of length three. However, the result is the same; i.e., 4 + 3 = 7. Thus, the order in which we add three and four does not matter; that is, 3 + 4 = 4 + 3. Version: Fall 2010

16

CHAPTER 1. THE WHOLE NUMBERS

This property of addition of whole numbers is known as the commutative property of addition. The Commutative Property of Addition. Let a and b represent two whole numbers. Then, a + b = b + a.

Grouping Symbols
In mathematics, we use grouping symbols to aect the order in which an expression is evaluated. Whether we use parentheses, brackets, or curly braces, the expression inside any pair of grouping symbols must be evaluated rst. For example, note how we rst evaluate the sum in the parentheses in the following calculation. (3 + 4) + 5 = 7 + 5 = 12 The rule is simple: Whatever is inside the parentheses is evaluated rst. Writing Mathematics. When writing mathematical statements, follow the mantra: One equal sign per line.

We can use brackets instead of parentheses. 5 + [7 + 9] = 5 + 16 = 21 Again, note how the expression inside the brackets is evaluated rst. We can also use curly braces instead of parentheses or brackets. {2 + 3} + 4 = 5 + 4 =9 Again, note how the expression inside the curly braces is evaluated rst. If grouping symbols are nested, we evaluate the innermost parentheses rst. For example, 2 + [3 + (4 + 5)] = 2 + [3 + 9] = 2 + 12 = 14. Version: Fall 2010

1.2. ADDING AND SUBTRACTING WHOLE NUMBERS

17

Grouping Symbols. Use parentheses, brackets, or curly braces to delimit the part of an expression you want evaluated rst. If grouping symbols are nested, evaluate the expression in the innermost pair of grouping symbols rst.

The Associative Property of Addition


Consider the evaluation of the expression (2+3)+4. We evaluate the expression in parentheses rst.

(2 + 3) + 4 = 5 + 4 =9 Now, suppose we change the order of addition to 2 + (3 + 4). Then,

2 + (3 + 4) = 2 + 7 = 9. Although the grouping has changed, the result is the same. That is, (2 + 3) + 4 = 2 + (3 + 4). This property of addition of whole numbers is called the associate property of addition. Associate Property of Addition. Let a, b, and c represent whole numbers. Then, (a + b) + c = a + (b + c).

Because of the associate property of addition, when presented with a sum of three numbers, whether you start by adding the rst two numbers or the last two numbers, the resulting sum is the same.

The Additive Identity


Imagine a number line visualization of the sum of four and zero; i.e., 4 + 0. In Figure 1.4, we start at zero, then draw an arrow of magnitude (length) four pointing to the right. Now, at the end of this arrow, attach a second arrow of length zero. Of course, that means that we remain right where we are, at 4. Hence the shaded dot at 4 is the sum. That is, 4 + 0 = 4.

Version: Fall 2010

18 Start 4 0 1 2 3

CHAPTER 1. THE WHOLE NUMBERS End

Figure 1.4: Adding zero to four.

The Additive Identity Property. The whole number zero is called the additive identity. If a is any whole number, then a + 0 = a.

The number zero is called the additive identity because if you add zero to any number, you get the identical number back.

Adding Larger Whole Numbers


For completeness, we include two examples of adding larger whole numbers. Hopefully, the algorithm is familiar from previous coursework. You Try It! Simplify: 1, 286 + 349 EXAMPLE 1. Simplify: 1, 234 + 498.

Solution. Align the numbers vertically, then add, starting at the furthest column to the right. Add the digits in the ones column, 4 + 8 = 12. Write the 2, then carry a 1 to the tens column. Next, add the digits in the tens column, 3 + 9 = 12, add the carry to get 13, then write the 3 and carry a 1 to the hundreds column. Continue in this manner, working from right to left.
1 1

1 2 34 + 4 98 1 7 32 Answer: 1,635 Therefore, 1, 234 + 498 = 1, 732.

Add three or more numbers in the same manner. You Try It! Simplify: 256 + 342 + 283 EXAMPLE 2. Simplify: Version: Fall 2010 256 + 322 + 418.

1.2. ADDING AND SUBTRACTING WHOLE NUMBERS

19

Solution. Align the numbers vertically, then add, starting at the furthest column to the right. Add the digits in the ones column, 6 + 2 + 8 = 16. Write the 6, then carry a 1 to the tens column. Continue in this manner, working from right to left.
1

2 3 + 4 9 Therefore, 256 + 322 + 418 = 996.

5 2 1 9

6 2 8 6 Answer: 881

Subtraction of Whole Numbers


The key idea is this: Subtraction is the opposite of addition. For example, consider the dierence 7 4 depicted on the number line in Figure 1.5. End Start 7 0 1 2 3 4 5 6 7 8 4

Figure 1.5: Subtraction means add the opposite. If we were adding 7 and 4, we rst draw an arrow starting at zero pointing to the right with magnitude (length) seven. Then, to add 4, we would draw a second arrow of magnitude (length) 4, attached to the end of the rst arrow and pointing to the right. However, because subtraction is the opposite of addition, in Figure 1.5 we attach an arrow of magnitude (length) four to the end of the rst arrow, but pointing in the opposite direction (to the left). Note that this last arrow ends at the answer, which is a shaded dot on the number line at 3. That is, 7 4 = 3. Note that subtraction is not commutative; that is, it make no sense to say that 7 5 is the same as 5 7. Subtraction is not associative. It is not the case that (9 5) 2 is the same as 9 (5 2). On the one hand, (9 5) 2 = 4 2 = 2, Version: Fall 2010

20 but

CHAPTER 1. THE WHOLE NUMBERS

9 (5 2) = 9 3 = 6.

Subtracting Larger Whole Numbers


Much as we did with adding larger whole numbers, to subtract two large whole numbers, align them vertically then subtract, working from right to left. You may have to borrow to complete the subtraction at any step. You Try It! Simplify: 5, 635 288. EXAMPLE 3. Simplify: 1, 755 328.

Solution. Align the numbers vertically, then subtract, starting at the ones column, then working right to left. At the ones column, we cannot subtract 8 from 5, so we borrow from the previous column. Now, 8 from 15 is 7. Continue in this manner, working from right to left.
4

1 7 5 15 3 2 8 1 4 2 7 Answer: 5,347 Therefore, 1, 755 328 = 1, 427.

Order of Operations
In the absence of grouping symbols, it is important to understand that addition holds no precedence over subtraction, and vice-versa. Perform all additions and subtractions in the order presented, moving left to right. Lets look at an example. You Try It! Simplify: 25 10 + 8. EXAMPLE 4. Simplify the expression 15 8 + 4. Solution. This example can be trickier than it seems. However, if we follow the rule (perform all additions and subtractions in the order presented, moving left to right), we should have no trouble. First comes fteen minus eight, which is seven. Then seven plus four is eleven. Version: Fall 2010

1.2. ADDING AND SUBTRACTING WHOLE NUMBERS

21

15 8 + 4 = 7 + 4 = 11. Answer: 23

Caution! Incorrect answer ahead! Note that it is possible to arrive at a different (but incorrect) answer if we favor addition over subtraction in Example 4. If we rst add eight and four, then 15 8 + 4 becomes 15 12, which is 3. However, note that this is incorrect, because it violates the rule perform all additions and subtractions in the order presented, moving left to right.

Applications Geometry
There are any number of applications that require a sum or dierence of whole numbers. Lets examine a few from the world of geometry. Perimeter of a Polygon. In geometry a polygon is a plane gure made up of a closed path of a nite sequence of segments. The segments are called the edges or sides of the polygon and the points where two edges meet are called the vertices of the polygon. The perimeter of any polygon is the sum of the lengths of its sides.

You Try It! EXAMPLE 5. A quadrilateral is a polygon with four sides. Find the perimeter of the quadrilateral shown below, where the sides are measured in yards. A quadrilateral has sides that measure 4 in., 3 in., 5 in., and 5 in. Find the perimeter.

3y 3y d

d
4 yd

5 yd Solution. To nd the perimeter of the quadrilateral, nd the sum of the lengths of the sides. Perimeter = 3 + 3 + 4 + 5 = 15 Hence, the perimeter of the quadrilateral is 15 yards. Version: Fall 2010 Answer: 17 inches

22

CHAPTER 1. THE WHOLE NUMBERS

You Try It! A rectangle has length 12 meters and width 8 meters. Find its perimeter. EXAMPLE 6. A quadrilateral (four sides) is a rectangle if all four of its angles are right angles. It can be shown that the opposite sides of a rectangle must be equal. Find the perimeter of the rectangle shown below, where the sides of the rectangle are measured in meters.

3m

5m Solution. To nd the perimeter of the rectangle, nd the sum of the four sides. Because opposite sides have the same length, we have two sides of length 5 meters and two sides of length 3 meters. Hence, Perimeter = 5 + 3 + 5 + 3 = 16. Answer: 40 meters Thus, the perimeter of the rectangle is 16 meters.

You Try It! A square has a side that measures 18 centimeters. Find its perimeter. EXAMPLE 7. A quadrilateral (four sides) is a square if all four of its sides are equal and all four of its angles are right angles. Pictured below is a square having a side of length 12 feet. Find the perimeter of the square.

12 ft

Solution. Because the quadrilateral is a square, all four sides have the same length, namely 12 feet. To nd the perimeter of the square, nd the sum of the four sides. Perimeter = 12 + 12 + 12 + 12 = 48 Answer: 72 centimeters Hence, the perimeter of the square is 48 feet.

Version: Fall 2010

1.2. ADDING AND SUBTRACTING WHOLE NUMBERS

23

Application Alternative Fuels


Automobiles that run on alternative fuels (other than gasoline) have increased in the United States over the years. You Try It! EXAMPLE 8. Table 1.2 show the number of cars (in thousands) running on compressed natural gas versus the year. Create a bar chart showing the number of cars running on compressed natural gas versus the year. Year Number 1992 23 1993 32 1994 41 1995 50 1996 60 1997 73 1998 78 1999 89 2000 101 The following table shows the number of hybrid cars (in thousands) by country. Country U.S. Japan Canada U.K. Netherlands Number 279 77 17 14 11

Table 1.2: Number of vehicles (in thousands) running on compressed natural gas.

Solution. Place the years on the horizontal axis. At each year, sketch a bar having height equal to the number of cars in that year that are running on compressed natural gas. Scale the vertical axis in thousands. 120 110 100 90 80 70 60 50 40 30 20 10 0

Create a bar chart showing the number of cars versus the country of use.

Vehicles (thousands)

1992 1993 1994 1995 1996 1997 1998 1999 2000 Year

Version: Fall 2010

24

CHAPTER 1. THE WHOLE NUMBERS

You Try It! The following table show Alphonsos percentage scores on his examinations in mathematics. Exam Exam #1 Exam #2 Exam #3 Exam #4 Exam #5 Percentage 52 45 72 889 76 EXAMPLE 9. Using the data in Table 1.2, create a table that shows the dierences in consecutive years, then create a line plot of the result. In what consecutive years did the United States see the greatest increase in cars powered by compressed natural gas? Solution. Table 1.3 shows the dierences in consecutive years. Years Dierence 92-93 9 93-94 9 94-95 9 95-96 10 96-97 13 97-98 5 98-99 11 99-00 12

Table 1.3: Showing the dierences in vehicles in consecutive years. Next, craft a line graph. Place consecutive years on the horizontal axis. At each consecutive year pair, plot a point at a height equal to the dierence in alternative fuel vehicles. Connect the points with straight line segments. 15 14 13 12 11 10 9 8 7 6 5 4 3 2 1 0 92-93 93-94 94-95 95-96 96-97 97-98 98-99 99-00 Consecutive Years Note how the line graph makes it completely clear that the greatest increase in vehicles powered by compressed natural gas occurred in the consecutive years 1996-1997, an increase of 13,000 vehicles.

Construct a line graph of Alphonsos exam scores versus exam number.

Version: Fall 2010

Dierence (thousands)

1.2. ADDING AND SUBTRACTING WHOLE NUMBERS

25

l l l

Exercises

l l l

1. Sketch a number line diagram depicting the sum 3 + 2, as shown in Figure 1.2 in the narrative of this section. 2. Sketch a number line diagram depicting the sum 3 + 5, as shown in Figure 1.2 in the narrative of this section. 3. Sketch a number line diagram depicting the sum 3 + 4, as shown in Figure 1.2 in the narrative of this section. 4. Sketch a number line diagram depicting the sum 2 + 4, as shown in Figure 1.2 in the narrative of this section. 5. Sketch a number line diagram depicting the sum 4 + 2, as shown in Figure 1.2 in the narrative of this section.

6. Sketch a number line diagram depicting the sum 4 + 3, as shown in Figure 1.2 in the narrative of this section. 7. Sketch a number line diagram depicting the sum 2 + 5, as shown in Figure 1.2 in the narrative of this section. 8. Sketch a number line diagram depicting the sum 4 + 5, as shown in Figure 1.2 in the narrative of this section. 9. Sketch a number line diagram depicting the sum 4 + 4, as shown in Figure 1.2 in the narrative of this section. 10. Sketch a number line diagram depicting the sum 3 + 3, as shown in Figure 1.2 in the narrative of this section.

In Exercises 11-28, determine which property of addition is depicted by the given identity. 11. 28 + 0 = 28 12. 53 + 0 = 53 13. 24 + 0 = 24 14. 93 + 0 = 93 15. (51 + 66) + 88 = 51 + (66 + 88) 16. (90 + 96) + 4 = 90 + (96 + 4) 17. 64 + 39 = 39 + 64 18. 68 + 73 = 73 + 68 19. (70 + 27) + 52 = 70 + (27 + 52) 20. (8 + 53) + 81 = 8 + (53 + 81) 21. 79 + 0 = 79 22. 42 + 0 = 42 23. 10 + 94 = 94 + 10 24. 55 + 86 = 86 + 55 25. 47 + 26 = 26 + 47 26. 62 + 26 = 26 + 62 27. (61 + 53) + 29 = 61 + (53 + 29) 28. (29 + 96) + 61 = 29 + (96 + 61)

29. Sketch a number line diagram depicting the dierence 82, as shown in Figure 1.5 in the narrative of this section.

30. Sketch a number line diagram depicting the dierence 84, as shown in Figure 1.5 in the narrative of this section.

Version: Fall 2010

26

CHAPTER 1. THE WHOLE NUMBERS 35. Sketch a number line diagram depicting the dierence 94, as shown in Figure 1.5 in the narrative of this section. 36. Sketch a number line diagram depicting the dierence 65, as shown in Figure 1.5 in the narrative of this section. 37. Sketch a number line diagram depicting the dierence 85, as shown in Figure 1.5 in the narrative of this section. 38. Sketch a number line diagram depicting the dierence 93, as shown in Figure 1.5 in the narrative of this section.

31. Sketch a number line diagram depicting the dierence 72, as shown in Figure 1.5 in the narrative of this section. 32. Sketch a number line diagram depicting the dierence 95, as shown in Figure 1.5 in the narrative of this section. 33. Sketch a number line diagram depicting the dierence 74, as shown in Figure 1.5 in the narrative of this section. 34. Sketch a number line diagram depicting the dierence 64, as shown in Figure 1.5 in the narrative of this section.

In Exercises 39-50, simplify the given expression. 39. 16 8 + 2 40. 17 3 + 5 41. 20 5 + 14 42. 14 5 + 6 43. 15 2 + 5 44. 13 4 + 2 45. 12 5 + 4 46. 19 4 + 13 47. 12 6 + 4 48. 13 4 + 18 49. 15 5 + 8 50. 13 3 + 11

In Exercises 51-58, the width W and length L of a rectangle are given. Find the perimeter P of the rectangle. 51. W = 7 in, L = 9 in 52. W = 4 in, L = 6 in 53. W = 8 in, L = 9 in 54. W = 5 in, L = 9 in 55. W = 4 cm, L = 6 cm 56. W = 5 in, L = 8 in 57. W = 4 cm, L = 7 cm 58. W = 4 in, L = 9 in

In Exercises 59-66, the length s of a side of a square is given. Find the perimeter P of the square. 59. s = 25 cm 60. s = 21 in 61. s = 16 cm Version: Fall 2010 62. s = 10 in 63. s = 18 in 64. s = 7 in

1.2. ADDING AND SUBTRACTING WHOLE NUMBERS 65. s = 3 in 66. s = 20 in

27

In Exercises 67-86, nd the sum. 67. 3005 + 5217 68. 1870 + 5021 69. 575 + 354 + 759 70. 140 + 962 + 817 71. 472 + (520 + 575) 72. 318 + (397 + 437) 73. 274 + (764 + 690) 74. 638 + (310 + 447) 75. 8583 + 592 76. 5357 + 9936 77. 899 + 528 + 116 78. 841 + 368 + 919 79. (466 + 744) + 517 80. (899 + 996) + 295 81. 563 + 298 + 611 + 828 82. 789 + 328 + 887 + 729 83. 607 + 29 + 270 + 245 84. 738 + 471 + 876 + 469 85. (86 + 557) + 80 86. (435 + 124) + 132

In Exercises 87-104, nd the dierence. 87. 3493 2034 227 88. 3950 1530 2363 89. 8338 7366 90. 2157 1224 91. 2974 2374 92. 881 606 93. 3838 (777 241) 94. 8695 (6290 4233) 95. 5846 541 4577 96. 5738 280 4280 97. 3084 (2882 614) 98. 1841 (217 28) 99. 2103 (1265 251) 100. 1471 (640 50) 101. 9764 4837 150 102. 9626 8363 1052 103. 7095 226 104. 4826 1199

105. Water Subsidies. Since the drought began in 2007, California farms have received $79 million in water subsidies. California cotton and rice farmers received an additional $439 million. How much

total water subsidies have farmers received? Associated Press Times-Standard 4/15/09

Version: Fall 2010

28

CHAPTER 1. THE WHOLE NUMBERS 111. Sunspot Temperature. The surface of the sun is about 10,000 degrees Fahrenheit. Sunspots are darker regions on the surface of the sun that have a relatively cooler temperature of 6,300 degrees Fahrenheit. How many degrees cooler are sunspots? 112. Jobs. The Times-Standard reports that over the next year, the credit- and debitcard processing business Humboldt Merchant Services expects to cut 36 of its 80 jobs, but then turn around and hire another 21. How many people will be working for the company then? TimesStandard 5/6/09 113. Wild tigers. The chart shows the estimated wild tiger population, by region. According to this chart, what is the total wild tiger population worldwide? Associated Press-Times-Standard 01/24/10 Pressure mounts to save the tiger. Region India, Nepal and Bhutan China and Russia Bangladesh Sumatra (Indonesia) Malaysia other SE Asia Tiger population 1650 450 250 400 500 350

106. War Budget. The 2010 Federal budget allocates $534 billion for the Department of Defense base programs and an additional $130 billion for the nations two wars. How much will the Department of Defense receive altogether? Associated Press Times-Standard 5/8/09 107. Sun Frost. Arcata, CA is home to Sun Frost, a manufacturer of highly efcient refrigerators and freezers. The AC model RF12 refrigerator/freezer costs $2,279 while an R16 model refrigerator/freezer costs $3,017. How much more does the R16 model cost? Source: www.sunfrost.com/retail pricelist.html 108. Shuttle Orbit. The space shuttle usually orbits at 250 miles above the surface of the earth. To service the Hubble Space Telescope, the shuttle had to go to 350 miles above the surface. How much higher did the shuttle have to orbit? 109. Earths Orbit. Earth orbits the sun in an ellipse. When earth is at its closest to the sun, called perihelion, earth is about 147 million kilometers. When earth is at its furthest point from the sun, called aphelion, earth is about 152 million kilometers from the sun. Whats the difference in millions of kilometers between aphelion and perihelion? 110. Plutos Orbit. Plutos orbit is highly eccentric. Find the dierence between Plutos closest approach to the sun and Plutos furthest distance from the sun if Plutos perihelion (closest point on its orbit about the sun) is about 7 billion kilometers and its aphelion (furthest point on its orbit about the sun) is about 30 billion kilometers.

Version: Fall 2010

1.2. ADDING AND SUBTRACTING WHOLE NUMBERS 114. Pirate Attacks. The following bar chart tracks pirate attacks o the coast of Somalia. 120 110 Number of pirate attacks 100 90 80 70 60 50 40 30 20 10 2003 2004 2005 2006 2007 2008 Year Source: ICC International Maritime Bureau, AP Times-Standard, 4/15/2009 a) How many pirate attacks were there in 2003, 2004, and 2005 combined? b) How many pirate attacks were there in 2006, 2007, and 2008 combined? c) How many more pirate attacks were there in 2008 than in 2007? 0

29

115. Emily shows improvement on each successive examination throughout the term. Her exam scores are recorded in the following table. Score Exam Exam #1 48 Exam #2 51 54 Exam #3 Exam #4 59 Exam #5 67 70 Exam #6 a) Create a bar plot for Emilys examination scores. Place the examination numbers on the horizontal axis in the same order shown in the table above. b) Create a table that shows successive dierences in examination scores. Make a line plot of these dierences. Between which two exams did Emily show the greatest improvement? 116. Jason shows improvement on each successive examination throughout the term. His exam scores are recorded in the following table. Score Exam Exam #1 34 42 Exam #2 Exam #3 45 50 Exam #4 Exam #5 57 Exam #6 62 a) Create a bar plot for Jasons examination scores. Place the examination numbers on the horizontal axis in the same order shown in the table above. b) Create a table that shows successive dierences in examination scores. Make a line plot of these dierences. Between which two exams did Jason show the greatest improvement? Version: Fall 2010

30

CHAPTER 1. THE WHOLE NUMBERS

l l l
1. 3 + 2 = 5. Start 3

Answers

l l l

23. Commutative property of addition End 2 25. Commutative property of addition 27. Associative property of addition 29. 8 2 = 6. End End 4 Start 8 0 1 2 3 4 5 6 7 8 9 31. 7 2 = 5. End Start 7 0 1 2 3 4 5 6 7 8 9 2 2

0 1 2 3 4 5 6 7 8 9 3. 3 + 4 = 7. Start 3

0 1 2 3 4 5 6 7 8 9 5. 4 + 2 = 6. Start 4

End 2

0 1 2 3 4 5 6 7 8 9 7. 2 + 5 = 7. Start 2

End 5

33. 7 4 = 3. End Start 7 0 1 2 3 4 5 6 7 8 9 4

0 1 2 3 4 5 6 7 8 9 9. 4 + 4 = 8. Start 4

End 4 35. 9 4 = 5. End Start 9 4

0 1 2 3 4 5 6 7 8 9 11. Additive identity property of addition.

0 1 2 3 4 5 6 7 8 9 13. Additive identity property of addition. 15. Associative property of addition 17. Commutative property of addition 19. Associative property of addition 21. Additive identity property of addition. Version: Fall 2010 39. 10 37. 8 5 = 3. End Start 8 0 1 2 3 4 5 6 7 8 9 5

1.2. ADDING AND SUBTRACTING WHOLE NUMBERS 41. 29 43. 18 45. 11 47. 10 49. 18 51. P = 32 in 95. 728 53. P = 34 in 55. P = 20 cm 97. 816 57. P = 22 cm 59. P = 100 cm 61. P = 64 cm 63. P = 72 in 65. P = 12 in 67. 8222 69. 1688 71. 1567 73. 1728 75. 9175 77. 1543 79. 1727 81. 2300 111. 3,700 degrees Fahrenheit 83. 1151 85. 723 113. 3600 87. 1232 109. 5 million kilometers 107. $738 105. $518 million 103. 6869 101. 4777 99. 1089 93. 3302 91. 600 89. 972

31

Version: Fall 2010

32 115. a) Bar chart. 80

CHAPTER 1. THE WHOLE NUMBERS

70 Exam Score

60

50

40

#1

#2

#3

#4

#5

#6

Exams b) Line plot of consecutive dierences. The line plot of consecutive examination score dierences.

1-2

2-3

3-4

4-5

The largest improvement was between Exam #4 and Exam #5, where Emily improved by 8 points.

Version: Fall 2010

5-6

10 9 8 7 6 5 4 3 2 1 0

Score Dierence

1.3. MULTIPLICATION AND DIVISION OF WHOLE NUMBERS

33

1.3

Multiplication and Division of Whole Numbers

We begin this section by discussing multiplication of whole numbers. The rst order of business is to introduce the various symbols used to indicate multiplication of two whole numbers. Mathematical symbols that indicate multiplication. Symbol ( ) Example 34 34 (3)(4) or 3(4) of (3)4

times symbol dot parentheses

Products and Factors. In the expression 3 4, the whole numbers 3 and 4 are called the factors and 3 4 is called the product. The key to understanding multiplication is held in the following statement. Multiplication is equivalent to repeated addition. Suppose, for example, that we would like to evaluate the product 3 4. Because multiplication is equivalent to repeated addition, 3 4 is equivalent to adding three fours. That is, 34=4+4+4
three fours

Thus, 3 4 = 12. You can visualize the product 3 4 as the sum of three fours on a number line, as shown in Figure 1.6. Start 4 0 1 2 3 4 5 4 6 7 8 9 4 10 11 12 End

Figure 1.6: Note that 3 4 = 4 + 4 + 4. That is, 3 4 = 12. Like addition, the order of the factors does not matter. 43 =3+3+3+3
four threes

Version: Fall 2010

34

CHAPTER 1. THE WHOLE NUMBERS

Thus, 4 3 = 12. Consider the visualization of 4 3 in Figure 1.7. Start 3 0 1 2 3 4 3 5 6 7 3 8 9 10 3 11 12 End

Figure 1.7: Note that 4 3 = 3 + 3 + 3 + 3. That is, 4 3 = 12. The evidence in Figure 1.6 and Figure 1.7 show us that multiplication is commutative. That is, 3 4 = 4 3. Commutative Property of Multiplication. If a and b are any whole numbers, then a b = b a.

The Multiplicative Identity


In Figure 1.8(a), note that ve ones equals 5; that is, 5 1 = 5. On the other hand, in Figure 1.8(b), we see that one ve equals ve; that is, 1 5 = 5. Start 1 0 1 1 2 1 3 1 4 End 1 5 Start 5 0 1 2 3 4 5 End

(a) Note that 5 1 = 1 + 1 + 1 + 1 + 1.

(b) Note that 1 5 = 5.

Figure 1.8: Note that 5 1 = 5 and 1 5 = 5. Because multiplying a whole number by 1 equals that identical number, the whole number 1 is called the multiplicative identity. The Multiplicative Identity Property. If a is any whole number, then a1 = a and 1 a = a.

Multiplication by Zero
Because 3 4 = 4 + 4 + 4, we can say that the product 3 4 represents 3 sets of 4, as depicted in Figure 1.9, where three groups of four boxes are each enveloped in an oval. Version: Fall 2010

1.3. MULTIPLICATION AND DIVISION OF WHOLE NUMBERS

35

Figure 1.9: Three sets of four: 3 4 = 12. Therefore, 0 4 would mean zero sets of four. Of course, zero sets of four is zero. Multiplication by Zero. If a represents any whole number, then a0=0 and 0 a = 0.

The Associative Property of Multiplication


Like addition, multiplication of whole numbers is associative. Indeed, 2 (3 4) = 2 12 = 24, and (2 3) 4 = 6 4 = 24.

The Associative Property of Multiplication. If a, b, and c are any whole numbers, then a (b c) = (a b) c.

Multiplying Larger Whole Numbers


Much like addition and subtraction of large whole numbers, we will also need to multiply large whole numbers. Again, we hope the algorithm is familiar from previous coursework.

Version: Fall 2010

36

CHAPTER 1. THE WHOLE NUMBERS

You Try It! Simplify: 56 335. EXAMPLE 1. Simplify: 35 127.

Solution. Align the numbers vertically. The order of multiplication does not matter, but well put the larger of the two numbers on top of the smaller number. The rst step is to multiply 5 times 127. Again, we proceed from right to left. So, 5 times 7 is 35. We write the 5, then carry the 3 to the tens column. Next, 5 times 2 is 10. Add the carry digit 3 to get 13. Write the 3 and carry the 1 to the hundreds column. Finally, 5 times 1 is 5. Add the carry digit to get 6.
1 3

1 2 7 3 5 6 3 5 The next step is to multiply 3 times 127. However, because 3 is in the tens place, its value is 30, so we actually multiply 30 times 126. This is the same as multiplying 127 by 3 and placing a 0 at the end of the result.
2

1 6 3 8

2 3 3 1

7 5 5 0

After adding the 0, 3 times 7 is 21. We write the 1 and carry the 2 above the 2 in the tens column. Then, 3 times 2 is 6. Add the carry digit 2 to get 8. Finally, 3 times 1 is 1. All that is left to do is to add the results. 1 6 3 8 4 4 Thus, 35 127 = 4, 445. Alternate Format. It does not hurt to omit the trailing zero in the second step of the multiplication, where we multiply 3 times 127. The result would look like this: 1 6 3 8 4 4 2 3 3 1 4 7 5 5 5 2 3 3 1 4 7 5 5 0 5

Version: Fall 2010

1.3. MULTIPLICATION AND DIVISION OF WHOLE NUMBERS

37

In this format, the zero is understood, so it is not necessary to have it physically present. The idea is that with each multiplication by a new digit, we indent the product one space from the right.

Answer: 18, 760

Division of Whole Numbers


We now turn to the topic of division of whole numbers. We rst introduce the various symbols used to indicate division of whole numbers. Mathematical symbols that indicate division. Symbol division symbol fraction bar ) division bar Example 12 4 12 4 4)12

Note that each of the following say the same thing; that is, 12 divided by 4 is 3. 3 12 =3 or 4)12 12 4 = 3 or 4

Quotients, Dividends, and Divisors. In the statement 3 4)12 the whole number 12 is called the dividend, the whole number 4 is called the divisor, and the whole number 3 is called the quotient. Note that this division bar notation is equivalent to 12 4 = 3 and 12 = 3. 4

The expression a/b means a divided by b, but this construct is also called a fraction.

Version: Fall 2010

38

CHAPTER 1. THE WHOLE NUMBERS

Fraction. The expression

a b is called a fraction. The number a on top is called the numerator of the fraction; the number b on the bottom is called the denominator of the fraction.

The key to understanding division of whole numbers is contained in the following statement. Division is equivalent to repeated subtraction. Suppose for example, that we would like to divide the whole number 12 by the whole number 4. This is equivalent to asking the question how many fours can we subtract from 12? This can be visualized in a number line diagram, such as the one in Figure 1.10. End 4 0 1 2 3 4 5 4 6 7 8 9 4 10 11 12 Start

Figure 1.10: Division is repeated subtraction. In Figure 1.10, note that we if we subtract three fours from twelve, the result is zero. In symbols, 12 4 4 4 = 0.
three fours

Equivalently, we can also ask How many groups of four are there in 12, and arrange our work as shown in Figure 1.11, where we can see that in an array of twelve objects, we can circle three groups of four ; i.e., 12 4 = 3.

Figure 1.11: There are three groups of four in twelve. In Figure 1.10 and Figure 1.11, note that the division (repeated subtraction) leaves no remainder. This is not always the case. Version: Fall 2010

1.3. MULTIPLICATION AND DIVISION OF WHOLE NUMBERS

39

You Try It! EXAMPLE 2. Divide 7 by 3. Solution. In Figure 1.12, we see that we can subtract two threes from seven, leaving a remainder of one. End 3 0 1 2 3 4 5 3 6 7 Start Use both the number line approach and the array of boxes approach to divide 12 by 5.

Figure 1.12: Division with a remainder. Alternatively, in an array of seven objects, we can circle two groups of three, leaving a remainder of one.

Figure 1.13: Dividing seven by three leaves a remainder of one. Both Figure 1.12 and Figure1.13 show that there are two groups of three in seven, with one left over. We say Seven divided by three is two, with a remainder of one.

Division is not Commutative


When dividing whole numbers, the order matters. For example, 12 4 = 3, but 4 12 is not even a whole number. Thus, if a and b are whole numbers, then a b does not have to be the same as b a.

Division is not Associative


When you divide three numbers, the order in which they are grouped will usually aect the answer. For example, (48 8) 2 = 6 2 = 3, Version: Fall 2010

40 but

CHAPTER 1. THE WHOLE NUMBERS

48 (8 2) = 48 4 = 12. Thus, if a, b, and c are whole numbers, (a b) c does not have to be the same as a (b c).

Division by Zero is Undened


Suppose that we are asked to divide six by zero; that is, we are asked to calculate 6 0. In Figure 1.14, we have an array of six objects.

Figure 1.14: How many groups of zero do you see? Now, to divide six by zero, we must answer the question How many groups of zero can we circle in Figure 1.14? Some thought will provide the answer: This is a meaningless request! It makes absolutely no sense to ask how many groups of zero can be circled in the array of six objects in Figure 1.14. Division by Zero. Division by zero is undened. Each of the expressions 60 is undened. On the other hand, it make sense to ask What is zero divided by six? If we create an array of zero objects, then ask how many groups of six we can circle, the answer is zero groups of six. That is, zero divided by six is zero. 06=0 and 0 =0 6 and 0 6)0 . and 6 0 and 0)6

Dividing Larger Whole Numbers


Well now provide a quick review of division of larger whole numbers, using an algorithm that is commonly called long division. This is not meant to be a thorough discussion, but a cursory one. Were counting on the fact that our readers have encountered this algorithm in previous courses and are familiar with the process. Version: Fall 2010

1.3. MULTIPLICATION AND DIVISION OF WHOLE NUMBERS

41

You Try It! EXAMPLE 3. Simplify: 575/23. Divide: 980/35

Solution. We begin by estimating how many times 23 will divide into 57, guessing 1. We put the 1 in the quotient above the 7, multiply 1 times 23, place the answer underneath 57, then subtract. 1 23)575 23 34 Because the remainder is larger than the divisor, our estimate is too small. We try again with an estimate of 2. 2 23)575 46 11 Thats the algorithm. Divide, multiply, then subtract. You may continue only when the remainder is smaller than the divisor. To continue, bring down the 5, estimate that 115 divided by 23 is 5, then multiply 5 times the divisor and subtract. 25 23)575 46 115 115 0 Because the remainder is zero, 575/23 = 25. Answer: 28

Application Counting Rectangular Arrays


Consider the rectangular array of stars in Figure 1.15. To count the number of stars in the array, we could use brute force, counting each star in the array one at a time, for a total of 20 stars. However, as we have four rows of ve stars each, it is much faster to multiply: 4 5 = 20 stars. Version: Fall 2010

42

CHAPTER 1. THE WHOLE NUMBERS

Figure 1.15: Four rows and ve columns. 1 ft 1 in 1 in 1 in


(a) One square inch.

1 in

1 ft

1 ft

1 ft
(b) One square foot.

Figure 1.16: Measures of area are in square units.

Application Area
In Figure 1.16(a), pictured is one square inch (1 in2 ), a square with one inch on each side. In Figure 1.16(b), pictured is one square foot (1 ft2 ), a square with one foot on each side. Both of these squares are measures of area. Now, consider the rectangle shown in Figure 1.17. The length of this rectangle is four inches (4 in) and the width is three inches (3 in).

3 in

One square inch (1 in2 )

4 in Figure 1.17: A rectangle with length 4 inches and width 3 inches. To nd the area of the gure, we can count the individual units of area that make up the area of the rectangle, twelve square inches (12 in2 ) in all. However, Version: Fall 2010

1.3. MULTIPLICATION AND DIVISION OF WHOLE NUMBERS

43

as we did in counting the stars in the array in Figure 1.15, it is much faster to note that we have three rows of four square inches. Hence, it is much faster to multiply the number of squares in each row by the number of squares in each column: 4 3 = 12 square inches. The argument presented above leads to the following rule for nding the area of a rectangle. Area of a Rectangle. Let L and W represent the length and width of a rectangle, respectively. L

L To nd the area of the rectangle, calculate the product of the length and width. That is, if A represents the area of the rectangle, then the area of the rectangle is given by the formula A = LW.

You Try It! EXAMPLE 4. A rectangle has width 5 feet and length 12 feet. Find the area of the rectangle. Solution. Substitute L = 12 ft and W = 5 ft into the area formula. A = LW = (12 ft)(5 ft) = 60 ft2 Hence, the area of the rectangle is 60 square feet. Answer: 561 square inches. A rectangle has width 17 inches and length 33 inches. Find the area of the rectangle.

Version: Fall 2010

44

CHAPTER 1. THE WHOLE NUMBERS

l l l

Exercises

l l l

In Exercises 1-4 use number line diagrams as shown in Figure 1.6 to depict the multiplication. 1. 2 4. 2. 3 4. 3. 4 2. 4. 4 3.

In Exercises 5-16, state the property of multiplication depicted by the given identity. 5. 9 8 = 8 9 6. 5 8 = 8 5 7. 8 (5 6) = (8 5) 6 8. 4 (6 5) = (4 6) 5 9. 6 2 = 2 6 10. 8 7 = 7 8 11. 3 (5 9) = (3 5) 9 12. 8 (6 4) = (8 6) 4 13. 21 1 = 21 14. 39 1 = 39 15. 13 1 = 13 16. 44 1 = 44

In Exercises 17-28, multiply the given numbers. 17. 78 3 18. 58 7 19. 907 6 20. 434 80 21. 128 30 22. 454 90 23. 799 60 24. 907 20 25. 14 70 26. 94 90 27. 34 90 28. 87 20

In Exercises 29-40, multiply the given numbers. 29. 237 54 30. 893 94 31. 691 12 32. 823 77 33. 955 89 Version: Fall 2010 34. 714 41 35. 266 61 36. 366 31 37. 365 73 38. 291 47

1.3. MULTIPLICATION AND DIVISION OF WHOLE NUMBERS 39. 955 57 40. 199 33

45

41. Count the number of objects in the array.

43. Count the number of objects in the array.

42. Count the number of objects in the array.

44. Count the number of objects in the array.

In Exercises 45-48, nd the area of the rectangle having the given length and width. 45. L = 50 in, W = 25 in 46. L = 48 in, W = 24 in 47. L = 47 in, W = 13 in 48. L = 19 in, W = 10 in

In Exercises 49-52, nd the perimeter of the rectangle having the given length and width. 49. L = 25 in, W = 16 in 50. L = 34 in, W = 18 in 51. L = 30 in, W = 28 in 52. L = 41 in, W = 25 in

Version: Fall 2010

46 53. A set of beads costs 50 cents per dozen. What is the cost (in dollars) of 19 dozen sets of beads? 54. A set of beads costs 60 cents per dozen. What is the cost (in dollars) of 7 dozen sets of beads? 55. If a math tutor worked for 47 hours and was paid $15 each hour, how much money would she have made? 56. If a math tutor worked for 46 hours and was paid $11 each hour, how much money would he have made?

CHAPTER 1. THE WHOLE NUMBERS 57. There are 12 eggs in one dozen, and 12 dozen in one gross. How many eggs are in a shipment of 24 gross? 58. There are 12 eggs in one dozen, and 12 dozen in one gross. How many eggs are in a shipment of 11 gross? 59. If bricks weigh 4 kilograms each, what is the weight (in kilograms) of 5000 bricks? 60. If bricks weigh 4 pounds each, what is the weight (in pounds) of 2000 bricks?

In Exercises 61-68, which of the following four expressions diers from the remaining three? 61. 30 , 30 5, 5)30, 5 30 5 12 62. , 12 2, 2)12, 2 12 2 8 63. , 8 2, 2)8, 8)2 2 8 64. , 8 4, 4)8, 8)4 4 65. 2)14, 66. 9)54, 67. 3)24, 68. 3)15, 14 , 14 2 2 54 , 54 9 54)9, 9 24 , 24 3 3 24, 3 15 , 15 3 3 15, 3 14)2,

In Exercises 69-82, simplify the given expression. If the answer doesnt exist or is undened, write undened. 69. 0 11 70. 0 5 71. 17 0 72. 24 0 73. 10 0 74. 20 0 7 75. 0 23 76. 0 80. 77. 16)0 78. 25)0 79. 0 24 0 22

81. 0)0 82. 0 0

Version: Fall 2010

1.3. MULTIPLICATION AND DIVISION OF WHOLE NUMBERS In Exercises 83-94, divide the given numbers. 83. 84. 85. 86. 87. 88. 2816 44 1998 37 2241 83 2716 97 3212 73 1326 17 89. 90. 91. 92. 93. 94. 8722 98 1547 91 1440 96 2079 27 8075 85 1587 23

47

In Exercises 95-106, divide the given numbers. 95. 96. 97. 98. 99. 100. 17756 92 46904 82 11951 19 22304 41 18048 32 59986 89 101. 102. 103. 104. 105. 106. 29047 31 33264 36 22578 53 18952 46 12894 14 18830 35

107. A concrete sidewalk is laid in square blocks that measure 6 feet on each side. How many blocks will there be in a walk that is 132 feet long? 108. A concrete sidewalk is laid in square blocks that measure 5 feet on each side. How many blocks will there be in a walk that is 180 feet long?

109. One boat to the island can take 5 people. How many trips will the boat have to take in order to ferry 38 people to the island? (Hint: Round up your answer.) 110. One boat to the island can take 4 people. How many trips will the boat have to take in order to ferry 46 people to the island? (Hint: Round up your answer.)

Version: Fall 2010

48 111. If street lights are placed at most 145 feet apart, how many street lights will be needed for a street that is 4 miles long, assuming that there are lights at each end of the street? (Note: 1 mile = 5280 feet.) 112. If street lights are placed at most 70 feet apart, how many street lights will be needed for a street that is 3 miles long, assuming that there are lights at each end of the street? (Note: 1 mile = 5280 feet.) 113. A concrete sidewalk is laid in square blocks that measure 4 feet on each side. How many blocks will there be in a walk that is 292 feet long? 114. A concrete sidewalk is laid in square blocks that measure 5 feet on each side. How many blocks will there be in a walk that is 445 feet long?

CHAPTER 1. THE WHOLE NUMBERS 115. One boat to the island can take 3 people. How many trips will the boat have to take in order to ferry 32 people to the island? (Hint: Round up your answer.) 116. One boat to the island can take 4 people. How many trips will the boat have to take in order to ferry 37 people to the island? (Hint: Round up your answer.) 117. If street lights are placed at most 105 feet apart, how many street lights will be needed for a street that is 2 miles long, assuming that there are lights at each end of the street? (Note: 1 mile = 5280 feet.) 118. If street lights are placed at most 105 feet apart, how many street lights will be needed for a street that is 3 miles long, assuming that there are lights at each end of the street? (Note: 1 mile = 5280 feet.)

119. Writing articles. Eli writes an average of 4 articles a day, ve days a week, to support product sales. How many articles does Eli write in one week? 120. Machine gun. A 0.50-caliber antiaircraft machine gun can re 800 rounds each minute. How many rounds could re in three minutes? Associated Press Times-Standard 4/15/09 121. Laps. The swimming pool at CalCourts is 25 yards long. If one lap is up and back again, how many yards has Wendell swam doing 27 laps? 122. Refrigerator wattage. A conventional refrigerator will run about 12 hours each day can use 150 Watts of power each hour. How many Watts of power will a refrigerator use over the day?

123. Horse hay. A full-grown horse should eat a minimum of 12 pounds of hay each day and may eat much more depending on their weight. How many pounds minimum would a horse eat over a year? 124. College costs. After a $662 hike in fees, Califormia residents who want to attend the University of California as an undergraduate should expect to pay $8,700 in for the upcoming academic year 20092010. If the cost were to remain the same for the next several years, how much should a student expect to pay for a fouryear degree program at a UC school? 125. Non-resident costs. Nonresident undergraduates who want to attend a University of California college should expect to pay about $22,000 for the upcoming academic year. Assuming costs remain the same, what can a four-year degree cost?

Version: Fall 2010

1.3. MULTIPLICATION AND DIVISION OF WHOLE NUMBERS 126. Student tax. The mayer of Providence, Rhode Island wants to tax its 25,000 Brown University students $150 each to contribute to tax receipts saying students should pay for the resources they use just like the town residents. How many dollars would the mayer generate? 127. New iceberg. A new iceberg, shaved o a glacier after a collision with another iceberg, measures about 48 miles long and 28 miles wide. Whats the approximate area of the new iceberg? Associated PressTimes-Standard 02/27/10 2 Huge icebergs set loose o Antarcticas coast. 128. Solar panels. One of the solar panels on the International Space Station is 34

49

meters long and 11 meters wide. If there are eight of these, whats the total area for solar collection? 129. Sidewalk. A concrete sidewalk is to be 80 foot long and 4 foot wide. How much will it cost to lay the sidewalk at $8 per square foot? 130. Hay bales. An average bale of hay weighs about 60 pounds. If a horse eats 12 pounds of hay a day, how many days will one bale feed a horse? 131. Sunspots. Sunspots, where the suns magnetic eld is much higher, usually occur in pairs. If the total count of sunspots is 72, how many pairs of sunspots are there?

l l l
1. 2 4 = 4 + 4 = 8 Start
2 times

Answers

l l l

11. Associative property of multiplication End 4 13. Multiplicative identity property 15. Multiplicative identity property 7 8 17. 234 19. 5442

4 0 1 2 3 4 5

3. 4 2 = 2 + 2 + 2 + 2 = 8
4 times

21. 3840 End 23. 47940 25. 980 2

Start 2 0 1 2 2 3 4 2 5 6

27. 3060 29. 12798

5. Commutative property of multiplication 31. 8292 7. Associative property of multiplication 9. Commutative property of multiplication 33. 84995 35. 16226 Version: Fall 2010

50 37. 26645 39. 54435 41. 64 43. 56 45. 1250 in2 47. 611 in2 49. 82 in 51. 116 in 53. 9.50 55. 705 57. 3456 59. 20000 61. 5 30 63. 8)2 65. 14)2 67. 3 24 69. 0 71. Undened 73. 0 75. Undened 77. 0 79. 0 81. Undened 83. 64 Version: Fall 2010

CHAPTER 1. THE WHOLE NUMBERS 85. 27 87. 44 89. 89 91. 15 93. 95 95. 193 97. 629 99. 564 101. 937 103. 426 105. 921 107. 22 109. 8 111. 147 113. 73 115. 11 117. 102 119. 20 articles 121. 1350 yards 123. 4380 pounds of hay 125. $88,000 127. 1344 mi2 129. $2,560 131. 36

1.4. PRIME FACTORIZATION

51

1.4

Prime Factorization

In the statement 3 4 = 12, the number 12 is called the product, while 3 and 4 are called factors. You Try It! EXAMPLE 1. Find all whole number factors of 18. Solution. We need to nd all whole number pairs whose product equals 18. The following pairs come to mind. 1 18 = 18 and 2 9 = 18 and 3 6 = 18. Answer: 1, 3, 7, and 21. Find all whole number factors of 21.

Hence, the factors of 18 are (in order) 1, 2, 3, 6, 9, and 18.

Divisibility
In Example 1, we saw 3 6 = 18, making 3 and 6 factors of 18. Because division is the inverse of multiplication, that is, divison by a number undoes the multiplication of that number, this immediately provides 18 6 = 3 and 18 3 = 6.

That is, 18 is divisible by 3 and 18 is divisible by 6. When we say that 18 is divisible by 3, we mean that when 18 is divided by 3, there is a zero remainder. Divisible. Let a and b be whole numbers. Then a is divisible by b if and only if the remainder is zero when a is divided by b. In this case, we say that b is a divisor of a.

You Try It! EXAMPLE 2. Find all whole number divisors of 18. Solution. In Example 1, we saw that 3 6 = 18. Therefore, 18 is divisible by both 3 and 6 (18 3 = 6 and 18 6 = 3). Hence, when 18 is divided by 3 or 6, the remainder is zero. Therefore, 3 and 6 are divisors of 18. Noting the other products in Example 1, the complete list of divisors of 18 is 1, 2, 3, 6, 9, and 18. Find all whole number divisors of 21.

Answer: 1, 3, 7, and 21.

Example 1 and Example 2 show that when working with whole numbers, the words factor and divisor are interchangeable. Version: Fall 2010

52

CHAPTER 1. THE WHOLE NUMBERS

Factors and Divisors. If c = a b, then a and b are called factors of c. Both a and b are also called divisors of c.

Divisibility Tests
There are a number of very useful divisibility tests. Divisible by 2. If a whole number ends in 0, 2, 4, 6, or 8, then the number is called an even number and is divisible by 2. Examples of even numbers are 238 and 1,246 (238 2 = 119 and 1, 246 2 = 623). A number that is not even is called an odd number. Examples of odd numbers are 113 and 2,339. Divisible by 3. If the sum of the digits of a whole number is divisible by 3, then the number itself is divisible by 3. An example is 141. The sum of the digits is 1 + 4 + 1 = 6, which is divisible by 3. Therefore, 141 is also divisible by 3 (141 3 = 47). Divisible by 4. If the number represented by the last two digits of a whole number is divisible by 4, then the number itself is divisible by 4. An example is 11,524. The last two digits represent 24, which is divisible by 4 (24 4 = 6). Therefore, 11,524 is divisible by 4 (11, 524 4 = 2, 881). Divisible by 5. If a whole number ends in a zero or a 5, then the number is divisible by 5. Examples are 715 and 120 (7155 = 143 and 1205 = 24). Divisible by 6. If a whole number is divisible by 2 and by 3, then it is divisible by 6. An example is 738. First, 738 is even and divisible by 2. Second, 7+3+8=18, which is divisible by 3. Hence, 738 is divisible by 3. Because 738 is divisible by both 2 and 3, it is divisible by 6 (738 6 = 123). Divisible by 8. If the number represented by the last three digits of a whole number is divisible by 8, then the number itself is divisible by 8. An example is 73,024. The last three digits represent the number 24, which is divisible by 8 (248 = 3). Thus, 73,024 is also divisible by 8 (73, 0248 = 9, 128). Divisible by 9. If the sum of the digits of a whole number is divisible by 9, then the number itself is divisible by 9. An example is 117. The sum of the digits is 1 + 1 + 7 = 9, which is divisible by 9. Hence, 117 is divisible by 9 (117 9 = 13). Version: Fall 2010

1.4. PRIME FACTORIZATION

53

Prime Numbers
We begin with the denition of a prime number. Prime Number. A whole number (other than 1) is a prime number if its only factors (divisors) are 1 and itself. Equivalently, a number is prime if and only if it has exactly two factors (divisors).

You Try It! EXAMPLE 3. Which of the whole numbers 12, 13, 21, and 37 are prime numbers? Solution. The factors (divisors) of 12 are 1, 2, 3, 4, 6, and 12. Hence, 12 is not a prime number. The factors (divisors) of 13 are 1 and 13. Because its only divisors are 1 and itself, 13 is a prime number. The factors (divisors) of 21 are 1, 3, 7, and 21. Hence, 21 is not a prime number. The factors (divisors) of 37 are 1 and 37. Because its only divisors are 1 and itself, 37 is a prime number. Answer: 23 and 59. Which of the whole numbers 15, 23, 51, and 59 are prime numbers?

You Try It! EXAMPLE 4. List all the prime numbers less than 20. Solution. The prime numbers less than 20 are 2, 3, 5, 7, 11, 13, 17, and 19. List all the prime numbers less than 100.

Composite Numbers. If a whole number is not a prime number, then it is called a composite number.

Version: Fall 2010

54

CHAPTER 1. THE WHOLE NUMBERS

You Try It! Is the whole number 2,571 prime or composite? Answer: Composite. EXAMPLE 5. Is the whole number 1,179 prime or composite? Solution. Note that 1 + 1 + 7 + 9 = 18, which is divisible by both 3 and 9. Hence, 3 and 9 are both divisors of 1,179. Therefore, 1,179 is a composite number.

Factor Trees
We will now learn how to express a composite number as a unique product of prime numbers. The most popular device for accomplishing this goal is the factor tree. You Try It! Express 36 as a product of prime factors. EXAMPLE 6. Express 24 as a product of prime factors. Solution. We use a factor tree to break 24 down into a product of primes. 24 4 2 2 2 6 3 24 = 4 6 4 = 2 2 and 6 = 2 3

At each level of the tree, break the current number into a product of two factors. The process is complete when all of the circled leaves at the bottom of the tree are prime numbers. Arranging the factors in the circled leaves in order, 24 = 2 2 2 3. The nal answer does not depend on product choices made at each level of the tree. Here is another approach. 24 8 2 2 4 2 3 24 = 8 3 8= 24 4= 22

The nal answer is found by including all of the factors from the circled leaves at the end of each branch of the tree, which yields the same result, namely 24 = 2 2 2 3. Alternate Approach. Some favor repeatedly dividing by 2 until the result is no longer divisible by 2. Then try repeatedly dividing by the next prime until Version: Fall 2010

1.4. PRIME FACTORIZATION

55

the result is no longer divisible by that prime. The process terminates when the last resulting quotient is equal to the number 1. 2 2 2 3 24 12 6 3 1 24 2 = 12 12 2 = 6 62=3 33=1 Answer: 2 2 3 3.

The rst column reveals the prime factorization; i.e., 24 = 2 2 2 3.

The fact that the alternate approach in Example 6 yielded the same result is signicant. Unique Factorization Theorem. Every whole number can be uniquely factored as a product of primes. This result guarantees that if the prime factors are ordered from smallest to largest, everyone will get the same result when breaking a number into a product of prime factors.

Exponents
We begin with the denition of an exponential expression. Exponents. The expression am is dened to mean am = a a . . . a .
m times

The number a is called the base of the exponential expression and the number m is called the exponent. The exponent m tells us to repeat the base a as a factor m times.

You Try It! EXAMPLE 7. Evaluate 25 , 33 and 52 . Solution. In the case of 25 , we have 25 = 2 2 2 2 2 = 32. Version: Fall 2010 Evaluate: 35 .

56 In the case of 33 , we have

CHAPTER 1. THE WHOLE NUMBERS

33 = 3 3 3 = 27. In the case of 52 , we have 52 = 5 5 = 25. Answer: 243.

You Try It! Prime factor 54. EXAMPLE 8. Express the solution to Example 6 in compact form using exponents. Solution. In Example 6, we determined the prime factorization of 24. 24 = 2 2 2 3 Because 2 2 2 = 23 , we can write this more compactly. 24 = 23 3 Answer: 2 3 3 3.

You Try It! Evaluate: 33 52 . EXAMPLE 9. Evaluate the expression 23 32 52 . Solution. First raise each factor to the given exponent, then perform the multiplication in order (left to right). 23 32 52 = 8 9 25 = 72 25 = 1800 Answer: 675

Version: Fall 2010

1.4. PRIME FACTORIZATION

57

Application
A square is a rectangle with four equal sides. Area of a Square. Let s represent the length of each side of a square. s

s Because a square is also a rectangle, we can nd the area of the square by multiplying its length and width. However, in this case, the length and width both equal s, so A = (s)(s) = s2 . Hence, the formula for the area of a square is A = s2 .

You Try It! EXAMPLE 10. The edge of a square is 13 centimeters. Find the area of the square. Solution. Substitute s = 13 cm into the area formula. A = s2 = (13 cm)2 = (13 cm)(13 cm) = 169 cm2 Hence, the area of the square is 169 cm2 ; i.e., 169 square centimeters. Answer: 225 square meters. The edge of a square is 15 meters. Find the area of the square.

Version: Fall 2010

58

CHAPTER 1. THE WHOLE NUMBERS

l l l

Exercises

l l l

In Exercises 1-12, nd all divisors of the given number. 1. 30 2. 19 3. 83 4. 51 5. 91 6. 49 7. 75 8. 67 9. 64 10. 87 11. 14 12. 89

In Exercises 13-20, which of the following numbers is not divisible by 2? 13. 117, 120, 342, 230 14. 310, 157, 462, 160 15. 30, 22, 16, 13 16. 382, 570, 193, 196 17. 105, 206, 108, 306 18. 60, 26, 23, 42 19. 84, 34, 31, 58 20. 66, 122, 180, 63

In Exercises 21-28, which of the following numbers is not divisible by 3? 21. 561, 364, 846, 564 22. 711, 850, 633, 717 23. 186, 804, 315, 550 24. 783, 909, 504, 895 25. 789, 820, 414, 663 26. 325, 501, 945, 381 27. 600, 150, 330, 493 28. 396, 181, 351, 606

In Exercises 29-36, which of the following numbers is not divisible by 4? 29. 3797, 7648, 9944, 4048 30. 1012, 9928, 7177, 1592 31. 9336, 9701, 4184, 2460 32. 2716, 1685, 2260, 9788 33. 9816, 7517, 8332, 7408 34. 1788, 8157, 7368, 4900 35. 1916, 1244, 7312, 7033 36. 7740, 5844, 2545, 9368

Version: Fall 2010

1.4. PRIME FACTORIZATION

59

In Exercises 37-44, which of the following numbers is not divisible by 5? 37. 8920, 4120, 5285, 9896 38. 3525, 7040, 2185, 2442 39. 8758, 3005, 8915, 3695 40. 3340, 1540, 2485, 2543 41. 2363, 5235, 4145, 4240 42. 9030, 8000, 5445, 1238 43. 1269, 5550, 4065, 5165 44. 7871, 9595, 3745, 4480

In Exercises 45-52, which of the following numbers is not divisible by 6? 45. 328, 372, 990, 528 46. 720, 288, 148, 966 47. 744, 174, 924, 538 48. 858, 964, 930, 330 49. 586, 234, 636, 474 50. 618, 372, 262, 558 51. 702, 168, 678, 658 52. 780, 336, 742, 312

In Exercises 53-60, which of the following numbers is not divisible by 8? 53. 1792, 8216, 2640, 5418 54. 2168, 2826, 1104, 2816 55. 8506, 3208, 9016, 2208 56. 2626, 5016, 1392, 1736 57. 4712, 3192, 2594, 7640 58. 9050, 9808, 8408, 7280 59. 9808, 1232, 7850, 7912 60. 3312, 1736, 9338, 3912

In Exercises 61-68, which of the following numbers is not divisible by 9? 61. 477, 297, 216, 991 62. 153, 981, 909, 919 63. 153, 234, 937, 675 64. 343, 756, 927, 891 65. 216, 783, 594, 928 66. 504, 279, 307, 432 67. 423, 801, 676, 936 68. 396, 684, 567, 388

Version: Fall 2010

60

CHAPTER 1. THE WHOLE NUMBERS

In Exercises 69-80, identify the given number as prime, composite, or neither. 69. 19 70. 95 71. 41 72. 88 73. 27 74. 61 75. 91 76. 72 77. 21 78. 65 79. 23 80. 36

In Exercises 81-98, nd the prime factorization of the natural number. 81. 224 82. 320 83. 108 84. 96 85. 243 86. 324 87. 160 88. 252 89. 32 90. 128 91. 360 92. 72 93. 144 94. 64 95. 48 96. 200 97. 216 98. 392

In Exercises 99-110, compute the exact value of the given exponential expression. 99. 52 41 100. 23 41 101. 01 102. 13 103. 33 02 104. 33 22 105. 41 106. 52 107. 43 108. 42 109. 33 12 110. 52 23

Version: Fall 2010

1.4. PRIME FACTORIZATION In Exercises 111-114, nd the area of the square with the given side. 111. 28 inches 112. 31 inches 113. 22 inches 114. 13 inches

61

Create factor trees for each number in Exercises 115-122. Write the prime factorization for each number in compact form, using exponents. 115. 12 116. 18 117. 105 118. 70 119. 56 120. 56 121. 72 122. 270

123. Sieve of Eratosthenes. This exercise introduces the Sieve of Eratosthenes, an ancient algorithm for nding the primes less than a certain number n, rst created by the Greek mathematician Eratosthenes. Consider the grid of integers from 2 through 100. 2 3 4 5 6 7 8 9 10 11

To nd the primes less than 100, proceed as follows. i) Strike out all multiples of 2 (4, 6, 8, etc.) ii) The lists next number that has not been struck out is a prime number. iii) Strike out from the list all multiples of the number you identied in step (ii). iv) Repeat steps (ii) and (iii) until you can no longer strike any more multiples. v) All unstruck numbers in the list are primes.

12 13 14 15 16 17 18 19 20 21 22 23 24 25 26 27 28 29 30 31 32 33 34 35 36 37 38 39 40 41 42 43 44 45 46 47 48 49 50 51 52 53 54 55 56 57 58 59 60 61 62 63 64 65 66 67 68 69 70 71 72 73 74 75 76 77 78 79 80 81 82 83 84 85 86 87 88 89 90 91 92 93 94 95 96 97 98 99 100

Version: Fall 2010

62

CHAPTER 1. THE WHOLE NUMBERS

l l l
1. 1, 2, 3, 5, 6, 10, 15, 30 3. 1, 83 5. 1, 7, 13, 91 7. 1, 3, 5, 15, 25, 75 9. 1, 2, 4, 8, 16, 32, 64 11. 1, 2, 7, 14

Answers

l l l

43. 1269 45. 328 47. 538 49. 586 51. 658 53. 5418 55. 8506

13. 117 57. 2594 15. 13 17. 105 19. 31 21. 364 23. 550 25. 820 27. 493 29. 3797 31. 9701 33. 7517 35. 7033 81. 2 2 2 2 2 7 37. 9896 39. 8758 41. 2363 Version: Fall 2010 83. 2 2 3 3 3 85. 3 3 3 3 3 87. 2 2 2 2 2 5 59. 7850 61. 991 63. 937 65. 928 67. 676 69. prime 71. prime 73. composite 75. composite 77. composite 79. prime

1.4. PRIME FACTORIZATION 89. 2 2 2 2 2 91. 2 2 2 3 3 5 93. 2 2 2 2 3 3 95. 2 2 2 2 3 97. 2 2 2 3 3 3 99. 100 101. 0 103. 0 105. 4 107. 64 109. 27 111. 784 in2 113. 484 in2 115. 12 = 22 3 117. 105 = 3 5 7 119. 56 = 23 7 121. 72 = 23 32

63

123. Unstruck numbers are primes: 2, 3, 5, 7, 11, 13, 17, 19, 23, 29, 31, 37, 41, 43, 47, 53, 59, 61, 67, 71, 73, 79, 83, 89, 97

Version: Fall 2010

64

CHAPTER 1. THE WHOLE NUMBERS

1.5

Order of Operations

The order in which we evaluate expressions can be ambiguous. Take for example, the expression 4 + 3 2. If we do the addition rst, then 4+32= 72 = 14. On the other hand, if we do the multiplication rst, then 4+32=4+6 = 10. So, what are we to do? Of course, grouping symbols can remove the ambiguity. Grouping Symbols. Parentheses, brackets, or curly braces can be used to group parts of an expression. Each of the following are equivalent: (4 + 3) 2 or [4 + 3] 2 or {4 + 3} 2

In each case, the rule is evaluate the expression inside the grouping symbols rst. If grouping symbols are nested, evaluate the expression in the innermost pair of grouping symbols rst. Thus, for example, (4 + 3) 2 = 7 2 = 14. Note how the expression contained in the parentheses was evaluated rst. Another way to avoid ambiguities in evaluating expressions is to establish an order in which operations should be performed. The following guidelines should always be strictly enforced when evaluating expressions. Rules Guiding Order of Operations. When evaluating expressions, proceed in the following order. 1. Evaluate expressions contained in grouping symbols rst. If grouping symbols are nested, evaluate the expression in the innermost pair of grouping symbols rst. 2. Evaluate all exponents that appear in the expression. 3. Perform all multiplications and divisions in the order that they appear in the expression, moving left to right.

Version: Fall 2010

1.5. ORDER OF OPERATIONS

65

4. Perform all additions and subtractions in the order that they appear in the expression, moving left to right.

You Try It! EXAMPLE 1. Evaluate 4 + 3 2. Solution. Because of the established Rules Guiding Order of Operations, this expression is no longer ambiguous. There are no grouping symbols or exponents, so we immediately go to rule three, evaluate all multiplications and divisions in the order that they appear, moving left to right. After that we invoke rule four, performing all additions and subtractions in the order that they appear, moving left to right. 4+32=4+6 = 10 Thus, 4 + 3 2 = 10. Answer: 18 Simplify: 8 + 2 5.

You Try It! EXAMPLE 2. Evaluate 18 2 + 3. Solution. Follow the Rules Guiding Order of Operations. Addition has no precedence over subtraction, nor does subtraction have precedence over addition. We are to perform additions and subtractions as they occur, moving left to right. 18 2 + 3 = 16 + 3 = 19 Thus, 18 2 + 3 = 19. Subtract: 18 2 = 16. Add: 16 + 3 = 19. Answer: 11 Simplify: 17 8 + 2.

You Try It! EXAMPLE 3. Evaluate 54 9 2. Solution. Follow the Rules Guiding Order of Operations. Division has no precedence over multiplication, nor does multiplication have precedence over division. We are to perform divisions and multiplications as they occur, moving Version: Fall 2010 Simplify: 72 9 2.

66 left to right. 54 9 2 = 6 2 = 12

CHAPTER 1. THE WHOLE NUMBERS

Divide: 54 9 = 6. Multiply: 6 2 = 12.

Answer: 16

Thus, 54 9 2 = 12.

You Try It! Simplify: 14 + 3 42 . EXAMPLE 4. Evaluate 2 32 12. Solution. Follow the Rules Guiding Order of Operations, exponents rst, then multiplication, then subtraction. 2 32 12 = 2 9 12 = 18 12 =6 Answer: 62 Thus, 2 32 12 = 6. Evaluate the exponent: 32 = 9. Perform the multiplication: 2 9 = 18. Perform the subtraction: 18 12 = 6.

You Try It! Simplify: 3(2 + 3 4)2 11. EXAMPLE 5. Evaluate 12 + 2(3 + 2 5)2 . Solution. Follow the Rules Guiding Order of Operations, evaluate the expression inside the parentheses rst, then exponents, then multiplication, then addition. 12 + 2(3 + 2 5)2 = 12 + 2(3 + 10)2 = 12 + 2(13) = 12 + 338 = 350 Answer: 577 Thus, 12 + 2(3 + 2 5)2 = 350.
2

Multiply inside parentheses: 2 5 = 10. Add inside parentheses: 3 + 10 = 13. Exponents are next: (13)2 = 169. Multiplication is next: 2(169) = 338. Time to add: 12 + 338 = 350.

= 12 + 2(169)

Version: Fall 2010

1.5. ORDER OF OPERATIONS

67

You Try It! EXAMPLE 6. Evaluate 2{2 + 2[2 + 2]}. Solution. When grouping symbols are nested, evaluate the expression between the pair of innermost grouping symbols rst. 2{2 + 2[2 + 2]} = 2{2 + 2[4]} = 2{2 + 8} = 2{10} = 20 Thus, 2{2 + 2[2 + 2]} = 20. Innermost grouping rst: 2 + 2 = 4. Multiply next: 2[4] = 8. Add inside braces: 2 + 8 = 10. Multiply: 2{10} = 20 Answer: 26 Simplify: 2{3 + 2[3 + 2]}.

Fraction Bars
Consider the expression 62 + 82 . (2 + 3)2 Because a fraction bar means division, the above expression is equivalent to (62 + 82 ) (2 + 3)2 . The position of the grouping symbols signals how we should proceed. We should simplify the numerator, then the denominator, then divide. Fractional Expressions. If a fractional expression is present, evaluate the numerator and denominator rst, then divide.

You Try It! EXAMPLE 7. Evaluate the expression 62 + 82 . (2 + 3)2 Solution. Simplify the numerator and denominator rst, then divide. 62 + 82 62 + 82 = (2 + 3)2 (5)2 36 + 64 = 25 100 = 25 =4 Parentheses in denominator rst: 2 + 3 = 5. Exponents are next: 62 = 36, 82 = 64, 52 = 25. Add in numerator: 36 + 64 = 100. Divide: 100 25 = 4. Version: Fall 2010 Simplify: 12 + 3 2 . 6

68 62 + 82 = 4. (2 + 3)2

CHAPTER 1. THE WHOLE NUMBERS

Answer: 3

Thus,

The Distributive Property


Consider the expression 2 (3 + 4). If we follow the Rules Guiding Order of Operations, we would evaluate the expression inside the parentheses rst. 2 (3 + 4) = 2 7 = 14 Parentheses rst: 3 + 4 = 7. Multiply: 2 7 = 14.

However, we could also choose to distribute the 2, rst multiplying 2 times each addend in the parentheses. 2 (3 + 4) = 2 3 + 2 4 =6+8 = 14 Multiply 2 times both 3 and 4. Multiply: 2 3 = 6 and 2 4 = 8. Add: 6 + 8 = 14.

The fact that we get the same answer in the second approach is an illustration of an important property of whole numbers.1 The Distributive Property. Let a, b, and c be any whole numbers. Then, a (b + c) = a b + a c. We say that multiplication is distributive with respect to addition. Multiplication is distributive with respect to addition. If you are not computing the product of a number and a sum of numbers, the distributive property does not apply. Caution! Wrong Answer Ahead! If you are calculating the product of a number and the product of two numbers, the distributive property must not be used. For example, here is a common misapplication of the distributive property. 2 (3 4) = (2 3) (2 4) =68 = 48
1 Later,

well see that this property applies to all numbers, not just whole numbers.

Version: Fall 2010

1.5. ORDER OF OPERATIONS

69

This result is quite distant from the correct answer, which is found by computing the product within the parentheses rst. 2 (3 4) = 2 12 = 24. In order to apply the distributive property, you must be multiplying times a sum.

You Try It! EXAMPLE 8. Use the distributive property to calculate 4 (5 + 11). Solution. This is the product of a number and a sum, so the distributive property may be applied. 4 (5 + 11) = 4 5 + 4 11 Distribute the 4 times each addend in the sum. = 20 + 44 Multiply: 4 5 = 20 and 4 11 = 44. = 64 Add: 20 + 44 = 64. Answer: 95 Distribute: 5 (11 + 8).

Readers should check that the same answer is found by computing the sum within the parentheses rst.

The distributive property is the underpinning of the multiplication algorithm learned in our childhood years. You Try It! EXAMPLE 9. Multiply: 6 43. Use the distributive property to evaluate 8 92.

Solution. Well express 43 as sum, then use the distributive property. 6 43 = 6 (40 + 3) = 6 40 + 6 3 = 240 + 18 = 258 Express 43 as a sum: 43 = 40 + 3. Distribute the 6. Multiply: 6 40 = 240 and 6 3 = 18. Add: 240 + 18 = 258.

Readers should be able to see this application of the distributive property in the more familiar algorithmic form: 43 6 18 240 258 Version: Fall 2010

70

CHAPTER 1. THE WHOLE NUMBERS

Or in the even more condensed form with carrying: 43 6 258 Answer: 736
1

Multiplication is also distributive with respect to subtraction. The Distributive Property (Subtraction). Let a, b, and c be any whole numbers. Then, a (b c) = a b a c. We say the multiplication is distributive with respect to subtraction.

You Try It! Distribute: 8 (9 2). EXAMPLE 10. Use the distributive property to simplify: 3 (12 8).

Solution. This is the product of a number and a dierence, so the distributive property may be applied. 3 (12 8) = 3 12 3 8 Distribute the 3 times each term in the dierence. = 36 24 = 12 Multiply: 3 12 = 36 and 3 8 = 24. Subtract: 36 24 = 12.

Alternate solution. Note what happens if we use the usual order of operations to evaluate the expression. 3 (12 8) = 3 4 = 12 Answer: 56 Same answer. Parentheses rst: 12 8 = 4. Multiply: 3 4 = 12.

Version: Fall 2010

1.5. ORDER OF OPERATIONS

71

l l l

Exercises

l l l

In Exercises 1-12, simplify the given expression. 1. 5 + 2 2 2. 5 + 2 8 3. 23 7 2 4. 37 3 7 5. 4 3 + 2 5 6. 2 5 + 9 7 7. 6 5 + 4 3 8. 5 2 + 9 8 9. 9 + 2 3 10. 3 + 6 6 11. 32 8 2 12. 24 2 5

In Exercises 13-28, simplify the given expression. 13. 45 3 5 14. 20 1 4 15. 2 9 3 18 16. 19 20 4 16 17. 30 2 3 18. 27 3 3 19. 8 6 + 1 20. 15 5 + 10 21. 14 16 16 19 22. 20 17 17 14 23. 15 17 + 10 10 12 4 24. 14 18 + 9 3 7 13 25. 22 10 + 7 26. 29 11 + 1 27. 20 10 + 15 5 7 6 28. 18 19 + 18 18 6 7

In Exercises 29-40, simplify the given expression. 29. 9 + 8 {4 + 4} 30. 10 + 20 {2 + 2} 31. 7 [8 5] 10 32. 11 [12 4] 10 33. (18 + 10) (2 + 2) 34. (14 + 7) (2 + 5) 35. 9 (10 + 7) 3 (4 + 10) 36. 9 (7 + 7) 8 (3 + 8) 37. 2 {8 + 12} 4 38. 4 {8 + 7} 3 39. 9 + 6 (12 + 3) 40. 3 + 5 (10 + 12)

Version: Fall 2010

72

CHAPTER 1. THE WHOLE NUMBERS

In Exercises 41-56, simplify the given expression. 41. 2 + 9 [7 + 3 (9 + 5)] 42. 6 + 3 [4 + 4 (5 + 8)] 43. 7 + 3 [8 + 8 (5 + 9)] 44. 4 + 9 [7 + 6 (3 + 3)] 45. 6 5[11 (2 + 8)] 46. 15 1[19 (7 + 3)] 47. 11 1[19 (2 + 15)] 48. 9 8[6 (2 + 3)] 49. 4{7[9 + 3] 2[3 + 2]} 50. 4{8[3 + 9] 4[6 + 2]} 51. 9 [3 + 4 (5 + 2)] 52. 3 [4 + 9 (8 + 5)] 53. 3{8[6 + 5] 8[7 + 3]} 54. 2{4[6 + 9] 2[3 + 4]} 55. 3 [2 + 4 (9 + 6)] 56. 8 [3 + 9 (5 + 2)]

In Exercises 57-68, simply the given expression. 57. (5 2)2 58. (5 3)4 59. (4 + 2)2 60. (3 + 5)2 61. 23 + 33 62. 54 + 24 63. 23 13 64. 32 12 65. 12 52 + 8 9 + 4 66. 6 32 + 7 5 + 12 67. 9 3 2 + 12 102 68. 11 2 3 + 12 42

In Exercises 69-80, simplify the given expression. 69. 42 (13 + 2) 70. 33 (7 + 6) 71. 33 (7 + 12) 72. 43 (6 + 5) 73. 19 + 3[12 (23 + 1)] 74. 13 + 12[14 (22 + 1)] 75. 17 + 7[13 (22 + 6)] 76. 10 + 1[16 (22 + 9)] 77. 43 (12 + 1) 78. 53 (17 + 15) 79. 5 + 7[11 (22 + 1)] 80. 10 + 11[20 (22 + 1)]

Version: Fall 2010

1.5. ORDER OF OPERATIONS In Exercises 81-92, simplify the given expression. 81. 82. 83. 84. 13 + 35 3(4) 35 + 28 7(3) 64 (8 6 3) 479 87. 17 + 14 98 16 + 2 88. 13 11 37 + 27 89. 8(2) 90. 91. 92. 16 + 38 6(3) 40 (3 7 9) 822 60 (8 6 3) 545

73

19 (4 3 2) 639 2 + 13 85. 41 7+1 86. 84

In Exercises 93-100, use the distributive property to evaluate the given expression. 93. 5 (8 + 4) 94. 8 (4 + 2) 95. 7 (8 3) 96. 8 (9 7) 97. 6 (7 2) 98. 4 (8 6) 99. 4 (3 + 2) 100. 4 (9 + 6)

In Exercises 101-104, use the distributive property to evaluate the given expression using the technique shown in Example 9. 101. 9 62 102. 3 76 103. 3 58 104. 7 57

l l l
1. 9 3. 9 5. 22

Answers
7. 42 9. 15 11. 16

l l l

Version: Fall 2010

74 13. 75 15. 108 17. 45 19. 3 21. 266 23. 208 25. 19 27. 161 29. 10 31. 11 33. 7 35. 111 37. 10 39. 99 41. 443 43. 367 45. 1 47. 9 49. 296 51. 279 53. 24 55. 186 57. 9

CHAPTER 1. THE WHOLE NUMBERS 59. 36 61. 35 63. 7 65. 376 67. 1203 69. 1 71. 8 73. 28 75. 38 77. 51 79. 47 81. 4 83. 1 85. 5 87. 31 89. 4 91. 2 93. 60 95. 35 97. 30 99. 20 101. 558 103. 174

Version: Fall 2010

1.6. SOLVING EQUATIONS BY ADDITION AND SUBTRACTION

75

1.6

Solving Equations by Addition and Subtraction

Lets start with the denition of a variable. Variable. A variable is a symbol (usually a letter) that stands for a value that may vary. Next we follow with the denition of an equation. Equation. An equation is a mathematical statement that equates two mathematical expressions. The key dierence between a mathematical expression and an equation is the presence of an an equals sign. So, for example, 2 + 3[5 4 2], x2 + 2x 3, and x + 2y + 3

are mathematical expressions (two of which contain variables), while 3 + 2(7 3) = 11, x + 3 = 4, and 3x = 9

are equations. Note that each of the equations contain an equals sign, but the expressions do not. Next we have the denition of a solution of an equation. What it Means to be a Solution. A solution of an equation is a numerical value that satises the equation. That is, when the variable in the equation is replaced by the solution, a true statement results.

You Try It! EXAMPLE 1. Show that 3 is a solution of the equation x + 8 = 11. Solution. Substitute 3 for x in the given equation and simplify. x + 8 = 11 3 + 8 = 11 11 = 11 The given equation. Substitute 3 for x. Simplify both sides. Show that 27 is a solution of the equation x 12 = 15.

Since the left- and right-hand sides of the last line are equal, this shows that when 3 is substituted for x in the equation a true statement results. Therefore, 3 is a solution of the equation.

Version: Fall 2010

76

CHAPTER 1. THE WHOLE NUMBERS

You Try It! Is 8 a solution of 5 = 12 y? EXAMPLE 2. Is 23 a solution of the equation 4 = y 11? Solution. Substitute 23 for y in the given equation and simplify. 4 = y 11 4 = 23 11 4 = 12 The given equation. Substitute 23 for y. Simplify both sides.

Answer: No.

Since the left- and right-hand sides of the last line are not equal, this shows that when 23 is substituted for y in the equation a false statement results. Therefore, 23 is not a solution of the equation.

Equivalent Equations
We start with the denition of equivalent equations. Equivalent Equations. Two equations are equivalent if they have the same solution set.

You Try It! Are the equations x = 4 and x + 8 = 3 equivalent? Answer: No. EXAMPLE 3. Are the equations x + 2 = 9 and x = 7 equivalent? Solution. The number 7 is the only solution of the equation x + 2 = 9. Similarly, 7 is the only solution of the equation x = 7. Therefore x + 2 = 9 and x = 7 have the same solution sets and are equivalent.

You Try It! Are the equations x = 2 and x2 = 2x equivalent? EXAMPLE 4. Are the equations x2 = x and x = 1 equivalent? Solution. By inspection, the equation x2 = x has two solutions, 0 and 1. On the other hand, the equation x = 1 has a single solution, namely 1. Hence, the equations x2 = x and x = 1 do not have the same solution sets and are not equivalent.

Answer: No.

Version: Fall 2010

1.6. SOLVING EQUATIONS BY ADDITION AND SUBTRACTION

77

Operations that Produce Equivalent Equations


There are many operations that will produce equivalent operations. In this section we look at two: addition and subtraction. Adding the Same Quantity to Both Sides of an Equation. Adding the same quantity to both sides of an equation does not change the solution set. That is, if a = b, then adding c to both sides of the equation produces the equivalent equation a + c = b + c.

Lets see if this works as advertised. Consider the equaton x 4 = 3. By inspection, 7 is the only solution of the equation. Now, lets add 4 to both sides of the equation to see if the resulting equation is equivalent to x 4 = 3. x4=3 x4+4=3+4 x=7 The given equation. Add 4 to both sides of the equation. Simplify both sides of the equation.

The number 7 is the only solution of the equation x = 7. Thus, the equation x = 7 is equivalent to the original equation x 4 = 3 (they have the same solutions). Important Point. Adding the same amount to both sides of an equation does not change its solutions. It is also a fact that subtracting the same quantity from both sides of an equation produces an equivalent equation. Subtracting the Same Quantity from Both Sides of an Equation. Subtracting the same quantity from both sides of an equation does not change the solution set. That is, if a = b, then subtracting c from both sides of the equation produces the equivalent equation a c = b c.

Version: Fall 2010

78

CHAPTER 1. THE WHOLE NUMBERS

Lets also see if this works as advertised. Consider the equation x + 4 = 9. By inspection, 5 is the only solution of the equation. Now, lets subtract 4 from both sides of the equation to see if the resulting equation is equivalent to x + 4 = 9. x+4=9 x+44=94 x=5 The given equation. Subtract 4 from both sides of the equation. Simplify both sides of the equation.

The number 5 is the only solution of the equation x = 5. Thus, the equation x = 5 is equivalent to the original equation x + 4 = 9 (they have the same solutions). Important Point. Subtracting the same amount from both sides of an equation does not change its solutions.

Writing Mathematics. When solving equations, observe the following rules to neatly arrange your work: 1. One equation per line. This means that you should not arrange your work like this: x+3=7 x+33=73 x=4

Thats three equations on a line. Rather, arrange your work one equation per line like this: x+3=7 x+33 = 73 x=4 2. Add and subtract inline. Dont do this: x 7 = 12 + 7 +7 x = 19 Instead, add 7 to both sides of the equation inline. x 7 = 12 x 7 + 7 = 12 + 7 x = 19

Version: Fall 2010

1.6. SOLVING EQUATIONS BY ADDITION AND SUBTRACTION

79

Wrap and Unwrap


Suppose that you are wrapping a gift for your cousin. You perform the following steps in order. 1. Put the gift paper on. 2. Put the tape on. 3. Put the decorative bow on. When we give the wrapped gift to our cousin, he politely unwraps the present, undoing each of our three steps in inverse order. 1. Take o the decorative bow. 2. Take o the tape. 3. Take o the gift paper. This seemingly frivolous wrapping and unwrapping of a gift contains some deeply powerful mathematical ideas. Consider the mathematical expression x + 4. To evaluate this expression at a particular value of x, we would start with the given value of x, then 1. Add 4. Suppose we started with the number 7. If we add 4, we arrive at the following result: 11. Now, how would we unwrap this result to return to our original number? We would start with our result, then 1. Subtract 4. That is, we would take our result from above, 11, then subtract 4, which returns us to our original number, namely 7. Addition and Subtraction as Inverse Operations. Two extremely important observations: The inverse of addition is subtraction. If we start with a number x and add a number a, then subtracting a from the result will return us to the original number x. In symbols, x + a a = x. The inverse of subtraction is addition. If we start with a number x and subtract a number a, then adding a to the result will return us to the original number x. In symbols, x a + a = x.

Version: Fall 2010

80

CHAPTER 1. THE WHOLE NUMBERS

You Try It! Solve x + 5 = 12 for x. EXAMPLE 5. Solve x 8 = 10 for x. Solution. To undo the eects of subtracting 8, we add 8 to both sides of the equation. x 8 = 10 x 8 + 8 = 10 + 8 x = 18 Original equation. Add 8 to both sides of the equation. On the left, adding 8 undoes the eect of subtracting 8 and returns x. On the right, 10+8=18.

Therefore, the solution of the equation is 18. Check. To check, substitute the solution 18 into the original equation. x 8 = 10 18 8 = 10 10 = 10 Original equation. Substitute 18 for x. Simplify both sides.

Answer: x = 7.

The fact that the last line of our check is a true statement guarantees that 18 is a solution of x 8 = 10.

You Try It! Solve y 8 = 11 for y. EXAMPLE 6. Solve 11 = y + 5 for y. Solution. To undo the eects of adding 5, we subtract 5 from both sides of the equation. 11 = y + 5 11 5 = y + 5 5 6=y Original equation. Subtract 5 from both sides of the equation. On the right, subtracting undoes the eect of adding 5 and returns y. On the left, 11-5=6.

Therefore, the solution of the equation is 6. Version: Fall 2010

1.6. SOLVING EQUATIONS BY ADDITION AND SUBTRACTION Check. To check, substitute the solution 6 into the original equation. 11 = y + 5 11 = 6 + 5 11 = 11 Original equation. Substitute 6 for y. Simplify both sides.

81

The fact that the last line of our check is a true statement guarantees that 6 is a solution of 11 = y + 5.

Answer: y = 19.

Word Problems
The solution of a word problem must incorporate each of the following steps. Requirements for Word Problem Solutions. 1. Set up a Variable Dictionary. You must let your readers know what each variable in your problem represents. This can be accomplished in a number of ways: Statements such as Let P represent the perimeter of the rectangle. Labeling unknown values with variables in a table. Labeling unknown quantities in a sketch or diagram. 2. Set up an Equation. Every solution to a word problem must include a carefully crafted equation that accurately describes the constraints in the problem statement. 3. Solve the Equation. You must always solve the equation set up in the previous step. 4. Answer the Question. This step is easily overlooked. For example, the problem might ask for Janes age, but your equations solution gives the age of Janes sister Liz. Make sure you answer the original question asked in the problem. Your solution should be written in a sentence with appropriate units. 5. Look Back. It is important to note that this step does not imply that you should simply check your solution in your equation. After all, its possible that your equation incorrectly models the problems situation, so you could have a valid solution to an incorrect equation. The important question is: Does your answer make sense based on the words in the original problem statement.

Version: Fall 2010

82

CHAPTER 1. THE WHOLE NUMBERS

Lets give these requirements a test drive. You Try It! 12 more than a certain number is 19. Find the number. EXAMPLE 7. Four more than a certain number is 12. Find the number. Solution. In our solution, we will carefully address each step of the Requirements for Word Problem Solutions. 1. Set up a Variable Dictionary. We can satisfy this requirement by simply stating Let x represent a certain number. 2. Set up an Equation. Four more than a certain number is 12 becomes a certain number x

4 4

more than +

is =

12 12

3. Solve the Equation. To undo the addition, subtract 4 from both sides of the equation.

4 + x = 12 4 + x 4 = 12 4 x=8

Original equation. Subtract 4 from both sides of the equation. On the left, subtracting 4 undoes the eect of adding 4 and returns x. On the right, 12 4 = 8.

4. Answer the Question. The number is 8. 5. Look Back. Does the solution 8 satisfy the words in the original problem? We were told that four more than a certain number is 12. Well, four more than 8 is 12, so our solution is correct. Answer: 7

You Try It! Fred withdraws $230 from his account, lowering his balance to $3,500. What was his original balance? EXAMPLE 8. Amelie withdraws $125 from her savings account. Because of the withdrawal, the current balance in her account is now $1,200. What was the original balance in the account before the withdrawal? Solution. In our solution, we will carefully address each step of the Requirements for Word Problem Solutions. Version: Fall 2010

1.6. SOLVING EQUATIONS BY ADDITION AND SUBTRACTION

83

1. Set up a Variable Dictionary. We can satisfy this requirement by simply stating Let B represent the original balance in Amelies account. 2. Set up an Equation. We can describe the situation in words and symbols.

Original Balance B

minus

Amelies Withdrawal 125

is =

Current Balance 1200

3. Solve the Equation. To undo the subtraction, add 125 to both sides of the equation.

B 125 = 1200 Original equation. B 125 + 125 = 1200 + 125 Add 125 to both sides of the equation. B = 1325 On the left, adding 125 undoes the eect of subtracting 125 and returns B. On the right, 1200+125=1325.

4. Answer the Question. The original balance was $1,325. 5. Look Back. Does the solution $1,325 satisfy the words in the original problem? Note that if Amelie withdraws $125 from this balance, the new balance will be $1,200. Hence, the solution is correct. Answer: $3,730.

You Try It! EXAMPLE 9. The perimeter of a triangle is 114 feet. Two of the sides of the triangle measure 30 feet and 40 feet, respectively. Find the measure of the third side of the triangle. Solution. In our solution, we will carefully address each step of the Requirements for Word Problem Solutions. 1. Set up a Variable Dictionary. When geometry is involved, we can create our variable dictionary by labeling a carefully constructed diagram. With this thought in mind, we draw a triangle, then label its known and unknown sides and its perimeter. Version: Fall 2010 The perimeter of a quadrilater is 200 meters. If three of the sides measure 20, 40, and 60 meters, what is the length of the fourth side?

84

CHAPTER 1. THE WHOLE NUMBERS

Perimeter = 114 ft

x 30 ft

40 ft The gure makes it clear that x represents the length of the unknown side of the triangle. The gure also summarizes information needed for the solution. 2. Set up an equation. We know that the perimeter of a triangle is found by nding the sum of its three sides; in words and symbols, First Side x Second Side 30 Third Side 40

Perimeter 114

is =

plus +

plus +

Simplify the right-hand side by adding 30 and 40; i.e., 30 + 40 = 70. 114 = x + 70 3. Solve the Equation. To undo adding 70, subtract 70 from both sides of the equation. 114 = x + 70 Our equation. 114 70 = x + 70 70 Subtract 70 from both sides. 44 = x On the right, subtracting 70 undoes the eect of adding 70 and returns to x. On the left, 114 70 = 44. 4. Answer the Question. The unknown side of the triangle is 44 feet. 5. Look Back. Does the solution 44 feet satisfy the words of the original problem? We were told that the perimeter is 114 feet and two of the sides have length 30 feet and 40 feet respectively. We found that the third side has length 44 feet. Now, adding the three sides, 30 + 40 + 44 = 114, which equals the given perimeter of 114 feet. The answer works! Answer: 80 meters.

Version: Fall 2010

1.6. SOLVING EQUATIONS BY ADDITION AND SUBTRACTION

85

l l l

Exercises

l l l

In Exercises 1-12, which of the numbers following the given equation are solutions of the given equation? Support your response with work similar to that shown in Examples 1 and 2. 1. x 4 = 6; 10, 17, 13, 11 2. x 9 = 7; 17, 23, 19, 16 3. x + 2 = 6; 5, 11, 7, 4 4. x + 3 = 9; 6, 9, 7, 13 5. x + 2 = 3; 8, 1, 4, 2 6. x + 2 = 5; 10, 3, 6, 4 7. x 4 = 7; 12, 11, 18, 14 8. x 6 = 7; 13, 16, 20, 14 9. x + 3 = 4; 8, 4, 2, 1 10. x + 5 = 9; 5, 11, 7, 4 11. x 6 = 8; 17, 21, 14, 15 12. x 2 = 9; 11, 14, 12, 18

In Exercises 13-52, solve the given equation for x. 13. x + 5 = 6 14. x + 6 = 19 15. 5 = 4 + x 16. 10 = 8 + x 17. 13 + x = 17 18. 7 + x = 15 19. 9 + x = 10 20. 14 + x = 17 21. 19 = x 3 22. 2 = x 11 23. x 18 = 1 24. x 20 = 8 25. x 3 = 11 26. x 17 = 18 27. 2 + x = 4 28. 1 + x = 16 29. x 14 = 12 30. x 1 = 17 31. x + 2 = 8 32. x + 11 = 14 33. 11 + x = 17 34. 11 + x = 18 35. x + 13 = 17 36. x + 1 = 16 37. 20 = 3 + x 38. 9 = 3 + x 39. 20 = 8 + x 40. 10 = 3 + x 41. 3 = x 20 42. 13 = x 15 43. x + 16 = 17 44. x + 6 = 12 45. 5 = x 6 46. 10 = x 7 47. 18 = x 6 48. 14 = x 4 49. 18 = 13 + x 50. 17 = 5 + x 51. x 9 = 15 52. x 11 = 17 Version: Fall 2010

86

CHAPTER 1. THE WHOLE NUMBERS

53. 12 less than a certain number is 19. Find the number. 54. 19 less than a certain number is 1. Find the number. 55. A triangle has a perimeter of 65 feet. It also has two sides measuring 19 feet and 17 feet, respectively. Find the length of the third side of the triangle. 56. A triangle has a perimeter of 55 feet. It also has two sides measuring 14 feet and 13 feet, respectively. Find the length of the third side of the triangle. 57. Burt makes a deposit to an account having a balance of $1900. After the deposit, the new balance in the account is $8050. Find the amount of the deposit. 58. Dave makes a deposit to an account having a balance of $3500. After the deposit, the new balance in the account is $4600. Find the amount of the deposit. 59. 8 more than a certain number is 18. Find the number. 60. 3 more than a certain number is 19. Find the number. 61. Michelle withdraws a $120 from her bank account. As a result, the new account balance is $1000. Find the account balance before the withdrawal. 62. Mercy withdraws a $430 from her bank account. As a result, the new account balance is $1200. Find the account balance before the withdrawal.

63. Foreclosures. Between January and March last year, 650,000 homes received a foreclosure notice. Between the rst three months of this year, there were 804,000 foreclosure notices. What was the increase in home foreclosure notices? Associated Press Times-Standard 4/22/09 64. Home Price. According to the Humboldt State University Economics Departments Humboldt Economic Index, the median home price in the US fell $1500 over the last month to $265,000. What was the median home price before the price drop? 65. Unmanned Areal Vehicle. Northrup Grummans Global Hawk unmanned drone can y at 65,000 feet, 40,000 feet higher than NASAs Ikhana unmanned aircraft. How high can the Ikhana y? 66. Tribal Land. The Yurok Tribe has the option to purchase 47,000 acres in order to increase its ancestral territory. The rst phase would include 22,500 acres in the Cappel and Pecman watersheds. The second phase plans for acreage in the Blue Creek area. How many acres could be purchased in the second phase? TimesStandard 4/15/09

Version: Fall 2010

1.6. SOLVING EQUATIONS BY ADDITION AND SUBTRACTION

87

l l l
1. 10 3. 4 5. 1 7. 11 9. 1 11. 14 13. 1 15. 1 17. 4 19. 1

Answers
35. 4 37. 17 39. 12 41. 23 43. 1 45. 11 47. 24 49. 5 51. 24 53. 31

l l l

21. 22 55. 29 23. 19 57. $6150 25. 14 59. 10 27. 2 61. $1120 29. 26 63. 154,000 31. 6 33. 6 65. 25,000 feet

Version: Fall 2010

88

CHAPTER 1. THE WHOLE NUMBERS

1.7

Solving Equations by Multiplication and Division

In Section 1.6, we stated that two equations that have the same solutions are equivalent. Furthermore, we saw that adding the same number to both sides of an equation produced an equivalent equation. Similarly, subtracting the same the number from both sides of an equation also produces an equivalent equation. We can make similar statements for multiplication and division. Multiplying both Sides of an Equation by the Same Quantity. Multiplying both sides of an equation by the same quantity does not change the solution set. That is, if a = b, then multiplying both sides of the equation by c produces the equivalent equation a c = b c, provided c = 0. A similar statement can be made about division. Dividing both Sides of an Equation by the Same Quantity. Dividing both sides of an equation by the same quantity does not change the solution set. That is, if a = b, then dividing both sides of the equation by c produces the equivalent equation b a = , c c provided c = 0. In Section 1.6, we saw that addition and subtraction were inverse operations. If you start with a number, add 4 and subtract 4, you are back to the original number. This concept also works for multiplication and division. Multiplication and Division as Inverse Operations. Two extremely important observations: The inverse of multiplication is division. If we start with a number x and multiply by a number a, then dividing the result by the number a returns us to the original number x. In symbols, ax = x. a

Version: Fall 2010

1.7. SOLVING EQUATIONS BY MULTIPLICATION AND DIVISION

89

The inverse of division is multiplication. If we start with a number x and divide by a number a, then multiplying the result by the number a returns us to the original number x. In symbols, a x = x. a

Lets put these ideas to work. You Try It! EXAMPLE 1. Solve the equation 3x = 24 for x. Solution. To undo the eects of multiplying by 3, we divide both sides of the equation by 3. 3x = 24 3x 24 = 3 3 x=8 Original equation. Divide both sides of the equation by 3. On the left, dividing by 3 undoes the eect of multiplying by 3 and returns to x. On the right, 24/3 = 8. Solution. To check, substitute the solution 8 into the original equation. 3x = 24 3(8) = 24 24 = 24 Original equation. Substitute 8 for x. Simplify both sides. Answer: 24 Solve for x: 5x = 120

That fact that the last line of our check is a true statement guarantees that 8 is a solution of 3x = 24.

You Try It! EXAMPLE 2. Solve the following equation for x. x = 12 7 Solution. To undo the eects of dividing by 7, we multiply both sides of the equation by 7. Version: Fall 2010 Solve for x: x/2 = 19

90

CHAPTER 1. THE WHOLE NUMBERS

x = 12 7 x 7 = 7 12 7 x = 84

Original equation. Multiply both sides of the equation by 7. On the left, multiplying by 7 undoes the eect of dividing by 7 and returns to x. On the right, 7 12 = 84.

Solution. To check, substitute the solution 84 into the original equation. x = 12 7 84 = 12 7 12 = 12

Original equation. Substitute 84 for x. Simplify both sides.

Answer: 38

That fact that the last line of our check is a true statement guarantees that 84 is a solution of x/7 = 12.

Word Problems
In Section 1.6 we introduced Requirements for Word Problem Solutions. Those requirements will be strictly adhered to in this section. You Try It! Seven times a certain number is one hundred ve. Find the unknown number. EXAMPLE 3. Fifteen times a certain number is 45. Find the unknown number. Solution. In our solution, we will carefully address each step of the Requirements for Word Problem Solutions. 1. Set up a Variable Dictionary. We can satisfy this requirement by simply stating Let x represent a certain number. 2. Set up an equation. Fifteen times a certain number is 45 becomes a certain number x

15 15 Version: Fall 2010

times

is =

45 45

1.7. SOLVING EQUATIONS BY MULTIPLICATION AND DIVISION

91

3. Solve the Equation. To undo the multiplication by 15, divide both sides of the equation by 15. 15x = 45 15x 45 = 15 15 x=3 Original equation. Write 15 x as 15x Divide both sides of the equation by 15. On the left, dividing by 15 undoes the eect of multiplying by 15 and returns to x. On the right, 45/15 = 3. 4. Answer the Question. The unknown number is 3. 5. Look Back. Does the solution 3 satisfy the words of the original problem? We were told that 15 times a certain number is 45. Well, 15 times 3 is 45, so our solution is correct. Answer: 15

You Try It! EXAMPLE 4. The area of a rectangle is 120 square feet. If the length of the rectangle is 12 feet, nd the width of the rectangle. Solution. In our solution, we will carefully address each step of the Requirements for Word Problem Solutions. 1. Set up a Variable Dictionary. When geometry is involved, we can create our variable dictionary by labeling a carefully constructed diagram. With this thought in mind, we draw a rectangle, then label its length, width, and area. The area of a rectangle is 3,500 square meters. If the width is 50 meters, nd the length.

Area = 120 ft2

12 ft The gure makes it clear that W represents the width of the rectangle. The gure also summarizes information needed for the solution. Version: Fall 2010

92

CHAPTER 1. THE WHOLE NUMBERS 2. Set up an equation. We know that the area of a rectangle is found by multiplying its length and width; in symbols, A = LW. (1.1)

Were given the area is A = 120 ft2 and the length is L = 12 ft. Substitute these numbers into the area formula (1.1) to get 120 = 12W. 3. Solve the Equation. To undo the multiplication by 12, divide both sides of the equation by 12. 120 = 12W 12W 120 = 12 12 10 = W Our equation. Divide both sides of the equation by 12. On the right, dividing by 12 undoes the eect of multiplying by 12 and returns to W . On the left, 120/12 = 10. 4. Answer the Question. The width is 10 feet. 5. Look Back. Does the found width satisfy the words of the original problem? We were told that the area is 120 square feet and the length is 12 feet. The area is found by multiplying the length and width, which gives us 12 feet times 10 feet, or 120 square feet. The answer works! Answer: 70 meters

You Try It! A class of 30 students averaged 75 points on an exam. How many total points were accumulated by the class as a whole? EXAMPLE 5. A class of 23 students averaged 76 points on an exam.How many total points were accumulated by the class as a whole? Solution. In our solution, we will carefully address each step of the Requirements for Word Problem Solutions. 1. Set up a Variable Dictionary. We can set up our variable dictionary by simply stating Let T represent the total points accumulated by the class. 2. Set up an equation. To nd the average score on the exam, take the total points accumulated by the class, then divide by the number of students in the class. In words and symbols, Version: Fall 2010

1.7. SOLVING EQUATIONS BY MULTIPLICATION AND DIVISION

93

Total Points T

divided by

Number of Students 23

equals =

Average Score 76

An equivalent representation is T = 76. 23 3. Solve the Equation. To undo the division by 23, multiply both sides of the equation by 23. T = 76 Our equation. 23 T 23 = 76 23 Multiply both sides of the equation by 23. 23 T = 1748 On the left, multiplying by 23 undoes the eect of dividing by 23 and returns to T . On the right, 76 23 = 1748. 4. Answer the Question. The total points accumulated by the class on the exam is 1,748. 5. Look Back. Does the solution 1,748 satisfy the words of the original problem? To nd the average on the exam, divide the total points 1,748 by 23, the number of students in the class. Note that this gives an average score of 1748 23 = 76. The answer works! Answer: 2,250

Version: Fall 2010

94

CHAPTER 1. THE WHOLE NUMBERS

l l l

Exercises

l l l

In Exercises 1-12, which of the numbers following the given equation are solutions of the given equation? 1. 2. 3. 4. 5. x = 4; 24, 25, 27, 31 6 x = 6; 49, 42, 43, 45 7 x = 3; 6, 9, 13, 7 2 x = 5; 45, 46, 48, 52 9 5x = 10; 9, 2, 3, 5 7. 5x = 25; 5, 6, 8, 12 8. 3x = 3; 1, 8, 4, 2 9. 2x = 2; 4, 8, 1, 2 10. 3x = 6; 2, 9, 5, 3 x 11. = 7; 57, 59, 63, 56 8 x 12. = 7; 24, 21, 28, 22 3

6. 4x = 36; 12, 16, 9, 10

In Exercises 13-36, solve the given equation for x. 13. x =7 6 x 14. = 6 8 15. 2x = 16 16. 2x = 10 17. 2x = 18 18. 2x = 0 19. 4x = 24 20. 2x = 4 x 21. = 9 4 x 22. = 6 5 23. 5x = 5 24. 3x = 15 25. 5x = 30 26. 4x = 28 x 27. = 4 3 x 28. = 4 9 x 29. = 9 8 x 30. = 2 8 x 31. = 8 7 x 32. = 6 4 33. 2x = 8 34. 3x = 9 x 35. = 5 8 x 36. = 4 5

Version: Fall 2010

1.7. SOLVING EQUATIONS BY MULTIPLICATION AND DIVISION 37. The price of one bookcase is $370. A charitable organization purchases an unknown number of bookcases and the total price of the purchase is $4,810. Find the number of bookcases purchased. 38. The price of one computer is $330. A charitable organization purchases an unknown number of computers and the total price of the purchase is $3,300. Find the number of computers purchased. 39. When an unknown number is divided by 3, the result is 2. Find the unknown number. 40. When an unknown number is divided by 8, the result is 3. Find the unknown number. 41. A class of 29 students averaged 80 points on an exam. How many total points were accumulated by the class as a whole? 42. A class of 44 students averaged 87 points on an exam. How many total points were accumulated by the class as a whole? 43. When an unknown number is divided by 9, the result is 5. Find the unknown number. 44. When an unknown number is divided by 9, the result is 2. Find the unknown number. 45. The area of a rectangle is 16 square cm. If the length of the rectangle is 2 cm, nd the width of the rectangle. 46. The area of a rectangle is 77 square ft. If the length of the rectangle is 7 ft, nd the width of the rectangle.

95

47. The area of a rectangle is 56 square cm. If the length of the rectangle is 8 cm, nd the width of the rectangle. 48. The area of a rectangle is 55 square cm. If the length of the rectangle is 5 cm, nd the width of the rectangle. 49. The price of one stereo is $430. A charitable organization purchases an unknown number of stereos and the total price of the purchase is $6,020. Find the number of stereos purchased. 50. The price of one computer is $490. A charitable organization purchases an unknown number of computers and the total price of the purchase is $5,880. Find the number of computers purchased. 51. A class of 35 students averaged 74 points on an exam. How many total points were accumulated by the class as a whole? 52. A class of 44 students averaged 88 points on an exam. How many total points were accumulated by the class as a whole? 53. 5 times an unknown number is 20. Find the unknown number. 54. 5 times an unknown number is 35. Find the unknown number. 55. 3 times an unknown number is 21. Find the unknown number. 56. 2 times an unknown number is 10. Find the unknown number.

Version: Fall 2010

96

CHAPTER 1. THE WHOLE NUMBERS

l l l
1. 24 3. 6 5. 2 7. 5 9. 1 11. 56 13. 42 15. 8 17. 9 19. 6 21. 36 23. 1 25. 6 27. 12

Answers
29. 72 31. 56 33. 4 35. 40 37. 13 39. 6

l l l

41. 2,320 43. 45 45. 8 cm 47. 7 cm 49. 14 51. 2,590 53. 4 55. 7

Version: Fall 2010

Chapter

The Integers

Today, much as we take for granted the fact that there exists a number zero, denoted by 0, such that a + 0 = a for any whole number a, we similarly take for granted that for any whole number a there exists a unique number a, called the negative or opposite of a, so that a + (a) = 0. In a natural way, or so it seems to modern-day mathematicians, this easily introduces the concept of a negative number. However, history teaches us that the concept of negative numbers was not embraced wholeheartedly by mathematicians until somewhere around the 17th century. In his work Arithmetica (c. 250 AD), the Greek mathematician Diophantus (c. 200-284 AD), who some call the Father of Algebra, described the equation 4 = 4x + 20 as absurd, for how could one talk about an answer less than nothing? Girolamo Cardano (1501-1576), in his seminal work Ars Magna (c. 1545 AD) referred to negative numbers as numeri cti, while the German mathematician Michael Stifel (1487-1567) referred to them as numeri absurdi. John Napier (1550-1617) (the creator of logarithms) called negative numbers defectivi, and Rene Descartes (1596-1650) (the creator of analytic geometry) labeled negative solutions of algebraic equations as false roots. On the other hand, there were mathematicians whose treatment of negative numbers resembled somewhat our modern notions of the properties held by negative numbers. The Indian mathematician Brahmagupta described arithmetical rules in terms of fortunes (positive number) and debts (negative numbers). Indeed, in his work Brahmasphutasiddhanta, he writes a fortune subtracted from zero is a debt, which in modern notation would resemble 0 4 = 4. If you nd the study of the integers somewhat dicult, do not be discouraged, as centuries of mathematicians have struggled mightily with the topic. With this thought it mind, lets begin the study of the integers.

97

98

CHAPTER 2. THE INTEGERS

2.1

An Introduction to the Integers

As we saw in the introduction to the chapter, negative numbers have a rich and storied history. One of the earliest applications of negative numbers had to do with credits and debits. For example, if $5 represents a credit or prot, then $5 represents a debit or loss. Of course, the ancients had a dierent monetary system than ours, but you get the idea. Note that if a vendor experiences a prot of $5 on a sale, then a loss of $5 on a second sale, the vendor breaks even. That is, the sum of $5 and $5 is zero. In much the same way, every whole number has an opposite or negative counterpart. The Opposite or Negative of a Whole Number. For every whole number a, there is a unique number a, called the opposite or negative of a, such that a + (a) = 0. The opposite or negative of any whole number is easily located on the number line. Number Line Location. To locate the opposite (or negative) of any whole number, rst locate the whole number on the number line. The opposite is the reection of the whole number through the origin (zero).

You Try It! Locate the number 7 and its opposite on the number line. EXAMPLE 1. Locate the whole number 5 and its opposite (negative) on the number line. Solution. Draw a number line, then plot the whole number 5 on the line as a shaded dot. 0 5

To nd its opposite, reect the number 5 through the origin. This will be the location of the opposite (negative) of the whole number 5, which we indicate by the symbol 5. 5

Note the symmetry. The whole number 5 is located ve units to the right of zero. Its negative is located ve units to the left of zero.

Version: Fall 2010

2.1. AN INTRODUCTION TO THE INTEGERS

99

Important Pronunciation. The symbol 5 is pronounced in one of two ways: (1) negative ve, or (2) the opposite of ve. In similar fashion, we can locate the opposite or negative of any whole number by reecting the whole number through the origin (zero), which leads to the image shown in Figure 2.1.

7 6 5 4 3 2 1 0

Figure 2.1: The opposite (negative) of any whole number is a reection of that number through the origin (zero).

The Integers
The collection of numbers arranged on the number line in Figure 2.1 extend indenitely to the right, and because the numbers on the left are reections through the origin, the numbers also extend indenitely to the left. This collection of numbers is called the set of integers. The Integers. The innite collection of numbers {. . . , 7, 6, 5, 4, 3, 2, 1, 0, 1, 2, 3, 4, 5, 6, 7, . . .} is called the set of integers. The ellipsis . . . at each end of this innite collection means etcetera, as the integers continue indenitely to the right and left. Thus, for example, both 23,456 and 117, 191 are elements of this set and are therefore integers.

Ordering the Integers


As we saw with the whole numbers, as you move to the right on the number line, the numbers get larger; as you move to the left, the numbers get smaller. Order on the Number Line. Let a and b be integers located on the number line so that the point representing the integer a lies to the left of the point representing the integer b.

Version: Fall 2010

100

CHAPTER 2. THE INTEGERS

Then the integer a is less than the integer b and we write a<b Alternatively, we can also say that the integer b is larger than the integer a and write b > a.

You Try It! Arrange the numbers 3, 5, 2, and 8 in order, from smallest to largest. EXAMPLE 2. Arrange the integers 4, 0, 4, and 2 in order, from smallest to largest. Solution. Place each of the numbers 4, 0, 4, and 2 as shaded dots on the number line.

7 6 5 4 3 2 1 0

Thus, 4 is the smallest integer, 2 is the next largest, followed by 0, then 4. Arranging these numbers in order, from smallest to largest, we have 4, 2, 0, 4. Answer: 8, 5, 3, 2

You Try It! Compare 12 and 11. EXAMPLE 3. Replace each shaded box with < (less than) or > (greater than) so the resulting inequality is a true statement. 3 5 2 4

Solution. For the rst case, locate 3 and 5 on the number line as shaded dots.

7 6 5 4 3 2 1 0 Note that 3 lies to the left of 5, so: 3 Version: Fall 2010 <

2.1. AN INTRODUCTION TO THE INTEGERS

101

That is, 3 is less than 5. In the second case, locate 2 and 4 as shaded dots on the number line.

7 6 5 4 3 2 1 0 Note that 2 lies to the right of 4, so: 2 That is, 2 is greater than 4. >

4 Answer: 12 < 11

Important Observation. In Example 3, note that the pointy end of the inequality symbol always points towards the smaller number.

Opposites of Opposites
We stated earlier that every integer has a unique number called its opposite or negative. Thus, the integer 5 is the opposite (negative) of the integer 5. Thus, we can say that the pair 5 and 5 are opposites. Each is the opposite of the other. Logically, this leads us to the conclusion that the opposite of 5 is 5. In symbols, we would write (5) = 5.

Opposites of Opposites. Let a be an integer. Then the opposite of the opposite of a is a. In symbols, we write (a) = a. We can also state that the negative of a negative a is a.

You Try It! EXAMPLE 4. Simplify (13) and (119). Solution. The opposite of the opposite of a number returns the original number. That is, (13) = 13 and (119) = 119. Answer: 50 Simplify: (50).

Version: Fall 2010

102

CHAPTER 2. THE INTEGERS

Positive and Negative


We now dene the terms positive integer and negative integer. Positive Integer. If a is an integer that lies to the right of zero (the origin) on the number line, then a is a positive integer. This means that a is a positive integer if and only if a > 0. Thus, 2, 5, and 117 are positive integers. Negative Integer. If a is an integer that lies to the left of zero (the origin) on the number line, then a is a negative integer. This means that a is a negative integer if and only if a < 0. Thus, 4, 8, and 1, 123 are negative integers. Zero. The integer zero is neither positive nor negative.

You Try It! Classify 11 as positive, negative, or neither EXAMPLE 5. Classify each of the following numbers as negative, positive, or neither: 4, 6, and 0. Solution. Locate 4, 6, and 0 on the number line.

7 6 5 4 3 2 1 0 Thus:

4 lies to the right of zero. That is, 4 > 0, making 4 a positive integer. 6 lies to the left of zero. That is, 6 < 0, making 6 a negative integer. The number 0 is neutral. It is neither negative nor positive. Answer: Negative

Absolute Value
We dene the absolute value of an integer.

Version: Fall 2010

2.1. AN INTRODUCTION TO THE INTEGERS

103

Absolute Value. The absolute value of an integer is dened as its distance from the origin (zero). It is important to note that distance is always a nonnegative quantity (not negative); i.e., distance is either positive or zero. As an example, weve shaded the integers 4 and 4 on a number line. a distance of 4 a distance of 4 4 0 4

The number line above shows two cases: The integer 4 is 4 units from zero. Because absolute value measures the distance from zero, | 4| = 4. The integer 4 is also 4 units from zero. Again, absolute value measures the distance from zero, so |4| = 4. Lets look at another example. You Try It! EXAMPLE 6. Determine the value of each expression: a) | 7|, b) |3|, and c) |0|. Solution. The absolute value of any integer is equal to the distance that number is from the origin (zero) on the number line. Thus: a) The integer 7 is 7 units from the origin; hence, | 7| = 7. b) The integer 3 is 3 units from the origin; hence, |3| = 3. c) The integer 0 is 0 units from the origin; hence, |0| = 0. Answer: 33 Simplify: | 33|.

You Try It! EXAMPLE 7. Determine the value of each expression: a) (8) and b) | 8| Solution. These are distinctly dierent problems. a) The opposite of 8 is 8. That is, (8) = 8. Version: Fall 2010 Simplify: | 50|.

104

CHAPTER 2. THE INTEGERS

b) However, in this case, we take the absolute value of 8 rst, which is 8, then the opposite of that result to get 8. That is, | 8| = (8) = 8 Answer: 50 First: | 8| = 8. Second: The opposite of 8 is 8.

Applications
There are a number of applications that benet from the use of integers. You Try It! The following table contains record low temperatures (degrees Fahrenheit) for Jackson Hole, Wyoming for the indicated months. Month Sept. Oct. Nov. Dec. Jan Temp 14 2 27 49 50 EXAMPLE 8. Prots and losses for the rst six months of the scal year for a small business are shown in Table 2.1. Prots and losses are measured in Month Prot/Loss Jan 10 Feb 12 Mar 7 Apr 2 May 4 Jun 5

Table 2.1: Prot and loss are measured in thousands of dollars. thousands of dollars. A positive number represents a prot, while a negative number represents a loss. Create a bar graph representing the prots and losses for this small business for each month of the rst half of the scal year. Solution. Start by labeling the horizontal axis with the months of the scal year. Once youve completed that task, scale the vertical axis to accommodate the Prot/Loss values recorded in Table 2.1. Finally, starting at the 0 level on the horizontal axis, sketch bars having heights equal to the prot and loss for each month. Prot/Loss (thousand dollars) 14 12 10 8 6 4 2 0 2 4 6

Create a bar graph of temperature versus months.

Apr Jan Feb Mar

May Jun

Figure 2.2: Prot and loss bar graph. Version: Fall 2010

2.1. AN INTRODUCTION TO THE INTEGERS

105

Note that the bars in Figure 2.2 for the months January, February, March, and June have heights greater than zero, representing a prot in each of those months. The bars for the months April and May have heights that are less than zero, representing a loss for each of those months.

You Try It! EXAMPLE 9. Table 2.2 contains the low temperature recordings (degrees Fahrenheit) on ve consecutive days in Fairbanks, Alaska, 1995. Create a line graph for the data in Table 2.2. Date Low Temp Jan 21 1 Jan 22 10 Jan 23 5 Jan 24 20 Jan 25 28 A man stands on the roof of a multistory building and throws a baseball vertically upward. The height (in feet) of the ball above the edge of the roof at measured times (in seconds) is given in the following table. Time 0 1 2 3 4 Height 0 24 16 24 96

Table 2.2: Temperature readings are in degrees Fahrenheit.

Solution. Start by labeling the horizontal axis with the days in January that the temperatures occurred. Scale the vertical axis to accommodate the temperatures given in Table 2.2. Finally, plot points on each day at a height that equals the temperature for that given day. Connect consecutive pairs of points with line segments to produce the line graph shown in Figure 2.3. 15 10 5 0 5 10 15 20 25 30 35 Temperature (degrees Fahrenheit)

Create a line graph of the height of the ball versus time in the air.

Jan 21 Jan 22 Jan 23 Jan 24 Jan 25 Figure 2.3: Plottting daily low temps versus the date they occurred. Note that the points in Figure 2.3 have heights greater than zero for January 21-23, representing temperatures greater than zero. The points representing January 24-45 have negative heights corresponding to the negative temperatures of Table 2.2.

Version: Fall 2010

106

CHAPTER 2. THE INTEGERS

l l l

Exercises

l l l

In Exercises 1-12, for each of the following questions, provide a number line sketch with your answer. 1. What number lies three units to the left of 4 on the number line? 2. What number lies three units to the left of 1 on the number line? 3. What number lies three units to the left of 6 on the number line? 4. What number lies four units to the left of 2 on the number line? 5. What number lies two units to the right of 0 on the number line? 6. What number lies four units to the right of 2 on the number line? 7. What number lies two units to the right of 1 on the number line? 8. What number lies two units to the right of 4 on the number line? 9. What number lies four units to the left of 6 on the number line? 10. What number lies two units to the left of 0 on the number line? 11. What number lies two units to the right of 5 on the number line? 12. What number lies three units to the right of 6 on the number line?

In Exercises 13-24, for each of the following sets of integers, perform the following tasks: i) Plot each of the integers on a numberline. ii) List the numbers in order, from smallest to largest. 13. 6, 3, 1, and 2 14. 3, 2, 4, and 6 15. 3, 1, 1, and 6 16. 6, 4, 1, and 5 17. 6, 4, 1, and 5 18. 6, 4, 2, and 6 19. 6, 2, 3, and 6 20. 6, 4, 2, and 2 21. 4, 1, 1, and 6 22. 5, 1, 3, and 6 23. 4, 2, 1, and 5 24. 3, 1, 3, and 6

In Exercises 25-36, in each of the following exercises, enter the inequality symbol < or the symbol > in the shaded box in order that the resulting inequality is a true statement. 25. 4 26. 4 27. 2 0 3 1 Version: Fall 2010 28. 3 29. 3 30. 6 5 0 1

2.1. AN INTRODUCTION TO THE INTEGERS 31. 3 32. 4 33. 3 6 2 6 34. 0 35. 1 36. 1 3 4 4

107

In Exercises 37-52, simplify each of the following expressions. 37. (4). 38. (6). 39. |7|. 40. |1|. 41. |5|. 42. |3|. 43. | 11|. 44. | 1|. 45. | 5|. 46. | 1|. 47. | 20|. 48. | 8|. 49. | 4|. 50. | 3|. 51. (2). 52. (17).

In Exercises 53-64, for each of the following exercises, provide a number line sketch with your answer. 53. Find two integers on the number line that are 2 units away from the integer 2. 54. Find two integers on the number line that are 2 units away from the integer 3. 55. Find two integers on the number line that are 4 units away from the integer 3. 56. Find two integers on the number line that are 2 units away from the integer 2. 57. Find two integers on the number line that are 3 units away from the integer 2. 58. Find two integers on the number line that are 4 units away from the integer 1. 59. Find two integers on the number line that are 2 units away from the integer 3. 60. Find two integers on the number line that are 3 units away from the integer 3. 61. Find two integers on the number line that are 3 units away from the integer 0. 62. Find two integers on the number line that are 4 units away from the integer 2. 63. Find two integers on the number line that are 2 units away from the integer 0. 64. Find two integers on the number line that are 3 units away from the integer 1.

Version: Fall 2010

108

CHAPTER 2. THE INTEGERS

65. Dam. Utahs lowest point of elevation is 2,350 feet above sea level and occurs at Beaver Dam Wash. Express the height as an integer.

66. Underwater Glider. The National Oceanic and Atmospheric Administrations underwater glider samples the bottom of the Atlantic Ocean at 660 feet below sea level. Express that depth as an integer. Associated Press Times-Standard 4/19/09

67. Prots and losses for the rst six months of the scal year for a small business are shown in the following bar chart. 14 12 10 8 6 4 2 Feb Mar Apr 0 Jan Feb Mar Apr May Jun 2 4 6 Create a table showing the prot and loss for each month. Use positive integers for the prot and negative integers for the loss. Create a line graph using the data in your table. Prot/Loss (thousand dollars) Prot/Loss (thousand dollars)

68. Prots and losses for the rst six months of the scal year for a small business are shown in the following bar chart. 14 12 10 8 6 4 2 Jan Feb Mar 0 Jan Feb Mar Apr May Jun 2 4 6 Create a table showing the prot and loss for each month. Use positive integers for the prot and negative integers for the loss. Create a line graph using the data in your table.

Version: Fall 2010

2.1. AN INTRODUCTION TO THE INTEGERS

109

69. The following line graph displays the low temperature recordings (degrees Fahrenheit) on ve consecutive days in Big Bear, California. What was the lowest temperature reading for the week and on what date did it occur? Temperature (degrees Fahrenheit) 15 10 5 0 5 10 15 1/13 1/14 1/15 1/16 1/17

70. The following line graph displays the low temperature recordings (degrees Fahrenheit) on ve consecutive days in Ogden, Utah. What was the lowest temperature reading for the week and on what date did it occur? Temperature (degrees Fahrenheit) 15 10 5 0 5 10 15 1/18 1/19 1/20 1/21 1/22

71. The following table contains the low temperature recordings (degrees Fahrenheit) on ve consecutive days in Littletown, Ohio. Create a line graph for the data. Date Low Temp Feb 11 10 Feb 12 2 Feb 13 5 Feb 14 12 Feb 15 8

72. The following table contains the low temperature recordings (degrees Fahrenheit) on ve consecutive days in MyTown, Ottawa. Create a line graph for the data. Date Low Temp Apr 20 10 Apr 21 2 Apr 22 8 Apr 23 5 Apr 24 5

Version: Fall 2010

110

CHAPTER 2. THE INTEGERS

l l l
1. 1 7 3. 3 7 5. 0 7 7. 1 7 9. 2 7 11. 7 13. 0 6 7 0 3 7 0 2 7 0 6 7 0 4 7

Answers
15.

l l l
i) Arrange the integers 3, 1, 1, and 6 on a number line. 3 1 1 7 ii) 3, 1, 1, 6 0 6 7

17.

i) Arrange the integers 6, 4, 1, and 5 on a number line. 6 4 7 ii) 6, 4, 1, 5 1 0 5 7

19.

i) Arrange the integers 6, 2, 3, and 6 on a number line. 6 7 ii) 6, 2, 3, 6 2 0 3 6 7

21. 5 3 0 7

i) Arrange the integers 4, 1, 1, and 6 on a number line. 4 7 ii) 4, 1, 1, 6 1 1 0 6 7

i) Arrange the integers 6, 3, 1, and 2 on a number line. 6 7 ii) 6, 3, 1, 2 3 1 0 2 7

23.

i) Arrange the integers 4, 2, 1, and 5 on a number line. 4 2 7 ii) 4, 2, 1, 5 0 1 5 7

Version: Fall 2010

2.1. AN INTRODUCTION TO THE INTEGERS 25. 4 < 0 27. 2 < 1 29. 3 < 1 31. 3 < 6 33. 3 > 6 35. 1 < 4 37. 4 39. 7 41. 5 43. 11 45. 5 47. 20 49. 4 51. 2 53. 0 and 4. 2 7 0 2 2 4 7 14 12 10 8 6 4 2 0 2 4 6 Temperature (degrees Fahrenheit) 65. 2,350 feet 67. Month Jan Feb Mar Apr May Jun 7 2 0 2 63. 2 and 2. 2 2 7 3 61. 3 and 3. 3 0 3 3 7 0 1 59. 1 and 5. 2 3

111

2 5 7

Prot/Loss 8 4 3 2 2 5

55. 7 and 1. 4 7 3

4 0 1 7

57. 5 and 1. 3 7 5 3 2 0 1 7

Jan Feb Mar Apr May Jun Version: Fall 2010

112 69. Approximately 8 Fahrenheit on January 16.

CHAPTER 2. THE INTEGERS 71. 15 10 5 0 5 10 15 2/11 2/12 2/13 2/14 2/15

Version: Fall 2010

Temperature (degrees Fahrenheit)

2.2. ADDING INTEGERS

113

2.2

Adding Integers

Like our work with the whole numbers, addition of integers is best explained through the use of number line diagrams. However, before we start, lets take a moment to discuss the concept of a vector. Vectors. A vector is a mathematical object that possesses two import qualities: (1) magnitude or length, and (2) directon. Vectors are a fundamental problem solving tool in mathematics, science, and engineering. In physics, vectors are used to represent forces, position, velocity, and acceleration, while engineers use vectors to represent both internal and external forces on structures, such as bridges and buildings. In this course, and in this particular section, we will concentrate on the use of vectors to help explain addition of integers.

Vectors on the Number Line


Consider the number line in Figure 2.4. 4 7 6 5 4 3 2 1

Figure 2.4: A vector representing positive four. Above the line weve drawn a vector with tail starting at the integer 0 and arrowhead nishing at the integer 4. There are two important things to note about this vector: 1. The magnitude (length) of the vector in Figure 2.4 is four. 2. The vector in Figure 2.4 points to the right. We will agree that the vector in Figure 2.4 represents positive four. It is not important that the vector start at the origin. Consider, for example, the vector pictured in Figure 2.5. 4 7 6 5 4 3 2 1

Figure 2.5: A vector representing positive four. Again, there are two important observations to be made: Version: Fall 2010

114

CHAPTER 2. THE INTEGERS

1. The magnitude (length) of the vector in Figure 2.5 is four. 2. The vector in Figure 2.5 points to the right. Hopefully, you have the idea. Any vector that has length 4 and points to the right will represent positive four, regardless of its starting or nishing point. Conversely, consider the vector in Figure 2.6, which starts at the integer 4 and nishes at the integer 3. 7 7 6 5 4 3 2 1

Figure 2.6: A vector representing negative seven. Two observations: 1. The magnitude (length) of the vector in Figure 2.6 is seven. 2. The vector in Figure 2.6 points to the left. We will agree that the vector in Figure 2.6 represents negative seven. We could select dierent starting and nishing points for our vector, but as long as the vector has length seven and points to the left, it represents the integer 7. Important Observation. A vector pointing to the right represents a positive number. A vector pointing to the left represents a negative number.

Magnitude and Absolute Value


In Figure 2.4 and Figure 2.5, the vectors pictured represent the integer positive four. Note that the absolute value of four is four; that is, |4| = 4. Note also that this absolute value is the magnitude or length of the vectors representing the integer positive four in Figure 2.4 and Figure 2.5. In Figure 2.6, the vector pictured represents the integer 7. Note that | 7| = 7. This shows that the absolute value represents the magnitude or length of the vector representing 7. Magnitude and Absolute Value. If a is an integer, then |a| gives the magnitude or length of the vector that represents the integer a.

Version: Fall 2010

2.2. ADDING INTEGERS

115

Adding Integers with Like Signs


Because the positive integers are also whole numbers, weve already seen how to add to them in Section 1.2. You Try It! EXAMPLE 1. Find the sum 3 + 4. Solution. To add the positive integers 3 and 4, proceed as follows. 1. Start at the integer 0, then draw a vector 3 units in length pointing to the right, as shown in Figure 2.7. This arrow has magnitude (length) three and represents the positive integer 3. 2. Draw a second vector of length four that points to the right, starting at the end of the rst vector representing the positive integer 3. This arrow has magnitude (length) four and represents the positive integer 4. 3. The sum of the positive integers 3 and 4 could be represented by a vector that starts at the integer 0 and ends at the positive integer 7. However, we prefer to mark this sum on the number line as a solid dot at the positive integer 7. This integer represents the sum of the positive integers 3 and 4. Start 3 7 6 5 4 3 2 1 0 1 2 3 4 4 5 6 7 End Use a number line diagram to show the sum 5 + 7.

Figure 2.7: Adding two positive integers on the number line. Thus, 3 + 4 = 7. Answer: 12.

Negative integers are added in a similar fashion. You Try It! EXAMPLE 2. Find the sum 3 + (4). Solution. To add the negative integers 3 and 4, proceed as follows. 1. Start at the integer 0, then draw a vector 3 units in length pointing to the left, as shown in Figure 2.8. This arrow has magnitude (length) three and represents the negative integer 3. 2. Draw a second vector of length four that points to the left, starting at the end of the rst vector representing the negative integer 3. This arrow has magnitude (length) four and represents the negative integer 4. Version: Fall 2010 Use a number line diagram to show the sum 7 + (3).

116

CHAPTER 2. THE INTEGERS

3. The sum of the negative integers 3 and 4 could be represented by a vector that starts at the integer 0 and ends at the negative integer 7. However, we prefer to mark this sum on the number line as a solid dot at the negative integer 7. This integer represents the sum of the negative integers 3 and 4. End 4 3 Start

7 6 5 4 3 2 1 0

Figure 2.8: Adding two positive integers on the number line. Thus, 3 + (4) = 7. Drawing on Physical Intuition. Imagine that you are walking the number line in Figure 2.8. You start at the origin (zero) and take 3 paces to the left. Next, you walk an additional four paces to the left, landing at the number 7.

Answer: 10.

It should come as no surprise that the procedure used to add two negative integers comprises two steps. Adding Two Negative Integers. To add two negative integers, proceed as follows: 1. Add the magnitudes of the integers. 2. Prex the common negative sign.

You Try It! Find the sum: 5 + (9). EXAMPLE 3. Find the sums: (a) 4 + (5), (b) 12 + (9), and (c) 2 + (16). Solution. Well examine three separate but equivalent approaches, as discussed in the narrative above. a) The number line schematic End 5 4 Start

9 8 7 6 5 4 3 2 1 0 shows that (4) + (5) = 9. Version: Fall 2010

2.2. ADDING INTEGERS

117

b) Drawing on physical intuition, start at zero, walk 12 units to the left, then an additional 9 units to the left. You should nd yourself 21 units to the left of the origin (zero). Hence, 12 + (9) = 21. c) Following the algorithm above in Adding Two Negative Integers, rst add the magnitudes of 2 and 16; that is, 2+16 = 18. Now prex the common sign. Hence, 2 + (16) = 18. Answer: 14.

Adding Integers with Unlike Signs


Adding integers with unlike signs is no harder than adding integers with like signs. You Try It! EXAMPLE 4. Find the sum 8 + 4. Solution. To nd the sum 8 + 4, proceed as follows: 1. Start at the integer 0, then draw a vector eight units in length pointing to the left, as shown in Figure 2.9. This arrow has magnitude (length) eight and represents the negative integer 8. 2. Draw a second vector of length four that points to the right, starting at the end of the rst vector representing the negative integer 8. This arrow (also shown in Figure 2.9) has magnitude (length) four and represents the positive integer 4. 3. The sum of the negative integers 8 and 4 could be represented by a vector that starts at the integer 0 and ends at the negative integer 4. However, we prefer to mark this sum on the number line as a solid dot at the negative integer 4. This integer represents the sum of the integers 8 and 4. 8 4 End 9 8 7 6 5 4 3 2 1 0 1 Use a number line diagram to show the sum 9 + 2.

Start

Figure 2.9: Adding 8 and 4 on the number line. Thus, 8 + 4 = 4. Version: Fall 2010

118

CHAPTER 2. THE INTEGERS

Answer: 7.

Drawing on Physical Intuition. Imagine you are walking the number line in Figure 2.9. You start at the origin (zero) and walk eight paces to the left. Next, turn around and walk four paces to the right, landing on the number 4.

Note that adding integers with unlike signs is a subtractive process. This is due to the reversal of direction experienced in drawing Figure 2.9 in Example 4. Adding Two Integers with Unlike Signs. To add two integers with unlike signs, proceed as follows: 1. Subtract the smaller magnitude from the larger magnitude. 2. Prex the sign of the number with the larger magnitude.

For example, to nd the sum 8 + 4 of Example 4, we would note that the integers 8 and 4 have magnitudes 8 and 4, respectively. We would then apply the process outlined in Adding Two Integers with Unlike Signs. 1. Subtract the smaller magnitude from the larger magnitude; that is, 8 4 = 4. 2. Prex the sign of the number with the larger magnitude. Because 8 has the larger magnitude and its sign is negative, we prex a negative sign to the dierence of the magnitudes. Thus, 8 + 4 = 4. You Try It! Use a number line diagram to show the sum 5 + (11). EXAMPLE 5. Find the sums: (a) 5 + (8), (b) 12 + 16, and (c) 117 + 115. Solution. Well examine three separate but equivalent approaches, as discussed in the narrative above. a) The number line schematic 5 Start End 4 3 2 1 0 shows that 5 + (8) = 3. b) Drawing on physical intuition, start at zero, walk 12 units to the left, then turn around and walk 16 units to the right. You should nd yourself 4 units to the right of the origin (zero). Hence, 12 + 16 = 4. Version: Fall 2010 1 2 3 4 5 6 8

2.2. ADDING INTEGERS

119

c) Following the algorithm in Adding Two Integers with Unlike Signs, subtract the smallar magnitude from the larger magnitude, thus 117 115 = 2. Because 117 has the larger magnitude and its sign is negative, we prex a negative sign to the dierence of the magnitudes. Thus, 117 + 115 = 2.

Answer: 6.

Properties of Addition of Integers


You will be pleased to learn that the properties of addition for whole numbers also apply to addition of integers. The Commutative Property of Addition. Let a and b represent two integers. Then, a + b = b + a.

You Try It! EXAMPLE 6. Show that 5 + (7) = 7 + 5. Solution. The number line schematic 5 Start End 8 7 6 5 4 3 2 1 0 1 2 3 4 5 6 7 Use a number line diagram to show that 8 + 6 is the same as 6 + (8).

shows that 5 + (7) = 2. On the other hand, the number line schematic 7 Start 5 End 8 7 6 5 4 3 2 1 0 1 2 3 4 5 6

shows that 7 + 5 = 2. Therefore, 5 + (7) = 7 + 5.

Addition of integers is also associative. The Associative Property of Addition. Let a, b, and c represent integers. Then, (a + b) + c = a + (b + c).

Version: Fall 2010

120

CHAPTER 2. THE INTEGERS

You Try It! Show that the expression (8 + 5) + 3 is the same as 8 + (5 + 3) by simplifying each of the two expressions independently. EXAMPLE 7. Show that (9 + 6) + 2 = 9 + (6 + 2). Solution. On the left, the grouping symbols demand that we add 9 and 6 rst. Thus, (9 + 6) + 2 = 3 + 2 = 1. On the right, the grouping symbols demand that we add 6 and 2 rst. Thus, 9 + (6 + 2) = 9 + 8 = 1. Both sides simplify to 1. Therefore, (9 + 6) + 2 = 9 + (6 + 2).

The Additive Identity Property. The integer zero is called the additive identity. If a is any integer, then a+0=a and 0 + a = a.

Thus, for example, 8 + 0 = 8 and 0 + (113) = 113. Finally, every integer has a unique opposite, called its additive inverse. The Additive Inverse Property. Let a represent any integer. Then there is a unique integer a, called the opposite or additive inverse of a, such that a + (a) = 0 and a + a = 0.

You Try It! Use a number line diagram to show that 9 + (9) = 0. EXAMPLE 8. Show that 5 + (5) = 0. Solution. The number line schematic Start 5 5 End 6 5 4 3 2 1 0 clearly shows that 5 + (5) = 0. Version: Fall 2010 1 2 3 4 5 6

2.2. ADDING INTEGERS

121

Important Observation. We have used several equivalent phrases to pronounce the integer a. Weve used the opposite of a, negative a, and the additive inverse of a. All are equivalent pronunciations.

Grouping for Eciency


Order of operations require that we perform all additions as they occur, working from left to right. You Try It! EXAMPLE 9. Simplify 7 + 8 + (9) + 12. Solution. We perform the additions as they occur, working left to right. 7 + 8 + (9) + 12 = 1 + (9) + 12 Working left to right, 7 + 8 = 1. = 8 + 12 =4 Thus, 7 + 8 + (9) + 12 = 4. Working left to right, 1 + (9) = 8. 8 + 12 = 4 Answer: 1 Simplify: 8 + 9 + (4) + 2.

The commutative property of addition tells us that changing the order of addition does not change the answer. The associative property of addition tells us that a sum is not aected by regrouping. Lets work Example 9 again, rst grouping positive and negative numbers together. You Try It! EXAMPLE 10. Simplify 7 + 8 + (9) + 12. Solution. The commutative and associative properties allows us to change the order of addition and regroup. 7 + 8 + (9) + 12 = 7 + (9) + 8 + 12 Use the commutative property to change the order. Simplify: 11 + 7 + (12) + 3.

= [7 + (9)] + [8 + 12] Use the associative property to regroup. = 16 + 20 =4 Thus, 7 + 8 + (9) + 12 = 4. Add the negatives. Add the positives. One nal addition. Answer: 13

Version: Fall 2010

122

CHAPTER 2. THE INTEGERS

At rst glance, there seems to be no advantage in using the technique in Example 10 over the technique used in Example 9. However, the technique in Example 10 is much quicker in practice, particularly if you eliminate some of the explanatory steps. Ecient Grouping. When asked to nd the sum of a number of integers, it is most ecient to rst add all the positive integers, then add the negatives, then add the results.

You Try It! Simplify: 11 + 3 + (2) + 7. EXAMPLE 11. Simplify 7 + 8 + (9) + 12. Solution. Add the positive integers rst, then the negatives, then add the results. 7 + 8 + (9) + 12 = 20 + (16) Add the positives: 8 + 12 = 20. Add the negatives: 7 + (9) = 16. =4 Answer: 7. Thus, 7 + 8 + (9) + 12 = 4. Add the results: 20 + (16) = 4.

Using Correct Notation. Never write +! That is, the notation 9 + 4 and 8 + 6

should not be used. Instead, use grouping symbols as follows: 9 + (4) and 8 + (6)

Version: Fall 2010

2.2. ADDING INTEGERS

123

l l l

Exercises

l l l

In Exercises 1-12, what integer is represented by the given vector? 1. 7.

7 2.

7 8.

7 3.

7 9.

7 4.

7 10.

7 5.

7 11.

7 6.

7 12.

Version: Fall 2010

124 In Exercises 13-36, nd the sum of the given integers. 13. 15 + 1 14. 1 + 18 15. 18 + (10) 16. 2 + (19) 17. 10 + (12) 18. 1 + (7) 19. 5 + 10 20. 1 + 12 21. 2 + 5 22. 14 + 1 23. 19 + (15) 24. 20 + (17)

CHAPTER 2. THE INTEGERS

25. 2 + (7) 26. 14 + (6) 27. 6 + 16 28. 2 + 14 29. 11 + (6) 30. 7 + (8) 31. 14 + (9) 32. 5 + (15) 33. 10 + 11 34. 14 + 11 35. 13 + 1 36. 8 + 2

In Exercises 37-52, state the property of addition depicted by the given identity. 37. 1 + (3 + (8)) = (1 + 3) + (8) 38. 4 + (6 + (5)) = (4 + 6) + (5) 39. 7 + (7) = 0 40. 14 + (14) = 0 41. 15 + (18) = 18 + 15 42. 14 + (8) = 8 + 14 43. 15 + 0 = 15 44. 11 + 0 = 11 45. 7 + (1 + (6)) = (7 + 1) + (6) 46. 4 + (8 + (1)) = (4 + 8) + (1) 47. 17 + (2) = 2 + 17 48. 5 + (13) = 13 + 5 49. 4 + 0 = 4 50. 7 + 0 = 7 51. 19 + (19) = 0 52. 5 + (5) = 0

In Exercises 53-64, state the additive inverse of the given integer. 53. 18 54. 10 55. 12 56. 15 Version: Fall 2010 57. 16 58. 4 59. 11 60. 13

2.2. ADDING INTEGERS 61. 15 62. 19 63. 18 64. 9

125

In Exercises 65-80, nd the sum of the given integers. 65. 6 + (1) + 3 + (4) 66. 6 + (3) + 2 + (7) 67. 15 + (1) + 2 68. 11 + (16) + 16 69. 17 + 12 + 3 70. 5 + (3) + 2 71. 7 + 20 + 19 72. 14 + (14) + (20) 73. 4 + (8) + 2 + (5) 74. 6 + (3) + 7 + (2) 75. 7 + (8) + 2 + (1) 76. 8 + (9) + 5 + (3) 77. 9 + (3) + 4 + (1) 78. 1 + (9) + 7 + (6) 79. 9 + 10 + 2 80. 6 + 15 + (18)

81. Bank Account. Gerry deposited a check into his bank account for $215. He then made several withdraws of $40, $75, and $20 before depositing another check for $185. How much is in Gerrys account now? 82. Dead Sea Sinking. Due to tectonic plate movement, the Dead Sea is sinking about 1 meter each year. If its currently 418 meters now, what will Dead Sea elevation be in 5 years? Write an expression that models this situation and compute the result. 83. Prot and Loss. Prots and losses for the rst six months of the scal year for a small business are shown in the following bar chart. Sum the prots and losses from each month. Was there a net prot or loss over the six-month period? How much? 14 12 10 8 6 4 2 0 2 4 6 Prot/Loss (thousand dollars)

Apr Jan Feb Mar Apr

May May Jun

Version: Fall 2010

126

CHAPTER 2. THE INTEGERS

84. Prot and Loss. Prots and losses for the rst six months of the scal year for a small business are shown in the following bar chart. Sum the prots and losses from each month. Was there a net prot or loss over the six-month period? How much? 14 12 10 8 6 4 2 0 2 4 6 Prot/Loss (thousand dollars)

Jan Jan

Feb Feb

Mar Mar

Apr Apr May Jun

l l l
1. 4 3. 6 5. 5 7. 6 9. 10 11. 7 13. 14 15. 8 17. 22 19. 15 21. 7 23. 4 25. 9 Version: Fall 2010

Answers
27. 10

l l l

29. 17 31. 5 33. 21 35. 12 37. Associative property of addition 39. Additive inverse property 41. Commutative property of addition 43. Additive identity property 45. Associative property of addition 47. Commutative property of addition 49. Additive identity property 51. Additive inverse property

2.2. ADDING INTEGERS 53. 18 55. 12 57. 16 59. 11 61. 15 63. 18 65. 4 67. 16 69. 2 71. 46 73. 7 75. 0 77. 9 79. 21 81. $265 83. Net Prot: $24,000

127

Version: Fall 2010

128

CHAPTER 2. THE INTEGERS

2.3

Subtracting Integers

In Section 1.2, we stated that Subtraction is the opposite of addition. Thus, to subtract 4 from 7, we walked seven units to the right on the number line, but then walked 4 units in the opposite direction (to the left), as shown in Figure 2.10. 7 Start 4 End 0 1 2 3 4 5 6 7 8

Figure 2.10: Subtraction requires that we reverse direction. Thus, 7 4 = 3. The key phrase is add the opposite. Thus, the subtraction 7 4 becomes the addition 7 + (4), which we would picture on the number line as shown in Figure 2.11. 7 Start End 0 1 2 3 4 5 6 7 8 4

Figure 2.11: Subtraction means add the opposite. Figure 2.10 and Figure 2.11 provide ample evidence that the subtraction 74 is identical to the addition 7+(4). Again, subtraction means add the opposite. That is, 7 4 = 7 + (4). Dening Subtraction. Subtraction means add the opposite. That is, if a and b are any integers, then a b = a + (b).

Thus, for example, 123150 = 123+(150) and 57(91) = 57+91. In each case, subtraction means add the opposite. In the rst case, subtracting 150 is the same as adding 150. In the second case, subtracting91 is the same as adding 91.

Version: Fall 2010

2.3. SUBTRACTING INTEGERS

129

You Try It! EXAMPLE 1. Find the dierences: (a) 4 8, (b) 15 13, and (c) 117 (115). Solution. In each case, subtraction means add the opposite. a) Change the subtraction to addition with the phrase subtraction means add the opposite. That is, 4 8 = 4 + (8). We can now perform this addition on the number line. 4 Start 8 End 6 5 4 3 2 1 0 1 2 3 4 5 Use each of the techniques in parts (a), (b), and (c) of Example 1 to evaluate the dierence 11 (9).

Thus, 4 8 = 4 + (8) = 4. b) First change the subtraction into addition by adding the opposite. That is, 15 13 = 15 + (13). We can now use physical intuition to perform the addition. Start at the origin (zero), walk 15 units to the left, then an additional 13 units to the left, arriving at the answer 28. That is, 15 13 = 15 + (13) = 28. c) First change the subtraction into addition by adding the opposite. That is, 117 (115) = 117 + 115. Using Adding Two Integers with Unlike Signs from Section 2.2, rst subtract the smaller magnitude from the larger magnitude; that is, 117 115 = 2. Because 117 has the larger magnitude and its sign is negative, prex a negative sign to the dierence in magnitudes. Thus, 117 (115) = 117 + 115 = 2. Answer: 2

Order of Operations
We will now apply the Rules Guiding Order of Operations from Section 1.5 to a number of example exercises.

Version: Fall 2010

130

CHAPTER 2. THE INTEGERS

You Try It! Simplify: 3 (9) 11. EXAMPLE 2. Simplify 5 (8) 7. Solution. We work from left to right, changing each subtraction by adding the opposite. 5 (8) 7 = 5 + 8 + (7) = 3 + (7) = 4 Answer: 5 Add the opposite of 8, which is 8. Add the opposite of 7, which is 7. Working left to right, 5 + 8 = 3. 3 + (7) = 4.

Grouping symbols say do me rst. You Try It! Simplify: 3 (3 3). EXAMPLE 3. Simplify 2 (2 4). Solution. Parenthetical expressions must be evaluated rst. 2 (2 4) = 2 (2 + (4)) Simplify the parenthetical expression rst. Add the opposite of 4, which is 4. = 2 (6) = 2 + 6 =4 Answer: 3 Inside the parentheses, 2 + (4) = 6. Subtracting a 6 is the same as adding a 6. Add: 2 + 6 = 4.

Change as a Dierence
Suppose that when I leave my house in the early morning, the temperature outside is 40 Fahrenheit. Later in the day, the temperature measures 60 Fahrenheit. How do I measure the change in the temperature? The Change in a Quantity. To measure the change in a quantity, always subtract the former measurement from the latter measurement. That is: Change in a Latter Former = Quantity Measurement Measurement

Version: Fall 2010

2.3. SUBTRACTING INTEGERS

131

Thus, to measure the change in temperature, I perform a subtraction as follows: Change in Temperature = = = Latter Measurement 60 F 20 F Former Measurement 40 F

Note that the positive answer is in accord with the fact that the temperature has increased. You Try It! EXAMPLE 4. Suppose that in the afternoon, the temperature measures then late evening the temperature drops to 44 Fahrenheit. Find the change in temperature. Solution. To measure the change in temperature, we must subtract the former measurement from the latter measurement. Change in Temperature = = = Latter Measurement 44 F 11 F Former Measurement 65 F Marianne awakes to a morning temperature of 54c irc Fahrenheit. A storm hits, dropping the temperature to 43 Fahrenheit.Find the change in temperature.

Note that the negative answer is in accord with the fact that the temperature has decreased. There has been a change of 11 Fahrenheit.

Answer: 11 Fahrenheit

You Try It! EXAMPLE 5. Sometimes a bar graph is not the most appropriate visualization for your data. For example, consider the bar graph in Figure 2.12 depicting the Dow Industrial Average for seven consecutive days in March of 2009. Because the bars are of almost equal height, it is dicult to detect uctuation or change in the Dow Industrial Average. Lets determine the change in the Dow Industrial average on a day-to-day basis. Remember to subtract the latter measurement minus the former (current day minus former day). This gives us the following changes. Consecutive Days Sun-Mon Mon-Tues Tues-Wed Wed-Thu Thu-Fri Fri-Sat Change in Dow Industrial Average 6900 7000 = 100 6800 6900 = 100 6800 6800 = 0 7000 6800 = 200 7100 7000 = 100 7200 7100 = 100 Version: Fall 2010

132 Dow Industrial Average 8000 7000 6000 5000 4000 3000 2000 1000 0

CHAPTER 2. THE INTEGERS

7000

6900

6800

6800

7000

7100

Sun Mon Tue Fri Wed Thu Figure 2.12: Prot and loss bar graph.

Sat

We will use the data in the table to construct a line graph. On the horizontal axis, we place the pairs of consecutive days (see Figure 2.13). On the vertical axis we place the Change in the Industrial Dow Average. At each pair of days we plot a point at a height equal to the change in Dow Industrial Average as calculated in our table. 250 Change in Dow Industrial Average 200 150 100 50 0 50 100 150 S-M M-T T-W W-Th Th-F F-S

Figure 2.13: Plotting change in Dow versus consecutive days. Note that the data as displayed by Figure 2.13 more readily shows the changes in the Dow Industrial Average on a day-to-day basis. For example, it is now easy to pick the day that saw the greatest increase in the Dow (from Wednesday to Thursday, the Dow rose 200 points).

Version: Fall 2010

7200

2.3. SUBTRACTING INTEGERS

133

l l l
In Exercises 1-24, nd the dierence. 1. 16 20 2. 17 2 3. 10 12 4. 16 8 5. 14 11 6. 5 8 7. 7 (16) 8. 20 (10) 9. 4 (9) 10. 13 (3) 11. 8 (3) 12. 14 (20)

Exercises

l l l

13. 2 11 14. 16 2 15. 8 (10) 16. 14 (2) 17. 13 (1) 18. 12 (13) 19. 4 (2) 20. 6 (8) 21. 7 (8) 22. 13 (14) 23. 3 (10) 24. 13 (9)

In Exercises 25-34, simplify the given expression. 25. 14 12 2 26. 19 (7) 11 27. 20 11 18 28. 7 (13) (1) 29. 5 (10) 20 30. 19 12 (8) 31. 14 12 19 32. 15 4 (6) 33. 11 (7) (6) 34. 5 (5) (14)

In Exercises 35-50, simplify the given expression. 35. 2 (6 (5)) 36. 6 (14 9) 37. (5 (8)) (3 (2)) 38. (6 (8)) (9 3) 39. (6 (9)) (3 (6)) 40. (2 (3)) (3 (6)) 41. 1 (10 (9)) 42. 7 (14 (8)) 43. 3 (8 17) 44. 1 (1 4) Version: Fall 2010

134 45. 13 (16 (1)) 46. 7 (3 (8)) 47. (7 (8)) (5 (2))

CHAPTER 2. THE INTEGERS 48. (6 5) (7 3) 49. (6 4) (8 2) 50. (2 (6)) (9 (3))

51. The rst recorded temperature is 42 F. Four hours later, the second temperature is 65 F. What is the change in temperature? 52. The rst recorded temperature is 79 F. Four hours later, the second temperature is 46 F. What is the change in temperature? 53. The rst recorded temperature is 30 F. Four hours later, the second temperature is 51 F. What is the change in temperature? 54. The rst recorded temperature is 109 F. Four hours later, the second temperature is 58 F. What is the change in temperature? 55. Typical temperatures in Fairbanks, Alaska in January are 2 degrees Fahrenheit in the daytime and 19 degrees Fahrenheit at night. What is the change in temperature from day to night? 56. Typical summertime temperatures in Fairbanks, Alaska in July are 79 degrees Fahrenheit in the daytime and 53 degrees Fahrenheit at night. What is the change in temperature from day to night? 57. Communication. A submarine 1600 feet below sea level communicates with a pilot ying 22,500 feet in the air directly above the submarine. How far is the communique traveling? 58. Highest to Lowest. The highest spot on earth is on Mount Everest in Nepal-Tibet at 8,848 meters. The lowest point on the earths crust is the Marianas Trench in the North Pacic Ocean at 10,923 meters below sea level. What is the distance between the highest and the lowest points on earth? Wikipedia http://en.wikipedia.org/wiki/Extremes on Earth 59. Lowest Elevation. The lowest point in North America is Death Valley, California at -282 feet. The lowest point on the entire earths landmass is on the shores of the Dead Sea along the IsraelJordan border with an elevation of -1,371 feet. How much lower is the Dead Sea shore from Death Valley? Version: Fall 2010

2.3. SUBTRACTING INTEGERS

135

60. Exam Scores. Freidas scores on her rst seven mathematics exams are shown in the following bar chart. Calculate the dierences between consecutive exams, then create a line graph of the dierences on each pair of consecutive exams. Between which two pairs of consecutive exams did Freida show the most improvement? 80 70 60 Exam Scores 50 70 69 68 68 70 71 72 40 30 20 10 0 Exam 1 Exam 2 Exam 3 Exam 4 Exam 5 Exam 6 Exam 7

l l l
1. 4 3. 2 5. 3 7. 23 9. 5 11. 11 13. 9 15. 2 17. 14 19. 2 21. 15 23. 7

Answers
25. 0

l l l

27. 49 29. 5 31. 45 33. 2 35. 1 37. 4 39. 6 41. 20 43. 28 45. 4 47. 8 49. 12 Version: Fall 2010

136 51. 23 F 53. 21 F 55. 17 degrees Fahrenheit 57. 24,100 feet 59. 1,089 feet lower

CHAPTER 2. THE INTEGERS

Version: Fall 2010

2.4. MULTIPLICATION AND DIVISION OF INTEGERS

137

2.4

Multiplication and Division of Integers

Before we begin, let it be known that the integers satisfy the same properties of multiplication as do the whole numbers. Integer Properties of Multiplication. Commutative Property. If a and b are integers, then their product commutes. That is, a b = b a.

Associative Property. If a, b, and c are integers, then their product is associative. That is, (a b) c = a (b c).

Multiplicative Identity Property. If a is any integer, then a1=a and 1 a = a.

Because multiplying any integer by 1 returns the identical integer, the integer 1 is called the multiplicative identity.

In Section 1.3, we learned that multiplication is equivalent to repeated addition. For example, 34 = 4+4+4.
three fours

On the number line, three sets of four is equivalent to walking three sets of four units to the right, starting from zero, as shown in Figure 2.14. Start 4 0 1 2 3 4 5 4 6 7 8 9 4 10 11 12 End

Figure 2.14: Note that 3 4 = 4 + 4 + 4. That is, 3 4 = 12. This example and a little thought should convince readers that the product of two positive integers will always be a positive integer. The Product of Two Positive Integers. If a and b are two positive integers, then their product ab is also a positive integer.

Version: Fall 2010

138

CHAPTER 2. THE INTEGERS

For example, 2 3 = 6 and 13 117 = 1521. In each case, the product of two positive integers is a positive integer.

The Product of a Positive Integer and a Negative Integer


If we continue with the idea that multiplication is equivalent to repeated addition, then it must be that 3 (4) = 4 + (4) + (4) .
three negative fours

Pictured on the number line, 3 (4) would then be equivalent to walking three sets of negative four units (to the left), starting from zero, as shown in Figure 2.15. End 4 4 4 Start

12 11 10 9 8 7 6 5 4 3 2 1

Figure 2.15: Note that 3 (4) = 4 + (4) + (4). That is, 3 (4) = 12. Note, at least in this particular case, that the product of a positive integer and a negative integer is a negative integer. Weve shown that 3 (4) = 12. However, integer multiplication is commutative, so it must also be true that 4 3 = 12. That is, the product of a negative integer and a positive integer is also a negative integer. Although not a proof, this argument motivates the following fact about integer multiplication. The Product of a Positive Integer and a Negative Integer. Two facts are true: 1. If a is a positive integer and b is a negative integer, then the product ab is a negative integer. 2. If a is a negative integer and b is a positive integer, then the product ab is a negative integer.

Thus, for example, 5 (12) = 60 and 13 2 = 26. In each case the answer is negative because we are taking a product where one of the factors is positive and the other is negative.

Version: Fall 2010

2.4. MULTIPLICATION AND DIVISION OF INTEGERS

139

The Distributive Property


The integers satisfy the distributive property. The Distributive Property. Let a, b, and c be integers. Then, a (b + c) = a b + a c. We say that multiplication is distributive with respect to addition. Note how the a is distributed. The a is multiplied times each term in the parentheses. For example, consider the expression 3 (4 + 5). We can evaluate this expression according to the order of operations, simplifying the expression inside the parentheses rst. 3 (4 + 5) = 3 9 = 27 (2.1) (2.2)

But we can also use the distributive property, multiplying each term inside the parentheses by three, then simplifying the result. 3 (4 + 5) = 3 4 + 3 5 = 12 + 15 = 27 Distribute the 3. Perform multiplications rst: 3 4 = 12 and 3 5 = 15. Add: 12 + 15 = 27.

Note that evaluating 3 (4 + 5) using the distributive property provides the same result as the evaluation (2.1) using the order of operations.

The Multiplicative Property of Zero


The distributive property can be used to provide proofs of a number of important properties of integers. One important property is the fact that if you multiply an integer by zero, the product is zero. Here is a proof of that fact that uses the distributive property. Let a be any integer. Then, a 0 = a (0 + 0) a0=a0+a0 Additive Identity Property: 0 + 0 = 0. Distribute a times each zero in the parentheses.

Next, to undo the eect of adding a 0, subtract a 0 from both sides of the equation. a0a0= a0+a0a0 0=a0 Subtract a 0 from both sides. a 0 a 0 = 0 on each side.

Version: Fall 2010

140

CHAPTER 2. THE INTEGERS

Multiplicative Property of Zero. Let a represent any integer. Then a0=0 and 0 a = 0.

Thus, for example, 18 0 = 0 and 0 122 = 0.

Multiplying by Minus One


Here is another useful application of the distributive property. (1)a + a = (1)a + 1a = (1 + 1)a = 0a =0 Replace a with 1a. Use the distributive property to factor out a. Replace 1 + 1 with 0. Replace 0a with 0.

Thus, (1)a + a = 0. That is, if you add (1)a to a you get zero. However, the Additive Inverse Property says that a is the unique number that you add to a to get zero. The conclusion must be that (1)a = a. Multiplying by Minus One. If a is any integer, then (1)a = a.

Thus, for example, 1(4) = 4 and 1(4) = (4) = 4. This property is rather important, as we will see in future work. Not only does it tell us that (1)a = a, but it also tells us that if we see a, then it can be interpreted to mean (1)a.

The Product of Two Negative Integers


We can employ the multiplicative property of 1, that is, (1)a = a to nd the product of two negative numbers. (4)(3) = [(1)(4)](3) = (1)[(4)(3)] = (1)(12) = (12) = 12 Replace 4 with (1)(4). Use the associative property to regroup. We know: (4)(3) = 12. (1)a = a. Here (1)(12) = (12). (a) = a. Here (12) = 12.

Thus, at least in the case of (4)(3), the product of two negative integers is a positive integer. This is true in general. Version: Fall 2010

2.4. MULTIPLICATION AND DIVISION OF INTEGERS

141

The Product of Two Negative Integers. If both a and b are negative integers, then their product ab is a positive integer. Thus, for example, (5)(7) = 35 and (112)(6) = 672. In each case the answer is positive, because the product of two negative integers is a positive integer.

Memory Device
Heres a simple memory device to help remember the rules for nding the product of two integers. Like and Unlike Signs. There are two cases: Unlike Signs. The product of two integers with unlike signs is negative. That is: (+)() = ()(+) = Like Signs. The product of two integers with like signs is positive. That is: (+)(+) = + ()() = +

You Try It! EXAMPLE 1. Simplify: (a) (3)(2), (b) (4)(10), and (c) (12)(3). Solution. In each example, we use the like and unlike signs approach. a) Like signs gives a positive result. Hence, (3)(2) = 6. b) Unlike signs gives a negative result. Hence, (4)(10) = 40. c) Unlike signs gives a negative result. Hence, (12)(3) = 36. Answer: (a) 48, (b) 33 Simplify: (a) (12)(4) and (b) (3)(11).

Version: Fall 2010

142

CHAPTER 2. THE INTEGERS

You Try It! Simplify: (2)(3)(4)(1). EXAMPLE 2. Simplify (3)(2)(4)(2). Solution. Order of operations demands that we work from left to right. (3)(2)(4)(2) = (6)(4)(2) = (24)(2) = 48 Answer: 24. Hence, (3)(2)(4)(2) = 48. Work left to right: (3)(2) = 6. Work left to right: (6)(4) = 24. Multiply: (24)(2) = 48.

You Try It! Simplify: 22 . (a) (2)2 and (b) EXAMPLE 3. Simplify: (a) (2)3 and (c) (3)4 . Solution. In each example, use am = a a a . . . a,
m times

then work left to right with the multiplication. a) Use the denition of an exponent, then order of operations. (2)3 = (2)(2)(2) = 4(2) = 8 b) Use the denition of an exponent, then order of operations. (3)4 = (3)(3)(3)(3) = 9(3)(3) = 27(3) = 81 Answer: (a) 4 and (b) 4. Write 3 as a factor four times. Work left to right: (3)(3) = 9. Work left to right: 9(3) = 27. Write 2 as a factor three times. Work left to right: (2)(2) = 4.

Example 3 motivates the following fact. Even and Odd. Two facts are apparent.

Version: Fall 2010

2.4. MULTIPLICATION AND DIVISION OF INTEGERS

143

1. If a product contains an odd number of negative factors, then the product is negative. 2. If a product contains an even number of negative factors, then the product is positive.

Thus, for example, (2)5 = (2)(2)(2)(2)(2) = 32 quickly evaluates as 32 as it has an odd number of negative factors. On the other hand, (2)6 = (2)(2)(2)(2)(2)(2) = 64 quickly evaluates as 64 as it has an even number of negative factors.

Division of Integers
Consider that 12 = 4 because 3(4) = 12 3 In like manner, 12 = 4 because 3(4) = 12. 3

and

12 12 = 4 because 3(4) = 12 and = 4 because 3(4) = 12. 3 3 Thus, the rules for dividing integers are the same as the rules for multiplying integers. Like and Unlike Signs. There are two cases: Unlike Signs. The quotient of two integers with unlike signs is negative. That is, (+) = () () = (+) Like Signs. The quotient of two integers with like signs is positive. That is, (+) =+ (+) () =+ ()

Version: Fall 2010

144

CHAPTER 2. THE INTEGERS

Thus, for example, 12/(6) = 2 and 44/(4) = 11. In the rst case, unlike signs gives a negative quotient. In the second case, like signs gives a positive quotient. One nal reminder. Division by Zero is Undened. If a is any integer, the quotient a 0 is undened. Division by zero is meaningless. See the discussion in Section 1.3 for a discussion on division by zero. You Try It! Simplify: (a) 24/4 and (b) 28/(7). EXAMPLE 4. Simplify: (a) 12/(4), (b) 6/(3), and (c) 15/0. Solution. In each example, we use the like and unlike signs approach. a) Like signs gives a positive result. Hence, 12 = 3. 4 b) Unlike signs gives a negative result. Hence, 6 = 2. 3 c) Division by zero is undened. Hence, 15 0 Answer: (a) 6, (b) 4 is undened.

Version: Fall 2010

2.4. MULTIPLICATION AND DIVISION OF INTEGERS

145

l l l

Exercises

l l l

In Exercises 1-16, state the property of multiplication depicted by the given identity. 1. (2) (16)(13) = (2)(16) (13) 2. (10) (15)(6) = (10)(15) (6) 3. (17)(10) = (10)(17) 4. (5)(3) = (3)(5) 5. (4)(11) = (11)(4) 6. (5)(11) = (11)(5) 7. 16 (8 + (15) = 16 8 + 16 (15) 8. 1 (16 + (6) = 1 (16) + 1 (6) 9. (17) (20)(11) = (17)(20) (11) 10. (14) (20)(18) = (14)(20) (18) 11. 19 1 = 19 12. 17 1 = 17 13. 8 1 = 8 14. 20 1 = 20 15. 14 (12 + 7 = 14 (12) + 14 7 16. 14 (3 + 6 = 14 (3) + (14) 6

In Exercises 17-36, simplify each given expression. 17. 4 7 18. 4 2 19. 3 (3) 20. 7 (9) 21. 1 10 22. 1 11 23. 1 0 24. 8 0 25. 1 (14) 26. 1 (13) 27. 1 (19) 28. 1 (17) 29. 2 0 30. 6 0 31. 3 8 32. 7 (3) 33. 7 9 34. 6 3 35. 1 5 36. 1 2

In Exercises 37-48, simplify each given expression. 37. (7)(1)(3) 38. (10)(6)(3) 39. (7)(9)(10)(10) 40. (8)(5)(7)(9) 41. (6)(5)(8) 42. (7)(1)(9) Version: Fall 2010

146 43. (10)(4)(3)(8) 44. (8)(2)(5)(2) 45. (6)(3)(8)

CHAPTER 2. THE INTEGERS 46. (5)(4)(1) 47. (2)(1)(3)(4) 48. (7)(5)(1)(4)

In Exercises 49-60, compute the exact value. 49. (4)4 50. (3)4 51. (5)4 52. (2)2 53. (5)2 54. (3)3 55. (6)2 56. (6)4 57. (4)5 58. (4)2 59. (5)3 60. (3)2

In Exercises 61-84, simplify each given expression. 61. 16 (8) 62. 33 (3) 63. 64. 65. 66. 67. 8 1 40 20 1 0 2 0 3 3 74. 75. 76. 77. 73. 63 21 6 1 78 13 84 14 05

78. 0 (16) 79. 17 0 20 80. 0 81. 45 15 82. 28 28 83. 12 3 84. 22 (22)

68. 58 29 56 69. 28 60 70. 12 71. 0 15 72. 0 (4)

Version: Fall 2010

2.4. MULTIPLICATION AND DIVISION OF INTEGERS 85. Scuba. A diver goes down 25 feet, and then dives down 5 times further. Write the nal depth of the diver as an integer.

147

86. Investing Loss. An investing club of ve friends has lost $4400 on a trade. If they share the loss equally, write each members loss as an integer.

l l l
1. Associative property of multiplication 3. Commutative property of multiplication 5. Commutative property of multiplication 7. Distributive property 9. Associative property of multiplication 11. Multiplicative identity property 13. Multiplicative identity property 15. Distributive property 17. 28 19. 9 21. 10 23. 0 25. 14 27. 19

Answers

l l l

45. 144 47. 24 49. 256 51. 625 53. 25 55. 36 57. 1024 59. 125 61. 2 63. 8 65. Division by zero is undened. 67. 1 69. 2 71. 0

29. 0 31. 24 33. 63 35. 5 37. 21 39. 6300 41. 240 43. 960 73. 3 75. 6 77. 0 79. Division by zero is undened. 81. 3 83. 4 85. 125 feet Version: Fall 2010

148

CHAPTER 2. THE INTEGERS

2.5

Order of Operations with Integers

For convenience, we repeat the Rules Guiding Order of Operations rst introduced in Section 1.5. Rules Guiding Order of Operations. When evaluating expressions, proceed in the following order. 1. Evaluate expressions contained in grouping symbols rst. If grouping symbols are nested, evaluate the expression in the innermost pair of grouping symbols rst. 2. Evaluate all exponents that appear in the expression. 3. Perform all multiplications and divisions in the order that they appear in the expression, moving left to right. 4. Perform all additions and subtractions in the order that they appear in the expression, moving left to right.

Lets look at a number of examples that require the use of these rules. You Try It! Simplify: 22 . EXAMPLE 1. Simplify: (a) (3)2 and (b) 32 . Solution. Recall that for any integer a, we have (1)a = a. Because negating is equivalent to multiplying by 1, the Rules Guiding Order of Operations require that we address grouping symbols and exponents before negation. a) Because of the grouping symbols, we negate rst, then square. That is, (3)2 = (3)(3) = 9. b) There are no grouping symbols in this example. Thus, we must square rst, then negate. That is, 32 = (3 3) = 9. Answer: 4

Version: Fall 2010

2.5. ORDER OF OPERATIONS WITH INTEGERS

149

You Try It! EXAMPLE 2. Simplify: 2 3(5 7). Solution. Grouping symbols rst, then multiplication, then subtraction. 2 3(5 7) = 2 3(2) = 2 (6) = 2 + 6 =4 Answer: 1 Perform subtraction within parentheses. Multiply: 3(2) = 6. Add the opposite. Simplify: 3 2(6 8).

You Try It! EXAMPLE 3. Simplify: 2(2 4)2 3(3 5)3 . Solution. Grouping symbols rst, then multiplication, and subtraction, in that order. 2(2 4)2 3(3 5)3 = 2(2)2 3(2)3 = 2(4) 3(8) = 8 (24) = 8 + 24 = 16 Answer: 10 Perform subtraction within parentheses rst. Exponents are next. Multiplications are next. Add the opposite. . Simplify: 2(5 6)3 3(5 7)2

You Try It! EXAMPLE 4. Simplify: 24 8(3). Solution. Division has no preference over multiplication, or vice versa. Divisions and multiplications must be performed in the order that they occur, moving left to right. 24 8(3) = 3(3) =9 Division rst: 24 8 = 3. Simplify: 48 6(2).

Note that if you multiply rst, which would be incorrect, you would get a completely dierent answer.

Answer: 16

Version: Fall 2010

150

CHAPTER 2. THE INTEGERS

You Try It! Simplify: (4)(2)2 (1)3 . EXAMPLE 5. Simplify: (2)(3)(2)3 . Solution. Exponents rst, then multiplication in the order that it occurs, moving left to right. (2)(3)(2)3 = (2)(3)(8) Exponent rst: (2)3 = 8. = 6(8) = 48 Answer: 16 Multiply from left to right: (2)(3) = 6.

Evaluating Fractions
If a fraction bar is present, evaluate the numerator and denominator separately according to the Rules Guiding Order of Operations, then perform the division in the nal step. You Try It! Simplify: 6 2(6) 2 (2)2 EXAMPLE 6. Simplify: 5 5(2 4)3 22 3(5) Solution. Evaluate numerator and denominator separately, then divide. 5 5(2)3 5 5(2 4)3 = 22 3(5) 22 (15) = = = 5 5(8) 22 + 15 5 (40) 7 Numerator: parentheses rst. Denominator: multiply 3(5) = 15. Numerator: exponent (2)3 = 8. Denominator: add the opposite. Numerator: multiply 5(8) = 40. Denominator: add 22 + 15 = 7. Numerator: add the opposite. Numerator: 5 + 40 = 35. Divide: 35/ 7 = 5

5 + 40 7 35 = 7 = 5 Answer: 3

Version: Fall 2010

2.5. ORDER OF OPERATIONS WITH INTEGERS

151

Absolute Value
Absolute value calculates the magnitude of the vector associated with an integer, which is equal to the distance between the number and the origin (zero) on the number line. Thus, for example, |4| = 4 and | 5| = 5. But absolute value bars also act as grouping symbols, and according to the Rules Guiding Order of Operations, you should evaluate the expression inside a pair of grouping symbols rst. You Try It! EXAMPLE 7. Simplify: (a) (3) and (b) | 3|. Solution. There is a huge dierence between simple grouping symbols and absolute value. a) This is a case of (a) = a. Thus, (3) = 3. b) This case is much dierent. The absolute value of 3 is 3, and then the negative of that is 3. In symbols, | 3| = 3. Answer: 8 Simplify: | 8|.

You Try It! EXAMPLE 8. Simplify: 3 2|5 7|. Solution. Evaluate the expression inside the absolute value bars rst. Then multiply, then subtract. 3 2|5 7| = 3 2| 2| = 3 2(2) = 3 4 = 7 Subtract inside absolute value bars. Take the absolute value: | 2| = 2. Multiply: 2(2) = 4. Subtract. Answer: 10 Simplify: 2 4|6 8|.

Version: Fall 2010

152

CHAPTER 2. THE INTEGERS

l l l

Exercises

l l l

In Exercises 1-40, compute the exact value of the given expression. 14 2 16 2. 2 4 18 3. 7 9 7 4. 6 7 1. 7 5. 54 6. 3


3

20. 4 40 5 4 + 9 21. 64 4 4 22. 18 6 1 23. 2 3(5) 24. 64 7(7) 25. 15 1 3 26. 30 3 5 27. 8 + 12 6 1 5 28. 9 + 16 2 4 9 29. 32 4 4 30. 72 4 6 16 31. 11 + 16 20 32. 4 + 4 33. 52 34. 43 35. 10 + 12(5) 36. 4 + 12(4) 37. 2 + 6 1 6 1 38. 1 + 12 2 2 6 39. 40 5 4 40. 30 6 5

7. 9 1(7) 8. 85 8(9) 9. 63 10. 3


5

11. 3 + 9(4) 12. 6 + 7(1) 13. 10 72 6 3 + 8 14. 8 120 5 6 + 7 14 15. 6 + 2 8 16. 16 + 2 17. 34 18. 2
2

19. 3 24 4 3 + 4

Version: Fall 2010

2.5. ORDER OF OPERATIONS WITH INTEGERS In Exercises 41-80, simplify the given expression. 41. 11 + | 1 (6)2 | 42. 13 + | 21 (4)2 | 43. |0(4)| 4(4) 44. |10(3)| 3(1) 45. (2 + 3 4) 8 46. (11 + 5 2) 10 47. (8 1 12) + 4 48. (9 6 1) + 3 49. (6 + 10 4) 6 50. (8 + 7 6) 12 51. 10 + (6 4)3 3 52. 5 + (12 7)2 6 53. (6 8)2 (4 7)3 54. (3 8) (4 9) 55. |0(10)| + 4(4) 56. |12(5)| + 7(5) 57. |8(1)| 8(7) 58. |6(11)| 7(1) 59. 3 + (3 8)2 7 60. 9 + (8 3) 6
3 2 3

153

61. (4 2)2 (7 2)3 62. (1 4)2 (3 6)3 63. 8 | 25 (4)2 | 64. 20 | 22 42 | 65. 4 |30 (5)2 | 66. 8 | 11 (6)2 | 67. (8 7)2 (2 6)3 68. (2 7)2 (4 7)3 69. 4 (3 6)3 + 4 70. 6 (7 8)3 + 2 71. 3 + | 22 52 | 72. 12 + |23 (6)2 | 73. (3 4 1) + 6 74. (12 1 6) + 4 75. 1 (1 5)2 + 11 76. 9 (3 1)3 + 10 77. (2 6)2 (8 6)3 78. (2 7)2 (2 4)3 79. |9(3)| + 12(2) 80. |0(3)| + 9(7)

In Exercises 81-104, simplify the given expression. 81. 82. 83. 84. 4(10) 5 9 4 6 (8) 4 102 42 2 6 10 32 92 275 85. 32 + 62 518 102 + 42 165 8 4 7 13 13 1 84 Version: Fall 2010

86. 87. 88.

154 22 + 62 11 4 4 72 + 32 10 8 1 12 52 915 52 72 2 2 12 42 82 632 72 62 635 102 + 22 7 2 10 22 + 102 10 2 7

CHAPTER 2. THE INTEGERS 16 (2) 19 1 8 20 15 (17) 15 (15) 13 (17) 7 (9) 1 1 4 5 (19) 3 10 7 (11) 3

89. 90. 91. 92. 93. 94. 95. 96.

97. 98. 99. 100. 101. 102. 103.

6 9 (4) 2 6 2 10 104. 11

l l l
1. 14 3. 5 5. 625 7. 16 9. 216 11. 39 13. 18 15. 13 17. 81 19. 11 21. 64 Version: Fall 2010

Answers
23. 13 25. 45 27. 5 29. 32

l l l

31. 10 33. 25 35. 50 37. 37 39. 32 41. 26 43. 16 45. 6

2.5. ORDER OF OPERATIONS WITH INTEGERS 47. 0 49. 40 51. 15 53. 31 55. 16 57. 64 59. 21 61. 121 63. 33 65. 9 67. 65 97. 1 69. 35 99. 1 71. 44 73. 5 75. 4 101. 13 103. 25 77. 8 79. 3 81. 5 83. 42 85. 15 87. 2 89. 8 91. 6 93. 3 95. 8

155

Version: Fall 2010

156

CHAPTER 2. THE INTEGERS

2.6

Solving Equations Involving Integers

Recall (see Section 1.6) that a variable is a symbol (usually a letter) that stands for a value that varies. If a variable in an equation is replaced by a number and a true statement results, then that number is called a solution of the equation. You Try It! Is 4 a solution of 8 2x = 5? EXAMPLE 1. Is 6 a solution of the equation 2x + 5 = 7? Solution. Substitute 6 for x in the equation. 2x + 5 = 7 2(6) + 5 = 7 12 + 5 = 7 7 = 7 Answer: No. Original equation. Substitute 6 for x. On the left, multiply rst. On the left, add.

Because the last statement is a true statement, 6 is a solution of the equation.

Adding or Subtracting the Same Amount


Two equations having the same set of solutions are equivalent. For example, 2x+5 = 7 and x = 6 have the same solutions. Therefore, they are equivalent equations. Certain algebraic operations produce equivalent equations. Producing Equivalent Equations. Adding the Same Quantity to Both Sides of an Equation. If we start with the equation a = b, then adding c to both sides of the equation produces the equivalent equation a + c = b + c. Subtracting the Same Quantity from Both Sides of an Equation. If we start with the equation a = b, then subtracting c from both sides of the equation produces the equivalent equation a c = b c.

Version: Fall 2010

2.6. SOLVING EQUATIONS INVOLVING INTEGERS

157

That is, adding or subtracting the same amount from both sides of an equation will not change the solutions of the equation. You Try It! EXAMPLE 2. Solve for x: x + 3 = 7. Solve for x: x + 9 = 11

Solution. To undo the eect of adding 3, subtract 3 from both sides of the equation. x + 3 = 7 x + 3 3 = 7 3 x = 7 + (3) x = 10 To check the solution, substitute 10 for x in the original equation and simplify. x + 3 = 7 10 + 3 = 7 7 = 7 Original equation. Substitute 10 for x. Simplify both sides. Original equation. Subtract 3 from both sides. Simplify the left hand side. On the right, express subtraction as adding the opposite.

Since the last line of the check is a true statement, this conrms that 10 is a solution.

Answer: x = 20

You Try It! EXAMPLE 3. Solve for x: x 8 = 11. Solve for x: x 2 = 7

Solution. To undo the eect of subtracting 8, add 8 to both sides of the equation. x 8 = 11 x 8 + 8 = 11 + 8 x = 3 Original equation. Add 8 to both sides. Simplify both sides.

To check the solution, substitute 3 for x in the original equation and simplify. x 8 = 11 3 8 = 11 11 = 11 Original equation. Substitute 3 for x. Simplify both sides. Answer: x = 5

Since the last line of the check is a true statement, this conrms that 3 is a solution.

Version: Fall 2010

158

CHAPTER 2. THE INTEGERS

Sometimes a bit of simplication is in order before you start the solution process. You Try It! Solve for y: y + 2(4) = 8 + 6 EXAMPLE 4. Solve for y: 8 + 2 = y 11(4) 6 = y (44) 6 = y + 44 8 + 2 = y 11(4).

Solution. First, simplify both sides of the equation. Original equation. Simplify. On the left, 8 + 2 = 6. On the right, 11(4) = 44. Express subtraction as adding the opposite.

6 44 = y + 44 44 Subtract 44 from both sides of the equation. 6 + (44) = y Express subtraction as addition. Simplify on the right. 50 = y To check the solution, substitute 50 for y in the original equation and simplify. 8 + 2 = y 11(4) 8 + 2 = 50 11(4) 6 = 50 (44) 6 = 50 + 44 6 = 6 Answer: y = 6 Original equation. Substitute 50 for y. On the left, add. On the right, multiply rst: 11(4) = 44. Express subtraction on the right as addition. On the right, add: 50 + 44 = 6.

Since the last line of the check is a true statement, this conrms that 50 is a solution.

Multiplying or Dividing by the Same Amount


Adding and subtracting is not the only way to produce an equivalent equation. Producing Equivalent Equations. Multiplying Both Sides of an Equation by the Same Quantity. If we start with the equation a = b, then multiplying both sides of the equation by c produces the equivalent equation a c = b c, or equivalently, ac = bc, provided c = 0.

Version: Fall 2010

2.6. SOLVING EQUATIONS INVOLVING INTEGERS

159

Dividing Both Sides of an Equation by the Same Quantity. If we start with the equation a = b, then dividing both sides of the equation by c produces the equivalent equation b a = , c c provided c = 0.

That is, multiplying or dividing both sides of an equation by the same amount will not change the solutions of the equation. You Try It! EXAMPLE 5. Solve for x: 3x = 30. Solve for z: 4z = 28

Solution. To undo the eect of multiplying by 3, divide both sides of the equation by 3. 3x = 30 3x 30 = 3 3 x = 10

Original equation. Divide both sides by 3. On the left, 3 times x, divided by 3 is x. On the right, 30/(3) = 10.

To check the solution, substitute 10 for x in the original equation and simplify. 3x = 30 3(10) = 30 30 = 30 Original equation. Substitute 10 for x. Simplify.

Because the last line of the check is a true statement, this conrms that 10 is a solution.

Answer: z = 7

You Try It! EXAMPLE 6. Solve for x: x = 20. 2 Solve for t: t = 11 3

Solution. To undo the eect of dividing by 2, multiply both sides of the equation by 2. Version: Fall 2010

160

CHAPTER 2. THE INTEGERS

x = 20 2 x = 2(20) 2 x = 40

Original equation. Multiply both sides by 2. On the left, x divided by 2, multiplied by 2, the result is x. On the right, 2(20) = 40.

To check the solution, substitute 40 foro x in the original equation and simplify. x = 20 2 40 = 20 2 20 = 20 Answer: t = 33 Original equation. Substitute 40 for x. Simplify both sides.

Because the last line of the check is a true statement, this conrms that 40 is a solution.

Combining Operations
Recall the Wrap and Unwrap discussion from Section 1.6. To wrap a present we: (1) put the gift paper on, (2) put the tape on, and (3) put the decorative bow on. To unwrap the gift, we must undo each of these steps in inverse order. Hence, to unwrap the gift we: (1) take o the decorative bow, (2) take o the tape, and (3) take o the gift paper. Now, imagine a machine that takes its input, then: (1) multiplies the input by 2, and (2) adds 3 to the result. This machine is pictured on the left in Figure 2.16. x 1. Multiply by 2. 2. Add 3. x 1. Subtract 3. 2. Divide by 2.

Figure 2.16: The second machine unwraps the rst machine. To unwrap the eect of the machine on the left, we need a machine that will undo each of the steps of the rst machine, but in inverse order. The unwrapping machine is pictured on the right in Figure 2.16. It will rst subtract three from its input, then divide the result by 2. Note that each of these operations undoes the corresponding operation of the rst machine, but in inverse order. Version: Fall 2010

2.6. SOLVING EQUATIONS INVOLVING INTEGERS

161

For example, drop the integer 7 into the rst machine on the left in Figure 2.16. First, we double 7, then add 3 to the result. The result is 2(7) + 3 = 17. Now, to unwrap this result, we drop 17 into the second machine. We rst subtract 3, then divide by 2. The result is (17 3)/2 = 7, the original integer input into the rst machine. Now, consider the equation 2x + 3 = 7. On the left, order of operations demands that we rst multiply x by 2, then add 3. To solve this equation for x, we must undo each of these operations in inverse order. Thus, we will (1) subtract three from both sides of the equation, then (2) divide both sides of the resulting equation by 2. 2x + 3 3 = 7 3 2x = 4 4 2x = 2 2 x=2 Subtract 3 from both sides. Simplify both sides. Divide both sides by 2. Simplify both sides.

Readers should check this solution in the original equation. You Try It! x 3 = 7. EXAMPLE 7. Solve for x: 4 Solution. On the left, order of operations demands that we rst divide x by 4, then subtract 3. To solve this equation for x, we must undo each of these operations in inverse order. Thus, we will (1) add 3 to both sides of the equation, then (2) multiply both sides of the resulting equation by 4. x 3 = 7 4 Original equation. Add 3 to both sides. Simplify both sides. Multiply both sides by 4. Simplify both sides. Solve for x: x +6=4 2

x 3 + 3 = 7 + 3 4 x = 4 4 x = 4(4) 4 4 x = 16

Check. Substitute 16 for x in the original equation. x 3 = 7 Original equation. 4 16 3 = 7 Substitute 16 for x. 4 4 3 = 7 Divide rst: 16/4 = 4. 7 = 7 Subtract: 4 3 = 7. Version: Fall 2010

162

CHAPTER 2. THE INTEGERS

Answer: x = 4

Because the last line of the check is a true statement, 16 is a solution of the original equation.

You Try It! Solve for r: 0 = 9 + 3r EXAMPLE 8. Solve for t: 0 = 8 2t.

Solution. On the right, order of operations demands that we rst multiply t by 2, then add 8. To solve this equation for t, we must undo each of these operations in inverse order. Thus, we will (1) subtract 8 from both sides of the equation, then (2) divide both sides of the resulting equation by 2. 0 = 8 2t 0 8 = 8 2t 8 8 = 2t 2t 8 = 2 2 4=t Original equation. Subtract 8 from both sides. Simplify both sides. Divide both sides by -2. Simplify both sides.

Check. Substitute 4 for t in the original equation. 0 = 8 2t 0 = 8 2(4) 0 = 88 0=0 Answer: r = 3 Original equation. Substitute 4 for t. Multiply rst: 2(4) = 8. Subtract: 8 8 = 0.

Because the last line in the check is a true statement, 4 is a solution of the original equation.

You Try It! Solve for q: q 9 = 8 + 3 2 EXAMPLE 9. Solve for p: 12 + 3 = 8 + 4 + Solution. Always simplify when possible. 12 + 3 = 8 + 4 + 9 = 4 + p 3 p 3 Original equation. Simplify both sides. p . 3

On the right, order of operations demands that we rst divide p by 3, then add 4. To solve this equation for p, we must undo each of these operations Version: Fall 2010

2.6. SOLVING EQUATIONS INVOLVING INTEGERS

163

in inverse order. Thus, we will (1) add a positive 4 to both sides of the equation, then (2) multiply both sides of the resulting equation by 3. 9 + 4 = 4 + 5 = p 3 p +4 3 p 3 Add 4 to both sides. Simplify both sides. Multiply both sides by 3. Simplify both sides.

3(5) = 3 15 = p

Check. Substitute 15 for p in the original equation. 12 + 3 = 8 + 4 + p 3 15 12 + 3 = 8 + 4 + 3 9 = 8 + 4 + (5) 9 = 4 + (5) 9 = 9 Original equation. Substitute 15 for p. On the left, add: 12 + 3 = 9. On the right, divide: 15/(3) = 5. On the right, add: 8 + 4 = 4. On the right, add: 4 + (5) = 9.

Because the last line in the check is a true statement, 15 is a solution of the original equation.

Answer: q = 8

Applications
Lets look at some applications of equations involving integers. First, we remind readers that a solution of a word problem must incorporate each of the following steps. Requirements for Word Problem Solutions. 1. Set up a Variable Dictionary. You must let your readers know what each variable in your problem represents. This can be accomplished in a number of ways: Statements such as Let P represent the perimeter of the rectangle. Labeling unknown values with variables in a table. Labeling unknown quantities in a sketch or diagram.

Version: Fall 2010

164

CHAPTER 2. THE INTEGERS

2. Set up an Equation. Every solution to a word problem must include a carefully crafted equation that accurately describes the constraints in the problem statement. 3. Solve the Equation. You must always solve the equation set up in the previous step. 4. Answer the Question. This step is easily overlooked. For example, the problem might ask for Janes age, but your equations solution gives the age of Janes sister Liz. Make sure you answer the original question asked in the problem. Your solution should be written in a sentence with appropriate units. 5. Look Back. It is important to note that this step does not imply that you should simply check your solution in your equation. After all, its possible that your equation incorrectly models the problems situation, so you could have a valid solution to an incorrect equation. The important question is: Does your answer make sense based on the words in the original problem statement.

You Try It! After withdrawing $125 from his account, Allen nds that he is overdrawn by $15. What was his account balance before his withdrawal? EXAMPLE 10. A students bank account is overdrawn. After making a deposit of $120, he nds that his account is still overdrawn by an amount of $75. What was his balance before he made his deposit? Solution. In our solution, we address each step of the Requirements for Word Problem Solutions. 1. Set up a Variable Dictionary. In this case, the unknown is the original balance in the students account. Let B represent this original balance. 2. Set up an Equation. A positive integer represents a healthy balance, while a negative number represents an account that is overdrawn. After the students deposit, the account is still overdrawn by $75. We will say that this balance is $75. Thus,

Original Balance B Version: Fall 2010

plus +

Student Deposit $120

equals =

Current Balance $75

2.6. SOLVING EQUATIONS INVOLVING INTEGERS

165

3. Solve the Equation. To undo the addition, subtract 120 from both sides of the equation. B + 120 = 75 B + 120 120 = 75 120 B = 195 Original equation. Subtract 120 from both sides. Simplify both sides.

4. Answer the Question. The original balance was overdrawn to the tune of $195. 5. Look Back. If the original balance was overdrawn by $195, then we let $195 represent this balance. The student makes a deposit of $120. Add this to the original balance to get $195 + $120 = $75, the correct current balance.

Answer: $110

You Try It! EXAMPLE 11. Three more than twice a certain number is 11. Find the unknown number. Solution. In our solution, we address each step of the Requirements for Word Problem Solutions. 1. Set up a Variable Dictionary. Let x represent the unknown number. 2. Set up an Equation. Three more than twice a certain number becomes: Twice a Certain Number 2x Five less than twice a certain number is 7. Find the unknown number.

Three 3

more than +

is =

11 11

3. Solve the Equation. On the left, order of operations requires that we rst multiply x by 2, then add 3. To solve this equation for x, we must undo each of these operations in inverse order. Thus, we will (1) subtract 3 from both sides of the equation, then (2) divide both sides of the resulting equation by 2. 3 + 2x = 11 3 + 2x 3 = 11 3 2x = 14 2x 14 = 2 2 x = 7 Original equation. Subtract 3 from both sides. Simplify both sides. Divide both sides by 2. Simplify both sides. Version: Fall 2010

166

CHAPTER 2. THE INTEGERS

4. Answer the Question. The unknown number is 7. Answer: 1 5. Look Back. Does the answer satisfy the problem constraints? Three more than twice 7 is three more than 14, or 11. So the solution is correct.

Version: Fall 2010

2.6. SOLVING EQUATIONS INVOLVING INTEGERS

167

l l l
1. Is 11 a solution of 2x + 3 = 19? 2. Is 8 a solution of 2x + 7 = 9? 3. Is 6 a solution of 3x + 1 = 19? 4. Is 6 a solution of 2x + 7 = 5? 5. Is 12 a solution of 4x + 5 = 8? 6. Is 8 a solution of 3x + 8 = 18?

Exercises

l l l

7. Is 15 a solution of 2x + 6 = 9? 8. Is 3 a solution of 4x + 1 = 20? 9. Is 15 a solution of 3x + 6 = 17? 10. Is 18 a solution of 3x + 9 = 9? 11. Is 6 a solution of 2x + 3 = 15? 12. Is 7 a solution of 3x + 5 = 16?

In Exercises 13-28, solve the given equation for x. 13. x 13 = 11 14. x 6 = 12 15. x 3 = 6 16. x 3 = 19 17. x + 10 = 17 18. x + 3 = 9 19. x 6 = 1 20. x 10 = 12 21. x 15 = 12 22. x 2 = 13 23. x + 11 = 19 24. x + 3 = 17 25. x + 2 = 1 26. x + 2 = 20 27. x + 5 = 5 28. x + 14 = 15

In Exercises 29-44, solve the given equation for x. 29. x = 20 30. 5x = 35 x = 10 31. 7 x 32. = 20 6 x = 12 33. 10 x 34. = 11 2 x 35. = 16 9 x 36. = 7 3 37. 10x = 20 38. 17x = 85 39. 14x = 84 40. 10x = 40 41. 2x = 28 42. 14x = 42 x = 15 43. 10 x 44. = 1 8 Version: Fall 2010

168

CHAPTER 2. THE INTEGERS

In Exercises 45-68, solve the given equation for x. 45. 4x 4 = 16 46. 6x 14 = 4 47. 4x 4 = 76 48. 5x 15 = 45 49. 5x 14 = 79 50. 15x 2 = 43 51. 10x 16 = 24 52. 2x 7 = 11 53. 9x + 5 = 85 54. 8x + 8 = 16 55. 7x + 15 = 55 56. 2x + 2 = 38 57. x + 8 = 13 58. 5x + 20 = 50 59. 12x 15 = 3 60. 19x 17 = 36 61. 4x 12 = 56 62. 7x 16 = 40 63. 19x + 18 = 113 64. 6x + 20 = 64 65. 14x + 12 = 2 66. 9x + 5 = 104 67. 14x + 16 = 44 68. 14x + 10 = 60

69. Two less than eight times an unknown number is 74. Find the unknown number. 70. Six less than three times an unknown number is 21. Find the unknown number. 71. Eight more than two times an unknown number is 0. Find the unknown number. 72. Five more than eight times an unknown number is 35. Find the unknown number. 73. The number 6 is 2 more than an unknown number. Find the unknown number. 74. The number 4 is 7 more than an unknown number. Find the unknown number. 75. Three more than eight times an unknown number is 29. Find the unknown number. Version: Fall 2010

76. Four more than nine times an unknown number is 85. Find the unknown number. 77. Alans scores on his rst three exams are 79, 61, and 54. What must Alan score on his next exam to average 71 for all four exams? 78. Bennys scores on his rst three exams are 54, 68, and 54. What must Benny score on his next exam to average 61 for all four exams? 79. The quotient of two integers is 5. One of the integers is 2. Find the other integer. 80. The quotient of two integers is 3. One of the integers is 7. Find the other integer. 81. The quotient of two integers is 9. One of the integers is 8. Find the other integer. 82. The quotient of two integers is 9. One of the integers is 2. Find the other integer.

2.6. SOLVING EQUATIONS INVOLVING INTEGERS 83. The number 5 is 8 more than an unknown number. Find the unknown number. 84. The number 6 is 8 more than an unknown number. Find the unknown number. 85. A students bank account is overdrawn. After making a deposit of $260, he nds that his account is still overdrawn by an amount of $70. What was his balance before he made his deposit? 86. A students bank account is overdrawn. After making a deposit of $300, he nds that his account is still overdrawn by an amount of $70. What was his balance before he made his deposit? 87. A students bank account is overdrawn. After making a deposit of $360, he nds that his account is still overdrawn by an amount of $90. What was his balance before he made his deposit? 88. A students bank account is overdrawn. After making a deposit of $260, he nds that his account is still overdrawn by an amount of $50. What was his balance before he made his deposit?

169

89. The number 10 is 5 times larger than an unknown number. Find the unknown number. 90. The number 3 is 3 times larger than an unknown number. Find the unknown number. 91. The number 15 is 5 times larger than an unknown number. Find the unknown number. 92. The number 16 is 4 times larger than an unknown number. Find the unknown number. 93. Two less than nine times an unknown number is 7. Find the unknown number. 94. Four less than two times an unknown number is 8. Find the unknown number. 95. Marks scores on his rst three exams are 79, 84, and 71. What must Mark score on his next exam to average 74 for all four exams? 96. Alans scores on his rst three exams are 85, 90, and 61. What must Alan score on his next exam to average 77 for all four exams?

l l l
1. Yes 3. Yes 5. No 7. No 9. No 11. Yes 13. 24

Answers
15. 9 17. 7 19. 7 21. 3

l l l

23. 30 25. 1 27. 10 Version: Fall 2010

170 29. 20 31. 70 33. 120 35. 144 37. 2 39. 6 41. 14 43. 150 45. 5 47. 20 49. 13 51. 4 53. 10 55. 10 57. 5 59. 1 61. 11

CHAPTER 2. THE INTEGERS 63. 5 65. 1 67. 2 69. 9 71. 4 73. 8 75. 4 77. 90 79. 10 81. 72 83. 13 85. $330 87. $450 89. 2 91. 3 93. 1 95. 62

Version: Fall 2010

Chapter

The Fundamentals of Algebra

As his name portends, Abu Jafr Muhammad ibn Musa al-Khwarizmi was one of the greatest Arab mathematician of his time. While living in Baghad during the ninth century AD he became the Chief Librarian at the House of Wisdom, a library and major center of intellectual study. In the year 820AD, al-Khwarizmi wrote Al-Kitab al-mukhtasar ti Hisab al-jabr wal-muqabala, translated to, The Compendious Book on Calculation by Restoration and Reduction, the rst book to generalize solving equations using the principles of equality. In fact, the word algebra itself comes from al-jabr, meaning reunion or completion. Many earlier cultures had employed what we might call algebra in the service of business, land management, inheritance, and trade. The Bablyonians were solving quadratic equations in 2000BC, but only specic equations, with specic numbers. Hindus on the Indian continent were also developing algebra along side their invention of the symbol for zero 0. But like al-Khwarizmi and the Moslem Arabs of the rst millenium, to write equations these early cultures did not use symbols like x or y, or even equal signs = that we use today. al-Khwarizmi wrote absolutely everything in words! 42 would be forty-two! Early algebra written all with words is called rhetorical algebra, and a thousand years ago, each mathematician had their own way of expressing it. Algebra was a language with many dierent dialects, and communicating it from one mathematician to another was dicult as they had to explain themselves with words. It wasnt until well after the Gutenberg printing press was invented in 1436 and print became standardized, that Rene Descartes, a Frenchman began to develop a modern symbolic algebra. In this section well learn how to manipulate symbols in order to al-muqabalah (combine like terms) and al-jabr (restore and balance equations). But well use modern notation to make it easier!

171

172

CHAPTER 3. THE FUNDAMENTALS OF ALGEBRA

3.1

Mathematical Expressions

Recall the denition of a variable presented in Section 1.6. Variable. A variable is a symbol (usually a letter) that stands for a value that may vary. Lets add the denition of a mathematical expression. Mathematical Expression. When we combine numbers and variables in a valid way, using operations such as addition, subtraction, multiplication, division, exponentiation, and other operations and functions as yet unlearned, the resulting combination of mathematical symbols is called a mathematical expression. Thus, 2a, x + 5, and

y2,

being formed by a combination of numbers, variables, and mathematical operators, are valid mathematical expressions. A mathematical expression must be well-formed. For example, 2 + 5x is not a valid expression because there is no term following the plus sign (it is not valid to write + with nothing between these operators). Similarly, 2 + 3(2 is not well-formed because parentheses are not balanced.

Translating Words into Mathematical Expressions


In this section we turn our attention to translating word phrases into mathematical expressions. We begin with phrases that translate into sums. There is a wide variety of word phrases that translate into sums. Some common examples are given in Table 3.1(a), though the list is far from complete. In like manner, a number of phrases that translate into dierences are shown in Table 3.1(b). Lets look at some examples, some of which translate into expressions involving sums, and some of which translate into expressions involving dierences. You Try It! Translate the following phrases into mathematical expressions: (a) 13 more than x, and (b) 12 fewer than y. EXAMPLE 1. Translate the following phrases into mathematical expressions: (a) 12 larger than x, (b) 11 less than y, and (c) r decreased by 9. Version: Fall 2010

3.1. MATHEMATICAL EXPRESSIONS Solution. Here are the translations. a) 12 larger than x becomes x + 12. b) 11 less than y becomes y 11. c) r decreased by 9 becomes r 9.

173

Answers: (a) x + 13 and (b) y 12

Phrase sum of x and 12 4 greater than b 6 more than y 44 plus r 3 larger than z

Translates to: x + 12 b+4 y+6 44 + r z+3

Phrase dierence of x and 12 4 less than b 7 subtracted from y 44 minus r 3 smaller than z

Translates to: x 12 b4 y7 44 r z3

(a) Phrases that are sums.

(b) Phrases that are dierences.

Table 3.1: Translating words into symbols.

You Try It! EXAMPLE 2. Let W represent the width of the rectangle. The length of a rectangle is 4 feet longer than its width. Express the length of the rectangle in terms of its width W . Solution. We know that the width of the rectangle is W . Because the length of the rectangle is 4 feet longer that the width, we must add 4 to the width to nd the length. Length Length is = 4 4 more than + the width W Answer: L 5 The width of a rectangle is 5 inches shorter than its length L. Express the width of the rectangle in terms of its length L.

Thus, the length of the rectangle, in terms of its width W , is 4 + W .

You Try It! EXAMPLE 3. A string measures 15 inches is cut into two pieces. Let x represent the length of one of the resulting pieces. Express the length of the second piece in terms of the length x of the rst piece. Solution. The string has original length 15 inches. It is cut into two pieces and the rst piece has length x. To nd the length of the second piece, we must subtract the length of the rst piece from the total length. Version: Fall 2010 A string is cut into two pieces, the rst of which measures 12 inches. Express the total length of the string as a function of x, where x represents the length of the second piece of string.

174

CHAPTER 3. THE FUNDAMENTALS OF ALGEBRA

Length of the second piece Length of the second piece Answer: 12 + x

is =

Total length 15

minus

the length of rst piece x

Thus, the length of the second piece, in terms of the length x of the rst piece, is 15 x.

There is also a wide variety of phrases that translate into products. Some examples are shown in Table 3.2(a), though again the list is far from complete. In like manner, a number of phrases translate into quotients, as shown in Table 3.2(b). Phrase product of x and 12 4 times b twice r Translates to: 12x 4b 2r Phrase quotient of x and 12 4 divided by b the ratio of 44 to r Translates to: x/12 4/b 44/r

(a) Phrases that are products.

(b) Phrases that are dierences.

Table 3.2: Translating words into symbols. Lets look at some examples, some of which translate into expressions involving products, and some of which translate into expressions involving quotients. You Try It! Translate into mathematical symbols: (a) the product of 5 and x and (b) 12 divided by y. EXAMPLE 4. Translate the following phrases into mathematical expressions: (a) 11 times x, (b) quotient of y and 4, and (c) twice a. Solution. Here are the translations. a) 11 times x becomes 11x. b) quotient of y and 4 becomes y/4, or equivalently, Answer: (a) 5x and (b) 12/y. c) twice a becomes 2a. y . 4

You Try It! A carpenter cuts a board of unknown length L into three equal pieces. Express the length of each piece in terms of L. EXAMPLE 5. A plumber has a pipe of unknown length x. He cuts it into 4 equal pieces. Find the length of each piece in terms of the unknown length x. Version: Fall 2010

3.1. MATHEMATICAL EXPRESSIONS

175

Solution. The total length is unknown and equal to x. The plumber divides it into 4 equal pieces. To nd the length of each pieces, we must divide the total length by 4. Length of each piece Length of each piece is = Total length x divided by 4 4

Thus, the length of each piece, in terms of the unknown length x, is x/4, or x equivalently, . 4

Answer: L/3.

You Try It! EXAMPLE 6. Mary invests A dollars in a savings account paying 2% interest per year. She invests ve times this amount in a certicate of deposit paying 5% per year. How much does she invest in the certicate of deposit, in terms of the amount A in the savings account? Solution. The amount in the savings account is A dollars. She invests ve times this amount in a certicate of deposit. Amount in CD Amount in CD is = 5 5 times Amount in savings A Answer:
1 2K

David invest K dollars in as savings account paying 3% per year. He invests half this amount in a mutual fund paying 4% per year. Express the amount invested in the mutual fund in terms of K, the amount invested in the savings account.

Thus, the amount invested in the certicate of deposit, in terms of the amount A in the savings account, is 5A.

Combinations
Some phrases require combinations of the mathematical operations employed in previous examples. You Try It! EXAMPLE 7. Let the rst number equal x. The second number is 3 more than twice the rst number. Express the second number in terms of the rst number x. Solution. The rst number is x. The second number is 3 more than twice the rst number. Second number Second number is = 3 3 more than + twice the rst number 2x Version: Fall 2010 A second number is 4 less than 3 times a rst number. Express the second number in terms of the rst number y.

176 Answer: 3y 4

CHAPTER 3. THE FUNDAMENTALS OF ALGEBRA

Therefore, the second number, in terms of the rst number x, is 3 + 2x.

You Try It! The width of a rectangle is W . The length is 7 inches longer than twice the width. Express the length of the rectangle in terms of its length L. EXAMPLE 8. The length of a rectangle is L. The width is 15 feet less than 3 times the length. What is the width of the rectangle in terms of the length L? Solution. The length of the rectangle is L. The width is 15 feet less than 3 times the length. 3 times the length 3L

Width Width Answer: 2W + 7

is =

less

15 15

Therefore, the width, in terms of the length L, is 3L 15.

Version: Fall 2010

3.1. MATHEMATICAL EXPRESSIONS

177

l l l

Exercises

l l l

In Exercises 1-20, translate the phrase into a mathematical expression involving the given variable. 1. 8 times the width n 2. 2 times the length z 3. 6 times the sum of the number n and 3 4. 10 times the sum of the number n and 8 5. the demand b quadrupled 6. the supply y quadrupled 7. the speed y decreased by 33 8. the speed u decreased by 30 9. 10 times the width n 10. 10 times the length z 11. 9 times the sum of the number z and 2 12. 14 times the sum of the number n and 10 13. the supply y doubled 14. the demand n quadrupled 15. 13 more than 15 times the number p 16. 14 less than 5 times the number y 17. 4 less than 11 times the number x 18. 13 less than 5 times the number p 19. the speed u decreased by 10 20. the speed w increased by 32

21. Representing Numbers. represents a whole number.

Suppose n

i) What does n + 1 represent? ii) What does n + 2 represent? iii) What does n 1 represent? 22. Suppose 2n represents an even whole number. How could we represent the next even number after 2n? 23. Suppose 2n + 1 represents an odd whole number. How could we represent the next odd number after 2n + 1? 24. There are b bags of mulch produced each month. How many bags of mulch are produced each year?

25. Steve sells twice as many products as Mike. Choose a variable and write an expression for each mans sales. 26. Find a mathematical expression to represent the values. i) How many quarters are in d dollars? ii) How many minutes are in h hours? iii) How many hours are in d days? iv) How many days are in y years? v) How many months are in y years? vi) How many inches are in f feet? vii) How many feet are in y yards?

Version: Fall 2010

178

CHAPTER 3. THE FUNDAMENTALS OF ALGEBRA

l l l
1. 8n 3. 6(n + 3) 5. 4b 7. y 33 9. 10n 11. 9(z + 2) 13. 2y 15. 15p + 13 17. 11x 4

Answers

l l l

19. u 10 21. i) n+1 represents the next whole number after n. ii) n+2 represents the next whole number after n + 1, or, two whole numbers after n. iii) n 1 represents the whole number before n. 23. 2n + 3 25. Let Mike sell p products. Then Steve sells 2p products.

Version: Fall 2010

3.2. EVALUATING ALGEBRAIC EXPRESSIONS

179

3.2

Evaluating Algebraic Expressions

In this section we will evaluate algebraic expressions for given values of the variables contained in the expressions. Here are some simple tips to help you be successful. Tips for Evaluating Algebraic Expressions. 1. Replace all occurrences of variables in the expression with open parentheses. Leave room between the parentheses to substitute the given value of the variable. 2. Substitute the given values of variables in the open parentheses prepared in the rst step. 3. Evaluate the resulting expression according to the Rules Guiding Order of Operations.

Lets begin with an example. You Try It! EXAMPLE 1. Evaluate the expression x2 2xy + y 2 at x = 3 and y = 2. Solution. Following Tips for Evaluating Algebraic Expressions, rst replace all occurrences of variables in the expression x2 2xy + y 2 with open parentheses. x2 2xy + y 2 = ( )2 2( )( ) + ( )2 Secondly, replace each variable with its given value, and thirdly, follow the Rules Guiding Order of Operations to evaluate the resulting expression. x2 2xy + y 2 = (3) 2(3)(2) + (2) = 9 2(3)(2) + 4 = 9 (6)(2) + 4 = 9 (12) + 4 = 9 + 12 + 4 = 25
2 2

If x = 2 and y = 1, evaluate x3 y 3 .

Original expression. Substitute 3 for x and 2 for y. Evaluate exponents rst. Left to right, multiply: 2(3) = 6. Left to right, multiply: (6)(2) = 12. Add the opposite. Add. Answer: 7

Version: Fall 2010

180

CHAPTER 3. THE FUNDAMENTALS OF ALGEBRA

You Try It! If a = 3 and b = 5, evaluate a2 b2 . EXAMPLE 2. Evaluate the expression (a b)2 at a = 3 and b = 5. Solution. Following Tips for Evaluating Algebraic Expressions, rst replace all occurrences of variables in the expression (a b)2 with open parentheses. (a b)2 = (( ) ( ))2 Secondly, replace each variable with its given value, and thirdly, follow the Rules Guiding Order of Operations to evaluate the resulting expression. (a b)2 = ((3) (5))2 = (3 + 5) =8 Answer: 16
2 2

Substitute 3 for a and 5 for b. Add the opposite: (3) (5) = 3 + 5 Simplify inside parentheses: 3 + 5 = 8 Evaluate exponent: 82 = 64

= 64

You Try It! If a = 5 and b = 7, evaluate 2|a| 3|b|. EXAMPLE 3. Evaluate the expression |a| |b| at a = 5 and b = 7. Solution. Following Tips for Evaluating Algebraic Expressions, rst replace all occurrences of variables in the expression |a| |b| with open parentheses. |a| |b| = |( )| |( )| Secondly, replace each variable with its given value, and thirdly, follow the Rules Guiding Order of Operations to evaluate the resulting expression. |a| |b| = |(5)| |(7)| =57 = 5 + (7) = 2 Answer: 11 Substitute 5 for a and 7 for b. Absolute values rst: |(5)| = 5 and |(7)| = 7 Add the opposite: 5 7 = 5 + (7). Add: 5 + (7) = 2.

You Try It! If a = 5 and b = 7, evaluate |2a 3b|. EXAMPLE 4. Evaluate the expression |a b| at a = 5 and b = 7. Solution. Following Tips for Evaluating Algebraic Expressions, rst replace all occurrences of variables in the expression |a b| with open parentheses. |a b| = |( ) ( )| Version: Fall 2010

3.2. EVALUATING ALGEBRAIC EXPRESSIONS

181

Secondly, replace each variable with its given value, and thirdly, follow the Rules Guiding Order of Operations to evaluate the resulting expression. |a b| = |(5) (7)| = |5 + 7| = |12| = 12 Substitute 5 for a and 7 for b. Add the opposite: 5 (7) = 5 + 7. Add: 5 + 7 = 12. Take the absolute value: |12| = 12. Answer: 31

You Try It! EXAMPLE 5. Evaluate the expression ad bc a+b at a = 5, b = 3, c = 2, and d = 4. Solution. Following Tips for Evaluating Algebraic Expressions, rst replace all occurrences of variables in the expression with open parentheses. ( )( ) ( )( ) ad bc = a+b ( )+( ) Secondly, replace each variable with its given value, and thirdly, follow the Rules Guiding Order of Operations to evaluate the resulting expression. (5)(4) (3)(2) ad bc = a+b (5) + (3) 20 (6) = 2 = 20 + 6 2 14 = 2 = 7 Substitute: 5 for a, 3 for b, 2 for c, 4 for d. Numerator: (5)(4) = 20, (3)(2) = 6. Denominator: 5 + (3) = 2. Numerator: Add the opposite. Numerator: 20 + 6 = 14. Divide. If a = 7, b = 3, c = 15, and d = 14, evaluate: a2 + b 2 c+d

Answer

Version: Fall 2010

182

CHAPTER 3. THE FUNDAMENTALS OF ALGEBRA

You Try It! The surface area of the prism pictured in this example is given by the following formula: S = 2(W H + LH + LW ) If L = 12, W = 4, and H = 6 feet, respectively, calculate the surface area. H EXAMPLE 6. Pictured below is a rectangular prism.

W L The volume of the rectangular prism is given by the formula V = LW H, where L is the length, W is the width, and H is the height of the rectangular prism. Find the volume of a rectangular prism having length 12 feet, width 4 feet, and height 6 feet. Solution. Following Tips for Evaluating Algebraic Expressions, rst replace all occurrences of of L, W , and H in the formula V = LW H with open parentheses. V = Next, substitute 12 ft for L, 4 ft for W , and 6 ft for H and simplify. V = 12 ft 4 ft 6 ft = 288 ft3 Answer: 288 square feet. Hence, the volume of the rectangular prism is 288 cubic feet.

Version: Fall 2010

3.2. EVALUATING ALGEBRAIC EXPRESSIONS

183

l l l

Exercises

l l l

In Exercises 1-12, evaluate the expression at the given value of x. 1. 3x2 6x + 3 at x = 7 2. 7x 7x + 1 at x = 8 3. 6x 6 at x = 3 4. 6x 1 at x = 10 5. 5x2 + 2x + 4 at x = 1 6. 4x 9x + 4 at x = 3
2 2

7. 9x 5 at x = 2 8. 9x + 12 at x = 5 9. 4x2 + 2x + 6 at x = 6 10. 3x2 + 7x + 4 at x = 7 11. 12x + 10 at x = 12 12. 6x + 7 at x = 11

In Exercises 13-28, evaluate the expression at the given values of x and y. 13. |x| |y| at x = 5 and y = 4 14. |x| |y| at x = 1 and y = 2 15. 5x2 + 2y 2 at x = 4 and y = 2 16. 5x 4y at x = 2 and y = 5 17. |x| |y| at x = 0 and y = 2 18. |x| |y| at x = 2 and y = 0 19. |x y| at x = 4 and y = 5 20. |x y| at x = 1 and y = 4
2 2

21. 5x2 4xy + 3y 2 at x = 1 and y = 4 22. 3x2 + 5xy + 3y 2 at x = 2 and y = 1 23. |x y| at x = 4 and y = 4 24. |x y| at x = 3 and y = 5 25. 5x2 3xy + 5y 2 at x = 1 and y = 2 26. 3x2 2xy 5y 2 at x = 2 and y = 5 27. 5x2 + 4y 2 at x = 2 and y = 2 28. 4x2 + 2y 2 at x = 4 and y = 5

In Exercises 29-40, evaluate the expression at the given value of x. 29. 30. 31. 32. 33. 9 + 9x at x = 3 x 9 2x at x = 1 x 8x + 9 at x = 10 9 + x 2x + 4 at x = 0 1+x 4 + 9x at x = 2 7x 1 9x at x = 1 x 12 7x at x = 1 35. x 12 + 11x at x = 6 36. 3x 6x 10 at x = 6 37. 5+x 34. 38. 11x + 11 at x = 5 4 + x Version: Fall 2010

184 10x + 11 at x = 4 5+x

CHAPTER 3. THE FUNDAMENTALS OF ALGEBRA 40. 6x + 12 at x = 2 3 + x

39.

41. The formula

d = 16t2

gives the distance (in feet) that an object falls from rest in terms of the time t that has elapsed since its release. Find the distance d (in feet) that an object falls in t = 4 seconds. 42. The formula

d = 16t2

gives the distance (in feet) that an object falls from rest in terms of the time t that has elapsed since its release. Find the distance d (in feet) that an object falls in t = 24 seconds. 43. The formula

5(F 32) 9 gives the Celcius temperature C in terms of the Fahrenheit temperature F . Use the formula to nd the Celsius temperature ( C) if the Fahrenheit temperature is F = 230 F. C=

44. The formula

5(F 32) 9 gives the Celcius temperature C in terms of the Fahrenheit temperature F . Use the formula to nd the Celsius temperature ( C) if the Fahrenheit temperature is F = 95 F. C=

45. The Kelvin scale of temperature is used in chemistry and physics. Absolute zero occurs at 0 K, the temperature at which molecules have zero kinetic energy. Water freezes at 273 K and boils at K = 373 K. To change Kelvin temperature to Fahrenheit temperature, we use the formula F = 9(K 273) + 32. 5

Use the formula to change 28 K to Fahrenheit. 46. The Kelvin scale of temperature is used in chemistry and physics. Absolute zero occurs at 0 K, the temperature at which molecules have zero kinetic energy. Water freezes at 273 K and boils at K = 373 K. To change Kelvin temperature to Fahrenheit temperature, we use the formula F = 9(K 273) + 32. 5

Use the formula to change 248 K to Fahrenheit.

Version: Fall 2010

3.2. EVALUATING ALGEBRAIC EXPRESSIONS

185

47. A ball is thrown vertically upward. Its velocity t seconds after its release is given by the formula v = v0 gt, where v0 is its initial velocity, g is the acceleration due to gravity, and v is the velocity of the ball at time t. The acceleration due to gravity is g = 32 feet per second per second. If the initial velocity of the ball is v0 = 272 feet per second, nd the speed of the ball after t = 6 seconds. 48. A ball is thrown vertically upward. Its velocity t seconds after its release is given by the formula v = v0 gt, where v0 is its initial velocity, g is the acceleration due to gravity, and v is the velocity of the ball at time t. The acceleration due to gravity is g = 32 feet per second per second. If the initial velocity of the ball is v0 = 470 feet per second, nd the speed of the ball after t = 4 seconds.

49. Even numbers. Evaluate the expression 2n for the following values: i) n = 1 ii) n = 2 iii) n = 3 iv) n = 4 v) n = 5 vi) Is the result always an even number? Explain.

50. Odd numbers. Evaluate the expression 2n + 1 for the following values: i) n = 1 ii) n = 2 iii) n = 3 iv) n = 4 v) n = 5 vi) Is the result always an odd number? Explain.

l l l
1. 186 3. 24 5. 7 7. 13

Answers
13. 1

l l l

15. 72 17. 2 19. 1 21. 69

9. 138 11. 134

23. 0 25. 9 Version: Fall 2010

186 27. 36 29. 6 31. 71 33. 1 35. 5 37. 46 39. 29 41. 256 feet 43. 110 degrees

CHAPTER 3. THE FUNDAMENTALS OF ALGEBRA 45. 409 F 47. 80 feet per second 49. i) 2 ii) 4 iii) 6 iv) 8 v) 10 vi) Yes, the result will always be an even number because 2 will always be a factor of the product 2n.

Version: Fall 2010

3.3. SIMPLIFYING ALGEBRAIC EXPRESSIONS

187

3.3

Simplifying Algebraic Expressions

Recall the commutative and associative properties of multiplication. The Commutative Property of Multiplication. If a and b are any integers, then a b = b a, or equivalently, ab = ba. The Associative Property of Multiplication. If a, b, and c are any integers, then (a b) c = a (b c), or equivalently, (ab)c = a(bc).

The commutative property allows us to change the order of multiplication without aecting the product or answer. The associative property allows us to regroup without aecting the product or answer. You Try It! EXAMPLE 1. Simplify: 2(3x). Solution. Use the associative property to regroup, then simplify. 2(3x) = (2 3)x = 6x Regrouping with the associative property. Simplify: 2 3 = 6. Answer: 35y Simplify: 5(7y)

The statement 2(3x) = 6x is an identity. That is, the left-hand side and right-hand side of 2(3x) = 6x are the same for all values of x. Although the derivation in Example 1 should be the proof of this statement, it helps the intuition to check the validity of the statement for one or two values of x. If x = 4, then 2(3x) = 2(3(4)) = 2(12) = 24 If x = 5, then 2(3x) = 2(3(5)) = 2(15) = 30 The above calculations show that 2(3x) = 6x for both x = 4 and x = 5. Indeed, the statement 2(3x) = 6x is true, regardless of what is substituted for x. Version: Fall 2010 and 6x = 6(5) = 30 and 6x = 6(4) = 24

188

CHAPTER 3. THE FUNDAMENTALS OF ALGEBRA

You Try It! Simplify: (8a)(5) EXAMPLE 2. Simplify: (3t)(5). Solution. In essence, we are multiplying three numbers, 3, t, and 5, but the grouping symbols ask us to multiply the 3 and the t rst. The associative and commutative properties allow us to change the order and regroup. (3t)(5) = ((3)(5))t = 15t Answer: 40a Change the order and regroup. Multiply: (3)(5) = 15.

You Try It! Simplify: (4a)(5b) EXAMPLE 3. Simplify: (3x)(2y). Solution. In essence, we are multiplying four numbers, 3, x, 2, and y, but the grouping symbols specify a particular order. The associative and commutative properties allow us to change the order and regroup. (3x)(2y) = ((3)(2))(xy) = 6xy Answer: 20ab Change the order and regroup. Multiply: (3)(2) = 6.

Speeding Things Up
The meaning of the expression 2 3 4 is clear. Parentheses and order of operations are really not needed, as the commutative and associative properties explain that it doesnt matter which of the three numbers you multiply together rst. You can multiply 2 and 3 rst: 2 3 4 = (2 3) 4 = 64 = 24. Or you can multiply 3 and 4 rst: 2 3 4 = 2 (3 4) = 2 12 = 24. Version: Fall 2010

3.3. SIMPLIFYING ALGEBRAIC EXPRESSIONS Or you can multiply 2 and 4 rst: 2 3 4 = (2 4) 3 = 83 = 24.

189

So, it doesnt matter which two factors you multiply rst. Of course, this would not be the case if there were a mixture of multiplication and other operators (division, addition, subtraction). Then we would have to strictly follow the Rules Guiding Order of Operations. But if the only operator is multiplication, the order of multiplication is irrelevant. Thus, when we see 2(3x), as in Example 1, we should think Its all multiplication and it doesnt matter which two numbers I multiply together rst, so Ill multiply the 2 and the 3 and get 2(3x) = 6x. Our comments apply equally well to a product of four or more factors. It simply doesnt matter how you group the multiplication. So, in the case of (3x)(2y), as in Example 3, nd the product of 2 and 3 and multiply the result by the product of x and y. That is, (3x)(2y) = 6xy. You Try It! EXAMPLE 4. Simplify: (2a)(3b)(4c). Solution. The only operator is multiplication, so we can order and group as we please. So, well take the product of 2, 3, and 4, and multiply the result by the product of a, b, and c. That is, (2a)(3b)(4c) = 24abc. Answer: 24xyz Simplify: (3x)(2y)(4z)

The Distributive Property


Multiplication is distributive with respect to addition. The Distributive Property. If a, b, and c are any integers, then a (b + c) = a b + a c, or equivalently, a(b + c) = ab + ac.

For example, if we follow the Rules Guiding Order of Operations and rst evaluate the expression inside the parentheses, then 3(4 + 5) = 3(9) = 27. Parens rst: 4 + 5 = 9. Multiply: 3(9) = 27. Version: Fall 2010

190

CHAPTER 3. THE FUNDAMENTALS OF ALGEBRA

But if we distribute the 3, we get the same answer. 3(4 + 5) = 3(4 + 5) = 3(4) + 3(5) = 12 + 15 = 27 Multiply rst: 3(4) = 12, 3(5) = 15. Add. Each number in parentheses is multiplied by the number 3 outside the parentheses.

You Try It! Use the distributive property to simplify: 2(5z + 7) EXAMPLE 5. Use the distributive property to simplify: 3(4x + 5). Solution. Distribute the 3. 3(4x + 5) = 3(4x) + 3(5) = 12x + 15 Answer: 10z + 14 Each number in parentheses is multiplied by the number 3 outside the parentheses. Multiply rst: 3(4x) = 12x, 3(5) = 15.

Multiplication is also distributive with respect to subtraction. The Distributive Property. If a, b, and c are any integers, then a (b c) = a b a c, or equivalently, a(b c) = ab ac.

The application of this form of the distributive property is identical to the rst, the only dierence being the subtraction symbol. You Try It! Use the distributive property to simplify: 7(4a 5) EXAMPLE 6. Use the distributive property to simplify: 5(3x 2). Solution. Distribute the 5. 5(3x 2) = 5(3x) 5(2) = 15x 10 Answer: 28a 35 Each number in parentheses is multiplied by the number 5 outside the parentheses. Multiply rst: 5(3x) = 15x, 5(2) = 10.

Version: Fall 2010

3.3. SIMPLIFYING ALGEBRAIC EXPRESSIONS

191

You Try It! EXAMPLE 7. Remove parentheses: (a) 9(2t + 7), and (b) 5(4 3y). Solution. a) Use the distributive property. 9(2t + 7) = 9(2t) + (9)(7) Distribute multiplication by 9. = 18t + (63) Multiply: 9(2t) = 18t and 9(7) = 63. = 18t 63 Write the answer in simpler form. Adding 63 is the same as subtracting 63. b) Use the distributive property. 5(4 3y) = 5(4) (5)(3y) = 20 (15y) = 20 + 15y Distribute multiplication by 5. Multiply: 5(4) = 20 and (5)(3y) = 15y. Write the answer in simpler form. Subtracting 15y is the same as adding 15y. Answer: 12t + 33 Remove parentheses: 3(4t 11)

Writing Mathematics. Example 7 stresses the importance of using as few symbols as possible to write your nal answer. Hence, 18t 63 is favored over 18t + (63) and 20 + 15y is favored over 20 (15y). You should always make these nal simplications.

Moving a Bit Quicker


Once youve applied the distributive property to a number of problems, showing all the work as in Example 7, you should try to eliminate some of the steps. For example, consider again Example 7(a). Its not dicult to apply the distributive property without writing down a single step, getting: 9(2t + 7) = 18t 63. Heres the thinking behind this technique: 1. First, multiply 9 times 2t, getting 18t. Version: Fall 2010

192

CHAPTER 3. THE FUNDAMENTALS OF ALGEBRA

2. Second, multiply 9 times +7, getting 63. Note that this provides exactly the same solution found in Example 7(a). Let try this same technique on Example 7(b). 5(4 3y) = 20 + 15y Heres the thinking behind this technique. 1. First, multiply 5 times 4, getting 20. 2. Second, multiply 5 times 3y, getting +15y. Note that this provides exactly the same solution found in Example 7(b).

Extending the Distributive Property


Suppose that we add an extra term inside the parentheses. Distributive Property. If a, b, c, and d are any integers, then a(b + c + d) = ab + ac + ad.

Note that we distributed the a times each term inside the parentheses. Indeed, if we added still another term inside the parentheses, we would distribute a times that term as well. You Try It! Remove parentheses: 3(4a 5b + 7) EXAMPLE 8. Remove parentheses: 5(2x 3y + 8). Solution. We will use the quicker technique, distributing 5 times each term in the parentheses mentally. 5(2x 3y + 8) = 10x + 15y 40 Here is our thought process: 1. First, multiply 5 times 2x, getting 10x. 2. Second, multiply 5 times 3y, getting +15y. 3. Third, multiply 5 times +8, getting 40. Answer: 12a + 15b 21

Version: Fall 2010

3.3. SIMPLIFYING ALGEBRAIC EXPRESSIONS

193

You Try It! EXAMPLE 9. Remove parentheses: 4(3a + 4b 5c + 12). Solution. We will use the quicker technique, distributing 4 times each term in the parentheses mentally. 4(3a + 4b 5c + 12) = 12a 16b + 20c 48 Here is our thought process: 1. First, multiply 4 times 3a, getting 12a. 2. Second, multiply 4 times +4b, getting 16b. 3. Third, multiply 4 times 5c, getting +20c. 4. Fourth, multiply 4 times +12, getting 48. Answer: 4x 8y + 10z + 22 Remove parentheses: 2(2x + 4y 5z 11)

Distributing a Negative
It is helpful to recall that negating is equivalent to multiplying by 1. Multiplying by 1. Let a be any integer, then (1)a = a and a = (1)a.

We can use this fact, combined with the distributive property, to negate a sum. You Try It! EXAMPLE 10. Remove parentheses: (a + b). Solution. Change the negative symbol into multiplying by 1, then distribute the 1. (a + b) = (1)(a + b) = a b Negating is equivalent to multiplying by 1. Distribute the 1. Remove parentheses: (x + 2y)

We chose to use the quicker technique of distributing the 1. Here is our thinking: Version: Fall 2010

194

CHAPTER 3. THE FUNDAMENTALS OF ALGEBRA

1. Multiply 1 times a, getting a. 2. Multiply 1 times +b, getting b. Answer: x 2y

You Try It! Remove parentheses: (4a 3c) EXAMPLE 11. Remove parentheses: (a b). Solution. Change the negative symbol into multiplying by 1, then distribute the 1. (a b) = (1)(a b) = a + b Negating is equivalent to multiplying by 1. Distribute the 1.

We chose to use the quicker technique of distributing the 1. Here is our thinking: 1. Multiply 1 times a, getting a. 2. Multiply 1 times b, getting +b. Answer: 4a + 3c

The results in Example 10 and Example 11 show us how to negate a sum: Simply negate each term of the sum. Positive terms change to negative, negative terms turn to positive. Negating a Sum. To negate a sum, simply negate each term of the sum. For example, if a and b are integers, then (a + b) = a b and (a b) = a + b.

You Try It! Remove parentheses: (5 2x + 4y 5z) EXAMPLE 12. Remove parentheses: (5 7u + 3t). Solution. Simply negate each term in the parentheses. (5 7u + 3t) = 5 + 7u 3t. Answer: 5 + 2x 4y + 5z

Version: Fall 2010

3.3. SIMPLIFYING ALGEBRAIC EXPRESSIONS

195

l l l

Exercises

l l l

In Exercises 1-20, use the associative and commutative properties of multiplication to simplify the expression. 1. 10(9x) 2. 10(10x) 3. 8(4x) 4. 2(9x) 5. 9(5x) 6. 4(8x) 7. (6x)4 8. (2x)(7) 9. (8x)4 10. (5x)3 11. 2(4x) 12. 8(8x) 13. (6x)(2) 14. (3x)(6) 15. (9x)(6) 16. (5x)(9) 17. (6x)9 18. (2x)(4) 19. 4(7x) 20. 7(6x)

In Exercises 21-44, simplify the expression. 21. 8(7x + 8) 22. 2(5x + 5) 23. 9(2 + 10x) 24. 9(4 + 9x) 25. (2x + 10y 6) 26. (6y + 9x 7) 27. 2(10 + x) 28. 2(10 6x) 29. 3(3 + 4x) 30. 3(4 + 6x) 31. (5 7x + 2y) 32. (4x 8 7y) 33. 4(6x + 7) 34. 6(4x + 9) 35. 4(8x 9) 36. 10(10x + 1) 37. (4 2x 10y) 38. (4x + 6 8y) 39. (5x + 1 + 9y) 40. (10 5x 4y) 41. (6x + 2 10y) 42. (6x + 4 10y) 43. (3y 4 + 4x) 44. (7 10x + 7y)

Version: Fall 2010

196

CHAPTER 3. THE FUNDAMENTALS OF ALGEBRA

l l l
1. 90x 3. 32x 5. 45x 7. 24x 9. 32x 11. 8x 13. 12x 15. 54x 17. 54x 19. 28x 21. 56x + 64

Answers

l l l

23. 18 + 90x 25. 2x 10y + 6 27. 20 + 2x 29. 9 + 12x 31. 5 + 7x 2y 33. 24x + 28 35. 32x 36 37. 4 + 2x + 10y 39. 5x 1 9y 41. 6x 2 + 10y 43. 3y + 4 4x

Version: Fall 2010

3.4. COMBINING LIKE TERMS

197

3.4

Combining Like Terms

We begin our discussion with the denition of a term. Term. A term is a single number or variable, or it can be the product of a number (called its coecient ) and one or more variables (called its variable part ). The terms in an algebraic expression are separated by addition symbols.

You Try It! EXAMPLE 1. Identify the terms in the algebraic expression 3x2 + 5xy + 9y 2 + 12. For each term, identify its coecient and variable part. Solution. In tabular form, we list each term of the expression 3x2 + 5xy + 9y 2 + 12, its coecient, and its variable part. Term 3x2 5xy 9y
2

How many terms are in the algebraic expression 3x2 + 2xy 3y 2 ?

Coecient 3 5 9 12

Variable Part x2 xy y2 None Answer: 3

12

You Try It! EXAMPLE 2. Identify the terms in the algebraic expression a3 3a2 b + 3ab2 b3 . For each term, identify its coecient and variable part. Solution. The rst step is to write each dierence as a sum, because the terms of an expression are dened above to be those items separated by addition symbols. a3 + (3a2 b) + 3ab2 + (b3 ) In tabular form, we list each term of the expression a3 +(3a2 b)+3ab2 +(b3 ), its coecient, and its variable part. Version: Fall 2010 How many terms are in the algebraic expression 11 a2 2ab + 3b2 ?

198

CHAPTER 3. THE FUNDAMENTALS OF ALGEBRA Term a3 3a2 b 3ab2 b3 Coecient 1 3 3 1 Variable Part a3 a2 b ab2 b3

Answer: 4

Like Terms
We dene what is meant by like terms and unlike terms. Like and Unlike Terms. The variable parts of two terms determine whether the terms are like terms or unlike terms. Like Terms. Two terms are called like terms if they have identical variable parts, which means that the terms must contain the same variables raised to the same exponential powers. Unlike Terms. Two terms are called unlike terms if their variable parts are dierent.

You Try It! Are 3xy and 11xy like or unlike terms? EXAMPLE 3. Classify each of the following pairs as either like terms or unlike terms: (a) 3x and 7x, (b) 2y and 3y 2 , (c) 3t and 5u, and (d) 4a3 and 3a3 . Solution. Like terms must have identical variable parts. a) 3x and 7x have identical variable parts. They are like terms. b) 2y and 3y 2 do not have identical variable parts (the exponents dier). They are unlike terms. c) 3t and 5u do not have identical variable parts (dierent variables). They are unlike terms. d) 4a3 and 3a3 have identical variable parts. They are like terms. Answer: Like terms

Version: Fall 2010

3.4. COMBINING LIKE TERMS

199

Combining Like Terms


When using the distributive property, it makes no dierence whether the multiplication is on the left or the right, one still distributes the multiplication times each term in the parentheses. Distributive Property. If a, b, and c are integers, then a(b + c) = ab + ac and (b + c)a = ba + ca.

In either case, you distribute a times each term of the sum. Like terms can be combined and simplied. The tool used for combining like terms is the distributive property. For example, consider the expression 3y + 7y, composed of two like terms with a common variable part. We can use the distributive property and write 3y + 7y = (3 + 7)y. Note that we are using the distributive property in reverse, factoring out the common variable part of each term. Checking our work, note that if we redistribute the variable part y times each term in the parentheses, we are returned to the original expression 3y + 7y. You Try It! EXAMPLE 4. Use the distributive property to combine like terms (if possible) in each of the following expressions: (a) 5x2 9x2 , (b) 5ab + 7ab, (c) 4y 3 7y 2 , and (d) 3xy 2 7xy 2 . Solution. If the terms are like terms, you can use the distributive property to factor out the common variable part. a) Factor out the common variable part x2 . 5x2 9x2 = (5 9)x2 = 14x
2

Simplify:

8z 11z

Use the distributive property. Simplify: 5 9 = 5 + (9) = 14.

b) Factor out the common variable part ab. 5ab + 7ab = (5 + 7)ab = 2ab Use the distributive property. Simplify: 5 + 7 = 2.

c) The terms in the expression 4y 3 7y 2 have dierent variable parts (the exponents are dierent). These are unlike terms and cannot be combined.

Version: Fall 2010

200

CHAPTER 3. THE FUNDAMENTALS OF ALGEBRA

d) Factor out the common variable part xy 2 . 3xy 2 7xy 2 = (3 7)xy 2 = 4xy Answer: 19z
2

Use the distributive property. Simplify: 3 7 = 3 + (7) = 4.

Speeding Things Up a Bit


Once youve written out all the steps for combining like terms, like those shown in Example 4, you can speed things up a bit by following this rule: Combining Like Terms. To combine like terms, simply add their coecients and keep the common variable part. Thus for example, when presented with the sum of two like terms, such as in 5x + 8x, simply add the coecients and repeat the common variable part; that is, 5x + 8x = 13x. You Try It! Combine: 3x2 4x2 EXAMPLE 5. Combine like terms: (a) 9y 8y, (b) 3y 5 + 4y 5 , and (c) 3u2 + 2u2 . Solution. a) Add the coecients and repeat the common variable part. Therefore, 9y 8y = 17y. b) Add the coecients and repeat the common variable part. Therefore, 3y 5 + 4y 5 = 1y 5 . However, note that 1y 5 = y 5 . Following the rule that the nal answer should use as few symbols as possible, a better answer is 3y 5 + 4y 5 = y 5 . c) Add the coecients and repeat the common variable part. Therefore, 3u2 + 2u2 = (1)u2 . However, note that (1)u2 = u2 . Following the rule that the nal answer should use as few symbols as possible, a better answer is 3u2 + 2u2 = u2 .

Answer: 7x2

Version: Fall 2010

3.4. COMBINING LIKE TERMS

201

Simplify
A frequently occurring instruction asks the reader to simplify an expression. Simplify. The instruction simplify is a generic term that means try to write the expression in its most compact form, using the fewest symbols possible. One way you can accomplish this goal is by combining like terms when they are present. You Try It! EXAMPLE 6. Simplify: 2x + 3y 5x + 8y. Simplify: 3a + 4b 7a 9b

Solution. Use the commutative property to reorder terms and the associative and distributive properties to regroup and combine like terms. 2x + 3y 5x + 8y = (2x 5x) + (3y + 8y) Reorder and regroup. = 3x + 11y Combine like terms: 2x 5x = 3x and 3y + 8y = 11y.

Alternate solution. Of course, you do not need to show the regrouping step. If you are more comfortable combining like terms in your head, you are free to present your work as follows: 2x + 3y 5x + 8y = 3x + 11y. Answer: 10a 5b

You Try It! EXAMPLE 7. Simplify: 2x 3 (3x + 4). (3x + 4) = 3x 4. Simplify: 9a 4 (4a 8)

Solution. First, distribute the negative sign. 2x 3 (3x + 4) = 2x 3 3x 4

Next, use the commutative property to reorder, then the associative property to regroup. Then combine like terms. = (2x 3x) + (3 4) = 5x + (7) = 5x 7 Reorder and regroup. Combine like terms: 2x 3x = 5x. Simplify: 5x + (7) = 5x 7. Version: Fall 2010

202

CHAPTER 3. THE FUNDAMENTALS OF ALGEBRA

Alternate solution. You may skip the second step if you wish, simply combining like terms mentally. That is, it is entirely possible to order your work as follows: 2x 3 (3x + 4) = 2x 3 3x 4 = 5x 7 Answer: 13a + 4 Distribute negative sign. Combine like terms.

You Try It! Simplify: 2(3a 4) 2(5 a) EXAMPLE 8. Simplify: 2(5 3x) 4(x + 3).

Solution. Use the distributive property to expand, then use the commutative and associative properties to group the like terms and combine them. 2(5 3x) 4(x + 3) = 10 6x 4x 12 Use the distributive property. = (6x 4x) + (10 12) Group like terms. = 10x 2 Combine like terms: 6x 4x = 10x and 10 12 = 2. Alternate solution. You may skip the second step if you wish, simply combining like terms mentally. That is, it is entirely possible to order your work as follows: 2(5 3x) 4(x + 3) = 10 6x 4x 12 = 10x 2 Distribute. Combine like terms.

Answer: 4a 2

You Try It! Simplify: (a2 2ab) 2(3ab + a2 ) EXAMPLE 9. Simplify: 8(3x2 y 9xy) 8(7x2 y 8xy).

Solution. We will proceed a bit quicker with this solution, using the distributive property to expand, then combining like terms mentally. 8(3x2 y 9xy) 8(7x2 y 8xy) = 24x2 y + 72xy + 56x2 y + 64xy = 32x2 y + 136xy

Answer: a2 8ab

Version: Fall 2010

3.4. COMBINING LIKE TERMS

203

Applications
We can simplify a number of useful formulas by combining like terms. You Try It! EXAMPLE 10. Find the perimeter P of the (a) rectangle and (b) square pictured below. Simplify your answer as much as possible. L s A regular hexagon has six equal sides, each with length x. Find its perimeter in terms of x.

s L Solution. The perimeter of any polygonal gure is the sum of the lengths of its sides. a) To nd the perimeter P of the rectangle, sum its four sides. P = L + W + L + W. Combine like terms. P = 2L + 2W. b) To nd the perimeter P of the square, sum its four sides. P = s + s + s + s. Combine like terms. P = 4s. Answer: P = 6x

Sometimes it is useful to replace a variable with an expression containing another variable. You Try It! EXAMPLE 11. The length of a rectangle is three feet longer than twice its width. Find the perimeter P of the rectangle in terms of its width alone. Solution. From the previous problem, the perimeter of the rectangle is given by P = 2L + 2W, (3.1) where L and W are the length and width of the rectangle, respectively. This equation gives the perimeter in terms of its length and width, but were asked to get the perimeter in terms of the width alone. However, were also given the fact that the length is three feet longer than twice the width. Version: Fall 2010 The length L of a rectangle is 5 meters longer than twice its width W . Find the perimeter P of the rectangle in terms of its width W .

204

CHAPTER 3. THE FUNDAMENTALS OF ALGEBRA

Length L

is =

Three Feet 3

longer than +

Twice the Width 2W

Because L = 3+2W , we can replace L with 3+2W in the perimeter equation 3.1. P = 2L + 2W P = 2(3 + 2W ) + 2W Use the distributive property, then combine like terms. P = 6 + 4W + 2W P = 6 + 6W. Answer: P = 6W + 10 This last equation gives the perimeter P in terms of the width W alone.

You Try It! The width W of a rectangle is 5 feet less than twice its width L. Find the perimeter P of the rectangle in terms of its length L. EXAMPLE 12. The width of a rectangle is two feet less than its length. Find the perimeter P of the rectangle in terms of its length alone. Solution. Again, the perimeter of a rectangle is given by the equation P = 2L + 2W, (3.2)

where L and W are the length and width of the rectangle, respectively. This equation gives the perimeter in terms of its length and width, but were asked to get the perimeter in terms of the length alone. However, were also given the fact that the width is two feet less than the length. Width W is = Length L minus Two feet 2

Because W = L2, we can replace W with L2 in the perimeter equation 3.2. P = 2L + 2W P = 2L + 2(L 2) Use the distributive property, then combine like terms. P = 2L + 2L 4 P = 4L 4. Answer: P = 6L 10 This last equation gives the perimeter P in terms of the length L alone.

Version: Fall 2010

3.4. COMBINING LIKE TERMS

205

l l l

Exercises

l l l

In Exercises 1-16, combine like terms by rst using the distributive property to factor out the common variable part, and then simplifying. 1. 17xy 2 + 18xy 2 + 20xy 2 2. 13xy 3xy + xy 3. 8xy 2 3xy 2 10xy 2 4. 12xy 2xy + 10xy 5. 4xy 20xy 6. 7y + 15y 7. 12r 12r 8. 16s 5s
3 3

9. 11x 13x + 8x 10. 9r 10r + 3r 11. 5q + 7q 12. 17n + 15n 13. r 13r 7r 14. 19m + m + 15m 15. 3x3 18x3 16. 13x2 y + 2x2 y

In Exercises 17-32, combine like terms by rst rearranging the terms, then using the distributive property to factor out the common variable part, and then simplifying. 17. 8 + 17n + 10 + 8n 18. 11 + 16s 14 6s 19. 2x3 19x2 y 15x2 y + 11x3 20. 9x2 y 10y 3 10y 3 + 17x2 y 21. 14xy 2x3 2x3 4xy 22. 4x + 12xy + 4xy 12x 23. 13 + 16m + m + 16 24. 9 11x 8x + 15
3 3

25. 14x2 y 2xy 2 + 8x2 y + 18xy 2 26. 19y 2 + 18y 3 5y 2 17y 3 27. 14x3 + 16xy + 5x3 + 8xy 28. 16xy + 16y 2 + 7xy + 17y 2 29. 9n + 10 + 7 + 15n 30. 12r + 5 + 17 + 17r 31. 3y + 1 + 6y + 3 32. 19p + 6 + 8p + 13

In Exercises 33-56, simplify the expression by rst using the distributive property to expand the expression, and then rearranging and combining like terms mentally. 33. 4(9x2 y + 8) + 6(10x2 y 6) 34. 4(4xy + 5y ) + 6(5xy 9y ) 35. 3(4x2 + 10y 2 ) + 10(4y 2 x2 ) 36. 7(7x3 + 6x2 ) 7(10x2 7x3 )
3 3

37. s + 7 (1 3s) 38. 10y 6 (10 10y) 39. 10q 10 (3q + 5) 40. 2n + 10 (7n 1) Version: Fall 2010

206 41. 7(8y + 7) 6(8 7y) 42. 6(5n 4) 9(3 + 4n)

CHAPTER 3. THE FUNDAMENTALS OF ALGEBRA 49. 8(n + 4) 10(4n + 3) 50. 3(8r 7) 3(2r 2) 51. 5 (10p + 5) 52. 1 (2p 8) 53. 7(1 + 7r) + 2(4 5r) 54. (5 s) + 10(9 + 5s) 55. 2(5 8x2 ) 6(6) 56. 8(10y 2 + 3x3 ) 5(7y 2 7x3 )

43. 7(10x2 8xy 2 ) 7(9xy 2 + 9x2 ) 44. 10(8x2 y 10xy 2 ) + 3(8xy 2 + 2x2 y) 45. 2(6 + 4n) + 4(n 7) 46. 6(2 6m) + 5(9m + 7) 47. 8 (4 + 8y) 48. 1 (8 + s)

57. The length L of a rectangle is 2 feet longer than 6 times its width W . Find the perimeter of the rectangle in terms of its width alone. 58. The length L of a rectangle is 7 feet longer than 6 times its width W . Find the perimeter of the rectangle in terms of its width alone. 59. The width W of a rectangle is 8 feet shorter than its length L. Find the perimeter of the rectangle in terms of its length alone.

60. The width W of a rectangle is 9 feet shorter than its length L. Find the perimeter of the rectangle in terms of its length alone. 61. The length L of a rectangle is 9 feet shorter than 4 times its width W . Find the perimeter of the rectangle in terms of its width alone. 62. The length L of a rectangle is 2 feet shorter than 6 times its width W . Find the perimeter of the rectangle in terms of its width alone.

l l l
1. 55xy 2 3. 21xy 2 5. 16xy 7. 0 9. 16x 11. 2q 13. 19r Version: Fall 2010

Answers

l l l

15. 15x3 17. 2 + 25n 19. 9x3 34x2 y 21. 18xy 4x3 23. 3 + 17m 25. 6x2 y + 16xy 2 27. 9x3 + 24xy

3.4. COMBINING LIKE TERMS 29. 24n + 17 31. 9y + 4 33. 24x2 y 68 35. 22x + 70y 37. 2s + 8 39. 7q 15 57. 4 + 14W 41. 98y + 1 43. 7x2 119xy 2 45. 40 12n 59. 4L 16 61. 10W 18
2 2

207 47. 4 8y 49. 48n 62 51. 10 10p 53. 15 + 39r 55. 26 + 16x2

Version: Fall 2010

208

CHAPTER 3. THE FUNDAMENTALS OF ALGEBRA

3.5

Solving Equations Involving Integers II

We return to solving equations involving integers, only this time the equations will be a bit more advanced, requiring the use of the distributive property and skill at combining like terms. Lets begin. You Try It! Solve for x: 6x 5x = 22 EXAMPLE 1. Solve for x: 7x 11x = 12.

Solution. Combine like terms. 7x 11x = 12 4x = 12 Original equation. Combine like terms: 7x 11x = 4x.

To undo the eect of multiplying by 4, divide both sides of the last equation by 4. 12 4x = 4 4 x = 3 Divide both sides by 4. Simplify: 12/(4) = 3.

Check. Substitute 3 for x in the original equation. 7x 11x = 12 7(3) 11(3) = 12 21 + 33 = 12 12 = 12 Answer: x = 2 Original equation. Substitute 3 for x. On the left, multiply rst. On the left, add.

Because the last line of the check is a true statement, 3 is a solution of the original equation.

You Try It! Solve for x: 11 = 3x (1 x) EXAMPLE 2. Solve for x: 12 = 5x (4 + x).

Solution. To take the negative of a sum, negate each term in the sum (change each term to its opposite). Thus, (4 + x) = 4 x. 12 = 5x (4 + x) 12 = 5x 4 x 12 = 4x 4 Original equation. (4 + x) = 4 x. Combine like terms: 5x x = 4x.

To undo the eect of subtracting 4, add 4 to both sides of the last equation. 12 + 4 = 4x 4 + 4 16 = 4x Version: Fall 2010 Add 4 to both sides. Simplify both sides.

3.5. SOLVING EQUATIONS INVOLVING INTEGERS II

209

To undo the eect of multiplying by 4, divide both sides of the last equation by 4. 16 4x = 4 4 4=x Divide both sides by 4. Simplify: 16/4 = 4.

Check. Substitute 4 for x in the original equation. 12 = 5x (4 + x) 12 = 5(4) (4 + 4) 12 = 20 8 12 = 12 Original equation. Substitute 4 for x. On the right, 5(4) = 20 and evaluate parentheses: 4 + 4 = 8. Simplify. Answer: x = 3

Because the last line of the check is a true statement, 4 is a solution of the original equation.

Variables on Both Sides


Variables can occur on both sides of the equation. Goal. Isolate the terms containing the variable you are solving for on one side of the equation.

You Try It! EXAMPLE 3. Solve for x: 5x = 3x 18. Solve for x: 4x 3 = x

Solution. To isolate the variables on one side of the equation, subtract 3x from both sides of the equation and simplify. 5x = 3x 18 5x 3x = 3x 18 3x 2x = 18 Original equation. Subtract 3x from both sides. Combine like terms: 5x 3x = 2x and 3x 3x = 0.

Note that the variable is now isolated on the left-hand side of the equation. To undo the eect of multiplying by 2, divide both sides of the last equation by 2. 2x 18 = 2 2 x = 9 Divide both sides by 2. Simplify: 18/2 = 9. Version: Fall 2010

210

CHAPTER 3. THE FUNDAMENTALS OF ALGEBRA

Check. Substitute 9 for x in the original equation. 5x = 3x 18 5(9) = 3(9) 18 45 = 27 18 45 = 45 Answer: x = 1 Original equation. Substitute 9 for x. Multiply rst on both sides. Subtract on the right: 27 18 = 45.

Because the last line of the check is a true statement, 9 is a solution of the original equation.

You Try It! Solve for x: 7x = 18 + 9x EXAMPLE 4. Solve for x: 5x = 3 + 6x.

Solution. To isolate the variables on one side of the equation, subtract 6x from both sides of the equation and simplify. 5x = 3 + 6x 5x 6x = 3 + 6x 6x x = 3 Original equation. Subtract 6x from both sides. Combine like terms: 5x 6x = x and 6x 6x = 0.

Note that the variable is now isolated on the left-hand side of the equation. There are a couple of ways we can nish this solution. Remember, x is the same as (1)x, so we could undo the eects of multiplying by 1 by dividing both sides of the equation by 1. Multiplying both sides of the equation by 1 will work equally well. But perhaps the easiest way to proceed is to simply negate both sides of the equation. (x) = 3 x = 3 Negate both sides. Simplify: (x) = x.

Check. Substitute 3 for x in the original equation. 5x = 3 + 6x 5(3) = 3 + 6(3) 15 = 3 18 15 = 15 Answer: x = 9 Original equation. Substitute 3 for x. Multiply rst on both sides. Subtract on the right: 3 18 = 15.

Because the last line of the check is a true statement, 3 is a solution of the original equation.

Version: Fall 2010

3.5. SOLVING EQUATIONS INVOLVING INTEGERS II

211

Dealing with x. If your equation has the form x = c, where c is some integer, note that this is equivalent to the equation (1)x = c. Therefore, dividing both sides by 1 will produce a solution for x. Multiplying both sides by 1 works equally well. However, perhaps the easiest thing to do is negate each side, producing (x) = c, which is equivalent to x = c.

You Try It! EXAMPLE 5. Solve for x: 6x 5 = 12x + 19. Solve for x: 2x + 3 = 18 3x

Solution. To isolate the variables on one side of the equation, subtract 12x from both sides of the equation and simplify. 6x 5 = 12x + 19 6x 5 12x = 12x + 19 12x 6x 5 = 19 Original equation. Subtract 12x from both sides. Combine like terms: 6x 12x = 6x and 12x 12x = 0. Note that the variable is now isolated on the left-hand side of the equation. Next, to undo subtracting 5, add 5 to both sides of the equation. 6x 5 + 5 = 19 + 5 6x = 24 6x 24 = 6 6 x = 4 Add 5 to both sides. Simplify: 5 + 5 = 0 and 19 + 5 = 24.

Finally, to undo multiplying by 6, divide both sides of the equation by 6. Divide both sides by 6. Simplify: 24/(6) = 4.

Check. Substitute 4 for x in the original equation. 6x 5 = 12x + 19 6(4) 5 = 12(4) + 19 24 5 = 48 + 19 29 = 29 Original equation. Substitute 4 for x. Multiply rst on both sides. Add: 24 5 = 29 and 48 + 19 = 29. Answer: x = 3

Because the last line of the check is a true statement, 4 is a solution of the original equation.

Version: Fall 2010

212

CHAPTER 3. THE FUNDAMENTALS OF ALGEBRA

You Try It! Solve for x: 3(2x 4) 2(5 x) = 18 EXAMPLE 6. Solve for x: 2(3x + 2) 3(4 x) = x + 8.

Solution. Use the distributive property to remove parentheses on the left-hand side of the equation. 2(3x + 2) 3(4 x) = x + 8 6x + 4 12 + 3x = x + 8 9x 8 = x + 8 Original equation. Use the distributive property. Combine like terms: 6x + 3x = 9x and 4 12 = 8. Isolate the variables on the left by subtracting x from both sides of the equation. 9x 8 x = x + 8 x 8x 8 = 8 Subtract x from both sides. Combine like terms: 9x x = 8x and x x = 0.

Note that the variable is now isolated on the left-hand side of the equation. Next, to undo subtracting 8, add 8 to both sides of the equation. 8x 8 + 8 = 8 + 8 8x = 16 Add 8 to both sides. Simplify: 8 + 8 = 0 and 8 + 8 = 16.

Finally, to undo multiplying by 8, divide both sides of the equation by 8. 8x 16 = 8 8 x=2 Divide both sides by 8. Simplify: 16/8 = 2.

Check. Substitute 2 for x in the original equation. 2(3x + 2) 3(4 x) = x + 8 2(3(2) + 2) 3(4 2) = 2 + 8 2(6 + 2) 3(2) = 10 2(8) 3(2) = 10 16 6 = 10 10 = 10 Original equation. Substitute 2 for x. Work parentheses on left, add on the right. Add in parentheses on left. Multiply rst on left. Subtract on left.

Answer: x = 5

Because the last line of the check is a true statement, 2 is a solution of the original equation.

Version: Fall 2010

3.5. SOLVING EQUATIONS INVOLVING INTEGERS II

213

l l l
In Exercises 1-16, solve the equation. 1. 9x + x = 8 2. 4x 5x = 3 3. 4 = 3x 4x 4. 6 = 5x + 7x 5. 27x + 51 = 84 6. 20x + 46 = 26 7. 9 = 5x + 9 6x 8. 6 = x + 3 4x

Exercises

l l l

9. 0 = 18x + 18 10. 0 = x + 71 11. 41 = 28x + 97 12. 65 = x 35 13. 8x 8 9x = 3 14. 6x + 7 9x = 4 15. 85x + 85 = 0 16. 17x 17 = 0

In Exercises 17-34, solve the equation. 17. 6x = 5x 9 18. 5x = 3x 2 19. 6x 7 = 5x 20. 3x + 8 = 5x 21. 4x 3 = 5x 1 22. x 2 = 9x 2 23. 3x + 5 = 3x 1 24. 5x + 9 = 4x 3 25. 5x = 3x + 6 26. 3x = 4x 6 27. 2x 2 = 4x 28. 6x 4 = 2x 29. 6x + 8 = 2x 30. 4x 9 = 3x 31. 6x = 4x 4 32. 8x = 6x + 8 33. 8x + 2 = 6x + 6 34. 3x + 6 = 2x 5

In Exercises 35-52, solve the equation. 35. 1 (x 2) = 3 36. 1 8(x 8) = 17 37. 7x + 6(x + 8) = 2 38. 8x + 4(x + 7) = 12 39. 8(6x 1) = 8 40. 7(2x 4) = 14 41. 7(4x 6) = 14 42. 2(2x + 8) = 8 43. 2 9(x 5) = 16 44. 7 2(x + 4) = 1 Version: Fall 2010

214 45. 7x + 2(x + 9) = 9 46. 8x + 7(x 2) = 14 47. 2(x + 8) = 10 48. 2(x 2) = 10

CHAPTER 3. THE FUNDAMENTALS OF ALGEBRA 49. 8 + 2(x 5) = 4 50. 5 + 2(x + 5) = 5 51. 9x 2(x + 5) = 10 52. 8x 5(x 3) = 15

In Exercises 53-68, solve the equation. 53. 4(7x + 5) + 8 = 3(9x 1) 2 54. 4(x + 9) + 5 = (5x 4) 2 55. 8(2x 6) = 7(5x 1) 2 56. 5(4x 8) = 9(6x + 4) 4 57. 2(2x 9) + 5 = 7(x 8) 58. 6(4x 9) + 4 = 2(9x 8) 59. 6(3x + 4) 6 = 8(2x + 2) 8 60. 5(5x 9) 3 = 4(2x + 5) 6 61. 2(2x 3) = 3(x + 2) 62. 2(7x + 1) = 2(3x 7) 63. 5(9x + 7) + 7 = (9x 8) 64. 7(2x 6) + 1 = 9(2x + 7) 65. 5(5x 2) = 4(8x + 1) 66. 5(x 4) = (x + 8) 67. 7(9x 6) = 7(5x + 7) 7 68. 8(2x + 1) = 2(9x + 8) 2

l l l
1. 1 3. 4 5. 5 7. 0 9. 1 11. 2 13. 5 15. 1

Answers
21. 2 23. 1 25. 3 27. 1 29. 2 31. 2 33. 2 35. 6 37. 50

l l l

17. 9 19. 7 Version: Fall 2010

39. 0 41. 2

3.5. SOLVING EQUATIONS INVOLVING INTEGERS II 43. 7 45. 3 47. 3 49. 1 51. 0 53. 33 55. 3 65. 2 67. 0 57. 23 59. 21 61. 12 63. 1

215

Version: Fall 2010

216

CHAPTER 3. THE FUNDAMENTALS OF ALGEBRA

3.6

Applications

Because weve increased our fundamental ability to simplify algebraic expressions, were now able to tackle a number of more advanced applications. Before we begin, we remind readers of required steps that must accompany solutions of applications.

Requirements for Word Problem Solutions. 1. Set up a Variable Dictionary. You must let your readers know what each variable in your problem represents. This can be accomplished in a number of ways: Statements such as Let P represent the perimeter of the rectangle. Labeling unknown values with variables in a table. Labeling unknown quantities in a sketch or diagram. 2. Set up an Equation. Every solution to a word problem must include a carefully crafted equation that accurately describes the constraints in the problem statement. 3. Solve the Equation. You must always solve the equation set up in the previous step. 4. Answer the Question. This step is easily overlooked. For example, the problem might ask for Janes age, but your equations solution gives the age of Janes sister Liz. Make sure you answer the original question asked in the problem. 5. Look Back. It is important to note that this step does not imply that you should simply check your solution in your equation. After all, its possible that your equation incorrectly models the problems situation, so you could have a valid solution to an incorrect equation. The important question is: Does your answer make sense based on the words in the original problem statement.

Consecutive Integers
The integers are consecutive, in the sense that one follows right after another. For example, 5 and 6 are a pair of consecutive integers. The important relation to notice is the fact that the second integer of this pair is one larger than its predecessor. That is, 6 = 5 + 1.

Version: Fall 2010

3.6. APPLICATIONS

217

Consecutive Integers. Let k represent an integer. The next consecutive integer is the integer k + 1. Thus, if k is an integer, then k + 1 is the next integer, k + 2 is the next integer after that, and so on. You Try It! EXAMPLE 1. The three sides of a triangle are consecutive integers and the perimeter is 72 inches. Find the measure of each side of the triangle. Solution. We follow the Requirements for Word Problem Solutions. 1. Set up a Variable Dictionary. In this case, a carefully labeled diagram is the best way to indicate what the unknown variable represents. The three sides of a triangle are consecutive integers and the perimeter is 57 centimeters. Find the measure of each side of the triangle.

k+2 k

k+1 In our schematic diagram, weve labeled the three sides of the triangle with expressions representing the consecutive integers k, k + 1, and k + 2. 2. Set up an Equation. To nd the perimeter P of the triangle, sum the three sides. P = k + (k + 1) + (k + 2) However, were given the fact that the perimeter is 72 inches. Thus, 72 = k + (k + 1) + (k + 2) 3. Solve the Equation. On the right, regroup and combine like terms. 72 = 3k + 3 Now, solve. 72 3 = 3k + 3 3 69 = 3k 69 3k = 3 3 23 = k Subtract 3 from both sides. Simplify. Divide both sides by 3. Simplify. Version: Fall 2010

218

CHAPTER 3. THE FUNDAMENTALS OF ALGEBRA

4. Answer the Question. Weve only found one side, but the question asks for the measure of all three sides. However, the remaining two sides can be found by substituting 23 for k into the expressions k + 1 and k + 2. k + 1 = 23 + 1 = 24 and k + 2 = 23 + 2 = 25

Hence, the three sides measure 23 inches, 24 inches, and 25 inches. 5. Look Back. Does our solution make sense? Well, the three sides are certainly consecutive integers, and their sum is 23 inches + 24 inches + 25 inches = 72 inches, which was the given perimeter. Therefore, our solution is correct.

Answer: 18, 19, and 20 cm

Consecutive Odd Integers


The integer pair 19 and 21 are an example of a pair of consecutive odd integers. The important relation to notice is the fact that the second integer of this pair is two larger than its predecessor. That is, 21 = 19 + 2. Consecutive Odd Integers. Let k represent an odd integer. The next consecutive odd integer is k + 2. Thus, if k is an odd integer, then k + 2 is the next odd integer, k + 4 is the next odd integer after that, and so on. You Try It! The length and width of a rectangle are consecutive odd integers and the perimeter is 120 meters. Find the length and width of the rectangle. EXAMPLE 2. The length and width of a rectangle are consecutive odd integers and the perimeter is 168 centimeters. Find the length and width of the rectangle. Solution. We follow the Requirements for Word Problem Solutions. 1. Set up a Variable Dictionary. In this case, a carefully labeled diagram is the best way to indicate what the unknown variable represents. k+2

k+2 In our schematic diagram, if the width k is an odd integer, then the length k + 2 is the next consecutive odd integer. Version: Fall 2010

3.6. APPLICATIONS

219

2. Set up an Equation. To nd the perimeter of the rectangle, sum the four sides. P = k + (k + 2) + k + (k + 2) However, were given the fact that the perimeter is 168 centimeters. Thus, 168 = k + (k + 2) + k + (k + 2) 3. Solve the Equation. On the right, regroup and combine like terms. 168 = 4k + 4 Now, solve. 168 4 = 4k + 4 4 164 = 4k 164 4k = 4 4 41 = k Subtract 4 from both sides. Simplify. Divide both sides by 4. Simplify.

4. Answer the Question. Weve only found the width, but the question asks for the measure of both the width and the length. However, the length can be found by substituting 41 for k into the expression k + 2. k + 2 = 41 + 2 = 43 Hence, the width is 41 centimeters and the length is 43 centimeters. 5. Look Back. Does our solution make sense? Well, the width is 41 cm and the length is 43 cm, certainly consecutive odd integers. Further, the perimeter would be 41 cm + 43 cm + 41 cm + 43 cm = 168 cm, so our solution is correct.

Answer: W = 29 cm, L = 31 cm

Tables
In the remaining applications in this section, we will strive to show how tables can be used to summarize information, dene variables, and construct equations to help solve the application. You Try It! EXAMPLE 3. Hue inherits $10,000 and decides to invest in two dierent types of accounts, a savings account paying 2% interest, and a certicate of deposit paying 4% interest. He decides to invest $1,000 more in the certicate of deposit than in savings. Find the amount invested in each account. Version: Fall 2010 Dylan invests a total of $2,750 in two accounts, a savings account paying 3% interest, and a mutual fund paying 5% interest. He invests $250 less in the mutual fund than in savings. Find the amount invested in each account.

220

CHAPTER 3. THE FUNDAMENTALS OF ALGEBRA

Solution. We follow the Requirements for Word Problem Solutions. 1. Set up a Variable Dictionary. Were going to use a table to summarize information and declare variables. In the table that follows, we let S represent the amount Hue invests in the savings account. Using a variable letter that sounds like the quantity it represents is an excellent strategy. Thus, in this case, letting S represent the amount invested in savings is far better than letting x represent the amount invested in savings. Account Type Savings Account (2%) Certicate of Deposit (4%) Totals Amount Deposited S S + 1000 10000

Because S represents the investment in savings, and were told that the investment in the certicate of deposit (CD) is $1,000 more than the investment in savings, the investment in the CD is therefore S + 1000, as indicated in the table. 2. Set up an Equation. The second column of the table reveals that the sum of the individual investments in the CD and savings totals $10,000. Hence, the equation that models this application is (S + 1000) + S = 10000. 3. Solve the Equation. On the left, regroup and combine like terms. 2S + 1000 = 10000 Now, solve. 2S + 1000 1000 = 10000 1000 2S = 9000 9000 2S = 2 2 S = 4500 Subtract 1000 from both sides. Simplify. Divide both sides by 2. Simplify.

4. Answer the Question. Weve only found the investment in savings, but the question also asks for the amount invested in the CD. However, the investment in the CD is easily found by substituting 4500 for S in the expression S + 1000. S + 1000 = 4500 + 1000 = 5500. Hence, the investment in savings is $4,500 and the investment in the CD is $5,500. Version: Fall 2010

3.6. APPLICATIONS

221

5. Look Back. Does our solution make sense? Well, the amount invested in the CD is $5,500, which is certainly $1,000 more than the $4,500 invested in savings. Secondly, the two investments total $5, 500 + $4, 500 = $10, 000, so our solution is correct.

Answer: $1,500 in savings, $1,250 in the mutual fund

You Try It! EXAMPLE 4. Jose cracks open his piggy bank and nds that he has $3.25 (325 cents), all in nickels and dimes. He has 10 more dimes than nickels. How many dimes and nickels does Jose have? Solution. We follow the Requirements for Word Problem Solutions. 1. Set up a Variable Dictionary. Were going to use a table to summarize information and declare variables. In the table that follows, we let N represent the number of nickels from the piggy bank. Using a variable letter that sounds like the quantity it represents is an excellent strategy. Thus, in this case, letting N represent the number of nickels is far better than letting x represent the number of nickels. Coins Nickels (5 cents apiece) Dimes (10 cents apiece) Totals Number of Coins N N + 10 Value (cents) 5N 10(N + 10) 325 David keeps his change in a bowl made by his granddaughter. There is $1.95 in change in the bowl, all in dimes and quarters. There are two fewer quarters than dimes. How many dimes and quarters does he have in the bowl?

Because there are 10 more dimes than nickels, the number of dimes is N + 10, recorded in the second column. In the third column, N nickels, worth 5 cents apiece, have a value of 5N cents. Next, N + 10 dimes, worth 10 cents apiece, have a value of 10(N + 10) cents. The nal entry in the column gives the total value of the coins as 325 cents. 2. Set up an Equation. The third column of the table reveals that the sum of the coin values is 325 cents. Hence, the equation that models this application is 5N + 10(N + 10) = 325, which sums the value of the nickels and the value of the dimes to a total of 325 cents. 3. Solve the Equation. On the left, use the distributive property to remove parentheses. 5N + 10N + 100 = 325 Combine like terms. 15N + 100 = 325 Version: Fall 2010

222 Now, solve.

CHAPTER 3. THE FUNDAMENTALS OF ALGEBRA

15N + 100 100 = 325 100 15N = 225 15N 225 = 15 15 N = 15

Subtract 100 from both sides. Simplify. Divide both sides by 15. Simplify.

4. Answer the Question. Weve only found the number of nickels, but the question also asks for the number of dimes. However, the number of dimes is easily found by substituting 15 for N in the expression N + 10. N + 10 = 15 + 10 = 25. Hence, Jose has 15 nickels and 25 dimes. 5. Look Back. Does our solution make sense? Well, the number of dimes is 25, which is certainly 10 more than 15 nickels. Also, the monetary value of 15 nickels is 75 cents and the monetary value of 25 dimes is 250 cents, a total of 325 cents, or $3.25, so our solution is correct.

Answer: 7 dimes, 5 quarters

You Try It! Emily purchase tickets to the IMAX theater for her family. An adult ticket cost $12 and a child ticket costs $4. She buys two more child tickets than adult tickets and the total cost is $136. How many adult and child tickets did she buy? EXAMPLE 5. A large childrens organization purchases tickets to the circus. The organization has a strict rule that every ve children must be accompanied by one adult guardian. Hence, the organization orders ve times as many child tickets as it does adult tickets. Child tickets are three dollars and adult tickets are six dollars. If the total cost of tickets is $4,200, how many child and adult tickets were purchased? Solution. We follow the Requirements for Word Problem Solutions. 1. Set up a Variable Dictionary. Were going to use a table to summarize information and declare variables. In the table that follows, we let A represent the number of adult tickets purchased. Using a variable letter that sounds like the quantity it represents is an excellent strategy. Thus, in this case, letting A represent the number of adult tickets is far better than letting x represent the number of adult tickets. Number of Tickets Adults ($6 apiece) Children ($3 apiece) Totals Version: Fall 2010 A 5A Cost (dollars) 6A 3(5A) 4200

3.6. APPLICATIONS

223

Because there are 5 times as many childrens tickets purchased than adult tickets, the number of childrens tickets purchased is 5A, recorded in the second column. In the third column, 5A childrens tickets at $3 apiece will cost 3(5A) dollars, and A adult tickets at $6 apiece will cost 6A dollars. The nal entry in the column gives the total cost of all tickets as $4,200. 2. Set up an Equation. The third column of the table reveals that the sum of the costs for both children and adult tickets is $4,200. Hence, the equation that models this application is 6A + 3(5A) = 4200 which sums the cost of children and adult tickets at $4,200. 3. Solve the Equation. On the left, use the associative property to remove parentheses. 6A + 15A = 4200 Combine like terms. 21A = 4200 Now, solve. 4200 21A = 21 21 A = 200 Divide both sides by 21. Simplify.

4. Answer the Question. Weve only found the number of adult tickets, but the question also asks for the number of childrens tickets. However, the number of childrens tickets is easily found by substituting 200 for A in the expression 5A. 5A = 5(200) = 1000. Hence, 1000 childrens tickets and 200 adult tickets were purchased. 5. Look Back. Does our solution make sense? Well, the number of childrens tickets purchased is 1000, which is certainly 5 times more than the 200 adult tickets purchased. Also, the monetary value of 1000 childrens tickets at $3 apiece is $3,000, and the monetary value of 200 adult tickets at $6 apiece is $1,200, a total cost of $4,200. Our solution is correct.

Answer: 8 adult and 10 child tickets

Version: Fall 2010

224

CHAPTER 3. THE FUNDAMENTALS OF ALGEBRA

l l l

Exercises

l l l

1. The three sides of a triangle are consecutive odd integers. If the perimeter of the triangle is 39 inches, nd the lengths of the sides of the triangle. 2. The three sides of a triangle are consecutive odd integers. If the perimeter of the triangle is 51 inches, nd the lengths of the sides of the triangle. 3. The width and length of a rectangle are consecutive integers. If the perimeter of the rectangle is 142 inches, nd the width and length of the rectangle. 4. The width and length of a rectangle are consecutive integers. If the perimeter of the rectangle is 166 inches, nd the width and length of the rectangle. 5. The three sides of a triangle are consecutive even integers. If the perimeter of the triangle is 240 inches, nd the lengths of the sides of the triangle. 6. The three sides of a triangle are consecutive even integers. If the perimeter of the triangle is 30 inches, nd the lengths of the sides of the triangle. 7. The width and length of a rectangle are consecutive integers. If the perimeter of the rectangle is 374 inches, nd the width and length of the rectangle. 8. The width and length of a rectangle are consecutive integers. If the perimeter of the rectangle is 318 inches, nd the width and length of the rectangle. 9. The width and length of a rectangle are consecutive odd integers. If the perimeter of the rectangle is 208 inches, nd the width and length of the rectangle. Version: Fall 2010

10. The width and length of a rectangle are consecutive odd integers. If the perimeter of the rectangle is 152 inches, nd the width and length of the rectangle. 11. The width and length of a rectangle are consecutive even integers. If the perimeter of the rectangle is 76 inches, nd the width and length of the rectangle. 12. The width and length of a rectangle are consecutive even integers. If the perimeter of the rectangle is 300 inches, nd the width and length of the rectangle. 13. The three sides of a triangle are consecutive even integers. If the perimeter of the triangle is 144 inches, nd the lengths of the sides of the triangle. 14. The three sides of a triangle are consecutive even integers. If the perimeter of the triangle is 198 inches, nd the lengths of the sides of the triangle. 15. The three sides of a triangle are consecutive integers. If the perimeter of the triangle is 228 inches, nd the lengths of the sides of the triangle. 16. The three sides of a triangle are consecutive integers. If the perimeter of the triangle is 216 inches, nd the lengths of the sides of the triangle. 17. The width and length of a rectangle are consecutive even integers. If the perimeter of the rectangle is 92 inches, nd the width and length of the rectangle. 18. The width and length of a rectangle are consecutive even integers. If the perimeter of the rectangle is 228 inches, nd the width and length of the rectangle.

3.6. APPLICATIONS 19. The three sides of a triangle are consecutive integers. If the perimeter of the triangle is 105 inches, nd the lengths of the sides of the triangle. 20. The three sides of a triangle are consecutive integers. If the perimeter of the triangle is 123 inches, nd the lengths of the sides of the triangle. 21. The width and length of a rectangle are consecutive odd integers. If the perimeter of the rectangle is 288 inches, nd the width and length of the rectangle.

225 22. The width and length of a rectangle are consecutive odd integers. If the perimeter of the rectangle is 352 inches, nd the width and length of the rectangle. 23. The three sides of a triangle are consecutive odd integers. If the perimeter of the triangle is 165 inches, nd the lengths of the sides of the triangle. 24. The three sides of a triangle are consecutive odd integers. If the perimeter of the triangle is 99 inches, nd the lengths of the sides of the triangle.

25. A large childrens organization purchases tickets to the circus. The organization has a strict rule that every 8 children must be accompanied by one adult guardian. Hence, the organization orders 8 times as many child tickets as it does adult tickets. Child tickets are $7 and adult tickets are $19. If the total cost of tickets is $975, how many adult tickets were purchased? 26. A large childrens organization purchases tickets to the circus. The organization has a strict rule that every 2 children must be accompanied by one adult guardian. Hence, the organization orders 2 times as many child tickets as it does adult tickets. Child tickets are $6 and adult tickets are $16. If the total cost of tickets is $532, how many adult tickets were purchased? 27. Judah cracks open a piggy bank and nds $3.30 (330 cents), all in nickels and dimes. There are 15 more dimes than nickels. How many nickels does Judah have? 28. Texas cracks open a piggy bank and nds $4.90 (490 cents), all in nickels and dimes. There are 13 more dimes than nickels. How many nickels does Texas have?

29. Steve cracks open a piggy bank and nds $4.00 (400 cents), all in nickels and dimes. There are 7 more dimes than nickels. How many nickels does Steve have? 30. Liz cracks open a piggy bank and nds $4.50 (450 cents), all in nickels and dimes. There are 15 more dimes than nickels. How many nickels does Liz have? 31. Jason inherits $20,300 and decides to invest in two dierent types of accounts, a savings account paying 2.5% interest, and a certicate of deposit paying 5% interest. He decides to invest $7,300 more in the certicate of deposit than in savings. Find the amount invested in the savings account. 32. Trinity inherits $24,300 and decides to invest in two dierent types of accounts, a savings account paying 2% interest, and a certicate of deposit paying 5.75% interest. She decides to invest $8,500 more in the certicate of deposit than in savings. Find the amount invested in the savings account. 33. Gina cracks open a piggy bank and nds $4.50 (450 cents), all in nickels and dimes. There are 15 more dimes than nickels. How many nickels does Gina have? Version: Fall 2010

226

CHAPTER 3. THE FUNDAMENTALS OF ALGEBRA 40. Amber inherits $26,000 and decides to invest in two dierent types of accounts, a savings account paying 2.25% interest, and a certicate of deposit paying 4.25% interest. She decides to invest $6,200 more in the certicate of deposit than in savings. Find the amount invested in the savings account. 41. A large childrens organization purchases tickets to the circus. The organization has a strict rule that every 8 children must be accompanied by one adult guardian. Hence, the organization orders 8 times as many child tickets as it does adult tickets. Child tickets are $6 and adult tickets are $16. If the total cost of tickets is $1024, how many adult tickets were purchased? 42. A large childrens organization purchases tickets to the circus. The organization has a strict rule that every 3 children must be accompanied by one adult guardian. Hence, the organization orders 3 times as many child tickets as it does adult tickets. Child tickets are $3 and adult tickets are $18. If the total cost of tickets is $351, how many adult tickets were purchased? 43. Alan inherits $25,600 and decides to invest in two dierent types of accounts, a savings account paying 3.5% interest, and a certicate of deposit paying 6% interest. He decides to invest $6,400 more in the certicate of deposit than in savings. Find the amount invested in the savings account. 44. Mercy inherits $27,100 and decides to invest in two dierent types of accounts, a savings account paying 3% interest, and a certicate of deposit paying 4% interest. She decides to invest $8,700 more in the certicate of deposit than in savings. Find the amount invested in the savings account.

34. Dylan cracks open a piggy bank and nds $4.05 (405 cents), all in nickels and dimes. There are 6 more dimes than nickels. How many nickels does Dylan have? 35. A large childrens organization purchases tickets to the circus. The organization has a strict rule that every 2 children must be accompanied by one adult guardian. Hence, the organization orders 2 times as many child tickets as it does adult tickets. Child tickets are $4 and adult tickets are $10. If the total cost of tickets is $216, how many adult tickets were purchased? 36. A large childrens organization purchases tickets to the circus. The organization has a strict rule that every 2 children must be accompanied by one adult guardian. Hence, the organization orders 2 times as many child tickets as it does adult tickets. Child tickets are $7 and adult tickets are $11. If the total cost of tickets is $375, how many adult tickets were purchased? 37. Connie cracks open a piggy bank and nds $3.70 (370 cents), all in nickels and dimes. There are 7 more dimes than nickels. How many nickels does Connie have? 38. Don cracks open a piggy bank and nds $3.15 (315 cents), all in nickels and dimes. There are 3 more dimes than nickels. How many nickels does Don have? 39. Mary inherits $22,300 and decides to invest in two dierent types of accounts, a savings account paying 2% interest, and a certicate of deposit paying 4% interest. She decides to invest $7,300 more in the certicate of deposit than in savings. Find the amount invested in the savings account.

Version: Fall 2010

3.6. APPLICATIONS 45. Tony inherits $20,600 and decides to invest in two dierent types of accounts, a savings account paying 2% interest, and a certicate of deposit paying 4% interest. He decides to invest $9,200 more in the certicate of deposit than in savings. Find the amount invested in the savings account. 46. Connie inherits $17,100 and decides to invest in two dierent types of accounts, a savings account paying 2% interest, and a certicate of deposit paying 5.5% interest. She decides to invest $6,100 more in the certicate of deposit than in savings. Find the amount invested in the savings account.

227 47. A large childrens organization purchases tickets to the circus. The organization has a strict rule that every 2 children must be accompanied by one adult guardian. Hence, the organization orders 2 times as many child tickets as it does adult tickets. Child tickets are $2 and adult tickets are $14. If the total cost of tickets is $234, how many adult tickets were purchased? 48. A large childrens organization purchases tickets to the circus. The organization has a strict rule that every 8 children must be accompanied by one adult guardian. Hence, the organization orders 8 times as many child tickets as it does adult tickets. Child tickets are $8 and adult tickets are $13. If the total cost of tickets is $1078, how many adult tickets were purchased?

l l l
1. 11 in., 13 in., 15 in. 3. 35 in., 36 in. 5. 78 in., 80 in., 82 in. 7. 93 in., 94 in. 9. 51 in., 53 in. 11. 18 in., 20 in. 13. 46 in., 48 in., 50 in. 15. 75 in., 76 in., 77 in. 17. 22 in., 24 in. 19. 34 in., 35 in., 36 in. 21. 71 in., 73 in. 23. 53 in., 55 in., 57 in.

Answers

l l l

25. 13 child tickets 27. 12 nickels 29. 22 nickels 31. $6, 500 33. 20 nickels 35. 12 child tickets 37. 20 nickels 39. $7, 500 41. 16 child tickets 43. $9, 600 45. $5, 700 47. 13 child tickets Version: Fall 2010

Chapter

Fractions
Around 3000BC, Egyptians were carving hierglyphs into stone monuments to their kings and queens. Hierglyphs are pictures that represent objects and they were used for words and numbers. Oddly, fractions were always written as sums of unit fractions, fractions whose numerator is always 1. For instance, instead of writing 3/5 , they would write a sum of unit fractions. 1 1 3 = + 5 2 10 Much of the ancient Egyptian math that we know of was in service to the agricultural and economic life of the people. in measuring dry goods such as grains, special glyphs were used to represent basic fractional amounts, glyphs that came together to represent the Eye of Horus.

Horus was a falcon-god whose father Osirus was murdered by his own brother Seth. When Horus attempted to avenge his fathers death, Seth ripped out Horus eye and cut it into six pieces, scattering them throughout Egypt. 229

230

CHAPTER 4. FRACTIONS

Taking pity on Horus, Thot, the god of learning and magic, found the pieces and put them back together making Horus healthy and whole again. Each piece of the Eye of Horus represents a dierent fraction of a hekat, or volume of grain. It was written that an apprentice scribe added the fractions one day and got 1 1 1 1 1 1 63 + + + + + = . 2 4 8 16 32 64 64 Asking where the missing 1/64 was, he was told that Thot would make up the dierence to anyone who sought and accepted is protection. In this chapter, youll learn how we use fractions.

Version: Fall 2010

4.1. EQUIVALENT FRACTIONS

231

4.1

Equivalent Fractions

In this section we deal with fractions, numbers or expressions of the form a/b. Fractions. A number of the form a b where a and b are numbers is called a fraction. The number a is called the numerator of the fraction, while the number b is called the denominator of the fraction. Near the end of this section, well see that the numerator and denominator of a fraction can also be algebraic expressions, but for the moment we restrict our attention to fractions whose numerators and denominators are integers. We start our study of fractions with the denition of equivalent fractions. Equivalent Fractions. Two fractions are equivalent if they represent the same numerical value. But how can we tell if two fractions represent the same number? Well, one technique involves some simple visualizations. Consider the image shown in Figure 4.1, where the shaded region represents 1/3 of the total area of the gure (one of three equal regions is shaded).

Figure 4.1: The shaded region is 1/3 of the whole region. In Figure 4.2, weve shaded 2/6 of the entire region (two of six equal regions are shaded).

Figure 4.2: The shaded region is 2/6 of the whole region.

Version: Fall 2010

232

CHAPTER 4. FRACTIONS

In Figure 4.3, weve shaded 4/12 of the entire region (four of twelve equal regions are shaded).

Figure 4.3: The shaded region is 4/12 of the whole region. Lets take the diagrams from Figure 4.1, Figure 4.2, and Figure 4.3 and stack them one atop the other, as shown in Figure 4.4.

Figure 4.4: One of three equals two of six equals four of twelve. Figure 4.4 provides solid visual evidence that the following fractions are equivalent. 2 4 1 = = 3 6 12 Key Observations 1. If we start with the fraction 1/3, then multiply both numerator and denominator by 2, we get the following result. 12 1 = 3 32 2 = 6 Multiply numerator and denominator by 2. Simplify numerator and denominator..

This is precisely the same thing that happens going from Figure 4.1 to 4.2, where we double the number of available boxes (going from 3 available to 6 available) and double the number of shaded boxes (going from 1 shaded to 2 shaded). 2. If we start with the fraction 1/3, then multiply both numerator and denominator by 4, we get the following result. 14 1 = 3 34 4 = 12 Version: Fall 2010 Multiply numerator and denominator by 4. Simplify numerator and denominator.

4.1. EQUIVALENT FRACTIONS

233

This is precisely the same thing that happens going from Figure 4.1 to 4.3, where we multiply the number of available boxes by 4 (going from 3 available to 12 available) and multiply the number of shaded boxes by 4 (going from 1 shaded to 4 shaded). The above discussion motivates the following fundamental result. Creating Equivalent Fractions. If you start with a fraction, then multiply both its numerator and denominator by the same number, the resulting fraction is equivalent (has the same numerical value) to the original fraction. In symbols, a ax = . b bx

Arguing in Reverse. Reversing the above argument also holds true. 1. If we start with the fraction 2/6, then divide both numerator and denominator by 2, we get the following result. 22 2 = 6 62 1 = 3 Divide numerator and denominator by 2. Simplify numerator and denominator.

This is precisely the same thing that happens going backwards from Figure 4.2 to 4.1, where we divide the number of available boxes by 2 (going from 6 available to 3 available) and dividing the number of shaded boxes by 2 (going from 2 shaded to 1 shaded). 2. If we start with the fraction 4/12, then divide both numerator and denominator by 4, we get the following result. 44 4 = 12 12 4 1 = 3 Multiply numerator and denominator by 4. Simplify numerator and denominator.

This is precisely the same thing that happens going backwards from Figure 4.3 to 4.1, where we divide the number of available boxes by 4 (going from 12 available to 3 available) and divide the number alignof shaded boxes by 4 (going from 4 shaded to 1 shaded). The above discussion motivates the following fundamental result.

Version: Fall 2010

234

CHAPTER 4. FRACTIONS

Creating Equivalent Fractions. If you start with a fraction, then divide both its numerator and denominator by the same number, the resulting fraction is equivalent (has the same numerical value) to the original fraction. In symbols, ax a = . b bx

The Greatest Common Divisor


We need a little more terminology. Divisor. If d and a are natural numbers, we say that d divides a if and only if when a is divided by d, the remainder is zero. In this case, we say that d is a divisor of a. For example, when 36 is divided by 4, the remainder is zero. In this case, we say that 4 is a divisor of 36. On the other hand, when 25 is divided by 4, the remainder is not zero. In this case, we say that 4 is not a divisor of 25. Greatest Common Divisor. Let a and b be natural numbers. The common divisors of a and b are those natural numbers that divide both a and b. The greatest common divisor is the largest of these common divisors.

You Try It! Find the greatest common divisor of 12 and 18 EXAMPLE 1. Find the greatest common divisor of 18 and 24. Solution. First list the divisors of each number, the numbers that divide each number with zero remainder. Divisors of 18 : 1, 2, 3, 6, 9, and 18 Divisors of 24 : 1, 2, 3, 4, 6, 8, 12, and 24 The common divisors are: Common Divisors : 1, 2, 3, and 6 The greatest common divisor is the largest of the common divisors. That is, Greatest Common Divisor = 6. Answer: 6 That is, the largest number that divides both 18 and 24 is the number 6.

Version: Fall 2010

4.1. EQUIVALENT FRACTIONS

235

Reducing a Fraction to Lowest Terms


First, a denition. Lowest Terms. A fraction is said to be reduced to lowest terms if the greatest common divisor of both numerator and denominator is 1. Thus, for example, 2/3 is reduced to lowest terms because the greatest common divisior of 2 and 3 is 1. On the other hand, 4/6 is not reduced to lowest terms because the greatest common divisor of 4 and 6 is 2. You Try It! EXAMPLE 2. Reduce the fraction 18/24 to lowest terms. Solution. One technique that works well is dividing both numerator and denominator by the greatest common divisor of the numerator and denominator. In Example 1, we saw that the greatest common divisor of 18 and 24 is 6. We divide both numerator and denominator by 6 to get 18 6 18 = 24 24 6 3 = 4 Divide numerator and denominator by 6. Simplify numerator and denominator. Reduce the fraction 12/18 to lowest terms.

Note that the greatest common divisor of 3 and 4 is now 1. Thus, 3/4 is reduced to lowest terms. There is a second way we can show division of numerator and denominator by 6. First, factor both numerator and denominator as follows: 36 18 = 24 46 Factor out a 6.

You can then show division of both numerator and denominator by 6 by crossing out or canceling a 6 in the numerator for a 6 in the denominator, like this: 3 6 46 3 = 4 = Cancel common factor.

Note that we get the same equivalent fraction, reduced to lowest terms, namely 3/4.

Answer: 2/3

Version: Fall 2010

236

CHAPTER 4. FRACTIONS

Important Point. In Example 2 we saw that 6 was both a divisor and a factor of 18. The words divisor and factor are equivalent. We used the following technique in our second solution in Example 2. Cancellation Rule. If you express numerator and denominator as a product, then you may cancel common factors from the numerator and denominator. The result will be an equivalent fraction. Because of the Cancellation Rule, one of the most eective ways to reduce a fraction to lowest terms is to rst nd prime factorizations for both numerator and denominator, then cancel all common factors. You Try It! Reduce the fraction 28/35 to lowest terms. EXAMPLE 3. Reduce the fraction 18/24 to lowest terms. Solution. Use factor trees to prime factor numerator and denominator. 18 6 2 3 3 2 4 2 2 24 6 3

Once weve factored the numerator and denominator, we cancel common factors. 233 18 = 24 2223 233 = 2 3 22 3 = 22 3 = 4 Answer: 4/5 Thus, 18/24 = 3/4. Prime factor numerator and denominator. Cancel common factors. Remaining factors. Simplify denominator.

You Try It! Reduce the fraction 36/60 to lowest terms. EXAMPLE 4. Reduce the fraction 28/42 to lowest terms. Solution. Use factor trees to prime factor numerator and denominator. Version: Fall 2010

4.1. EQUIVALENT FRACTIONS 28 4 2 2 7 2 6 3 42 7

237

Now we can cancel common factors. 227 28 = Prime factor numerator and denominator. 42 237 2 7 2 Cancel common factors. = 237 2 = 3 Thus, 28/42 = 2/3. Answer: 3/5

Reducing Fractions with Variables


We use exactly the same technique to reduce fractions whose numerators and denominators contain variables. You Try It! EXAMPLE 5. Reduce 56x2 y 60xy 2 to lowest terms. Solution. Use factor trees to factor the coecients of numerator and denominator. 56 4 2 2 2 14 7 2 6 3 2 60 10 5 Reduce: 25a3 b 40a2 b3

Now cancel common factors. 2227xxy 56x2 y = 60xy 2 2235xyy 2227xxy = 2 2 35xyy 27x = 35y 14x = 15y Prime factor numerator and denominator. Cancel common factors. Remaining factors. Simplify numerator and denominator. Answer: Version: Fall 2010 5a 8b2

Thus, 56x2 y/(60xy 2 ) = 14x/(15y).

238

CHAPTER 4. FRACTIONS

A Word on Mathematical Notation.


There are two types of mathematical notation: (1) inline mathematical notation, and (2) displayed mathematical notation. Inline Mathematical Notation. The notation 14x/(15y) is called inline mathematical notation. When the same expression is centered on its own line, as in 14x , 15y this type of notation is called displayed mathematical notation. When you work a problem by hand, using pencil and paper calculations, the preferred format is displayed notation, like the displayed notation used to simplify the given expression in Example 5. However, computers and calculators require that you enter your expressions using inline mathematical notation. Therefore, it is extremely important that you are equally competent with either mathematical notation: displayed or inline. By the way, order of operations, when applied to the inline expression 14x/(15y), requires that we perform the multiplication inside the parentheses rst. Then we must perform multiplications and divisions as they occur, as we move from left to right through the expression. This is why the inline notation 14x/(15y) is equivalent to the displayed notation 14x . 15y However, the expression 14x/15y is a dierent beast. There are no parentheses, so we perform multiplication and division as they occur, moving left to right through the expression. Thus, we must rst take the product of 14 and x, divide the result by 15, then multiply by y. In displayed notation, this result is equivalent to 14x y, 15 which is a dierent result. Some readers might wonder why we did not use the notation (14x)/(15y) to describe the solution in Example 5. After all, this inline notation is also equivalent to the displayed notation 14x . 15y However, the point is that we dont need to, as order of operations already requires that we take the product of 14 and x before dividing by 15y. If this is hurting your head, know that its quite acceptable to use the equivalent notation (14x)/(15y) instead of 14x/(15y). Both are correct. Version: Fall 2010

4.1. EQUIVALENT FRACTIONS

239

Equivalent Fractions in Higher Terms


Sometimes the need arises to nd an equivalent fraction with a dierent, larger denominator. You Try It! EXAMPLE 6. Express 3/5 as an equivalent fraction having denominator 20. Solution. The key here is to remember that multiplying numerator and denominator by the same number produces an equivalent fraction. To get an equivalent fraction with a denominator of 20, well have to multiply numerator and denominator of 3/5 by 4. 34 3 = 5 54 12 = 20 Multiply numerator and denominator by 4. Simplify numerator and denominator. Answer: 14/21 Express 2/3 as an equivalent fraction having denominator 21.

Therefore, 3/5 equals 12/20.

You Try It! EXAMPLE 7. Express 8 as an equivalent fraction having denominator 5. Solution. The key here is to note that 8= 8 1 Understood denominator is 1. Express 5 as an equivalent fraction having denominator 7.

To get an equivalent fraction with a denominator of 5, well have to multiply numerator and denominator of 8/1 by 5. = 85 15 40 = 5 Multiply numerator and denominator by 5. Simplify numerator and denominator. Answer: 35/7

Therefore, 8 equals 40/5.

Version: Fall 2010

240

CHAPTER 4. FRACTIONS

You Try It! Express 3/8 as an equivalent fraction having denominator 24a. EXAMPLE 8. Express 2/9 as an equivalent fraction having denominator 18a. Solution. To get an equivalent fraction with a denominator of 18a, well have to multiply numerator and denominator of 2/9 by 2a. 2 2a 2 = 9 9 2a 4a = 18a Answer: 9a 24a Multiply numerator and denominator by 2a. Simplify numerator and denominator.

Therefore, 2/9 equals 4a/(18a), or equivalently, (4a)/(18a).

Negative Fractions
We have to also deal with fractions that are negative. First, lets discuss placement of the negative sign. Positive divided by negative is negative, so 3 3 = . 5 5 But it is also true that negative divided by positive is negative. Thus, 3 3 = . 5 5 These two observations imply that all three of the following fractions are equivalent (the same number): 3 3 3 = = . 5 5 5 Note that there are three possible placements for the negative sign: (1) the denominator, (2) the fraction bar, or (3) the numerator. Any one of these placements produces an equivalent fraction. Fractions and Negative Signs. Let a and b be any integers. All three of the following fractions are equivalent (same number): a a a = = . b b b

Version: Fall 2010

4.1. EQUIVALENT FRACTIONS

241

Mathematicians prefer to place the negative sign either in the numerator or on the fraction bar. The use of a negative sign in the denominator is discouraged. You Try It! EXAMPLE 9. Reduce: 50x3 75x5 to lowest terms. Solution. Prime factor numerator and denominator and cancel. 255xxx 50x3 = 75x5 3 5 5 x x x x x 255xxx = 3 x x x x x 5 5 2 = 3 x x 2 = 3x2 However, it is preferred that there be no negative signs in the denominator, so lets place the negative sign on the fraction bar (the numerator would suit as well). Thus, 2 50x3 = 2 75x5 3x Answer: Reduce: 14y 5 35y 3

2y 2 5

We also have the following result. Fractions and Negative Signs. Let a and b be any integers. Then, a a = . b b

You Try It! EXAMPLE 10. Reduce: 12xy 2 18x2 y Solution. Unlike Example 9, some like to take care of the sign of the answer rst. 12xy 2 12xy 2 = 2y 18x 18x2 y Version: Fall 2010 Reduce: 21a2 b3 56a3b

242

CHAPTER 4. FRACTIONS

Now we can factor numerator and denominator and cancel common factors. = 223xyy 233xxy 22xyy 3 = 233xxy 2y = 3x

Thus, 3b2 8a

12xy 2 2y = . 2y 18x 3x

Answer:

Version: Fall 2010

4.1. EQUIVALENT FRACTIONS

243

l l l

Exercises

l l l

In Exercises 1-12, nd the GCD of the given numbers. 1. 72, 8 2. 76, 52 3. 52, 20 4. 56, 96 5. 36, 63 6. 63, 21 7. 72, 44 8. 10, 40 9. 16, 56 10. 54, 66 11. 84, 24 12. 75, 45

In Exercises 13-28, reduce the given fraction to lowest terms. 13. 14. 15. 16. 17. 18. 19. 20. 22 98 28 56 93 15 90 39 69 21 74 62 74 12 66 10 21. 22. 23. 24. 25. 26. 27. 28. 66 57 34 30 33 99 20 58 69 24 18 96 46 44 92 24

29. Express 3 as an equivalent fraction having denominator 24. 30. Express 3 as an equivalent fraction having denominator 8.

25 as an equivalent fraction hav31. Express 19 ing denominator 57. 29 as an equivalent fraction hav32. Express 22 ing denominator 44.

Version: Fall 2010

244 33. Express 2 as an equivalent fraction having denominator 2. 34. Express 2 as an equivalent fraction having denominator 8. 18 35. Express as an equivalent fraction hav19 ing denominator 95. 17 36. Express as an equivalent fraction hav22 ing denominator 44.

CHAPTER 4. FRACTIONS 1 37. Express as an equivalent fraction having 3 denominator 24. 15 38. Express as an equivalent fraction hav19 ing denominator 95. 39. Express 16 as an equivalent fraction having denominator 4. 40. Express 5 as an equivalent fraction having denominator 2.

In Exercises 41-56, reduce the given fraction to lowest terms. 41. 42. 43. 44. 45. 46. 47. 48. 34 86 48 14 72 92 27 75 92 82 44 62 21 33 57 99 49. 50. 51. 52. 53. 54. 55. 56. 22 98 33 69 42 88 100 48 94 6 36 38 10 86 100 46

3 57. Express as an equivalent fraction having 2 denominator 62n. 6 58. Express as an equivalent fraction hav25 ing denominator 50a. 13 59. Express as an equivalent fraction hav10 ing denominator 60m.

1 as an equivalent fraction hav60. Express 16 ing denominator 80p. 3 61. Express as an equivalent fraction having 2 denominator 50n. 43 62. Express as an equivalent fraction hav38 ing denominator 76a.

Version: Fall 2010

4.1. EQUIVALENT FRACTIONS 63. Express 11 as an equivalent fraction having denominator 4m. 64. Express 13 as an equivalent fraction having denominator 6n. 65. Express 3 as an equivalent fraction having denominator 10m.

245 66. Express 10 as an equivalent fraction having denominator 8b. 67. Express 6 as an equivalent fraction having denominator 5n. 68. Express 16 as an equivalent fraction having denominator 2y.

In Exercises 69-84, reduce the given fraction to lowest terms. 69. 70. 71. 72. 73. 74. 75. 76. 82y 5 48y 40y 5 55y 77x5 44x4 34x6 80x 14y 5 54y 2 96y 4 40y 2 42x 81x3 26x2 32x6 77. 78. 79. 80. 81. 82. 83. 84. 12x5 14x6 28y 4 72y 6 74x 22x2 56x2 26x3 12y 5 98y 6 96x2 14x4 18x6 54x2 32x6 62x2

In Exercises 85-100, reduce the given fraction to lowest terms. 85. 26y 2 x4 62y 6x2 6x2 y 3 40x3 y 2 2y 6 x4 94y 2x5 90y 6 x3 39y 3 x5 89. 30y 5 x5 26yx4 74x6 y 4 52xy 3 36x3 y 2 98x4 y 5 84x3 y 16x4 y 2 Version: Fall 2010

86.

90.

87.

91.

88.

92.

246 8x6 y 3 54x3 y 5 70y 5 x2 16y 4 x5 34yx6 58y 5 x4 99y 2 x3 88y 6 x

CHAPTER 4. FRACTIONS 36y 3x5 51y 2 x 44y 5 x5 88y 4x 91y 3 x2 28y 5x5 76y 2 x 57y 5x6

93. 94. 95. 96.

97. 98. 99. 100.

101. Hurricanes. According to the National Atmospheric and Oceanic Administration, in 2008 there were 16 named storms, of which 8 grew into hurricanes and 5 were major. i) What fraction of named storms grew into hurricanes? Reduce your answer to lowest terms. ii) What fraction of named storms were major hurricanes? Reduce your answer to lowest terms. iii) What fraction of hurricanes were major? Reduce your answer to lowest terms. 102. Tigers. Tigers are in critical decline because of human encroachment, the loss of more than nine-tenths of their habitat, and the growing trade in tiger skins and body parts. Associated Press-Times-Standard 01/24/10 Pressure mounts to save the tiger. i) Write the loss of habitat as a fraction. ii) Describe in words what the numerator and denominator of this fraction represent. iii) If the fraction represents the loss of the whole original habitat, how much of the original habitat remains?

l l l
1. 8 3. 4 5. 9

Answers
13. 15. 17. 11 49 31 5 23 7 37 6 22 19

l l l

7. 4 19. 9. 8 11. 12 Version: Fall 2010 21.

4.1. EQUIVALENT FRACTIONS 23. 25. 27. 29. 31. 33. 1 3 23 8 23 22 72 24 75 57 4 2 57. 59. 61. 63. 65. 67. 93n 62n 78m 60m 75n 50n 44m 4m 30m 10m 30n 5n 41y 4 24 7x 4 7y 3 27

247

90 35. 95 37. 39. 8 24 64 4 17 43

69. 71. 73. 75.

41. 43.

14 27x2 6 7x 37 11x 6 49y x4 3 13x2 31y 4

18 23

77. 79. 81. 83. 85.

46 45. 41 7 47. 11 11 49. 49 51. 53. 55. 21 44 47 3 5 43

87.

y4 47x 15y 4 x 13 Version: Fall 2010

89.

248 91. 93. 95. 97. 18 49xy 3 4x3 27y 2 17x2 29y 4 12yx 17
4

CHAPTER 4. FRACTIONS 99. 13 4y 2 x3 i) ii) iii) 1 2 5 16 5 8

101.

Version: Fall 2010

4.2. MULTIPLYING FRACTIONS

249

4.2

Multiplying Fractions

Consider the image in Figure 4.5, where the vertical lines divide the rectangular region into three equal pieces. If we shade one of the three equal pieces, the shaded area represents 1/3 of the whole rectangular region.

Figure 4.5: The shaded region is 1/3 of the whole region. Wed like to visualize taking 1/2 of 1/3. To do that, we draw an additional horizontal line which divides the shaded region in half horizontally. This is shown in Figure 4.6. The shaded region that represented 1/3 is now divided into two smaller rectangular regions, one of which is shaded with a dierent color. This region represents 1/2 of 1/3.

Figure 4.6: Shading 1/2 of 1/3. Next, extend the horizontal line the full width of the rectangular region, as shown in Figure 4.7.

Figure 4.7: Shading 1/2 of 1/3. Note that drawing the horizontal line, coupled with the three original vertical lines, has succeeded in dividing the full rectangular region into six smaller but equal pieces, only one of which (the one representing 1/2 of 1/3) is shaded in a new color. Hence, this newly shaded piece represents 1/6 of the whole region. The conclusion of our visual argument is the fact that 1/2 of 1/3 equals 1/6. In symbols, 1 1 1 = . 2 3 6 You Try It! EXAMPLE 1. Create a visual argument showing that 1/3 of 2/5 is 2/15. Solution. First, divide a rectangular region into ve equal pieces and shade two of them. This represents the fraction 2/5. Version: Fall 2010 Create a visual argument showing that 1/2 of 1/4 is 1/8.

250

CHAPTER 4. FRACTIONS

Next, draw two horizontal lines that divide the shaded region into three equal pieces and shade 1 of the three equal pieces. This represents taking 1/3 of 2/5.

Next, extend the horizontal lines the full width of the region and return the original vertical line from the rst image.

Note that the three horizontal lines, coupled with the ve original vertical lines, have succeeded in dividing the whole region into 15 smaller but equal pieces, only two of which (the ones representing 1/3 of 2/5) are shaded in the new color. Hence, this newly shaded piece represents 2/15 of the whole region. The conclusion of this visual argument is the fact that 1/3 of 2/5 equals 2/15. In symbols, 2 1 2 = . 3 5 15 Answer:

Multiplication Rule
In Figure 4.7, we saw that 1/2 of 1/3 equals 1/6. Note what happens when we multiply the numerators and multiply the denominators of the fractions 1/2 and 1/3. 11 1 1 = 2 3 23 1 = 6 Multiply numerators; multiply denominators. Simplify numerators and denominators.

We get 1/6! Could this be coincidence or luck? Lets try that again with the fractions from Example 1, where we saw that 1/3 of 2/5 equals 2/15. Again, multiply Version: Fall 2010

4.2. MULTIPLYING FRACTIONS the numerators and denominators of 1/3 and 2/5. 1 2 12 = Multiply numerators; multiply denominators. 3 5 35 2 = Simplify numerators and denominators. 15 Again, we get 2/15! These two examples motivate the following denition.

251

Multiplication Rule. To nd the product of the fractions a/b and c/d, multiply their numerators and denominators. In symbols, ac a c = b d bd

You Try It! EXAMPLE 2. Multiply 1/5 and 7/9. Solution. Multiply numerators and multiply denominators. 1 7 17 = 5 9 59 7 = 45 Multiply numerators; multiply denominators. Simplify numerators and denominators. Multiply: 6 15 Multiply: 1 2 3 5

You Try It! EXAMPLE 3. Find the product of 2/3 and 7/9. Solution. The usual rules of signs apply to products. Unlike signs yield a negative result. 2 7 27 = Multiply numerators; multiply denominators. 3 9 39 14 = Simplify numerators and denominators. 27 It is not required that you physically show the middle step. If you want to do that mentally, then you can simply write 2 7 14 = . 3 9 27 Answer: Multiply: 3 2 5 7

6 35

Version: Fall 2010

252

CHAPTER 4. FRACTIONS

Multiply and Reduce


After multiplying two fractions, make sure your answer is reduced to lowest terms (see Section 4.1). You Try It! Multiply: 3 14 7 9 EXAMPLE 4. Multiply 3/4 times 8/9. Solution. After multiplying, divide numerator and denominator by the greatest common divisor of the numerator and denominator. 38 3 8 = 4 9 49 24 = 36 24 12 = 36 12 2 = 3 Multiply numerators and denominators. Simplify numerator and denominator. Divide numerator and denominator by GCD. Simplify numerator and denominator.

Alternatively, after multiplying, you can prime factor both numerator and denominator, then cancel common factors. 24 3 8 = 4 9 36 2223 = 2233 222 3 = 2233 2 = 3 Answer: 2 3 Multiply numerators and denominators. Prime factor numerator and denominator. Cancel common factors.

You Try It! Multiply: 6 3x 2 21x3 EXAMPLE 5. Multiply 7x/2 and 5/(14x2 ). Solution. After multiplying, prime factor both numerator and denominator, then cancel common factors. Note that unlike signs yields a negative product. 5 7x 35x = 2 14x2 28x2 57x = 227xx 57x = 22xx 7 5 = 4x Version: Fall 2010 Multiply numerators and denominators. Prime factor numerator and denominator. Cancel common factors.

Answer:

3 7x2

4.2. MULTIPLYING FRACTIONS

253

Multiply and Cancel or Cancel and Multiply


When you are working with larger numbers, it becomes a bit harder to multiply, factor, and cancel. Consider the following argument. 630 18 35 = 30 6 180 23357 = 22335 2 3 3 5 7 = 2 2 3 3 5 7 = 2 Multiply numerators; multiply denominators. Prime factor numerators and denominators. Cancel common factors. Remaining factors.

There are a number of diculties with this approach. First, you have to multiply large numbers, and secondly, you have to prime factor the even larger results. One possible workaround is to not bother multiplying numerators and denominators, leaving them in factored form. 18 35 18 35 = 30 6 30 6 Multiply numerators; multiply denominators.

Finding the prime factorization of these smaller factors is easier. = (2 3 3) (5 7) (2 3 5) (2 3) Prime factor.

Now we can cancel common factors. Parentheses are no longer needed in the numerator and denominator because both contain a product of prime factors, so order and grouping do not matter. 2 3 3 5 7 2 3 5 3 2 7 = 2 = Cancel common factors. Remaining factors.

Another approach is to factor numerators and denominators in place, cancel common factors, then multiply. 18 35 233 = 30 6 235 233 = 2 3 5 7 = 2 57 23 57 2 3 Factor numerators and denominators. Cancel common factors. Remaining factors.

Note that this yields exactly the same result, 7/2.

Version: Fall 2010

254

CHAPTER 4. FRACTIONS

Cancellation Rule. When multiplying fractions, cancel common factors according to the following rule: Cancel a factor in a numerator for an identical factor in a denominator.

You Try It! Multiply: 6 70 35 36 EXAMPLE 6. Find the product of 14/15 and 30/140. Solution. Multiply numerators and multiply denominators. Prime factor, cancel common factors, then multiply. 14 30 14 30 = 15 140 15 140 (2 7) (2 3 5) = (3 5) (2 2 5 7) 2 7 2 3 5 = 352257 1 = 5 Answer: 1 3 Multiply numerators; multiply denominators. Prime factor numerators and denominators. Cancel common factors. Multiply.

Note: Everything in the numerator cancels because youve divided the numerator by itself. Hence, the answer has a 1 in its numerator.

When Everything Cancels. When all the factors in the numerator cancel, this means that you are dividing the numerator by itself. Hence, you are left with a 1 in the numerator. The same rule applies to the denominator. If everything in the denominator cancels, youre left with a 1 in the denominator.

You Try It! Simplify: 35b2 6a 15b 10a2 EXAMPLE 7. Simplify the product: 110y 2 6x . 55y 105x2

Solution. The product of two negatives is positive. 110y 2 6x 55y 105x2 = 6x 110y 2 55y 105x2 Like signs gives a positive.

Version: Fall 2010

4.2. MULTIPLYING FRACTIONS Prime factor numerators and denominators, then cancel common factors. 23x 5 11 y 2x 3 = 5 11 y 22y = 57x 4y = 35x = 2 5 11 y y 357xx 2 1 y y 5 1 357xx Prime factor numerators & denominators. Cancel common factors. Remaining factors

255

Multiply numerators; multiply denominators. Answer: 11b 5a

Parallelograms
In this section, we are going to learn how to nd the area of a parallelogram. Lets begin with the denition of a parallelogram. Recall that a quadrilateral is a polygon having four sides. A parallelogram is a very special type of quadrilateral. Parallelogram. A parallelogram is a quadrilateral whose opposite sides are parallel.

b The side on which the parallelogram rests is called its base (labeled b in the gure) and the distance from its base to the opposite side is called its height (labeled h in the gure). Note that the altitude is perpendicular to the base (meets the base at a 90 angle). Figure 4.8 shows a rectangle having length b and width h. Therefore, the area of the rectangle in Figure 4.8 is A = bh, which is found by taking the product of the length and width. Take a pair of scissors and cut a triangle from the right end of the rectangle as shown in Figure 4.9(a), then paste the cut triangle to the left end as shown in Figure 4.9(b). The result, seen in Figure 4.9(b) is a parallelogram having base b and height h. Because weve thrown no material away in creating the parallelogram from the rectangle, the parallelogram has the same area as the original rectangle. That is, the area of the parallelogram is A = bh. Version: Fall 2010

256

CHAPTER 4. FRACTIONS

b Figure 4.8: The area of the rectangle is A = bh.

(a) Cut a triangle from right end.

(b) Paste the triangle on left end.

Figure 4.9: Creating a parallelogram from a rectangle.

Area of a Parallelogram. A parallelogram having base b and height h has area A = bh. That is, to nd the area of a parallelogram, take the product of its base and height.

You Try It! The base of a parallelogram measures 14 inches. The height is 8/7 of an inch. What is the area of the parallelogram? EXAMPLE 8. Find the area of the parallelogram pictured below.

5/3 ft

6 ft Solution. The area of the parallelogram is equal to the product of its base and height. That is, A = bh = (6 ft) = 30 2 ft . 3 = 10 ft2 . 5 ft 3 Area formula for parallelogram. Substitute: 6 ft for b, 5/3 ft for h. Multiply numerators and denominators. Divide.

Answer: 16 square inches

Thus, the area of the parallelogram is 10 square feet. Version: Fall 2010

4.2. MULTIPLYING FRACTIONS

257

Triangles
Lets turn our attention to learning how to nd the area of a triangle. Triangle. A triangle is a three-sided polygon. It is formed by plotting three points and connecting them with three line segments. Each of the three points is called a vertex of the triangle and each of the three line segments is called a side of the triangle.

b The side on which the triangle rests is called its base, and the distance between its base and opposite vertex is called its height of altitude. The altitude is always perpendicular to the base; that is, it forms a 90 angle with the base. Its easily seen that a triangle has half the area of a parallelogram.

b The parallelogram has area A = bh. Therefore, the triangle has one-half that area. That is, the area of the triangle is A = (1/2)bh. Area of a Triangle. A triangle having base b and height h has area A = (1/2)bh. That is, to nd the area of a triangle, take one-half the product of the base and height.

You Try It! EXAMPLE 9. Find the area of the triangle pictured below. The base of a triangle measures 15 meters. The height is 12 meters. What is the area of the triangle?

6 cm 13 cm Version: Fall 2010

258

CHAPTER 4. FRACTIONS

Solution. To nd the area of the triangle, take one-half the product of the base and height. A= 1 bh 2 1 = (13 cm)(6 cm) 2 78 cm2 = 2 = 39 cm2 . Area of a triangle formula. Substitute: 13 cm for b, 6 cm for h. Multiply numerators; multiply denominators. Simplify.

Answer: 90 square meters

Therefore, the area of the triangle is 39 square centimeters.

Identifying the Base and Altitude. Sometimes it can be a bit dicult to determine the base and altitude (height) of a triangle. For example, consider the triangle in Figure 4.10(a). Lets say we choose the bottom edge of the triangle as the base and denote its length with the variable b, as shown in Figure 4.10(a).

b
(a) The base is easy.

b
(b) Adding the altitude (height).

Figure 4.10: Identifying the base and altitude (height) of a triangle. The altitude (height) of the triangle is dened as the distance between the base of the triangle and its opposite vertex. To identify this altitude, we must rst extend the base, as seen in the dashed extension in Figure 4.10(b), then drop a perpendicular dashed line from the opposite vertex to the extended base, also shown in Figure 4.10(b). This perpendicular is the altitude (height) of the triangle and we denote its length by h. But we can go further. Any of the three sides of a triangle may be designated as the base of the triangle. Suppose, as shown in Figure 4.11(a), we identify a dierent side as the base, with length denoted by the variable b. The altitude to this new base will be a segment from the opposite vertex, perpendicular to the base. Its length in Figure 4.11(b) is denoted by h. In like manner, there is a third side of the triangle that could also be used as the base. The altitude to this third side is found by dropping a perpendicular from the vertex of the triangle directly opposite from this base. This would also require extending the base. We leave this to our readers to explore. Version: Fall 2010

4.2. MULTIPLYING FRACTIONS

259

b h

(a) A dierent base.

(b) Adding the altitude (height).

Figure 4.11: Identifying the base and altitude (height) of a triangle.

Key Point. Any of the three sides of a triangle may be used as the base. The altitude is drawn by dropping a perpendicular from the opposite vertex to the chosen base. This sometimes requires that we extend the base. Regardless of which side we use for the base, the formula A = bh/2 will produce the same area result.

Version: Fall 2010

260

CHAPTER 4. FRACTIONS

l l l

Exercises

l l l

1. Create a diagram, such as that shown in Figure 4.7, to show that 1/3 of 1/3 is 1/9. 2. Create a diagram, such as that shown in Figure 4.7, to show that 1/2 of 1/4 is 1/8.

3. Create a diagram, such as that shown in Figure 4.7, to show that 1/3 of 1/4 is 1/12. 4. Create a diagram, such as that shown in Figure 4.7, to show that 2/3 of 1/3 is 2/9.

In Exercises 1-28, multiply the fractions, and simplify your result. 5. 6. 7. 8. 9. 10. 11. 12. 13. 14. 15. 16. 21 22 4 19 4 21 19 8 20 17 11 22 9 6 2 7 21 14 8 15 17 3 18 4 5 7 11 20 5 20 2 19 8 1 13 6 12 5 7 9 2 9 15 8 2 21 11 8 17. 18. 19. 20. 21. 22. 23. 24. 25. 26. 27. 28. 17 3 12 4 7 10 13 21 6 9 23 10 12 5 11 2 23 6 24 17 4 21 9 19 24 5 7 2 20 1 23 2 1 8 2 11 11 20 18 3 24 7 13 18 21 4 20 5

Version: Fall 2010

4.2. MULTIPLYING FRACTIONS In Exercises 29-40, multiply the fractions, and simplify your result. 29. 30. 31. 32. 33. 34. 12y 3 2 6 13 9y 8x3 6 5 3 5x 11y 3 6 24 5y 5 11y 21 18 17y 6 8x2 18 21 19x 2y 4 7 11 18y 35. 36. 37. 38. 39. 40. 9 13x6 15 16x2 17 22x6 15 16x3 6y 3 20 5 7y 6 21y 8 2 5 3y 3y 3 23 4 12y 16y 6 21 15 13y 4

261

In Exercises 41-56, multiply the fractions, and simplify your result. 41. 42. 43. 44. 45. 46. 47. 48. 13y 6 2x 20x4 7y 2 8y 3 7x2 13x6 10y 2 23y 4 7x6 21x 4y 2 2x6 y 5 9y 4 20x 11y 6 2x4 12x6 7y 2 16x3 11y 2 13y 4 18x x6 7y 4 21y 3 9x5 3y 3 14x5 5x 15y 2 49. 50. 51. 52. 53. 54. 55. 56. 19y 2 10x3 18x 7y 3 20x y 6 9y 3 4x3 4y 3 10x 5x5 21y 4 11y 2 22x 14x4 21y 3 16x 7y 3 21y 2 5x2 4y 10x3 5x 7y 6 17x3 12y 2 3y 6 7x4 6x4 13y 2 11y 3 8x5

Version: Fall 2010

262

CHAPTER 4. FRACTIONS

In Exercises 57-62, nd the area of the parallelogram having the given base and altitude. 57. base = 8 cm, altitude = 7 cm 58. base = 2 cm, altitude = 11 cm 59. base = 6 cm, altitude = 13 cm 60. base = 2 cm, altitude = 6 cm 61. base = 18 cm, altitude = 14 cm 62. base = 20 cm, altitude = 2 cm

In Exercises 63-68, nd the area of the triangle shown in the gure. (Note: Figures are not drawn to scale.) 63. 66.

9 ft 14 ft 64. 6 cm 67.

15 ft 10 ft

4 cm

65.

8 cm 12 in 5 in

68.

5 cm 14 in 6 in

69. Weight on the Moon. On the moon, you would only weigh 1/6 of what you weigh on earth. If you weigh 138 pounds on earth, what would your weight on the moon be?

Version: Fall 2010

4.2. MULTIPLYING FRACTIONS

263

l l l
1. This shows that 1/3 of 1/3 is 1/9.

Answers
27. 28 39

l l l

29. 31. 33. 35. 37.

8 39y 3

3. This shows that 1/3 of 1/4 is 1/12.

11 20y 2 48x 133 39x4 80 24 7y 3 23y 2 16

5. 7. 9. 11. 13. 15. 17.

231 38 170 121 49 20 7 44 4 39 3 20

39. 41.

13y 4 70x3 23y 2 x5 12 11y 4 42x2 xy 27

43. 45. 47. 49. 51. 53.

17 16 27 115

19. 21. 23.

95x2 63y 8 21yx4 16y 15x 68 7xy 4

23 68 60 7 4 11

55.

25.

57. 56 cm2 Version: Fall 2010

264 59. 78 cm2 61. 252 cm2 63. 63 ft2

CHAPTER 4. FRACTIONS 65. 30 in2 67. 10 cm2 69. 23 pounds

Version: Fall 2010

4.3. DIVIDING FRACTIONS

265

4.3

Dividing Fractions

Suppose that you have four pizzas and each of the pizzas has been sliced into eight equal slices. Therefore, each slice of pizza represents 1/8 of a whole pizza.

Figure 4.12: One slice of pizza is 1/8 of one whole pizza. Now for the question: How many one-eighths are there in four? This is a division statement. To nd how many one-eighths there are in 4, divide 4 by 1/8. That is, 1 Number of one-eighths in four = 4 . 8 On the other hand, to nd the number of one-eights in four, Figure 4.12 clearly demonstrates that this is equivalent to asking how many slices of pizza are there in four pizzas. Since there are 8 slices per pizza and four pizzas, Number of pizza slices = 4 8. The conclusion is the fact that 4 (1/8) is equivalent to 4 8. That is, 4 1 = 48 8 = 32.

Therefore, we conclude that there are 32 one-eighths in 4.

Reciprocals
The number 1 is still the multiplicative identity for fractions.

Version: Fall 2010

266

CHAPTER 4. FRACTIONS

Multiplicative Identity Property. Let a/b be any fraction. Then, a a 1= b b and 1 a a = . b b

The number 1 is called the multiplicative identity because the identical number is returned when you multiply by 1. Next, if we invert 3/4, that is, if we turn 3/4 upside down, we get 4/3. Note what happens when we multiply 3/4 by 4/3. 3 4 34 = 4 3 43 12 = 12 =1 Multiply numerators; multiply denominators. Simplify numerators and denominators. Divide.

The number 4/3 is called the multiplicative inverse or reciprocal of 3/4. The product of reciprocals is always 1. Multiplicative Inverse Property. Let a/b be any fraction. The number b/a is called the multiplicative inverse or reciprocal of a/b. The product of reciprocals is 1. a b =1 b a Note: To nd the multiplicative inverse (reciprocal) of a number, simply invert the number (turn it upside down). For example, the number 1/8 is the multiplicative inverse (reciprocal) of 8 because 1 8 = 1. 8 Note that 8 can be thought of as 8/1. Invert this number (turn it upside down) to nd its multiplicative inverse (reciprocal) 1/8. You Try It! Find the reciprocals of: (a) 3/7 and (b) 15 EXAMPLE 1. Find the multiplicative inverses (reciprocals) of: (a) 2/3, (b) 3/5, and (c) 12. Solution. a) Because 2 3 = 1, 3 2 the multiplicative inverse (reciprocal) of 2/3 is 3/2. Version: Fall 2010

4.3. DIVIDING FRACTIONS b) Because 3 5 5 3 = 1,

267

the multiplicative inverse (reciprocal) of 3/5 is 5/3. Again, note that we simply inverted the number 3/5 to get its reciprocal 5/3. c) Because 12 1 12 = 1,

the multiplicative inverse (reciprocal) of 12 is 1/12. Again, note that we simply inverted the number 12 (understood to equal 12/1) to get its reciprocal 1/12.

Answer: (a) 7/3, (b) 1/15

Division
Recall that we computed the number of one-eighths in four by doing this calculation: 4 1 = 48 8 = 32.

Note how we inverted the divisor (second number), then changed the division to multiplication. This motivates the following denition of division. Division Denition. If a/b and c/d are any fractions, then a c a d = . b d b c That is, we invert the divisor (second number) and change the division to multiplication. Note: We like to use the phrase invert and multiply as a memory aid for this denition.

You Try It! EXAMPLE 2. Divide 1/2 by 3/5. Solution. To divide 1/2 by 3/5, invert the divisor (second number), then multiply. 1 3 1 5 = 2 5 2 3 5 = 6 Invert the divisor (second number). Multiply. Version: Fall 2010 Answer: 1/5 Divide: 2 10 3 3

268

CHAPTER 4. FRACTIONS

You Try It! Divide: 15 5 7 EXAMPLE 3. Simplify the following expressions: (a) 3 4 2 and (b) 5. 3 5 Solution. In each case, invert the divisor (second number), then multiply. a) Note that 3 is understood to be 3/1. 3 3 3 2 = 3 1 2 9 = 2 Invert the divisor (second number). Multiply numerators; multiply denominators.

b) Note that 5 is understood to be 5/1. 4 1 4 5= 5 5 5 4 = 25 Answer: 3 7 Invert the divisor (second number). Multiply numerators; multiply denominators.

After inverting, you may need to factor and cancel, as we learned to do in Section 4.2. You Try It! Divide: 42 6 15 35 EXAMPLE 4. Divide 6/35 by 33/55. Solution. Invert, multiply, factor, and cancel common factors. 33 6 55 6 = 35 55 35 33 6 55 = 35 33 (2 3) (5 11) = (5 7) (3 11) & 235& 11 = 573& 11 & 2 = 7 Invert the divisor (second number). Multiply numerators; multiply denominators. Factor numerators and denominators. Cancel common factors. Remaining factors.

Answer: 1/3

Note that unlike signs produce a negative answer.

Of course, you can also choose to factor numerators and denominators in place, then cancel common factors. Version: Fall 2010

4.3. DIVIDING FRACTIONS

269

You Try It! EXAMPLE 5. Divide 6/x by 12/x2 . Solution. Invert, factor numerators and denominators, cancel common factors, then multiply. 12 6 2 x x = x2 6 x 12 xx 23 = x 223 2 3 xx = x 223 x = 2 Invert second number. Factor numerators and denominators. Cancel common factors. Multiply. Answer: 4a2 5 Divide: 12 15 3 a a

Note that like signs produce a positive answer.

Version: Fall 2010

270

CHAPTER 4. FRACTIONS

l l l

Exercises

l l l

In Exercises 1-16, nd the reciprocal of the given number. 1. 16/5 2. 3/20 3. 17 4. 16 5. 15/16 6. 7/9 7. 30 8. 28 9. 46 10. 50 11. 9/19 12. 4/7 13. 3/17 14. 3/5 15. 11 16. 48

In Exercises 17-32, determine which property of addition is depicted by the given identity. 17. 18. 19. 20. 21. 2 9 =1 9 2 12 19 =1 19 12 19 19 1= 12 12 19 19 1= 8 8 1 6 =1 6 1 19 =1 1 4 25. 1 4 26. 27. 28. 29. 30. 31. 32. 10 9 10 9 =1 =1

22. 19 23.

16 16 1= 11 11 7 7 1 = 24. 6 6

8 8 1 = 1 1 13 13 1= 15 15 1 =1 14 14 1 4 =1 4 13 13 1= 8 8 1 1 1= 13 13

Version: Fall 2010

4.3. DIVIDING FRACTIONS In Exercises 33-56, divide the fractions, and simplify your result. 33. 34. 35. 36. 37. 38. 39. 40. 41. 42. 43. 44. 6 8 23 11 10 6 21 5 18 16 19 23 13 17 10 18 6 4 21 5 2 12 9 19 1 8 9 3 1 15 2 8 3 21 11 10 23 7 24 2 12 2 7 3 9 6 16 7 45. 46. 47. 48. 49. 50. 51. 52. 53. 54. 55. 56. 24 2 19 23 7 10 3 21 9 24 5 19 14 22 17 21 18 14 11 9 5 20 6 19 13 4 18 9 3 7 2 12 11 21 2 10 9 13 2 22 12 3 10 5 22 18 7 17

271

In Exercises 57-68, divide the fractions, and simplify your result. 20 5 17 21 58. 7 8 21 59. 7 20 12 60. 3 17 8 2 61. 21 3 62. (6) 4 57. 63. 8 64. 65. 66. 67. 68. 10 17 20 6 21 18 8 5 21 6 8 3 (9) 4 2 (8) 9 Version: Fall 2010

272

CHAPTER 4. FRACTIONS

In Exercises 69-80, divide the fractions, and simplify your result. 69. 70. 71. 72. 73. 74. 8x4 11x2 12 3 4x2 11x6 3 6 17y 10y 6 9 3 5y 3y 5 12 2 12x 22x4 13 11 24y 5 9y 6 4 13 75. 76. 77. 78. 79. 80. 4x 3x4 10 5 18y 4 4y 2 11 7 15y 2 10y 5 14 13 3x 2x3 20 5 15x5 20x2 13 19 18y 6 14y 4 7 9

In Exercises 81-96, divide the fractions, and simplify your result. 81. 82. 83. 84. 85. 86. 87. 88. 11y 4 9y 2 14x2 7x3 5x2 22x 3 12y 21y 5 10x4 7x5 3y 4 24y 2 20x3 5x5 3 5 11y 6y 22y 4 5y 2 21x5 6x4 7y 5 21y 5 8x6 5x 22x4 17x3 3 21y 3y 4 15y 7y 4 4x 22x4 89. 90. 91. 92. 93. 94. 95. 96. 16y 2 2y 6 3x3 11x5 20x 22x5 21y 2 y6 x 23x3 4 12y 16y 3 20x2 8x3 17y 3 15y 9y 5 y2 4x 8x3 10y 4 5y 6 13x2 6x3 18x6 3x 2 13y 4 y 20x4 14x2 9y 6 17y 4

Version: Fall 2010

4.3. DIVIDING FRACTIONS

273

l l l
1. 3. 5. 7. 5 16 1 17

Answers
37. 39. 41. 43. 45. 47. 49. 51.

l l l
10 63 1 24 70 11 18 7

16 15 1 30 1 46 19 9

9. 11. 13. 15.

23 228 57 40 81 77 13 8 55 21

17 3 1 11

17. multiplicative inverse property 19. multiplicative identity property

53. 55. 1 8

21. multiplicative inverse property 23. multiplicative identity property 25. multiplicative inverse property 27. multiplicative identity property 61. 29. multiplicative inverse property 31. multiplicative identity property 33. 35. 44 69 207 152 57.

4 17 20 3

59.

4 21 68 5 20 9 1 12 Version: Fall 2010

63. 65. 67.

274 11 32x2 17 30y 5 121x3 78

CHAPTER 4. FRACTIONS 83. 80 7xy 2 44y 2 35x

69. 71.

85. 87.

73.

22xy 119 88x2 3y 4

3x3 75. 8 77. 39 28y 3 57x3 52 11y 2 x 18

89. 91.

4 69x2 y 2x2 9y 3 6x5 13y 2

79. 81.

93. 95.

Version: Fall 2010

4.4. ADDING AND SUBTRACTING FRACTIONS

275

4.4

Adding and Subtracting Fractions

Paul and Tony order a pizza which has been cut into eight equal slices. Thus, each slice is 1/8 of the whole pizza. Paul eats two slices (shaded in light gray in Figure 4.13), or 2/8 of the whole pizza. Tony eats three slices (shaded in light red (or a darker shade of gray in black-and-white printing) in Figure 4.13), or 3/8 of the whole pizza.

Figure 4.13: Paul eats two slices (2/8) and Tony eats three slices (3/8). It should be clear that together Paul and Tony eat ve slices, or 5/8 of the whole pizza. This reects the fact that 5 2 3 + = . 8 8 8 This demonstrates how to add two fractions with a common (same) denominator. Keep the common denominator and add the numerators. That is, 2 3 2+3 + = 8 8 8 5 = 8 Keep denominator; add numerators. Simplify numerator.

Adding Fractions with Common Denominators. Let a/c and b/c be two fractions with a common (same) denominator. Their sum is dened as a+b a b + = . c c c That is, to add two fractions having common denominators, keep the common denominator and add their numerators. A similar rule holds for subtraction. Subtracting Fractions with Common Denominators. Let a/c and b/c be two fractions with a common (same) denominator. Their dierence is dened as ab a b = . c c c

Version: Fall 2010

276

CHAPTER 4. FRACTIONS

That is, to subtract two fractions having common denominators, keep the common denominator and subtract their numerators.

You Try It! Add: 1 2 + 8 8 EXAMPLE 1. Find the sum of 4/9 and 3/9. Solution. Keep the common denominator and add the numerators. 4 3 4+3 + = 9 9 9 7 = 9 Answer: 3/8 Keep denominator; add numerators. Simplify numerator.

You Try It! Subtract: 11 7 12 12 EXAMPLE 2. Subtract 5/16 from 13/16. Solution. Keep the common denominator and subtract the numerators. 13 5 13 5 = 16 16 16 8 = 16 Keep denominator; subtract numerators. Simplify numerator.

Of course, as we learned in Section 4.1, we should always reduce our nal answer to lowest terms. One way to accomplish that in this case is to divide numerator and denominator by 8, the greatest common divisor of 8 and 16. 88 16 8 1 = 2 = Answer: 1/3 Divide numerator and denominator by 8. Simplify numerator and denominator.

You Try It! Subtract: 2 5 y y EXAMPLE 3. Simplify: 7 3 . x x

Version: Fall 2010

4.4. ADDING AND SUBTRACTING FRACTIONS Solution. Both fractions share a common denominator. 3 7 x x 7 3 + x x 3+7 = x 10 = x = Add the opposite. Keep denominator, add numerators. Simplify.

277

Answer: 3/y

Adding Fractions with Dierent Denominators


Consider the sum 4 1 + . 9 6 We cannot add these fractions because they do not have a common denominator. So, what to do?

Goal. In order to add two fractions with dierent denominators, we need to: 1. Find a common denominator for the given fractions. 2. Make fractions with the common denominator that are equivalent to the original fractions.

If we accomplish the two items in the Goal, we will be able to nd the sum of the given fractions. So, how to start? We need to nd a common denominator, but not just any common denominator. Lets agree that we want to keep the numbers as small as possible and nd a least common denominator. Least Common Denominator. The least common denominator (LCD) for a set of fractions is the smallest number divisible by each of the denominators of the given fractions. Consider again the sum we wish to nd: 4 1 + . 9 6 The denominators are 9 and 6. We wish to nd a least common denominator, the smallest number that is divisible by both 9 and 6. A number of candidates Version: Fall 2010

278

CHAPTER 4. FRACTIONS

come to mind: 36, 54, and 72 are all divisible by 9 and 6, to name a few. But the smallest number that is divisible by both 9 and 6 is 18. This is the least common denominator for 9 and 6. We now proceed to the second item in Goal. We need to make fractions having 18 as a denominator that are equivalent to 4/9 and 1/6. In the case of 4/9, if we multiply both numerator and denominator by 2, we get 42 4 = 9 92 8 = . 18 Multiply numerator and denominator by 2. Simplify numerator and denominator.

In the case of 1/6, if we multiply both numerator and denominator by 3, we get 1 13 = 6 63 3 = . 18 Multiply numerator and denominator by 3. Simplify numerator and denominator.

Typically, well arrange our work as follows. 4 1 42 13 + = + 9 6 92 63 8 3 = + 18 18 8+3 = 18 11 = 18 Equivalent fractions with LCD = 18. Simplify numerators and denominators. Keep common denominator; add numerators. Simplify numerator.

Lets summarize the procedure. Adding or Subtracting Fractions with Dierent Denominators. 1. Find the LCD, the smallest number divisible by all the denominators of the given fractions. 2. Create fractions using the LCD as the denominator that are equivalent to the original fractions. 3. Add or subtract the resulting equivalent fractions. Simplify, including reducing the nal answer to lowest terms.

Version: Fall 2010

4.4. ADDING AND SUBTRACTING FRACTIONS

279

You Try It! EXAMPLE 4. Simplify: 3 2 . 5 3 Solution. The smallest number divisible by both 5 and 3 is 15. 33 25 3 2 = 5 3 53 35 9 10 = 15 15 9 10 = 15 1 = 15 Equivalent fractions with LCD = 15. Simplify numerators and denominators. Keep LCD; subtract numerators. Simplify numerator. Subtract: 3 7 4 5

Although this answer is perfectly acceptable, negative divided by positive gives us a negative answer, so we could also write = 1 . 15 Answer: 13/20

You Try It! EXAMPLE 5. Simplify: 1 5 . 4 6 Solution. The smallest number divisible by both 4 and 6 is 12. 1 5 13 52 = 4 6 43 62 10 3 = 12 12 3 10 = 12 13 = 12 Equivalent fractions with LCD = 12. Simplify numerators and denominators. Keep LCD; subtract numerators. Simplify numerator. Answer: 11/24 Subtract: 3 1 8 12

Version: Fall 2010

280

CHAPTER 4. FRACTIONS

You Try It! Add: EXAMPLE 6. Simplify: 5 2 + z 3 5 3 + . x 4

Solution. The smallest number divisible by both 4 and x is 4x. 5 3 54 3x + = + x 4 x4 4x 20 3x = + 4x 4x 20 + 3x = 4x Equivalent fractions with LCD = 4x. Simplify numerators and denominators. Keep LCD; add numerators.

Answer:

15 + 2z 3z

You Try It! Simplify: 3 y 7 4 EXAMPLE 7. Simplify: 2 x . 3 5

Solution. The smallest number divisible by both 3 and 5 is 15. 25 x3 2 x = 3 5 35 53 10 3x = 15 15 10 3x = 15 Equivalent fractions with LCD = 15. Simplify numerators and denominators. Keep LCD; subtract numerators.

Answer:

3y 28 7y

Least Common Multiple


First we dene the multiple of a number. Multiples. The multiples of a number d are 1d, 2d, 3d, 4d, etc. That is, the multiples of d are the numbers nd, where n is a natural number. For example, the multiples of 8 are 1 8, 2 8, 3 8, 4 8, etc., or equivalently, 8, 16, 24, 32, etc.

Version: Fall 2010

4.4. ADDING AND SUBTRACTING FRACTIONS

281

Least Common Multiple. The least common multiple (LCM) of a set of numbers is the smallest number that is a multiple of each number of the given set. The procedure for nding an LCM follows: 1. List all of the multiples of each number in the given set of numbers. 2. List the multiples that are in common. 3. Pick the least of the multiples that are in common.

You Try It! EXAMPLE 8. Find the least common multiple (LCM) of 12 and 16. Solution. List the multiples of 12 and 16. Multiples of 12 : Multiples of 16 : Pick the common multiples. Common Multiples : 48, 96, . . . 12, 24, 36, 48, 60, 72, 84, 96, . . . 16, 32, 48, 64, 80, 96, 112, . . . Find the least common denominator of 6 and 9.

The LCM is the least of the common multiples. LCM(12,16) = 48 Answer: 18

Important Observation. The least common denominator is the least common multiple of the denominators. For example, suppose your problem is 5/12 + 5/16. The LCD is the smallest number divisible by both 12 and 16. That number is 48, which is also the LCM of 12 and 16. Therefore, the procedure for nding the LCM can also be used to nd the LCD.

Least Common Multiple Using Prime Factorization


You can also nd the LCM using prime factorization.

Version: Fall 2010

282

CHAPTER 4. FRACTIONS

LCM by Prime Factorization. To nd an LCM for a set of numbers, follow this procedure: 1. Write down the prime factorization for each number in compact form using exponents. 2. The LCM is found by writing down every factor that appears in step 1 to the highest power of that factor that appears.

You Try It! Use prime factorization to nd the least common denominator of 18 and 24. EXAMPLE 9. Use prime factorization to nd the least common multiple (LCM) of 12 and 16. Solution. Prime factor 12 and 16. 12 = 2 2 3 16 = 2 2 2 2 Write the prime factorizations in compact form using exponents. 12 = 22 31 16 = 24 To nd the LCM, write down each factor that appears to the highest power of that factor that appears. The factors that appear are 2 and 3. The highest power of 2 that appears is 24 . The highest power of 3 that appears is 31 . LCM = 24 31 Keep highest power of each factor.

Now we expand this last expression to get our LCM. = 16 3 = 48. Expand: 24 = 16 and 31 = 3. Multiply.

Answer: 72

Note that this answer is identical to the LCM found in Example 8 that was found by listing multiples and choosing the smallest multiple in common.

You Try It! Simplify: 5 5 + 24 36 EXAMPLE 10. Simplify: Version: Fall 2010 5 11 + . 28 42

4.4. ADDING AND SUBTRACTING FRACTIONS

283

Solution. Prime factor the denominators in compact form using exponents. 28 = 2 2 7 = 22 7 42 = 2 3 7 = 21 31 71 To nd the LCD, write down each factor that appears to the highest power of that factor that appears. The factors that appear are 2, 3, and 7. The highest power of 2 that appears is 22 . The highest power of 3 that appears is 31 . The highest power of 7 that appears is 71 . LCM = 22 31 71 = 437 = 84 Keep highest power of each factor. Expand: 22 = 4, 31 = 3, 71 = 7. Multiply.

Create equivalent fractions with the new LCD, then add. 5 11 53 11 2 + = + 28 42 28 3 42 2 15 22 = + 84 84 37 = 84 Equivalent fractions with LCD=84. Simplify numerators and denominators. Keep LCD; add numerators. Answer: 25/72

You Try It! EXAMPLE 11. Simplify: 1 11 . 24 18 Simplify: 11 5 24 36

Solution. Prime factor the denominators in compact form using exponents. 24 = 2 2 2 3 = 23 31 18 = 2 3 3 = 21 32 To nd the LCD, write down each factor that appears to the highest power of that factor that appears. The factors that appear are 2 and 3. The highest power of 2 that appears is 23 . The highest power of 3 that appears is 32 . LCM = 23 32 = 89 = 72. Keep highest power of each factor. Expand: 23 = 8 and 32 = 9. Multiply. Version: Fall 2010

284

CHAPTER 4. FRACTIONS

Create equivalent fractions with the new LCD, then subtract. 11 1 11 3 14 = 24 18 24 3 18 4 33 4 = 72 72 33 4 = 72 37 = 72 Equivalent fractions with LCD=72. Simplify numerators and denominators. Keep LCD; subtract numerators. Simplify numerator.

Of course, negative divided by positive yields a negative answer, so we can also write our answer in the form Answer: 37/72 11 1 37 = . 24 18 72

Comparing Fractions
The simplest way to compare fractions is to create equivalent fractions. You Try It! Compare 3/8 and 1/2. EXAMPLE 12. Arrange the fractions 1/2 and 4/5 on a number line, then compare them by using the appropriate inequality symbol. Solution. The least common denominator for 2 and 5 is the number 10. First, make equivalent fractions with a LCD equal to 10. 15 5 1 = = 2 25 10 4 42 8 = = 5 52 10

To plot tenths, subdivide the interval between 1 and 0 into ten equal increments. 8 5 10 10 1

4 5

1 2

Answer:

1 3 < 2 8

Because 4/5 lies to the left of 1/2, we have that 4/5 is less than 1/2, so we write 1 4 < . 5 2 Version: Fall 2010

4.4. ADDING AND SUBTRACTING FRACTIONS

285

l l l

Exercises

l l l

In Exercises 1-10, list the multiples the given numbers, then list the common multiples. Select the LCM from the list of common mulitples. 1. 9 and 15 2. 15 and 20 3. 20 and 8 4. 15 and 6 5. 16 and 20 6. 6 and 10 7. 20 and 12 8. 12 and 8 9. 10 and 6 10. 10 and 12

In Exercises 11-20, for the given numbers, calculate the LCM using prime factorization. 11. 54 and 12 12. 108 and 24 13. 18 and 24 14. 36 and 54 15. 72 and 108 16. 108 and 72 17. 36 and 24 18. 18 and 12 19. 12 and 18 20. 12 and 54

In Exercises 21-32, add or subtract the fractions, as indicated, and simplify your result. 21. 22. 23. 24. 25. 26. 1 7 12 12 3 5 7 7 1 1 + 9 9 1 3 + 7 7 1 4 5 5 3 2 5 5 27. 28. 29. 30. 31. 32. 3 4 7 7 6 2 7 7 9 4 + 11 11 4 10 + 11 11 4 3 + 11 11 3 2 + 7 7

Version: Fall 2010

286

CHAPTER 4. FRACTIONS

In Exercises 33-56, add or subtract the fractions, as indicated, and simplify your result. 33. 34. 35. 36. 37. 38. 39. 40. 41. 42. 43. 44. 1 6 7 9 1 5 7 9 2 3 3 7 4 7 3 5 2 3 2 5 6 7 1 2 + + + + 1 8 2 3 2 3 2 3 5 8 5 9 5 9 7 8 3 8 1 8 1 6 1 4 45. 46. 47. 48. 49. 50. 51. 52. 53. 54. 55. 56. 1 6 4 9 7 9 1 6 1 3 5 6 1 2 1 3 5 6 1 2 1 3 1 6 + + + + + + + + 2 3 7 8 1 8 1 7 1 7 1 4 2 7 1 8 4 5 1 9 1 8 7 9

In Exercises 57-68, add or subtract the fractions, as indicated, by rst using prime factorization to nd the least common denominator. 57. 58. 59. 60. 61. 62. 7 36 7 54 7 18 5 54 7 36 5 72 + + + + 11 54 7 24 5 12 7 12 7 54 5 108 Version: Fall 2010 63. 64. 65. 66. 67. 68. 7 24 11 54 11 12 11 24 11 54 7 54 + + + 5 36 7 72 5 18 11 108 5 24 5 24

4.4. ADDING AND SUBTRACTING FRACTIONS

287

In Exercises 69-80, add or subtract the fractions, as indicated, and simplify your result. 3 3 69. + 7 7 1 5 70. + 9 9 71. 72. 73. 74. 7 1 9 9 4 8 9 9 2 7 + 9 9 1 2 + 3 3 3 75. 5 7 76. 9 7 77. + 8 2 78. + 3 4 5 1 9 1 8 1 3

1 2 79. 3 3 5 7 80. 8 8

In Exercises 81-104, add or subtract the fractions, as indicated, and simplify your result. 2 81. + 7 1 82. + 4 4 5 2 7 91. 92. 93. 94. 1 1 + 9 3 1 1 + 8 2 1 2 + 3 9 2 3 + 4 3

4 1 83. 4 9 1 3 84. 4 8 2 85. + 7 1 86. + 3 4 87. 9 5 88. 6 3 4 5 8 1 3 1 3

6 1 95. + 2 7 1 4 96. + 5 2 3 1 97. + 2 4 1 3 98. + 5 2 1 99. 4 8 100. 9 1 2 2 3 Version: Fall 2010

1 5 89. 7 5 1 6 90. 7 8

288 5 3 8 4 3 3 4 8

CHAPTER 4. FRACTIONS 1 1 8 3 4 1 2 9

101. 102.

103. 104.

In Exercises 105-120, add or subtract the fractions, as indicated, and write your answer is lowest terms. 105. 106. 107. 108. 109. 110. 111. 112. 1 2 4 7 4 9 4 9 2 s 2 s 1 3 1 2 + + + 3q 5 b 3 3a 4 b 2 1 3 3 7 7 b 9 s 113. 4b 2 + 7 3 2a 5 + 114. 5 8 2 9 115. 3 t 4 1 116. 7 y 9 7 + s 8 6 1 118. t 9 7b 5 119. 8 9 3p 1 120. 4 8 117.

In Exercises 121-132, determine which of the two given statements is true. 121. 122. 123. 124. 125. 126. 8 2 < 3 7 8 1 < 7 9 7 6 < or 7 3 2 1 < or 2 7 2 9 < 4 3 9 3 < 7 2 or or 6 7 1 2 or or 2 8 > 3 7 1 8 > 7 9 7 > 3 2 > 7 9 2 > 4 3 3 9 > 7 2 127. 5 5 5 5 < or > 7 9 7 9 1 1 1 1 or > 128. < 2 3 2 3 1 7 1 7 or > 129. < 2 5 2 5 5 3 5 3 or > 130. < 4 9 4 9 6 5 6 5 or > 131. < 9 5 9 5 7 3 7 3 or > 132. < 2 9 2 9

Version: Fall 2010

4.4. ADDING AND SUBTRACTING FRACTIONS

289

l l l
1. 45 3. 40

Answers
37. 39. 31 24 1 63 7 24 29 42 5 6 65 72 10 21 3 14 1 30 11 24

l l l

5. 80 7. 60 9. 30 11. 108 13. 72 15. 216 17. 72 19. 36 21. 23. 1 2 2 9 3 5 1 7 47. 49. 51. 53. 55. 57. 41. 43. 45.

25. 27. 29. 31. 33. 35.

43 108 1 36

59. 61. 63. 65.

13 11 7 11 1 24 13 15

35 108 11 72 43 36 1 216 Version: Fall 2010

67.

290 69. 71. 73. 8 9 5 9 7 5 3 4 6 7

CHAPTER 4. FRACTIONS 101. 103. 105. 107. 109. 111. 113. 115. 117. 119. 11 8 11 24 5 + 6q 10 16 27a 36 6+s 3s b 21 3b 12b + 14 21 2t 27 3t 72 + 7s 8s 63b 40 72 2 8 > 3 7

75. 77. 79. 81. 83. 85. 1 3

18 35 7 36 13 28 7 9 18 35 2 9

87. 89. 91. 93. 5 9

121. 123.

7 6 < 7 3 2 9 < 4 3

125. 19 14 5 4 3 4 127.

95. 97. 99.

5 5 > 7 9 1 7 < 2 5

129. 131.

6 5 < 9 5

Version: Fall 2010

4.5. MULTIPLYING AND DIVIDING MIXED FRACTIONS

291

4.5

Multiplying and Dividing Mixed Fractions

We begin with denitions of proper and improper fractions.

Proper and Improper Fractions. A proper fraction is a fraction whose numerator is smaller than its denominator. An improper fraction is a fraction whose numerator is larger than its denominator.

For example, 2 23 , , 3 39 and 119 127

are all examples of proper fractions. On the other hand, 317 4 , , 3 123 and 233 101

are all examples of improper fractions. A mixed fraction 1 is part whole number, part fraction.

Mixed Fractions. The number

3 4 is called a mixed fraction. It is dened to mean 5 5 3 3 =5+ . 4 4

In the mixed fraction 5 3 , the 5 is the whole number part and the 3/4 is the 4 fractional part.

Changing Mixed Fractions to Improper Fractions


We have all the tools required to change a mixed fraction into an improper fraction. We begin with an example. You Try It!
7 EXAMPLE 1. Change the mixed fraction 4 8 into an improper fraction.
1A

Change 5 3 to an improper 4 fraction.

mixed fractions is sometimes called a mixed number.

Version: Fall 2010

292

CHAPTER 4. FRACTIONS

Solution. We employ the denition of a mixed fraction, make an equivalent fraction for the whole number part, then add. 4 7 7 =4+ 8 8 48 7 = + 8 8 48+7 = 8 39 = 8 By denition. Equivalent fraction with LCD = 8. Add numerators over common denominator. Simplify the numerator.

Answer: 23/4

7 Thus, 4 8 is equal to 39/8.

There is a quick technique you can use to change a mixed fraction into an improper fraction. Quick Way to Change a Mixed Fraction to an Improper Fraction. To change a mixed fraction to an improper fraction, multiply the whole number part by the denominator, add the numerator, then place the result over the denominator.
7 Thus, to quickly change 4 8 to an improper fraction, multiply the whole number 4 by the denominator 8, add the numerator 7, then place the result over the denominator. In symbols, this would look like this:

48+7 7 = . 8 8

This is precisely what the third step in Example 1 looks like; were just eliminating a lot of the work. You Try It!
3 Change 7 8 to an improper fraction.

EXAMPLE 2. Change 4 2 to an improper fraction. 3 Solution. Take 4 2 , multiply the whole number part by the denominator, add 3 the numerator, then put the result over the denominator. 4 Thus, the result is 4 2 14 = . 3 3 43+2 2 = 3 3

Answer: 59/8 Version: Fall 2010

4.5. MULTIPLYING AND DIVIDING MIXED FRACTIONS

293

It is very easy to do the intermediate step in Example 2 mentally, allowing you to skip the intermediate step and go directly from the mixed fraction to the improper fraction without writing down a single bit of work. You Try It! EXAMPLE 3. Without writing down any work, use mental arithmetic to 3 change 2 5 to an improper fraction.
3 Solution. To change 2 5 to an improper fraction, ignore the minus sign, proceed as before, then prex the minus sign to the resulting improper fraction. So, multiply 5 times 2 and add 3. Put the result 13 over the denominator 5, then prex the resulting improper fraction with a minus sign. That is, 5 Change 3 12 to an improper fraction.

13 3 = . 5 5 Answer: 41/12

Changing Improper Fractions to Mixed Fractions


The rst step in changing the improper fraction 27/5 to a mixed fraction is to write the improper fraction as a sum. 27 25 2 = + 5 5 5 Simplifying equation 4.1, we get 27 2 =5+ 5 5 2 =5 . 5 Comment. You cant just choose any sum. The sum used in equation 4.1 is constructed so that the rst fraction will equal a whole number and the second fraction is proper. Any other sum will fail to produce the correct mixed fraction. For example, the sum 23 4 27 = + 5 5 5 is useless, because 23/5 is not a whole number. Likewise, the sum 27 20 7 = + 5 5 5 is no good. Even though 20/5 = 4 is a whole number, the second fraction 7/5 is still improper. Version: Fall 2010 (4.1)

294

CHAPTER 4. FRACTIONS

You Try It! Change 25/7 to a mixed fraction. EXAMPLE 4. Change 25/9 to a mixed fraction. Solution. Break 25/9 into the appropriate sum. 25 18 7 = + 9 9 9 7 =2+ 9 7 =2 9
4 Answer: 3 7

Comment. A pattern is emerging. In the case of 27/5, note that 27 divided by 5 is equal to 5 with a re2 mainder of 2. Compare this with the mixed fraction result: 27/5 = 5 5 . In the case of Example 4, note that 25 divided by 9 is 2 with a remainder of 7. Compare this with the mixed fraction result: 25/9 = 2 7 . 9 These observations motivate the following technique. Quick Way to Change an Improper Fraction to a Mixed Fraction. To change an improper fraction to a mixed fraction, divide the numerator by the denominator. The quotient will be the whole number part of the mixed fraction. If you place the remainder over the denominator, this will be the fractional part of the mixed fraction.

You Try It! Change 38/9 to a mixed fraction. EXAMPLE 5. Change 37/8 to a mixed fraction. Solution. 37 divided by 8 is 4, with a remainder of 5. That is: 4 8)37 32 5 The quotient becomes the whole number part and we put the remainder over the divisor. Thus, 37 5 =4 . 8 8 Version: Fall 2010

4.5. MULTIPLYING AND DIVIDING MIXED FRACTIONS

295

Note: You can check your result with the Quick Way to Change a Mixed Fraction to an Improper Fraction. 8 times 4 plus 5 is 37. Put this over 8 to get 37/8.

2 Answer: 4 9

You Try It! EXAMPLE 6. Change 43/5 to a mixed fraction. Solution. Ignore the minus sign and proceed in the same manner as in Example 5. 43 divided by 5 is 8, with a remainder of 3. 8 5)43 40 3 The quotient is the whole number part, then we put the remainder over the divisor. Finally, prex the minus sign. 3 43 = 8 . 5 5
3 Answer: 3 8

Change 27/8 to a mixed fraction.

Multiplying and Dividing Mixed Fractions


You have all the tools needed to multiply and divide mixed fractions. First, change the mixed fractions to improper fractions, then multiply or divide as you did in previous sections. You Try It! EXAMPLE 7. Simplify: 4 25 14 1 2 2 = 12 5 12 5 25 14 = 12 5 (5 5) (2 7) (2 2 3) (5) 5 2 57 = 2235 35 = 6 =
1 2 12 2 4 . 5

Simplify: 3 2 3 2 4 5

Solution. Change to improper fractions, factor, cancel, and simplify. Change to improper fractions. Multiply numerators; multiply denominators. Unlike signs; product is negative. Prime factor. Cancel common factors. Multiply numerators and denominators. Version: Fall 2010

296

CHAPTER 4. FRACTIONS

This is a perfectly good answer, but if you want a mixed fraction answer, 35 divided by 6 is 5, with a remainder of 5. Hence, 2 Answer: 9 1 4 5 2 = 5 . 12 5 6

You Try It! Simplify: 4 2 2 3 9 3 EXAMPLE 8. Simplify:


4 4 5 5 3 . 5

Solution. Change to improper fractions, invert and multiply, factor, cancel, and simplify. 3 24 28 4 4 5 = 5 5 5 5 24 5 = 5 28 2223 = 5 2 2 23 = 5 6 = 7 Change to improper fractions. Invert and multiply. 5 227 5 227 Prime factor. Cancel common factors. Multiply numerators and denominators.

Answer: 2/3

Version: Fall 2010

4.5. MULTIPLYING AND DIVIDING MIXED FRACTIONS

297

l l l

Exercises

l l l

In Exercises 1-12, convert the mixed fraction to an improper fraction. 1. 2 2. 3. 4. 5. 6. 1 3 8 1 11 1 1 19 1 1 5 3 1 7 3 1 17 7. 1 8. 9. 10. 11. 12. 1 9 5 1 11 1 1 2 5 1 8 1 1 3 5 1 7

In Exercises 13-24, convert the improper fraction to a mixed fraction. 13. 14. 15. 16. 17. 18. 13 7 17 9 13 5 10 3 16 5 16 13 19. 20. 21. 22. 23. 24. 9 8 16 5 6 5 17 10 3 2 7 4

In Exercises 25-48, multiply the numbers and express your answer as a mixed fraction. 1 1 25. 1 2 7 2 27. 4 1 1 6

1 1 26. 1 1 8 6

28. 1

7 4 10 Version: Fall 2010

298 1 3 1 12 1 2 3 4

CHAPTER 4. FRACTIONS 2 1 8

29. 30.

3 3

39.

(6) 1 6 2 3 4 1 9 2 1 5 9 7 11 7 12 1 2 2 5 3 4

1 3

40. (9) 3 41. 42. 4 1 2 1 2 3 7 1 6

1 1 31. 7 1 2 13 5 1 32. 2 1 4 11 33. 34. 35. 36. 1 1 1 1 2 13 1 14 3 7 4 5 4 2 3 3 2 15 5 6 3 4 3 4 2 3 2 5

43.

44. (6) 1 45. 46. 47. 48. 1 1 2 1 4 15 1 5 1 2 7 11

37. 9 1 38. 4 2

In Exercises 49-72, divide the mixed fractions and express your answer as a mixed fraction. 49. 8 2 2 9 56. 57. 1 1 1 16 1 15 58. 59. 60. 4 5 2 6 1 2 3 2 3 1 2 1 2 1 6 4 2 2 4 1 1 4 1 8 1 6 2 9

2 50. 4 4 3 51. 52. 3 1 1 2 2 5

7 1 53. 6 1 2 12 9 1 54. 5 1 2 10 55. (4) 1 5 9

61. (6) 1 62. 6 2 3

3 11

(6)

Version: Fall 2010

4.5. MULTIPLYING AND DIVIDING MIXED FRACTIONS 2 3 2 3 3 4 4 7 (4) (6) 1 1 1 12 1 5 2 2 68. 1 1 3 9 1 2 69. 7 2 2 5 70. 71. 72. 5 3 8 2 3 1 2 1 3 2 1 1 1 9 3 4 5 6

299

63. 64. 65. 66.

4 6 1 2

1 2 67. 5 1 3 9

73. Small Lots. How many quarter-acre lots can be made from 6 1 acres of land? 2
1 74. Big Field. A eld was formed from 17 2 half-acre lots. How many acres was the resulting eld ?

75. Jewelry. To make some jewelry, a bar of 1 1 silver 4 2 inches long was cut into pieces 12 inch long. How many pieces were made? 76. Muns. This recipe will make 6 muns: 2 1 cup milk, 1 3 cups our, 2 eggs, 1/2 tea1 spoon salt, 1 2 teaspoons baking powder. Write the recipe for six dozen muns.

l l l
1. 3. 7 3 20 19 10 7

Answers
15. 2 17. 3 19. 1 1 8

l l l
3 5 1 5

5. 7.

10 9 3 2

21. 1 23. 1 25. 2 27. 4 6 7 2 3

1 5 1 2

9. 4 11. 3

6 13. 1 7

Version: Fall 2010

300 29. 4 31. 8 1 16

CHAPTER 4. FRACTIONS 53. 4 2 19 4 7

1 13 5 13 5 14 1 5

55. 2 57. 2

33. 5 35. 5

8 13 9 17

59. 1 61. 4 5 7

37. 10 39. 12 41. 10 3 4 4 5 2 3 1 6

63. 1 65. 1 67. 5

1 6 8 13

43. 8 45. 3 47. 4 49. 3

1 10 1 8 3 10

69. 3

1 11 3 5 5 17

71. 3

73. 26 quarter-acre lots 75. 54 pieces

51. 3

Version: Fall 2010

4.6. ADDING AND SUBTRACTING MIXED FRACTIONS

301

4.6

Adding and Subtracting Mixed Fractions

In this section, we will learn how to add and subtract mixed fractions.

Adding Mixed Fractions


We can use tools weve already developed to add two or more mixed fractions. You Try It! EXAMPLE 1. Simplify: 27 + 13. 8 4 Simplify: 1 2 3 +4 3 8

Solution. Change the mixed fractions to improper fractions, make equivalent fractions with a common denominator, then add. 2 3 23 7 7 +1 = + 8 4 8 4 23 7 2 = + 8 42 23 14 = + 8 8 37 = 8 Change to improper fractions. Equivalent fractions with LCD = 8. Simplify numerators and denominators. Add numerators over common denominator.

Although this answer is perfectly acceptable, lets change the answer to a mixed fraction: 37 divided by 8 is 4, with a remainder of 5. Thus, 2 3 5 7 +1 =4 . 8 4 8
19 Answer: 7 24

You Try It! EXAMPLE 2. Simplify: 31 + 21. 4 3 Simplify: 2 1 8 +2 2 3

Solution. Change the mixed fractions to improper fractions, make equivalent fractions with a common denominator, then add. 3 1 13 7 1 +2 = + 4 3 4 3 13 3 7 4 = + 43 34 39 28 = + 12 12 67 = 12 Change to improper fractions. Equivalent fractions with LCD = 12. Simplify numerators and denominators. Add numerators over common denominator.

Version: Fall 2010

302

CHAPTER 4. FRACTIONS

Although this answer is perfectly acceptable, lets change the answer to a mixed fraction: 67 divided by 12 is 5, with a remainder of 7. Thus, 3
1 Answer: 11 6

1 1 7 +2 =5 . 4 3 12

Mixed Fraction Approach. There is another possible approach, based on the fact that a mixed fraction is a sum. Lets revisit Example 2. You Try It! Simplify: 1 2 7 +3 5 8 EXAMPLE 3. Simplify:
1 1 34 + 23.

Solution. Use the commutative and associative properties to change the order of addition, make equivalent fractions with a common denominator, then add. 3 1 1 +2 = 4 3 3+ 1 4 + 2+ 1 3 Mixed fractions as sums. Reorder and regroup. Add whole numbers: 3 + 2 = 5. Equivalent fractions; LCD = 12. Simplify numerators and denominators. Add numerators over common denominator.

= (3 + 2) + =5+

1 1 + 4 3 13 14 + 43 34

=5+

4 3 + 12 12 7 =5+ 12

This result can be written in mixed fraction form. Thus, 3


21 Answer: 10 40

1 1 7 +2 =5 . 4 3 12

Note that this solution is identical to the result found in Example 2.

Example 3 leads us to the following result.

Adding Mixed Fractions. To add two mixed fractions, add the whole number parts, then add the fractional parts.

Version: Fall 2010

4.6. ADDING AND SUBTRACTING MIXED FRACTIONS

303

Working in Vertical Format


When adding mixed fractions, many prefer to work in a vertical format. For example, here is how we would arrange the solution from Example 2 and Example 3 in vertical format. We create equivalent fractions, then add the whole number parts and fractional parts. 3 +2 1 4 1 3 = = 3 +2 13 43 14 34 = = 3 +2 3 12 4 12 7 12

Note that the answer is identical to the answer found in Example 2 and Example 3. That is, 1 7 1 3 +2 =5 . 4 3 12 You Try It! EXAMPLE 4. Sarah is making window curtains for two rooms in her house. The kitchen will require 5 2 yards of material and the dining room will require 3 6 5 yards of material. How much total material is required? 8 Solution. To nd the total material required for the two rooms, we must add 5 5 2 and 6 8 . Create equivalent fractions with a common denominator, then add 3 whole number parts and fractional parts. 5 +6 2 3 5 8 = = 5 +6 28 38 53 83 = = 5 +6 16 24 15 24 31 24 Jim is working on a project that requires two boards, the 1 rst cut to a length of 6 2 feet, the second cut to a 7 length of 5 8 feet. How many total feet of board is required?

11

An answer that is part mixed fraction, part improper fraction, is not allowed. To nish, we need to change the improper fractional part to a mixed fraction, 7 then add. 31 divided by 24 is 1, with a remainder of 7. That is, 31/24 = 1 24 . Now we can add whole number parts and fractional parts. 7 31 = 11 + 1 11 24 24 7 = 12 . 24 7 Thus, the total material required is 12 24 yards. Version: Fall 2010

3 Answer: 12 8 feet

304

CHAPTER 4. FRACTIONS

Subtracting Mixed Fractions


Lets look at some examples that subtract two mixed fractions. You Try It! Simplify: 1 2 5 3 3 5 EXAMPLE 5. Simplify:
5 1 4 8 2 16 .

Solution. Change the mixed fractions to improper fractions, make equivalent fractions with a common denominator, then subtract. 4 5 1 37 33 2 = 8 16 8 16 37 2 33 = 82 16 74 33 = 16 16 41 = 16 Change to improper fractions. Equivalent fractions with LCD = 16. Simplify numerators and denominators. Add numerators over common denominator.

Although this answer is perfectly acceptable, lets change the answer to a mixed fraction: 41 divided by 16 is 2, with a remainder of 9. Thus, 4
7 Answer: 2 15

5 1 9 2 =2 . 8 16 16

You Try It! Simplify: 3 7 4 2 9 18 EXAMPLE 6. Simplify:


3 1 54 23.

Solution. Change the mixed fractions to improper fractions, make equivalent fractions with a common denominator, then subtract. 5 1 23 7 3 2 = 4 3 4 3 23 3 7 4 = 43 34 69 28 = 12 12 41 = 12 Change to improper fractions. Equivalent fractions with LCD = 12. Simplify numerators and denominators. Add numerators over common denominator.

Although this answer is perfectly acceptable, lets change the answer to a mixed fraction: 41 divided by 12 is 3, with a remainder of 5. Thus, 5
11 Answer: 2 18

3 1 5 2 =3 . 4 3 12

Version: Fall 2010

4.6. ADDING AND SUBTRACTING MIXED FRACTIONS

305

Mixed Fraction Approach. There is another possible approach, based on the fact that a mixed fraction is a sum. Lets revisit Example 6. You Try It! EXAMPLE 7. Simplify: 53 21. 4 3 Simplify: 3 5 8 4 6 8

Solution. A mixed fraction is a sum. 5 1 3 2 = 4 3 5+ 3 4 2+ 1 3

Distribute the negative sign. = 5+ 3 1 2 4 3

We could change the subtraction to adding the opposite, change the order of addition, then change the adding of opposites back to subtraction. However, it is much easier if we look at this last line as a request to add four numbers, two of which are positive and two of which are negative. Changing the order does not aect the answer. = (5 2) + 3 1 4 3

Note that we did not change the signs of any of the four numbers. We just changed the order. Subtract the whole number parts. Make equivalent fractions with a common denominator, then subtract the fractional parts. 33 14 43 34 9 4 = 3+ 12 12 5 = 3+ 12 = 3+ Thus, 5 3 1 5 2 =3 . 4 3 12
11 Answer: 4 24

Create equivalent fractions. Simplify numerators and denominators. Subtract fractional parts.

Note that this is exactly the same answer as that found in Example 6.

In Example 6, we see that we handle subtraction of mixed fractions in exactly the same manner that we handle addition of mixed fractions.

Version: Fall 2010

306

CHAPTER 4. FRACTIONS

Subtracting Mixed Fractions. To subtract two mixed fractions, subtract their whole number parts, then subtract their fractional parts.

Working in Vertical Format


When subtracting mixed fractions, many prefer to work in a vertical format. For example, here is how we would arrange the solution from Example 6 and Example 7 in vertical format. We create equivalent fractions, then subtract the whole number parts and fractional parts. 3 4 1 3 33 43 14 34 9 12 4 12 5 12

5 2

= =

5 2

= =

5 2 3

Note that the answer is identical to the answer found in Example 6 and Example 7. That is, 3 1 5 5 2 =3 . 4 3 12

Borrowing in Vertical Format


Consider the following example. You Try It! Simplify: 7 5 1 2 14 21 EXAMPLE 8. Simplify:
1 5 84 56.

Solution. Create equivalent fractions with a common denominator. 1 4 5 6 13 43 52 62 3 12 10 12

8 5

= =

8 5

= =

8 5

You can see the diculty. On the far right, we cannot subtract 10/12 from 3/12. The x is to borrow 1 from 8 in the form of 12/12 and add it to the 3/12. Version: Fall 2010

4.6. ADDING AND SUBTRACTING MIXED FRACTIONS

307

8 5

3 12 10 12

= =

7+

12 3 + 12 12 5 10 12

= =

7 5 2

15 12 10 12 5 12
5 Answer: 4 6

1 5 5 Now we can subtract. Hence, 8 4 5 6 = 2 12 .

You Try It! EXAMPLE 9. Jim has a metal rod of length 10 inches. He cuts a length 7 from the metal rod measuring 2 8 inches. What is the length of the remaining piece?
7 Solution. To nd the length of the remaining piece, we must subtract 2 8 from 10. There is no fractional part on the rst number. To remedy this absence, we borrow 1 from 10 in the form of 8/8. Then we can subtract.

Sarah has a length of curtain material that measures 12 2 feet. She cuts a length of 6 3 feet from her curtain material. What is the length of the remaining piece?

10 2 7 8

= =

9+ 2

8 8 7 8

= =

9 2 7

8 8 7 8 1 8
1 Answer: 5 3 feet

Hence, the length of the remaining piece of the metal rod is 7 1 inches. 8

Version: Fall 2010

308

CHAPTER 4. FRACTIONS

l l l

Exercises

l l l

In Exercises 1-24, add or subtract the mixed fractions, as indicated, by rst converting each mixed fraction to an improper fraction. Express your answer as a mixed fraction. 1. 9 2. 3. 4. 5. 6. 7. 8. 9. 10. 11. 12. 1 1 +9 4 2 1 1 2 +9 3 2 1 1 6 1 2 3 3 1 5 1 3 4 1 1 9 +7 2 4 3 1 1 +9 3 4 1 2 5 +4 3 2 3 9 +2 1 16 4 1 1 3 1 3 4 1 1 2 1 2 4 1 1 8 1 2 3 2 1 5 1 2 3 13. 4 14. 15. 16. 17. 18. 19. 20. 21. 22. 23. 24. 1 2 1 2 2 7 4 8 1 1 8 1 2 3 1 5 3 1 9 2 1 5 2 2 4 3 1 1 4 1 9 2 1 1 4 1 8 1 1 3 3 +1 4 1 +5 2 1 1 4 1 1 4 3 1 4 3 1 16 1 +1 4 1 +1 3 1 +3 8 2 +1 3 1

In Exercises 25-48, add or subtract the mixed fractions, as indicated, by using vertical format. Express your answer as a mixed fraction. 25. 3 3 1 +3 2 4 1 2 +2 2 3 3 1 +1 8 4 Version: Fall 2010 28. 2 2 1 +1 4 3 7 1 +1 8 2 3 1 +4 4 2

26. 1

29. 1

27. 1

30. 1

4.6. ADDING AND SUBTRACTING MIXED FRACTIONS 31. 8 32. 33. 34. 35. 36. 37. 38. 39. 1 2 1 8 2 1 7 2 1 5 2 1 9 2 1 2 2 1 5 3 1 4 4 1 9 2 2 3 2 1 3 3 1 16 1 1 3 1 1 3 3 1 16 1 2 2 1 1 2 2 2 3 5 40. 7 41. 42. 43. 44. 45. 46. 47. 48. 2 1 4 2 3 3 1 +1 1 16 4 1 1 1 +1 4 3 2 1 8 +3 2 3 1 2 1 +2 3 2 3 1 6 1 2 16 1 1 4 1 2 3 1 2 2 +1 3 4 1 1 1 +1 2 16

309

l l l
1. 18 3. 5 1 6 3 4 1 6 3 4

Answers
17. 1 19. 7 21. 5

l l l
1 12 3 4 11 12 5 8

5. 16 7. 10 9. 2 11. 7

23. 12 25. 7 27. 2 29. 3 31. 2 1 4 5 8 3 8 5 6

1 12 1 6

3 13. 3 8 5 15. 6 8

Version: Fall 2010

310 5 16 1 6 5 6 5 6

CHAPTER 4. FRACTIONS 41. 2 13 16 1 6

33. 6 35. 8 37. 2 39. 6

43. 12 45. 5 47. 3

5 16 11 12

Version: Fall 2010

4.7. ORDER OF OPERATIONS WITH FRACTIONS

311

4.7

Order of Operations with Fractions

Lets begin by taking powers of fractions. Recall that am = a a . . . a


m times

You Try It! EXAMPLE 1. Simplify: Solution. By denition, 3 4


2

(3/4)2 .

Simplify: 2 5
2

3 4 33 = 44 = = 9 16

3 4

Fact: a2 = a a. Multiply numerators and denominators. Product of even number of negative factors is positive. Simplify.

Answer: 4/25

You Try It! EXAMPLE 2. Simplify: Solution. By denition, 2 3


3

(2/3)3 .

Simplify: 1 6
3

2 2 3 3 222 = 333 = 8 27

2 3

Fact: a3 = a a a. Multiply numerators and denominators. Product of odd number of negative factors is negative. Simplify.

Answer: 1/216

Version: Fall 2010

312

CHAPTER 4. FRACTIONS

The last two examples reiterate a principle learned earlier. Odd and Even. The product of an even number of negative factors is positive. The product of an odd number of negative factors is negative.

Order of Operations
For convenience, we repeat here the rules guiding order of operations. Rules Guiding Order of Operations. When evaluating expressions, proceed in the following order. 1. Evaluate expressions contained in grouping symbols rst. If grouping symbols are nested, evaluate the expression in the innermost pair of grouping symbols rst. 2. Evaluate all exponents that appear in the expression. 3. Perform all multiplications and divisions in the order that they appear in the expression, moving left to right. 4. Perform all additions and subtractions in the order that they appear in the expression, moving left to right.

You Try It! Simplify: 2 3 + 3 4 1 2 1 1 + 2 4 Solution. Multiply rst, then add. EXAMPLE 3. Simplify: 1 1 + 2 4 1 3 1 1 = + 2 12 16 1 = + 26 12 1 6 = + 12 12 7 = 12 1 . 3 1 4 1 3 1 . 12

Multiply:

Equivalent fractions, LCD = 12. Simplify numerator and denominator. Add over common denominator.

Answer: 25/24

Version: Fall 2010

4.7. ORDER OF OPERATIONS WITH FRACTIONS

313

You Try It! EXAMPLE 4. Simplify: 2 1 2


2

+4

1 . 2

Simplify: 3 1 3
2

Solution. Exponents rst, then multiply, then add.


2 2

1 3

1 2

+4

1 2

=2 =

1 4

+4

1 2

Exponent rst:

1 2 + 2 1

1 2 1 1 Multiply: 2 = 4 2 1 2 and 4 = . 2 1

1 . 4

1 22 + 2 12 4 1 = + 2 2 3 = 2 =

Equivalent fractions, LCD = 2. Simplify numerator and denominator. Add over common denominator.

Answer: 1

You Try It! EXAMPLE 5. Given a = 3/4, b = 1/2, c = 1/3, and d = 1/4, evaluate the expression ab cd. Given a = 1/2, b = 1/3, and c = 1/5, evaluate a + bc.

Solution. Recall that it is good practice to prepare parentheses before substituting. ad bc = ( )( )( )( )

Substitute the given values into the algebraic expression, then simplify using Version: Fall 2010

314 order of operations. ab cd = 3 4 1 2 1 3 1 4

CHAPTER 4. FRACTIONS

Substitute: 3/4 for a, 1/2 for b, 1/3 for c, and 1/4 for d. Multiply rst: and 1 3 1 4 3 4 1 2 1 . 12 = 3 8

1 3 = 8 12

3 1 = + 8 12 33 12 = + 8 3 12 2 9 2 = + 24 24 7 = 24 Answer: 17/30

Subtract by adding opposite. Equivalent fractions; LCD = 24. Simplify numerators and denominators. Add over common denominator.

You Try It! Give a = 1/2 and b = 1/3, evaluate ab (a + b). EXAMPLE 6. Given a = 1/4 and b = 1/2, evaluate (a2 b2 ) (a + b). Solution. Recall that it is good practice to prepare parentheses before substituting. (a2 b2 ) (a + b) = ( )2 ( )2 ( )+( )

Substitute the given values into the algebraic expression, then evaluate exponents rst. (a2 b2 ) (a + b) = = 1 4
2

1 2

1 4

1 2

1 1 16 4

1 1 + 4 2

We must evaluate parentheses rst. Inside each set of parentheses, create equivalent fractions and perform subtractions and additions next. 1 14 1 12 + 16 4 4 4 22 4 1 1 2 + = 16 16 4 4 1 3 = 16 4 = Version: Fall 2010

4.7. ORDER OF OPERATIONS WITH FRACTIONS Invert and multiply. = 3 4 16 1 12 = 16

315

Reduce. 12 4 16 4 3 = 4 = Note: In the last step, you could also reduce by prime factoring numerator and denominator and canceling common factors. Answer: 1/5

Complex Fractions

Complex Fractions. When the numerator and denominator of a fraction contain fractions themselves, such an expression is called a complex fraction.

You can use the standard order of operations to simplify a complex fraction. Recall the advice when a fraction is present.

Fractional Expressions. If a fractional expression is present, simplify the numerator and denominator separately, then divide.

You Try It! EXAMPLE 7. Simplify: 1 1 + 2 3 3 3 4 2 Version: Fall 2010 Simplify: 1 1 4 3 1 1 + 4 3

316

CHAPTER 4. FRACTIONS

Solution. We have addition in the numerator, subtraction in the denominator. In each case, we need equivalent fractions with a common denominator. 1 1 13 12 + + 2 3 23 32 = 3 3 3 32 4 2 4 22 3 2 + 6 6 = 3 6 4 4 1 6 = 3 4

Create equivalent fractions.

Simplify numerator and denominator.

1 3 2 Numerator: + = . 6 6 6 3 3 6 Denominator: = . 4 4 4

The last expression asks us to divide. Invert and multiply. 3 1 = 6 4 4 1 = 6 3 A complex fraction means divide. Invert and multiply.

Like signs (two negatives) give a positive product. Multiply numerators and denominators, then reduce. = = 4 18 Like signs yields positive answer. Multiply numerators and denominators. Divide both numerator and denominator by 2. Simplify.

42 18 2 2 = 9

Alternatively, one could prime factor and cancel to reduce to lowest terms; that is, 22 4 = 18 233 22 = 2 33 2 = 9 Answer: 1/7 Version: Fall 2010 Prime factor. Cancel common factors. Simplify.

4.7. ORDER OF OPERATIONS WITH FRACTIONS

317

Clearing Fractions. An alternate technique for simplifying complex fractions is available. Clearing Fractions from Complex Fractions. You can clear fractions from a complex fraction using the following algorithm: 1. Determine an LCD1 for the numerator. 2. Determine an LCD2 for the denominator. 3. Determine an LCD for both LCD1 and LCD2 . 4. Multiply both numerator and denominator by this combined LCD.

Lets apply this technique to the complex fraction of Example 7. You Try It! EXAMPLE 8. Simplify: 1 1 + 2 3 3 3 4 2 Solution. As we saw in the solution in Example 7, common denominators of 6 and 4 were used for the numerator and denominator, respectively. Thus, a common denominator for both numerator and denominator would be 12. We begin the alternate solution technique by multiplying both numerator and denominator by 12. 1 1 1 1 12 + + 2 3 2 3 = 3 3 3 3 12 4 2 4 2 1 1 12 + 12 2 3 = 3 3 12 12 4 2 6 + 4 = 9 18 = 2 9 2 = 9 Simplify: 2 1 + 3 5 4 1 5 2

Multiply numerator and denominator by 12.

Distribute the 12.

Multiply: 12(1/2) = 6, 12(1/2) = 4. 12(3/4) = 9, and 12(3/2) = 18. Simplify. Like signs yields positive. Version: Fall 2010 Answer: 14/9

318

CHAPTER 4. FRACTIONS

Application Trapezoid
A trapezoid is a special type of quadrilateral (four-sided polygon). Trapezoid. A quadrilateral with one pair of parallel opposite sides is called a trapezoid. b2 h b1 The pair of parallel sides are called the bases of the trapezoid. Their lengths are marked by the variables b1 and b2 in the gure above. The distance between the parallel bases is called the height or altitude of the trapezoid. The height is marked by the variable h in the gure above. Mathematicians use subscripts to create new variables. Thus, b1 (b sub 1) and b2 (b sub 2) are two distinct variables, used in this case to represent the length of the bases of the trapezoid. By drawing in a diagonal, we can divide the trapezoid into two triangles (see Figure 4.14). b2 h b1 Figure 4.14: Dividing the trapezoid into two triangles. We can nd the area of the trapezoid by summing the areas of the two triangles. The shaded triangle in Figure 4.14 has base b1 and height h. Hence, the area of the shaded triangle is (1/2)b1 h. The unshaded triangle in Figure 4.14 has base b2 and height h. Hence, the area of the unshaded triangle is (1/2)b2 h. Summing the areas, the area of the trapezoid is Area of Trapezoid = 1 1 b1 h + b2 h. 2 2

We can use the distributive property to factor out a (1/2)h.

Version: Fall 2010

4.7. ORDER OF OPERATIONS WITH FRACTIONS

319

Area of a Trapezoid. A trapezoid with bases b1 and b2 and height h has area 1 A = h (b1 + b2 ) . 2 That is, to nd the area, sum the bases, multiply by the height, and take one-half of the result.

You Try It! EXAMPLE 9. Find the area of the trapezoid pictured below.
1 22

in 3 in

A trapezoid has bases measuring 6 and 15 feet, respectively. The height of the trapezoid is 5 feet. Find the area of the trapezoid.

1 4 4 in

Solution. The formula for the area of a trapezoid is A= 1 h (b1 + b2 ) 2

Substituting the given bases and height, we get A= 1 1 1 (3) 4 + 2 2 4 2 .

Simplify the expression inside the parentheses rst. Change mixed fractions to improper fractions, make equivalent fractions with a common denominator, then add. A= 17 5 1 (3) + 2 4 2 17 5 2 1 + = (3) 2 4 22 17 10 1 + = (3) 2 4 4 27 1 3 = 2 1 4

Multiply numerators and denominators. = 81 8

Version: Fall 2010

320

CHAPTER 4. FRACTIONS

This improper fraction is a perfectly good answer, but lets change this result to a mixed fraction (81 divided by 8 is 10 with a remainder of 1). Thus, the area of the trapezoid is A = 10
1 Answer: 52 2 square feet

1 square inches. 8

Version: Fall 2010

4.7. ORDER OF OPERATIONS WITH FRACTIONS

321

l l l
In Exercises 1-8, simplify the expression. 1. 2. 3. 4. 1 2 5 3 3 5 7 3
3 3

Exercises

l l l

5. 6.
4

1 2 3 4 4 3 8 5

7.
4

8.

9. If a = 7/6, evaluate a3 . 10. If e = 1/6, evaluate e3 . 11. If e = 2/3, evaluate e2 . 12. If c = 1/5, evaluate c2 .

13. If b = 5/9, evaluate b2 . 14. If c = 5/7, evaluate c2 . 15. If b = 1/2, evaluate b3 . 16. If a = 2/9, evaluate a3 .

In Exercises 17-36, simplify the expression. 17. 18. 19. 20. 21. 22. 3 2 1 2 1 2 3 4 9 8
2 2

1 6 1 2

3 2

7 8 3 5 7 3 1 2 1 2

1 4

7 9

4 8 8 24. 9 5 9 25. 26. 27. 3 9 + 4 7 3 1 + 2 4 1 3 2 9


2

+
2

7 6 9 8 7 8 2 3 1 2 1 3

7 8 7 4

7 4

28. 29.

1 9 5 4 7 1 7 23. 6 7 9

5 5 7 + 9 9 9 1 9 1 30. + 2 8 3 Version: Fall 2010

322 7 4 4 2 3 9 5 6 6 5 3 8 + + 3 4 2 5 7 9 8 3 3 4

CHAPTER 4. FRACTIONS 1 1 34. 3 5 35. 36. 1 4 5 9 1 6 1 2 4 3 + + 5 6 1 6


2 2

31. 32. 33.

37. Given a = 5/4, b = 1/2, and c = 3/8, evaluate a + bc. 38. Given a = 3/5, b = 1/5, and c = 1/3, evaluate a + bc. 39. Given x = 1/8, y = 5/2, and z = 1/2, evaluate the expression x + yz. 40. Given x = 5/9, y = 1/4, and z = 2/3, evaluate the expression x + yz. 41. Given a = 3/4, b = 5/7, and c = 1/2, evaluate the expression a bc. 42. Given a = 5/9, b = 2/3, and c = 2/9, evaluate the expression a bc. 43. Given x = 3/2, y = 1/4, and z = 5/7, evaluate x2 yz. 44. Given x = 3/2, y = 1/2, and z = 5/3, evaluate x2 yz. 45. Given a = 6/7, b = 2/3, c = 8/9, and d = 6/7, evaluate ab + cd. 46. Given a = 4/9, b = 3/2, c = 7/3, and d = 8/9, evaluate ab + cd. 47. Given w = 1/8, x = 2/7, y = 1/2, and z = 8/7, evaluate wx yz. 48. Given w = 2/7, x = 9/4, y = 3/4, and z = 9/2, evaluate wx yz. 49. Given x = 3/8, y = 3/5, and z = 3/2, evaluate xy + z 2 . 50. Given x = 1/2, y = 7/5, and z = 3/2, evaluate xy + z 2 . 51. Given u = 9/7, v = 2/3, and w = 3/7, evaluate uv w2 . 52. Given u = 8/7, v = 4/3, and w = 2/3, evaluate uv w2 . 53. Given a = 7/8, b = 1/4, and c = 3/2, evaluate a2 + bc. 54. Given a = 5/8, b = 3/2, and c = 3/2, evaluate a2 + bc. 55. Given u = 1/3, v = 5/2, and w = 2/9, evaluate the expression u vw. 56. Given u = 1/2, v = 1/4, and w = 1/4, evaluate the expression u vw.

Version: Fall 2010

4.7. ORDER OF OPERATIONS WITH FRACTIONS In Exercises 57-68, simplify the complex rational expression. 57. 8 7 + 3 6 9 1 2 4 7 1 + 8 9 8 1 9 6 3 4 + 4 3 1 5 + 9 3 9 6 8 5 7 1 + 4 2 7 5 + 5 2 1 1 + 4 2 5 2 + 6 3 3 2 + 5 3 63. 3 2 7 4 2 3 2 3

323

58.

64.

8 3 + 9 4 2 1 3 6 1 2 5 + 7 4 7 1 6

59.

65.

60.

66.

3 5 2 8 3 1 4 2 3 1 7 3 1 6 3 7 5 8 5 4 6 5 3 8

61.

67.

62.

68.

69. A trapezoid has bases measuring 3 3 and 8 5 1 feet, respectively. The height of the 2 trapezoid is 7 feet. Find the area of the trapezoid. 70. A trapezoid has bases measuring 2 1 and 2 6 7 feet, respectively. The height of the 8 trapezoid is 3 feet. Find the area of the trapezoid.

1 71. A trapezoid has bases measuring 2 4 and 3 7 8 feet, respectively. The height of the trapezoid is 7 feet. Find the area of the trapezoid. 1 72. A trapezoid has bases measuring 3 8 and 1 6 2 feet, respectively. The height of the trapezoid is 3 feet. Find the area of the trapezoid.

Version: Fall 2010

324
3 73. A trapezoid has bases measuring 2 4 and 5 6 8 feet, respectively. The height of the trapezoid is 3 feet. Find the area of the trapezoid.

CHAPTER 4. FRACTIONS 74. A trapezoid has bases measuring 2 1 and 4 7 1 feet, respectively. The height of the 8 trapezoid is 5 feet. Find the area of the trapezoid.

l l l
1. 3. 5. 7. 9. 343 27

Answers
29. 31. 33. 80 81 13 48 3 2 1 4

l l l

625 81 1 32 16 9 343 216 4 9

35. 37. 39. 41. 43. 45. 47. 49.

17 16 11 8

11. 13. 15. 17. 19. 21.

25 81 1 8 43 72 305 64 5 8 23 18 3 4 13 72 Version: Fall 2010

11 28 17 7 4 3 17 28 99 40 33 49 73 64 8 9

23. 25. 27.

51. 53. 55.

4.7. ORDER OF OPERATIONS WITH FRACTIONS 57. 59. 61. 63. 65. 46 57 67. 16 11 1 16 11 16 1 16

325

75 64 78 5 26 29 45 23

69. 31

71. 33

73. 14

Version: Fall 2010

326

CHAPTER 4. FRACTIONS

4.8

Solving Equations with Fractions

Undoing Subtraction
We can still add the same amount to both sides of an equation without changing the solution. You Try It! Solver for x: x 2 1 = 3 5 5 = 6 Solution. To undo subtracting 5/6, and simplify. EXAMPLE 1. Solve for x: x 1 5 = 6 3 5 5 1 5 x + = + 6 6 3 6 12 5 x= + 32 6 2 5 x= + 6 6 7 x= 6 x 1 . 3 add 5/6 to both sides of the equation

Original equation. Add 5 to both sides. 6

Equivalent fractions, LCD = 6. Simplify. Add.

It is perfectly acceptable to leave your answer as an improper fraction. If you desire, or if you are instructed to do so, you can change your answer to a mixed fraction (7 divided by 6 is 1 with a remainder of 1). That is x = 1 1 . 6 Checking the Solution. Substitute 7/6 for x in the original equation and simplify. 5 6 7 5 6 6 2 6 1 3 x Answer: 13/15 1 3 1 = 3 1 = 3 1 = 3 = Original equation. Substitute 7/6 for x. Subtract. Reduce.

Because the last statement is true, we conclude that 7/6 is a solution of the equation x 5/6 = 1/3.

Undoing Addition
You can still subtract the same amount from both sides of an equation without changing the solution. Version: Fall 2010

4.8. SOLVING EQUATIONS WITH FRACTIONS

327

You Try It! EXAMPLE 2. Solve for x: 3 2 = . 3 5 Solution. To undo adding 2/3, subtract 2/3 from both sides of the equation and simplify. x+ 3 2 = 3 5 2 2 3 2 x+ = 3 3 5 3 33 25 x= 53 35 9 10 x= 15 15 19 x= 15 x+ Original equation. Subtract 2 from both sides. 3 Solve for x: x+ 3 1 = 4 2

Equivalent fractions, LCD = 15. Simplify. Subtract. Answer: 5/4

Readers are encouraged to check this solution in the original equation.

Undoing Multiplication
We undo multiplication by dividing. For example, to solve the equation 2x = 6, we would divide both sides of the equation by 2. In similar fashion, we could divide both sides of the equation 4 3 x= 5 10 by 3/5. However, it is more ecient to take advantage of reciprocals. For convenience, we remind readers of the Multiplicative Inverse Property. Multiplicative Inverse Property. Let a/b be any fraction. The number b/a is called the multiplicative inverse or reciprocal of a/b. The product of reciprocals is 1. a b = 1. b a

Lets put our knowledge of reciprocals to work. You Try It! EXAMPLE 3. Solve for x: 3 4 x= . 5 10 Version: Fall 2010 Solve for y: 2 4 y= 3 5

328

CHAPTER 4. FRACTIONS

Solution. To undo multiplying by 3/5, multiply both sides by the reciprocal 5/3 and simplify. 3 4 x= 5 10 3 5 4 x = 5 3 10 3 20 x= 5 30 1x = 2 3 2 3

Original equation. Multiply both sides by 5/3. On the left, use the associative property to regroup. On the right, multiply. 5 3 On the left, = 1. 3 5 2 20 = . On the right, reduce: 30 3 On the left, 1x = x.

5 3 5 3

x=

Checking the Solution. Substitute 2/3 for x in the original equation and simplify. 3 x= 5 2 = 3 6 = 15 2 = 5 4 10 4 10 4 10 2 5

Original equation. Substitute 2/3 for x. Multiply numerators; multiply denominators. Reduce both sides to lowest terms.

3 5

Answer: 6/5

Because this last statement is true, we conclude that 2/3 is a solution of the equation (3/5)x = 4/10.

You Try It! Solve for z: 2 4 z= 7 21 EXAMPLE 4. Solve for x: 5 8 . x= 9 18

Solution. To undo multiplying by 8/9, multiply both sides by the recipVersion: Fall 2010

4.8. SOLVING EQUATIONS WITH FRACTIONS rocal 9/8 and simplify. 8 5 x= 9 18 9 9 5 8 x = 8 9 8 18 8 5 33 9 x= 8 9 222 233 1x = 5 33 222 2 3 3 5 16 Original equation. Multiply both sides by 9/8.

329

On the left, use the associative property to regroup. On the right, prime factor. 9 8 On the left, = 1. 8 9 On the right, cancel common factors. On the left, 1x = x. Multiply on right. Answer: 2/3

x=

Readers are encouraged to check this solution in the original equation.

Clearing Fractions from the Equation


Although the technique demonstrated in the previous examples is a solid mathematical technique, working with fractions in an equation is not always the most ecient use of your time.

Clearing Fractions from the Equation. To clear all fractions from an equation, multiply both sides of the equation by the least common denominator of the fractions that appear in the equation.

Lets put this idea to work. You Try It! EXAMPLE 5. In Example 1, we were asked to solve the following equation for x: 1 5 x = . 6 3 Take a moment to review the solution technique in Example 1. We will now solve this equation by rst clearing all fractions from the equation. Version: Fall 2010 Solve for t: t 2 1 = 7 4

330

CHAPTER 4. FRACTIONS

Solution. Multiply both sides of the equation by the least common denominator for the fractions appearing in the equation. x 6 x 6x 6 5 6 5 6 5 1 = 6 3 =6 =6 1 3 1 3 Original equation. Multiply both sides by 6. Distribute the 6. On each side, multiply rst. 6 5 6 = 5 and 6 1 3 = 2.

6x 5 = 2

Note that the equation is now entirely clear of fractions, making it a much simpler equation to solve. 6x 5 + 5 = 2 + 5 6x = 7 7 6x = 6 6 7 x= 6 Answer: 1/28 Add 5 to both sides. Simplify both sides. Divide both sides by 6. Simplify.

Note that this is the same solution found in Example 1.

You Try It! Solve for u: 7 14 u= 9 27 EXAMPLE 6. In Example 4, we were asked to solve the following equation for x. 5 8 x= 9 18 Take a moment to review the solution in Example 4. We will now solve this equation by rst clearing all fractions from the equation. Solution. Multiply both sides of the equation by the least common denominator for the fractions that appear in the equation. 5 8 x= 9 18 5 8 18 x = 18 9 18 16x = 5 Original equation. Multiply both sides by 18. On each side, cancel and multiply. 18 Version: Fall 2010 8 9 = 16 and 18 5 18 = 5.

4.8. SOLVING EQUATIONS WITH FRACTIONS Note that the equation is now entirely free of fractions. Continuing, 16x 5 = 16 16 5 x= 16 Divide both sides by 16. Simplify.

331

Note that this is the same as the solution found in Example 4.

Answer: 2/3

You Try It! EXAMPLE 7. Solve for x: 3 1 2 x+ = . 3 4 2 Solution. Multiply both sides of the equation by the least common denominator for the fractions appearing in the equation. 2 3 1 x+ = 3 4 2 2 3 12 x+ = 12 3 4 3 2 x + 12 = 12 3 4 8x + 9 = 6 Original equation. 1 2 1 2 Multiply both sides by 12. On the left, distribute 12. Multiply: 12 and 12 1 2 2 x 3 = 6. = 8x, 12 3 4 = 9, Solve for r: 3 2 1 r+ = 4 3 2

12

Note that the equation is now entirely free of fractions. We need to isolate the terms containing x on one side of the equation. 8x + 9 9 = 6 9 8x = 3 3 8x = 8 8 3 x= 8 Subtract 9 from both sides. Simplify both sides. Divide both sides by 8. Simplify both sides. Answer: 2/9

Readers are encouraged to check this solution in the original equation.

You Try It! EXAMPLE 8. Solve for x: 2 3x x 1 = . 3 4 2 8 Version: Fall 2010 Solve for s: s 1 3 2s = 2 5 3 5

332

CHAPTER 4. FRACTIONS

Solution. Multiply both sides of the equation by the least common denominator for the fractions in the equation. 2 3x x 1 = 3 4 2 8 2 3x x 1 24 = 24 3 4 2 8 3x 2 x 24 24 = 24 3 4 2 16 18x = 12x 3 Original equation. Multiply both sides by 24. 1 8 On both sides, distribute 24. Left: 24 3x = 18x. 4 1 x = 12x, 24 = 3. Right: 24 2 8 = 16, 24 2 3

24

Note that the equation is now entirely free of fractions. We need to isolate the terms containing x on one side of the equation. 16 18x 12x = 12x 3 12x 16 30x = 3 16 30x 16 = 3 16 30x = 19 30x 19 = 30 30 19 x= 30 Answer: 51/22 Subtract 12x from both sides. Left: 18x 12x = 30x. Right: 12x 12x = 0. Subtract 16 from both sides. Left: 16 16 = 0. Right: 3 16 = 19. Divide both sides by 30. Simplify both sides.

Readers are encouraged to check this solution in the original equation.

Applications
Lets look at some applications that involve equations containing fractions. For convenience, we repeat the Requirements for Word Problem Solutions. Requirements for Word Problem Solutions. 1. Set up a Variable Dictionary. You must let your readers know what each variable in your problem represents. This can be accomplished in a number of ways: Statements such as Let P represent the perimeter of the rectangle.

Version: Fall 2010

4.8. SOLVING EQUATIONS WITH FRACTIONS

333

Labeling unknown values with variables in a table. Labeling unknown quantities in a sketch or diagram. 2. Set up an Equation. Every solution to a word problem must include a carefully crafted equation that accurately describes the constraints in the problem statement. 3. Solve the Equation. You must always solve the equation set up in the previous step. 4. Answer the Question. This step is easily overlooked. For example, the problem might ask for Janes age, but your equations solution gives the age of Janes sister Liz. Make sure you answer the original question asked in the problem. Your solution should be written in a sentence with appropriate units. 5. Look Back. It is important to note that this step does not imply that you should simply check your solution in your equation. After all, its possible that your equation incorrectly models the problems situation, so you could have a valid solution to an incorrect equation. The important question is: Does your answer make sense based on the words in the original problem statement.

You Try It! EXAMPLE 9. In the third quarter of a basketball game, announcers informed the crowd that attendance for the game was 12,250. If this is two-thirds of the capacity, nd the full seating capacity for the basketball arena. Solution. We follow the Requirements for Word Problem Solutions. 1. Set up a Variable Dictionary. Let F represent the full seating capacity. Note: It is much better to use a variable that sounds like the quantity that it represents. In this case, letting F represent the full seating capacity is much more descriptive than using x to represent the full seating capacity. 2. Set up an Equation. Two-thirds of the full seating capacity is 12,250. Attendance for the Celtics game was 9,510. If this is 3/4 of capacity, what is the capacity of the Celtics arena?

Two-thirds 2 3

of

Full Seating Capacity F

is =

12,250 12,250 Version: Fall 2010

334 Hence, the equation is

CHAPTER 4. FRACTIONS

2 F = 12250. 3 3. Solve the Equation. Multiply both sides by 3 to clear fractions, then solve. 2 F = 12250 Original equation. 3 2 3 F = 3(12250) Multiply both sides by 3. 3 2F = 36750 2F 36750 = 2 2 F = 18375 Simplify both sides. Divide both sides by 2. Simplify both sides.

4. Answer the Question. The full seating capacity is 18,375. 5. Look Back. The words of the problem state that 2/3 of the seating capacity is 12,250. Lets take two-thirds of our answer and see what we get. 2 2 18375 18375 = 3 3 1 2 3 6125 = 3 1 2 3 6125 = 1 3 = 12250 Answer: 12,680 This is the correct attendance, so our solution is correct.

You Try It! The area of a triangle is 161 square feet. If the base of 1 the triangle measures 40 4 feet, nd the height of the triangle. EXAMPLE 10. The area of a triangle is 20 square inches. If the length of 1 the base is 2 2 inches, nd the height (altitude) of the triangle. Solution. We follow the Requirements for Word Problem Solutions. 1. Set up a Variable Dictionary. Our variable dictionary will take the form of a well labeled diagram.

1 2 2 in

Version: Fall 2010

4.8. SOLVING EQUATIONS WITH FRACTIONS

335

2. Set up an Equation. The area A of a triangle with base b and height h is A=


1 Substitute A = 20 and b = 2 2 .

1 bh. 2 1 2

20 =

1 2

h.

3. Solve the Equation. Change the mixed fraction to an improper fraction, then simplify. 1 2 1 20 = 2 20 = 20 = 20 = 2 5 2 1 5 2 2 5 h 4 1 2 h h h Original equation. Mixed to improper: 2 Associative property. Multiply: 1 5 5 = . 2 2 4 5 1 = . 2 2

Now, multiply both sides by 4/5 and solve. 4 4 (20) = 5 5 16 = h 5 h 4 Multiply both sides by 4/5. Simplify: and 4 (20) = 16 5

4 5 = 1. 5 4

4. Answer the Question. The height of the triangle is 16 inches.


1 5. Look Back. If the height is 16 inches and the base is 2 2 inches, then the area is 1 1 2 (16) A= 2 2 1 5 16 = 2 2 1 5 16 = 22 (5) (2 2 2 2) = (2) (2) 52222 = 2 2 = 20

This is the correct area (20 square inches), so our solution is correct. Version: Fall 2010

Answer: 8 feet

336

CHAPTER 4. FRACTIONS

l l l
1. Is 1/4 a solution of the equation x+ 5 5 = ? 8 8

Exercises

l l l

7. Is 3/8 a solution of the equation x 13 5 = ? 9 72

2. Is 1/4 a solution of the equation x+ 5 1 = ? 3 12

8. Is 1/2 a solution of the equation x 1 3 = ? 5 10

3. Is 8/15 a solution of the equation 1 1 x= ? 4 15 4. Is 18/7 a solution of the equation 3 25 x= ? 8 28 5. Is 1/2 a solution of the equation x+ 4 17 = ? 9 18

9. Is 2/7 a solution of the equation x 8 4 = ? 9 63

10. Is 1/9 a solution of the equation x 4 31 = ? 7 63

11. Is 8/5 a solution of the equation 11 44 x= ? 14 35 12. Is 16/9 a solution of the equation 13 104 x= ? 18 81

6. Is 1/3 a solution of the equation x+ 3 13 = ? 4 12

In Exercises 13-24, solve the equation and simplify your answer. 13. 2x 3 = 6x + 7 14. 9x 8 = 9x 3 15. 7x + 4 = 3x 16. 6x + 9 = 6x 17. 2x = 9x 4 18. 6x = 9x + 8 19. 8x = 7x 7 20. 6x = 5x + 4 21. 7x + 8 = 2x 22. x 7 = 3x 23. 9x + 4 = 4x 6 24. 2x + 4 = x 7

Version: Fall 2010

4.8. SOLVING EQUATIONS WITH FRACTIONS In Exercises 25-48, solve the equation and simplify your answer. 25. x + 26. 27. 28. 29. 30. 31. 32. 33. 34. 35. 36. 1 3 = 2 2 1 3 x = 4 4 1 9 x= 5 2 7 7 x= 3 2 8 3 x= 8 7 3 1 x= 9 5 1 2 x= 5 6 2 1 x= 6 9 8 3 x= 2 7 7 3 x= 2 5 5 3 x+ = 4 9 3 1 x = 9 2 37. x 38. x + 39. x + 40. x 41. x + 42. x + 43. 44. 45. 46. 47. 48. 4 7 4 9 8 9 5 6 5 2 1 2 = 7 8 3 4

337

= 2 3 1 = 4 = = =

9 8

5 3 7 8 x= 5 9 5 3 x= 2 9 1 1 x = 4 8 7 9 x = 2 2 1 1 x= 4 2 8 8 x= 9 3

In Exercises 49-72, solve the equation and simplify your answer. 2 3 2 7 49. x = x + 3 3 4 3 1 3 3 1 50. x = x + 2 2 2 4 5 4 7 51. x = 2 4 5 5 8 7 52. x + = 6 6 9 9 5 9 53. x + = 7 2 2 7 1 5 54. x = 9 2 4 55. 56. 57. 58. 59. 60. 4 2 1 x = 4 3 3 3 5 8 x+ = 7 7 3 3 1 5 x+ = 3 2 4 8 2 1 x = 2 3 5 4 9 5 1 x+ = x 3 5 5 6 3 4 3 2 x = x 9 5 5 2 Version: Fall 2010

338 4 8 1 61. x = x 9 9 2 5 8 5 62. x = x + 4 3 7 1 1 1 63. x = x + 2 8 8 8 7 3 64. x + = x 2 3 9 1 1 3 65. x = 7 3 9 2 9 2 66. x + = 3 9 5 1 2 7 3 5 7 1 2

CHAPTER 4. FRACTIONS 2 8 1 3 67. x + = x 4 7 7 3 1 5 1 1 68. x + = x 2 3 2 4 2 2 1 3 69. x = x 4 3 3 2 5 3 4 1 70. x = x + 3 7 2 3 9 5 5 71. x + = 2 5 8 4 1 9 72. x + = 4 3 6

73. At a local soccer game, announcers informed the crowd that attendance for the game was 4,302. If this is 2/9 of the capacity, nd the full seating capacity for the soccer stadium. 74. At a local basketball game, announcers informed the crowd that attendance for the game was 5,394. If this is 2/7 of the capacity, nd the full seating capacity for the basketball stadium. 75. The area of a triangle is 51 square inches. If the length of the base is 8 1 inches, nd 2 the height (altitude) of the triangle. 76. The area of a triangle is 20 square inches. If the length of the base is 2 1 inches, nd 2 the height (altitude) of the triangle.

77. The area of a triangle is 18 square inches. If the length of the base is 4 1 inches, nd 2 the height (altitude) of the triangle. 78. The area of a triangle is 44 square inches. If the length of the base is 5 1 inches, nd 2 the height (altitude) of the triangle. 79. At a local hockey game, announcers informed the crowd that attendance for the game was 4,536. If this is 2/11 of the capacity, nd the full seating capacity for the hockey stadium. 80. At a local soccer game, announcers informed the crowd that attendance for the game was 6,970. If this is 2/7 of the capacity, nd the full seating capacity for the soccer stadium.

81. Pirates. About one-third of the worlds pirate attacks in 2008 occurred o the Somali coast. If there were 111 pirate attacks o the Somali coast, estimate the number of pirate attacks worldwide in 2008. 82. Nuclear arsenal. The U.S. and Russia agreed to cut nuclear arsenals of long-range nuclear weapons by about a third, down to 1, 550. How many long-range nuclear weapons are there now? Associated Press-Times-Standard 04/04/10 Nuclear heartland anxious about missile cuts.

Version: Fall 2010

4.8. SOLVING EQUATIONS WITH FRACTIONS

339

83. Seed vault. The Svalbard Global Seed Vault has amassed half a million seed samples, and now houses at least one-third of the worlds crop seeds. Estimate the total number of worlds crop seeds. Associated Press-Times-Standard 03/15/10 Norway doomsday seed vault hits half-million mark.
1 84. Freight train. The three and one-half mile long Union Pacic train is about 2 2 times the length of a typical freight train. How long is a typical freight train? Associated Press-Times-Standard 01/13/10 Unususally long train raises safety concerns.

l l l
1. No 3. No 5. Yes 7. Yes 9. No 11. Yes 13. 15. 17. 19. 21. 23. 2 5 4 11 7 15 8 9 10 13 5 2

Answers
29. 64 21

l l l

31. 33. 35. 37.

5 12 16 21 7 36

81 56 2 9 29 8 35 72

39. 41. 43. 45. 1 8

47. 2 49. 51. 53. 55. 16 37 41 70

25. 1 27. 5 18

49 9 8 3 Version: Fall 2010

340 57. 59. 61. 63. 21 20 49 44 7 17

CHAPTER 4. FRACTIONS 71. 47 100

73. 19,359 75. 12 77. 8 79. 24,948 81. There were about 333 pirate attacks worldwide. 83. 1, 500, 000

47 35

14 65. 27 67. 52 159

69. 2

Version: Fall 2010

Chapter

Decimals
On January 29, 2001, the New York Stock exchange ended its 200-year tradition of quoting stock prices in fractions and switched to decimals. It was said that pricing stocks the same way other consumer items were priced would make it easier for investors to understand and compare stock prices. Foreign exchanges had been trading in decimals for decades. Supporters of the change claimed that trading volume, the number of shares of stock traded, would increase and improve eciency. But switching to decimals would have another eect of narrowing the spread. The spread is the dierence between the best price oered by buyers, called the bid, and the price requested by sellers called the ask. Stock brokers make commissions as a percentage of the spread which, using fractions, could be anywhere upwards from 12 cents per share. When the New York Stock Exchange began back in 1792, the dollar was based on the Spanish real, (pronounced ray-al), also called pieces of eight as these silver coins were often cut into quarters or eighths to make change. This is what led to stock prices rst denominated in eighths. Thus, the smallest spread that could occur would be 1/8 of a dollar, or 12.5 cents. That may seem like small change, but buying 1000 shares for $1 per share with a $0.125 spread is a $125.00 commision. Not bad for a quick trade! Decimalization of stock pricing allowed for spreads as small as 1 cent. Since the number of shares traded on stock market exchanges have skyrocketed, with trillions of shares traded daily, stock broker commissions have not suered. And the ease with which investors can quickly grasp the price of stock shares has contributed to the opening of markets for all classes of people. In this chapter, well learn about how to compute and solve problems with decimals, and see how they relate to fractions.

341

342

CHAPTER 5. DECIMALS

5.1

Introduction to Decimals

Recall that whole numbers are constructed by using digits.

The Digits. The set {0, 1, 2, 3, 4, 5, 6, 7, 8, 9} is called the set of digits.

As an example, the whole number 55,555 (fty-ve thousand ve hundred fty-ve) is constructed by using a single digit. However, the position of the digit 5 determines its value in the number 55,555. The rst occurrence of the 5 ten thousands 5 5 5 5

thousands

hundreds

10,000

1,000

100

10

Table 5.1: Place value. digit 5 happens in the ten thousands place, so its value is 5 ten thousands, or 50,000. The next occurrence of the digit 5 is in the thousands place, so its value is 5 thousands, or 5,000. Indeed, the whole number 55,555 in expanded form is 50000 + 5000 + 500 + 50 + 5, which reects the value of the digit 5 in each place.

Decimal Notation
In Table 5.1, each time you move one column to the left, the place value is 10 times larger than the place value of the preceding column. Vice-versa, each time you move one column to the right, the place value is 1/10 of the place value of the preceding column. Now, consider the decimal number 12.3456, which consists of three parts: the whole number part, the decimal point, and the fractional part. Version: Fall 2010

ones

tens

5.1. INTRODUCTION TO DECIMALS

343

12 . 3456 Whole number part Decimal point Fractional part

The whole number part of the decimal number is the part that lies strictly to the left of the decimal point, and the place value of each digit in the whole number part is given by the columns shown in Table 5.1. The fractional part of the decimal number is the part that lies strictly to the right of the decimal point. As we saw in Table 5.1, each column has a value equal to 1/10 of the value of the column that lies to its immediate left. Thus, it should come as no surprise that: The rst column to the right of the decimal point has place value 1/10 (tenths). The second column to the right of the decimal point has place value 1/100 (hundredths). The third column to the right of the decimal point has place value 1/1000 (thousandths). The fourth column to the right of the decimal point has place value 1/10000 (ten-thousandths). These results are summarized for the decimal number 12.3456 in Table 5.2. 1 2 . decimal point 3 4 5 6 ten-thousandths 1/10000 Version: Fall 2010

Table 5.2: Place value.

1/1000

1/100

1/10

10

thousandths

hundredths

tenths

ones

tens

344

CHAPTER 5. DECIMALS

Pronouncing Decimal Numbers


The decimal number 12.3456 is made up of 1 ten, 2 ones, 3 tenths, 4 hundredths, 5 thousandths, and 6 ten-thousandths (see Table 5.2), and can be written in expanded form as 12.3456 = 10 + 2 + 4 5 6 3 + + + . 10 100 1000 10000

Note that the whole numbers can be combined and the fractions can be written with a common denominator and summed. 12.3456 = 12 + 4 100 5 10 6 3 1000 + + + 10 1000 100 100 1000 10 10000 3000 400 50 6 = 12 + + + + 10000 10000 10000 10000 3456 = 12 + 10000

The result tells us how to pronounce the number 12.3456. It is pronounced twelve and three thousand, four hundred fty-six ten-thousandths. You Try It! Place the decimal number 3,502.23 in expanded form, then combine the whole number part and sum the fractional part over a common denominator EXAMPLE 1. Place the decimal number 1,234.56 in expanded form, then combine the whole number part and sum the fractional part over a common denominator. Use the result to help pronounce the decimal number. Solution. In expanded form, 1, 234.56 = 1, 000 + 200 + 30 + 4 + 6 5 + 10 100

Sum the whole number parts. Express the fractional parts as equivalent fractions and combine over one common denominator. 6 5 10 + 10 10 100 50 6 = 1, 234 + + 100 100 56 = 1, 234 + 100 = 1, 234 + Answer: 3, 502 +
23 100

Hence, 1,234.56 is pronounced one thousand, two hundred thirty-four and fty-six hundredths.

Version: Fall 2010

5.1. INTRODUCTION TO DECIMALS

345

You Try It! EXAMPLE 2. Place the decimal number 56.128 in expanded form, then combine the whole number part and sum the fractional part over a common denominator. Use the result to help pronounce the decimal number. Solution. In expanded form, 56.128 = 50 + 6 + 2 8 1 + + 10 100 1000 Place the decimal number 235.568 in expanded form, then combine the whole number part and sum the fractional part over a common denominator

Sum the whole number parts. Express the fractional parts as equivalent fractions and combine over one common denominator. 2 10 8 1 100 + + 10 100 100 10 1000 100 20 8 = 56 + + + 1000 1000 1000 128 = 56 + 1000 = 56 + Thus, 56.128 is pronounced fty-six and one hundred twenty-eight thousandths. Answer: 235 +
568 1000

The discussion and example leads to the following result. How to Read a Decimal Number 1. Pronounce the whole number part to the left of the decimal as you would any whole number. 2. Say the word and for the decimal point. 3. State the fractional part to the right of the decimal as you would any whole number, followed by the place value of the digit in the rightmost column.

You Try It! EXAMPLE 3. Pronounce the decimal number 34.12. Pronounce 28.73

Solution. The rightmost digit in the fractional part of 34.12 is in the hundredths column. Thus, 34.12 is pronounced thirty-four and twelve hundredths. Answer: Twenty-eight and seventy-three hundredths

Version: Fall 2010

346

CHAPTER 5. DECIMALS

Important Point. In pronouncing decimal numbers, the decimal point is read as and. No other instance of the word and should appear in the pronunciation.

You Try It! Pronounce 286.9. EXAMPLE 4. Explain why four hundred and thirty-four and two tenths is an incorrect pronunciation of the decimal number 434.2. Solution. The decimal point is read as and. No other occurrence of the word and is allowed in the pronunciation. The correct pronunciation should be four hundred thirty-four and two tenths.

Answer: Four hundred thirty-four and two tenths

You Try It! Pronounce 7, 002.207. EXAMPLE 5. Pronounce the decimal number 5,678.123. Solution. The rightmost digit in the fractional part of 5,678.123 is in the thousandths column. Hence, 5,678.123 is pronounced 5 thousand six hundred seventy-eight and one hundred twenty-three thousandths.

Answer: Seven thousand two and two hundred seven thousandths.

You Try It! Pronounce 500.1205. EXAMPLE 6. Pronounce the decimal number 995.4325. Solution. The rightmost digit in the fractional part of 995.4325 is in the tenthousandths column. Hence, 995.4325 is pronounced nine hundred ninety-ve and four thousand three hundred twenty-ve ten-thoudandths.

Asnwer: Five hundred and one thousand two hundred ve ten-thousandths.

Decimals to Fractions
Because we now have the ability to pronounce decimal numbers, it is a simple exercise to change a decimal to a fraction.1 For example, 134.12 is pronounced
1 Changing

fractions to decimals will be covered in Section 5.5.

Version: Fall 2010

5.1. INTRODUCTION TO DECIMALS

347

one hundred thirty-four and twelve hundredths, so it can easily be written as a mixed fraction. 12 134.12 = 134 100 But this mixed fraction can be changed to an improper fraction. 134 100 134 + 12 12 = 100 100 13400 + 12 = 100 13412 = 100

Note that the numerator is our original number without the decimal point. There are two decimal places in the original number and the denominator of the nal improper fraction contains two zeros. This discussion leads to the following result. Changing Decimals to Improper Fractions. To change a decimal number to an improper fraction, proceed as follows: 1. Create a fraction. 2. Place the decimal number in the numerator without the decimal point. 3. Count the number of decimal places. Place an equal number of zeros in the denominator.

You Try It! EXAMPLE 7. Change the following decimal numbers to improper fractions: (a) 1.2345, and (b) 27.198. Solution. In each case, place the number in the numerator without the decimal point. In the denominator, add a number of zeros equal to the number of decimal places. a) The decimal number 1.2345 has four decimal places. Hence, 1.2345 = 12345 10000 Change 17.205 to an improper fraction.

b) The decimal number 27.198 has three decimal places. Hence, 27.198 = 27198 1000 Version: Fall 2010 Answer: 17205 100

348

CHAPTER 5. DECIMALS

You Try It! Change 0.375 to a fraction, reduced to lowest terms. EXAMPLE 8. Change each of the following decimals to fractions reduced to lowest terms: (a) 0.35, and (b) 0.125. Solution. Place each number in the numerator without the decimal point. Place a number of zeros in the denominator equal to the number of decimal places. Reduce to lowest terms. a) First, place 35 over 100. 0.35 = 35 100

We can divide both numerator and denominator by the greatest common divisor. 35 5 100 5 7 = 20 = Divide numerator and denominator by 5. Simplify numerator and denominator.

b) First, place 125 over 1000. 0.125 = 125 1000

Prime factor and cancel common factors. 555 222555 555 = 222 5 5 5 1 = 8 = Answer: 3/8 Prime factor numerator and denominator. Cancel common factors. Simplify.

Rounding
The rules for rounding decimal numbers are almost identical to the rules for rounding whole numbers. First, a bit of terminology.

Version: Fall 2010

5.1. INTRODUCTION TO DECIMALS

349

Rounding Digit and Test Digit. The digit in the place to which we wish to round is called the rounding digit and the digit that follows on its immediate right is called the test digit. If we want to round the decimal number 12.254 to the nearest hundredth, then the rounding digit is 5 and the test digit is 4. Test digit 12.2 5 4 Rounding digit If we used the rules for rounding whole numbers, because the test digit 4 is less than 5, we would replace all digits to the right of the rounding digit with zeros to obtain the following approximation. 12.254 12.250 However, because 250 25 = 12 , 1000 100 the trailing zero at the end of the fractional part is irrelevant. Hence, we truncate every digit after the rounding digit and use the following approximation. 12.250 = 12 12.254 12.25 Important Observation. Deleting trailing zeros from the end of the fractional part of a decimal number does not change its value. The above discussion motivates the following algorithm for rounding decimal numbers. Rounding Decimal Numbers. Locate the rounding digit and the test digit. If the test digit is greater than or equal to 5, add 1 to the rounding digit and truncate all digits to the right of the rounding digit. If the test digit is less than 5, simply truncate all digits to the right of the rounding digit.

Version: Fall 2010

350

CHAPTER 5. DECIMALS

You Try It! Round 9.2768 to the nearest hundredth. EXAMPLE 9. Round 8.7463 to the nearest hundredth. Solution. Locate the rounding digit in the hundredths place and the test digit to its immediate right. Test digit 8.7 4 6 3 Rounding digit Because the test digit is greater than 5, add 1 to the rounding digit and truncate all digits to the right of the rounding digit. Hence, to the nearest hundredth: 8.7463 8.75 Answer: 9.28

You Try It! Round 58.748 to the nearest tenth. EXAMPLE 10. Round 113.246 to the nearest tenth. Solution. Locate the rounding digit in the tenths place and the test digit to its immediate right. Test digit 113. 2 4 6 Rounding digit Because the test digit is less than 5, truncate all digits to the right of the rounding digit. Hence, to the nearest tenth: 113.246 113.2 Answer: 58.7

Comparing Decimals
We can compare two positive decimals by comparing digits in each place as we move from left to right, place by place. For example, suppose we wish to compare the decimal numbers 5.234 and 5.2357. First, add enough trailing Version: Fall 2010

5.1. INTRODUCTION TO DECIMALS

351

zeros to the decimal number with the fewer decimal places so that the numbers have the same number of decimal places. In this case, note that 5.234 = 5 234 2340 =5 = 5.2340. 1000 10000

Important Observation. Adding trailing zeros to the end of the fractional part of a decimal number does not change its value. Next, align the numbers as follows. 5.2340 5.2357 Same digit Same digit Same digit

Dierent digit

As you scan the columns, moving left to right, the rst place that has dierent digits occurs in the thousandths place, where the digit 5 is the second number is greater than the digit 4 in the rst number in the same place. Because 5 is greater than 4, the second number is larger than the rst. That is: 5.234 < 5.2357 This discussion suggests the following algorithm. Comparing Positive Decimal Numbers. 1. Add enough trailing zeros so that both numbers have the same number of decimal places. 2. Compare the digits in each place, moving left to right. 3. As soon as you nd two digits in the same place that are dierent, the decimal number with the greatest digit in this place is the larger number.

You Try It! EXAMPLE 11. Compare 4.25 and 4.227. Solution. Add a trailing zero to the rst decimal number and align the numbers as follows. 4.250 4.227 Same digit Same digit Dierent digit Version: Fall 2010 Compare 8.34 and 8.348.

352

CHAPTER 5. DECIMALS

The rst dierence is in the hundredths place, where the digit 5 in the rst number is greater than the digit 2 in the same place of the second number. Hence, the rst number is larger than the second; that is: 4.25 > 4.227 Answer: 8.34 < 8.348

When comparing negative numbers, the number with the larger magnitude is the smaller number. Hence, we have to adjust our algorithm for comparing negative decimal numbers. Comparing Negative Decimal Numbers. 1. Add enough trailing zeros so that both numbers have the same number of decimal places. 2. Compare the digits in each place, moving left to right. 3. As soon as you nd two digits in the same place that are dierent, the decimal number with the greatest digit in this place is the smaller number.

You Try It! Compare 7.86 and 7.85. EXAMPLE 12. Compare 4.25 and 4.227. Solution. Add a trailing zero to the rst decimal number and align the numbers as follows. 4.250 4.227 Same digit Same digit Dierent digit

The rst dierence is in the hundredths place, where the digit 5 in the rst number is greater than the digit 2 in the same place of the second number. Hence, the rst number is smaller than the second; that is: 4.25 < 4.227 Answer: 7.86 < 7.85

Version: Fall 2010

5.1. INTRODUCTION TO DECIMALS

353

l l l

Exercises

l l l

1. Which digit is in the tenths column of the number 4,552.0908? 2. Which digit is in the thousandths column of the number 7,881.6127? 3. Which digit is in the tenths column of the number 4,408.2148? 4. Which digit is in the tenths column of the number 9,279.0075? 5. Which digit is in the ten-thousandths column of the number 2,709.5097? 6. Which digit is in the hundredths column of the number 1,743.1634?

7. Which digit is in the hundredths column of the number 3,501.4456? 8. Which digit is in the ten-thousandths column of the number 9,214.3625? 9. Which digit is in the hundredths column of the number 5,705.2193? 10. Which digit is in the hundredths column of the number 7,135.2755? 11. Which digit is in the tenths column of the number 8,129.3075? 12. Which digit is in the thousandths column of the number 6,971.4289?

In Exercises 13-20, write the given decimal number in expanded form. 13. 46.139 14. 68.392 15. 643.19 16. 815.64 17. 14.829 18. 45.913 19. 658.71 20. 619.38

In Exercises 21-28, follow the procedure shown in Examples 1 and 2 to write the decimal number in expanded form, then combine the whole number part and sum the fractional part over a common denominator. 21. 32.187 22. 35.491 23. 36.754 24. 89.357 25. 596.71 26. 754.23 27. 527.49 28. 496.15

Version: Fall 2010

354

CHAPTER 5. DECIMALS

In Exercises 29-40, pronounce the given decimal number. Write your answer out in words. 29. 0.9837 30. 0.6879 31. 0.2653 32. 0.8934 33. 925.47 34. 974.35 35. 83.427 36. 32.759 37. 63.729 38. 85.327 39. 826.57 40. 384.72

In Exercises 41-52, convert the given decimal to a mixed fraction. Do not simplify your answer. 41. 98.1 42. 625.591 43. 781.7 44. 219.999 45. 915.239 46. 676.037 47. 560.453 48. 710.9 49. 414.939 50. 120.58 51. 446.73 52. 653.877

In Exercises 53-60, convert the given decimal to an improper fraction. Do not simplify your answer. 53. 8.7 54. 3.1 55. 5.47 56. 5.27 57. 2.133 58. 2.893 59. 3.9 60. 1.271

In Exercises 61-68, convert the given decimal to a fraction. Reduce your answer to lowest terms. 61. 0.35 62. 0.38 63. 0.06 64. 0.84 65. 0.98 66. 0.88 67. 0.72 68. 0.78

Version: Fall 2010

5.1. INTRODUCTION TO DECIMALS 69. Round 79.369 to the nearest hundredth. 70. Round 54.797 to the nearest hundredth. 71. Round 71.2427 to the nearest thousandth. 72. Round 59.2125 to the nearest thousandth. 73. Round 29.379 to the nearest tenth. 74. Round 42.841 to the nearest tenth.

355 75. Round 89.3033 to the nearest thousandth. 76. Round 9.0052 to the nearest thousandth. 77. Round 20.655 to the nearest tenth. 78. Round 53.967 to the nearest tenth. 79. Round 19.854 to the nearest hundredth. 80. Round 49.397 to the nearest hundredth.

In Exercises 81-92, determine which of the two given statements is true. 81. 0.30387617 < 0.3036562 or 0.30387617 > 0.3036562 82. 8.5934 < 8.554 or 8.5934 > 8.554 83. 0.034 < 0.040493 or 0.034 > 0.040493 84. 0.081284 < 0.08118 or 0.081284 > 0.08118 85. 8.3527 < 8.36553 or 8.3527 > 8.36553 86. 0.00786 < 0.0051385 or 0.00786 > 0.0051385 92. 0.049785 < 0.012916 or 0.049785 > 0.012916 Version: Fall 2010 91. 15.188392 < 15.187157 or 15.188392 > 15.187157 90. 0.000681 < 0.00043174 or 0.000681 > 0.00043174 89. 36.8298 < 36.8266595 or 36.8298 > 36.8266595 88. 514.873553 < 514.86374 or 514.873553 > 514.86374 87. 18.62192 < 18.6293549 or 18.62192 > 18.6293549

356

CHAPTER 5. DECIMALS

93. Write the decimal number in words. i) A recently discovered 7.03-carat blue diamond auctioned at Sothebys. ii) The newly launched European Planck telescope will stay in orbit 1.75 years measuring radiation from the Big Bang. iii) The sun composes 0.9985 of the mass in our solar system. iv) Clay particles are small - only 0.0001 inch. 94. Light speed. The index of refraction for a given material is a value representing the number of times slower a light wave travels in that particular material than it travels in the vacuum of space. i) Reorder the materials by their index of refraction from lowest to highest. ii) How many times slower is a lightwave in a diamond compared with a vacuum? Material Diamond Vacuum Plexiglas Air Water Zircon Crown Glass Ice Index of Refraction 2.417 1.0000 1.51 1.0003 1.333 1.923 1.52 1.31

95. Shorter day? Scientists at NASAs Jet Propulsion Laboratory calculated that the earthquake in Chile may have shortened the length of a day on Earth by 1.26 millionths of a second. i) Write that number completely as a decimal. ii) Actual observations of the length of the day are accurate to ve millionths of a second. Write this fraction as a decimal. iii) Comparing the two decimals above and determine which is smaller. Do you think scientists can observe and measure the calculated slowing of the earth?

l l l
1. 0 3. 2 5. 7 7. 4 Version: Fall 2010

Answers
9. 1 11. 3

l l l

13. 40 + 6 +

1 10

3 100 1 10

+ +

9 1000 9 100

15. 600 + 40 + 3 +

5.1. INTRODUCTION TO DECIMALS 17. 10 + 4 +


8 10

357 87 10 547 100 2133 1000 39 10 7 20 3 50 49 50 18 25

2 100 7 10

+ +

9 1000 1 100

53. 55. 57. 59. 61. 63.

19. 600 + 50 + 8 + 21. 32 + 23. 36 +


187 1000 754 1000 71 100 49 100

25. 596 + 27. 527 +

29. nine thousand eight hundred thirty-seven ten-thousandths 31. two thousand six hundred fty-three tenthousandths

65. 33. nine hundred twenty-ve and forty-seven hundredths 67. 35. eighty-three and four hundred twentyseven thousandths 37. sixty-three and seven hundred twenty-nine thousandths 39. eight hundred twenty-six and fty-seven hundredths 41. 98 1 10 7 10

69. 79.37 71. 71.243 73. 29.4 75. 89.303 77. 20.7 79. 19.85 81. 0.30387617 > 0.3036562

43. 781

239 45. 915 1000 47. 560 49. 414 453 1000 939 1000

83. 0.034 > 0.040493 85. 8.3527 > 8.36553 87. 18.62192 < 18.6293549 89. 36.8298 > 36.8266595 91. 15.188392 < 15.187157 Version: Fall 2010

73 51. 446 100

358 93. i) seven and three hundredths ii) one and seventy-ve hundredths iii) nine thousand nine hundred eightyve ten-thousandths iv) one ten-thousandth of an inch 95.

CHAPTER 5. DECIMALS i) 0.00000126 ii) 0.000005 iii) 0.00000126 < 0.000005; scientists would be unable to measure the calculated change in the length of a day.

Version: Fall 2010

5.2. ADDING AND SUBTRACTING DECIMALS

359

5.2

Adding and Subtracting Decimals

Adding Decimals
Addition of decimal numbers is quite similar to addition of whole numbers. For example, suppose that we are asked to add 2.34 and 5.25. We could change these decimal numbers to mixed fractions and add. 2.34 + 5.25 = 2 25 34 +5 100 100 59 =7 100

However, we can also line the decimal numbers on their decimal points and add vertically, as follows. 2.34 + 5.25 7.59 Note that this alignment procedure produces the same result, seven and fty nine hundredths. This motivates the following procedure for adding decimal numbers. Adding Decimals. To add decimal numbers, proceed as follows: 1. Place the numbers to be added in vertical format, aligning the decimal points. 2. Add the numbers as if they were whole numbers. 3. Place the decimal point in the answer in the same column as the decimal points above it.

You Try It! EXAMPLE 1. Add 3.125 and 4.814. Solution. Place the numbers in vertical format, aligning on their decimal points. Add, then place the decimal point in the answer in the same column as the decimal points that appear above the answer. 3.125 + 4.814 7.939 Thus, 3.125 + 4.814 = 7.939. Version: Fall 2010 Answer: 5.893 Add: 2.864 + 3.029

360

CHAPTER 5. DECIMALS

You Try It! Alice has $8.63 in her purse and Joanna has $2.29. If they combine sum their money, what is the total? EXAMPLE 2. Jane has $4.35 in her purse. Jim has $5.62 in his wallet. If they sum their money, what is the total? Solution. Arrange the numbers in vertical format, aligning decimal points, then add. $4.35 + $5.62 $9.97 Answer: $10.91 Together they have $9.97, nine dollars and ninety seven cents.

Before looking at another example, lets recall an important observation. Important Observation. Adding zeros to the end of the fractional part of a decimal number does not change its value. Similarly, deleting trailing zeros from the end of a decimal number does not change its value. For example, we could add two zeros to the end of the fractional part of 7.25 to obtain 7.2500. The numbers 7.25 and 7.2500 are identical as the following argument shows: 7.2500 = 7 2500 10000 25 =7 100 = 7.25

You Try It! Add: 9.7 + 15.86 EXAMPLE 3. Add 7.5 and 12.23. Solution. Arrange the numbers in vertical format, aligning their decimal points in a column. Note that we add a trailing zero to improve columnar alignment. 7.50 + 12.23 19.73 Answer: 25.56 Hence, 7.5 + 12.23 = 19.73.

Version: Fall 2010

5.2. ADDING AND SUBTRACTING DECIMALS

361

You Try It! EXAMPLE 4. Find the sum: 12.2 + 8.352 + 22.44. Add: 12.9 + 4.286 + 33.97

Solution. Arrange the numbers in vertical format, aligning their decimal points in a column. Note that we add trailing zeros to improve the columnar alignment. 12.200 8.352 + 22.440 42.992 Hence, 12.2 + 8.352 + 22.44 = 42.992. Answer: 51.156

Subtracting Decimals
Subtraction of decimal numbers proceeds in much the same way as addition of decimal numbers. Subtracting Decimals. To subtract decimal numbers, proceed as follows: 1. Place the numbers to be subtracted in vertical format, aligning the decimal points. 2. Subtract the numbers as if they were whole numbers. 3. Place the decimal point in the answer in the same column as the decimal points above it.

You Try It! EXAMPLE 5. Subtract 12.23 from 33.57. Solution. Arrange the numbers in vertical format, aligning their decimal points in a column, then subtract. Note that we subtract 12.23 from 33.57. 33.57 12.23 21.34 Hence, 33.57 12.23 = 21.34. Asnwer: 19.81 Subtract: 58.76 38.95

As with addition, we add trailing zeros to the fractional part of the decimal numbers to help columnar alignment. Version: Fall 2010

362

CHAPTER 5. DECIMALS

You Try It! Subtract: 15.2 8.756 EXAMPLE 6. Find the dierence: 13.3 8.572. Solution. Arrange the numbers in vertical format, aligning their decimal points in a column. Note that we add trailing zeros to the fractional part of 13.3 to improve columnar alignment. 13.300 8.572 4.728 Answer: 6.444 Hence, 13.3 8.572 = 4.728.

Adding and Subtracting Signed Decimal Numbers


We use the same rules for addition of signed decimal numbers as we did for the addition of integers. Adding Two Decimals with Like Signs. To add two decimals with like signs, proceed as follows: 1. Add the magnitudes of the decimal numbers. 2. Prex the common sign.

You Try It! Simplify: 5.7 + (83.85) EXAMPLE 7. Simplify: 3.2 + (18.95). 3.20 + 18.95 22.15 Answer: 89.55 Prex the common sign. Hence, 3.2 + (18.95) = 22.15

Solution. To add like signs, rst add the magnitudes.

We use the same rule as we did for integers when adding decimals with unlike signs. Adding Two Decimals with Unlike Signs. To add two decimals with unlike signs, proceed as follows:

Version: Fall 2010

5.2. ADDING AND SUBTRACTING DECIMALS

363

1. Subtract the smaller magnitude from the larger magnitude. 2. Prex the sign of the decimal number with the larger magnitude.

You Try It! EXAMPLE 8. Simplify: 3 + 2.24. Simplify: 8 + 5.74

Solution. To add unlike signs, rst subtract the smaller magnitude from the larger magnitude. 3.00 2.24 0.76 Prex the sign of the decimal number with the larger magnitude. Hence, 3 + 2.24 = 0.76. Answer: 2.26

Subtraction still means add the opposite. You Try It! EXAMPLE 9. Simplify: 8.567 (12.3). Simplify: 2.384 (15.2)

Solution. Subtraction must rst be changed to addition by adding the opposite. 8.567 (12.3) = 8.567 + 12.3 We have unlike signs. First, subtract the smaller magnitude from the larger magnitude. 12.300 8.567 3.733 Prex the sign of the decimal number with the larger magnitude. Hence: 8.567 (12.3) = 8.567 + 12.3 = 3.733 Answer: 12.816

Order of operations demands that we simplify expressions contained in parentheses rst. You Try It! EXAMPLE 10. Simplify: 11.2 (8.45 + 2.7). Simplify: 12.8 (7.44 + 3.7)

Solution. We need to add inside the parentheses rst. Because we have unlike Version: Fall 2010 signs, subtract the smaller magnitude from the larger magnitude.

364 8.45 2.70 5.75

CHAPTER 5. DECIMALS

Prex the sign of the number with the larger magnitude. Therefore, 11.2 (8.45 + 2.7) = 11.2 (5.75) Subtraction means add the opposite. 11.2 (5.75) = 11.2 + 5.75 Again, we have unlike signs. Subtract the smaller magnitude from the larger magnitude. 11.20 5.75 5.45 Prex the sign of the number with the large magnitude. 11.2 + 5.75 = 5.45 Answer: 9.06

Writing Mathematics. The solution to the previous example should be written as follows: 11.2 (8.45 + 2.7) = 11.2 (5.75) = 11.2 + 5.75 = 5.45 Any scratch work, such as the computations in vertical format in the previous example, should be done in the margin or on a scratch pad.

Version: Fall 2010

5.2. ADDING AND SUBTRACTING DECIMALS

365

You Try It! EXAMPLE 11. Simplify: 12.3 | 4.6 (2.84)|. Simplify: 8.6 | 5.5 (8.32)|

Solution. We simplify the expression inside the absolute value bars rst, take the absolute value of the result, then subtract. 12.3 | 4.6 (2.84)| = 12.3 | 4.6 + 2.84| = 12.3 | 1.76| = 12.3 1.76 = 12.3 + (1.76) = 14.06 Add the opposite. Add: 4.6 + 2.84 = 1.76. | 1.76| = 1.76. Add the opposite. Add: 12.3 + (1.76) = 14.06.

Answer: 11.42

Version: Fall 2010

366

CHAPTER 5. DECIMALS

l l l
In Exercises 1-12, add the decimals. 1. 31.9 + 84.7 2. 9.39 + 7.7 3. 4 + 97.18 4. 2.645 + 2.444 5. 4 + 87.502 6. 23.69 + 97.8

Exercises

l l l

7. 95.57 + 7.88 8. 18.7 + 7 9. 52.671 + 5.97 10. 9.696 + 28.2 11. 4.76 + 2.1 12. 1.5 + 46.4

In Exercises 13-24, subtract the decimals. 13. 9 2.261 14. 98.14 7.27 15. 80.9 6 16. 9.126 6 17. 55.672 3.3 18. 4.717 1.637 19. 60.575 6 20. 8.91 2.68 21. 39.8 4.5 22. 8.210 3.7 23. 8.1 2.12 24. 7.675 1.1

In Exercises 25-64, add or subtract the decimals, as indicated. 25. 19.13 7 26. 8 79.8 27. 6.08 76.8 28. 5.76 36.8 29. 34.7 + (56.214) 30. 7.5 + (7.11) 31. 8.4 + (6.757) 32. 1.94 + 72.85 33. 50.4 + 7.6 34. 1.4 + (86.9) Version: Fall 2010 35. 43.3 + 2.2 36. 0.08 + (2.33) 37. 0.19 0.7 38. 9 18.01 39. 7 1.504 40. 4.28 2.6 41. 4.47 + (2) 42. 9 + (43.67) 43. 71.72 (6) 44. 6 (8.4)

5.2. ADDING AND SUBTRACTING DECIMALS 45. 9.829 (17.33) 46. 95.23 (71.7) 47. 2.001 4.202 48. 4 11.421 49. 2.6 2.99 50. 3.57 84.21 51. 4.560 2.335 52. 4.95 96.89 53. 54.3 3.97 54. 2 29.285 55. 6.32 + (48.663) 56. 8.8 + (34.27) 57. 8 (3.686) 58. 2.263 (72.3) 59. 9.365 + (5) 60. 0.12 + 6.973 61. 2.762 (7.3) 62. 65.079 (52.6) 63. 96.1 + (9.65) 64. 1.067 + (4.4)

367

In Exercises 65-80, simplify the given expression. 65. 12.05 |17.83 (17.16)| 66. 15.88 | 5.22 (19.94)| 67. 6.4 + |9.38 (9.39)| 68. 16.74 + |16.64 2.6| 69. 19.1 (1.51 (17.35)) 70. 17.98 (10.07 (10.1)) 71. 11.55 + (6.3 (1.9)) 72. 8.14 + (16.6 (15.41)) 73. 1.7 (1.9 (16.25)) 74. 4.06 (4.4 (10.04)) 75. 1.2 + |8.74 16.5| 76. 18.4 + |16.5 7.6| 77. 12.4 |3.81 16.4| 78. 13.65 |11.55 (4.44)| 79. 11.15 + (11.6 (16.68)) 80. 8.5 + (3.9 6.98)

81. Big Banks. Market capitalization of nations four largest banks (as of April 23, 2009 ) JPMorgan Chase & Co $124.8 billion Wells Fargo & Co. $85.3 billion Goldman Sachs Group Inc. $61.8 billion Bank of America $56.4 billion What is the total value of the nations four largest banks? Associated Press Times-Standard 4/22/09 82. Telescope Mirror. The newly launched Herschel Telescope has a mirror 11.5 feet in diameter while Hubbles mirror is 7.9 feet in diameter. How much larger is Herschels mirror in diameter than Hubbles? Version: Fall 2010

368

CHAPTER 5. DECIMALS

83. Average Temperature. The average temperatures in Sacramento, California in July are a high daytime temperature of 93.8 degrees Fahrenheit and a low nighttime temperature of 60.9 degrees Fahrenheit. What is the change in temperature from day to night? Hint: See Section 2.3 for the formula for comparing temperatures. 84. Average Temperature. The average temperatures in Redding, California in July are a high daytime temperature of 98.2 degrees Fahrenheit and a low nighttime temperature of 64.9 degrees Fahrenheit. What is the change in temperature from day to night? Hint: See Section 2.3 for the formula for comparing temperatures. 85. Net Worth. Net worth is dened as assets minus liabilities. Assets are everything of value that can be converted to cash while liabilities are the total of debts. General Growth Properties, the owners of the Bayshore Mall, have $29.6 billion in assets and $27 billion in liabilities, and have gone bankrupt. What was General Growth Properties net worth before bankruptcy? Times-Standard 4/17/2009 86. Grape crush. The California Department of Food and Agricultures preliminary grape crush report shows that the state produced 3.69 million tons of wine grapes in 2009. Thats just shy of the record 2005 crush of 3.76 million tons. By how many tons short of the record was the crush of 2009? Associated Press-Times-Standard Calif. winegrapes harvest jumped 23% in 09. 87. Turnover. The Labor Departments Job Openings and Labor Turnover Survey claims that employers hired about 4.08 million people in January 2010 while 4.12 million people were red or otherwise left their jobs. How many more people lost jobs than were hired? Convert your answer to a whole number. Associated Press-Times-Standard 03/10/10 Job openings up sharply in January to 2.7M.

l l l
1. 116.6 3. 101.18 5. 91.502 7. 103.45 9. 58.641 11. 6.86 13. 6.739 15. 74.9 Version: Fall 2010

Answers

l l l

17. 52.372 19. 54.575 21. 35.3 23. 5.98 25. 26.13 27. 70.72 29. 90.914 31. 1.643 33. 42.8

5.2. ADDING AND SUBTRACTING DECIMALS 35. 41.1 37. 0.51 39. 8.504 41. 6.47 43. 77.72 45. 7.501 47. 2.201 75. 8.96 49. 0.39 51. 6.895 53. 58.27 55. 54.983 57. 4.314 59. 4.365 61. 10.062 77. 24.99 79. 17.13 81. $328.3 billion 83. 32.9 degrees Fahrenheit 85. $2.6 billion 87. 40, 000 63. 105.75 65. 47.04 67. 12.37 69. 37.96 71. 19.75 73. 19.85

369

Version: Fall 2010

370

CHAPTER 5. DECIMALS

5.3

Multiplying Decimals

Multiplying decimal numbers involves two steps: (1) multiplying the numbers as whole numbers, ignoring the decimal point, and (2) placing the decimal point in the correct position in the product or answer. For example, consider (0.7)(0.08), which asks us to nd the product of seven tenths and eight hundredths. We could change these decimal numbers to fractions, then multiply. (0.7)(0.08) = 8 7 10 100 56 = 1000 = 0.056

The product is 56/1000, or fty six thousandths, which as a decimal is written 0.056. Important Observations. There are two very important observations to be made about the example (0.7)(0.08).

1. In fractional form 7 8 56 = , 10 100 1000 note that the numerator of the product is found by taking the product of the whole numbers 7 and 8. That is, you ignore the decimal points in 0.7 and 0.08 and multiply 7 and 8 as if they were whole numbers. 2. The rst factor, 0.7, has one digit to the right of the decimal point. Its fractional equivalent, 7/10, has one zero in its denominator. The second factor, 0.08, has two digits to the right of the decimal point. Its fractional equivalent, 8/100, has two zeros in its denominator. Therefore, the product 56/1000 is forced to have three zeros in its denominator and its decimal equivalent, 0.056, must therefore have three digits to the right of the decimal point.

Lets look at another example. You Try It! Multiply: (1.86)(9.5) EXAMPLE 1. Simplify: Version: Fall 2010 (2.34)(1.2).

5.3. MULTIPLYING DECIMALS

371

Solution. Change the decimal numbers two and thirty four hundredths and one and two tenths to fractions, then multiply. (2.34)(1.2) = 2 34 2 1 100 10 234 12 = 100 10 2808 = 1000 808 =2 1000 = 2.808 Change decimals to fractions. Change mixed to improper fractions. Multiply numerators and denominators. Change to mixed fraction. Change back to decimal form. Answer: 17.67

Important Observations. We make the same two observations as in the previous example. 1. If we treat the decimal numbers as whole numbers without decimal points, then (234)(12) = 2808, which is the numerator of the fraction 2808/1000 in the solution shown in Example 1. These are also the same digits shown in the answer 2.808. 2. There are two digits to the right of the decimal point in the rst factor 2.34 and one digit to the right of the decimal point in the second factor 1.2. This is a total of three digits to the right of the decimal points in the factors, which is precisely the same number of digits that appear to the right of the decimal point in the answer 2.808. The observations made at the end of the previous two examples lead us to the following method. Multiplying Decimal Numbers. To multiply two decimal numbers, perform the following steps: 1. Ignore the decimal points in the factors and multiply the two factors as if they were whole numbers. 2. Count the number of digits to the right of the decimal point in each factor. Sum these two numbers. 3. Place the decimal point in the product so that the number of digits to the right of the decimal point equals the sum found in step 2.

Version: Fall 2010

372

CHAPTER 5. DECIMALS

You Try It! Multiply: (5.98)(3.7) EXAMPLE 2. Use the steps outlined in Multiplying Decimal Numbers to nd the product in Example 1. Solution. We follow the steps outlined in Multiplying Decimal Numbers. 1. The rst step is to multiply the factors 2.34 and 1.2 as whole numbers, ignoring the decimal points. 234 12 468 234 2808 2. The second step is to nd the sum of the number of digits to the right of the decimal points in each factor. Note that 2.34 has two digits to the right of the decimal point, while 1.2 has one digit to the right of the decimal point. Thus, we have a total of three digits to the right of the decimal points in the factors. 3. The third and nal step is to place the decimal point in the product or answer so that there are a total of three digits to the right of the decimal point. Thus, (2.34)(1.2) = 2.808. Note that this is precisely the same solution found in Example 1. What follows is a convenient way to arrange your work in vertical format. 2.34 1.2 468 2 34 2.808 Answer: 22.126

You Try It! Multiply: (9.582)(8.6) EXAMPLE 3. Simplify: (8.235)(2.3).

Solution. We use the convenient vertical format introduced at the end of Example 2. Version: Fall 2010

5.3. MULTIPLYING DECIMALS 8.235 2.3 2 4705 16 470 18.9405

373

The factor 8.235 has three digits to the right of the decimal point; the factor 2.3 has one digit to the right of the decimal point. Therefore, there must be a total of four digits to the right of the decimal point in the product or answer.

Answer: 82.4052

Multiplying Signed Decimal Numbers


The rules governing multiplication of signed decimal numbers are identical to the rules governing multiplication of integers. Like Signs. The product of two decimal numbers with like signs is positive. That is: (+)(+) = + and ()() = + Unike Signs. The product of two decimal numbers with unlike signs is negative. That is: (+)() = and ()(+) =

You Try It! EXAMPLE 4. Simplify: (2.22)(1.23). Multiply: (3.86)(5.77)

Solution. Ignore the signs to do the multiplication on the left, then consider the signs in the nal answer on the right. As each factor has two digits to the right of the decimal point, there should be a total of 4 decimals to the right of the decimal point in the product. 2.22 1.23 666 444 1 11 1.6206 Like signs give a positive product. Hence: (2.22)(1.23) = 1.6206

Answer: 22.2722

Version: Fall 2010

374

CHAPTER 5. DECIMALS

You Try It! Multiply: (9.23)(0.018) EXAMPLE 5. Simplify: (5.68)(0.012).

Solution. Ignore the signs to do the multiplication on the left, then consider the signs in the nal answer on the right. The rst factor has two digits to the right of the decimal point, the second factor has three. Therefore, there must be a total of ve digits to the right of the decimal point in the product or answer. This necessitates prepending an extra zero in front of our product. 5.68 0.012 1136 568 0.06816 Unlike signs give a negative product. Hence: (5.68)(0.012) = 0.06816

Answer: 0.16614

Order of Operations
The same Rules Guiding Order of Operations also apply to decimal numbers. Rules Guiding Order of Operations. When evaluating expressions, proceed in the following order. 1. Evaluate expressions contained in grouping symbols rst. If grouping symbols are nested, evaluate the expression in the innermost pair of grouping symbols rst. 2. Evaluate all exponents that appear in the expression. 3. Perform all multiplications and divisions in the order that they appear in the expression, moving left to right. 4. Perform all additions and subtractions in the order that they appear in the expression, moving left to right.

You Try It! If a = 3.8 and b = 4.6, evaluate the expression: 2.5a2 b2 EXAMPLE 6. If a = 3.1 and b = 2.4, evaluate a2 3.2b2 . Version: Fall 2010

5.3. MULTIPLYING DECIMALS Solution. Prepare the expression for substitution using parentheses. a2 3.2b2 =
2

375

3.2

Substitute 3.1 for a and 2.4 for b and simplify. a2 3.2b2 = (3.1)2 3.2(2.4)2 = 9.61 3.2(5.76) = 9.61 18.432 = 8.822 Substitute: 3.1 for a, 2.4 for b. Exponents rst: (3.1)2 = 9.61, (2.4)2 = 5.76 Multiply: 3.2(5.76) = 18.432 Subtract: 9.61 18.432 = 8.822 Answer: 14.94

Powers of Ten
Consider: 101 = 10 102 = 10 10 = 100 103 = 10 10 10 = 1, 000 104 = 10 10 10 10 = 10, 000 Note the answer for 104 , a one followed by four zeros! Do you see the pattern? Powers of Ten. In the expression 10n , the exponent matches the number of zeros in the answer. Hence, 10n will be a 1 followed by n zeros.

You Try It! EXAMPLE 7. Simplify: 109 . Solution. 109 should be a 1 followed by 9 zeros. That is, 109 = 1, 000, 000, 000, or one billion. Answer: 1, 000, 000 Simplify: 106

Version: Fall 2010

376

CHAPTER 5. DECIMALS

Multiplying Decimal Numbers by Powers of Ten


Lets multiply 1.234567 by 103 , or equivalently, by 1,000. Ignore the decimal point and multiply the numbers as whole numbers. 1.234567 1000 1234.567000 The sum total of digits to the right of the decimal point in each factor is 6. Therefore, we place the decimal point in the product so that there are six digits to the right of the decimal point. However, the trailing zeros may be removed without changing the value of the product. That is, 1.234567 times 1000 is 1234.567. Note that the decimal point in the product is three places further to the right than in the original factor. This observation leads to the following result. Multiplying a Decimal Number by a Power of Ten. Multiplying a decimal number by 10n will move the decimal point n places to the right.

You Try It! Simplify: 1.234567 102 EXAMPLE 8. Simplify: 1.234567 104

Answer: 123.4567

Solution. Multiplying by 104 (or equivalently, by 10,000) moves the decimal 4 places to the right. Thus, 1.234567 10, 000 = 12345.67.

The Circle
Lets begin with a denition. The Circle. A circle is the collection of all points equidistant from a given point O, called the center of the circle.
O

The segment joining any point on the circle to the center of the circle is called a radius of the circle. In the gure above, the variable r represents the length of the radius of the circle.

Version: Fall 2010

5.3. MULTIPLYING DECIMALS We need another term, the diameter of a circle.

377

The Diameter of a Circle. If two points on a circle are connected with a line segment, then that segment is called a chord of the circle. If the chord passes through the center of the circle, then the chord is called the diameter of the circle. O

In the gure above, the variable d represents the length of the diameter of the circle. Note that the diameter is twice the length of the radius; in symbols, d = 2r.

The Circumference of a Circle When we work with polygons, the perimeter of the polygon is found by summing the lengths of its edges. The circle uses a dierent name for its perimeter. The Circumference of a Circle. The length of the circle is called its circumference. We usually use the variable C to denote the circumference of a circle. That is, if one were to walk along the circle, the total distance traveled in one revolution is the circumference of the circle. The ancient mathematicians of Egypt and Greece noted a striking relation between the circumference of a circle and its diameter. They discovered that whenever you divide a circles circumference by its diameter, you get a constant. That is, if you take a very large circle and divide its circumference by its diameter, you get exactly the same number if you take a very small circle and divide its circumference by its diameter. This common constant was named (pi). Relating the Circumference and Diameter. Whenever a circles circumference is divided by its diameter, the answer is the constant . That is, if C is the circumference of the circle and d is the circles diameter, then C = . d In modern times, we usually multiply both sides of this equation by d to obtain Version: Fall 2010 the formula for the circumference of a circle. C = d

378

CHAPTER 5. DECIMALS

Because the diameter of a circle is twice the length of its radius, we can substitute d = 2r in the last equation to get an alternate form of the circumference equation. C = (2r) = 2r

The number has a rich and storied history. Ancient geometers from Egypt, Babylonia, India, and Greece knew that was slightly larger than 3. The earliest known approximations date from around 1900 BC (Wikipedia); they are 25/8 (Babylonia) and 256/81 (Egypt). The Indian text Shatapatha Brahmana gives as 339/108 3.139. Archimedes (287-212 BC) was the rst to estimate rigorously, approximating the circumference of a circle with inscribed and circumscribed polygons. He was able to prove that 223/71 < < 22/7. Taking the average of these values yields 3.1419. Modern mathematicians have proved that is an irrational number, an innite decimal that never repeats any pattern. Mathematicians, with the help of computers, routinely produce approximations of with billions of digits after the decimal point. Digits of Pi. Here is , correct to the rst fty digits. = 3.14159265358979323846264338327950288419716939937510 . . .

The number of digits of used depends on the application. Working at very small scales, one might keep many digits of , but if you are building a circular garden fence in your backyard, then fewer digits of are needed. You Try It! Find the radius of a circle having radius 14 inches. Use 3.14 EXAMPLE 9. Find the circumference of a circle given its radius is 12 feet. Solution. The circumference of the circle is given by the formula C = d, or, because d = 2r, C = 2r. Substitute 12 for r. C = 2r = 2(12) = 24 Therefore, the circumference of the circle is exactly C = 24 feet. We can approximate the circumference by entering an approximation for . Lets use 3.14. Note: The symbol is read approximately equal to. C = 24 24(3.14) 75.36 feet Answer: 87.92 inches It is important to understand that the solution C = 24 feet is the exact circumference, while C 75.36 feet is only an approximation. Version: Fall 2010

5.3. MULTIPLYING DECIMALS The Area of a Circle

379

Heres a reasonable argument used to help develop a formula for the area of a circle. Start with a circle of radius r and divide it into 8 equal wedges, as shown in the gure that follows.

Rearrange the pieces as shown in the following gure. r r Note that the rearranged pieces almost form a rectangle with length approximately half the circumference of the circle, r, and width approximately r. The area would be approximately A (length)(width) (r)(r) r2 . This approximation would be even better if we doubled the number of wedges of the circle. r r If we doubled the number of wedges again, the resulting gure would even more closely resemble a rectangle with length r and width r. This leads to the following conclusion. The Area of a Circle. The area of a circle of radius r is given by the formula A = r2 .

You Try It! EXAMPLE 10. Find the area of a circle having a diameter of 12.5 meters. Use 3.14 for and round the answer for the area to the nearest tenth of a square meter. Solution. The diameter is twice the radius. d = 2r Version: Fall 2010 Find the area of a circle having radius 12.2 centimeters. Use 3.14

380 Substitute 12.5 for d and solve for r. 12.5 = 2r 12.5 2r = 2 2 6.25 = r

CHAPTER 5. DECIMALS

Substitute 12.5 for d. Divide both sides by 2. Simiplify.

Hence, the radius is 6.25 meters. To nd the area, use the formula A = r2 and substutite: 3.14 for and 6.25 for r. A = (3.14)(6.25)2 = (3.14)(39.0625) = 122.65625 Substitute: 3.14 for , 6.25 for r. Square rst: (6.25)2 = 39.0625. Multiply: (3.14)(39.0625) = 122.65625.

Hence, the approximate area of the circle is A = 122.65625 square meters. To round to the nearest tenth of a square meter, identify the rounding digit and the test digit. Test digit 122. 6 5 625 Rounding digit Because the test digit is greater than or equal to 5, add 1 to the rounding digit and truncate. Thus, correct to the nearest tenth of a square meter, the area of the circle is approximately A 122.7 m2 . Answer: 467.3576 cm2

Version: Fall 2010

5.3. MULTIPLYING DECIMALS

381

l l l
In Exercises 1-28, multiply the decimals. 1. (6.7)(0.03) 2. (2.4)(0.2) 3. (28.9)(5.9) 4. (33.5)(2.1) 5. (4.1)(4.6) 6. (2.6)(8.2) 7. (75.3)(0.4) 8. (21.4)(0.6) 9. (6.98)(0.9) 10. (2.11)(0.04) 11. (57.9)(3.29) 12. (3.58)(16.3) 13. (47.3)(0.9) 14. (30.7)(0.4)

Exercises

l l l

15. (9.9)(6.7) 16. (7.2)(6.1) 17. (19.5)(7.9) 18. (43.4)(8.9) 19. (6.9)(0.3) 20. (7.7)(0.7) 21. (35.3)(3.81) 22. (5.44)(9.58) 23. (2.32)(0.03) 24. (4.48)(0.08) 25. (3.02)(6.7) 26. (1.26)(9.4) 27. (4.98)(6.2) 28. (3.53)(2.9)

In Exercises 29-56, multiply the decimals. 29. (9.41)(0.07) 30. (4.45)(0.4) 31. (7.4)(0.9) 32. (6.9)(0.05) 33. (8.2)(3.7) 34. (7.5)(6.6) 35. (9.72)(9.1) 36. (6.22)(9.4) 37. (6.4)(2.6) 38. (2.3)(4.4) 39. (39.3)(0.8) 40. (57.7)(0.04) 41. (63.1)(0.02) 42. (51.1)(0.8) 43. (90.8)(3.1) 44. (74.7)(2.9) 45. (47.5)(82.1) 46. (14.8)(12.7) 47. (31.1)(4.8) 48. (28.7)(6.8) Version: Fall 2010

382 49. (2.5)(0.07) 50. (1.3)(0.05) 51. (1.02)(0.2) 52. (7.48)(0.1)

CHAPTER 5. DECIMALS 53. (7.81)(5.5) 54. (1.94)(4.2) 55. (2.09)(37.9) 56. (20.6)(15.2)

In Exercises 57-68, multiply the decimal by the given power of 10. 57. 24.264 10 58. 65.722 100 59. 53.867 104 60. 23.216 10 61. 5.096 103 62. 60.890 10
3 4

63. 37.968 103 64. 43.552 103 65. 61.303 100 66. 83.837 1000 67. 74.896 1000 68. 30.728 100

In Exercises 69-80, simplify the given expression. 69. (0.36)(7.4) (2.8)2 70. (8.88)(9.2) (2.3)2 71. 9.4 (7.7)(1.2)2 72. 0.7 (8.7)(9.4)2 73. 5.94 (1.2)(1.8)2 74. 2.6 (9.8)(9.9)2 75. 6.3 4.2(9.3)2 76. 9.9 (4.1)(8.5)2 77. (6.3)(1.88) (2.2)2 78. (4.98)(1.7) 3.52 79. (8.1)(9.4) 1.82 80. (3.63)(5.2) 0.82

81. Given a = 6.24, b = 0.4, and c = 7.2, evaluate the expression a bc2 . 82. Given a = 4.1, b = 1.8, and c = 9.5, evaluate the expression a bc2 . 83. Given a = 2.4, b = 2.1, and c = 4.6, evaluate the expression ab c2 . 84. Given a = 3.3, b = 7.3, and c = 3.4, evaluate the expression ab c2 .

85. Given a = 3.21, b = 3.5, and c = 8.3, evaluate the expression a bc2 . 86. Given a = 7.45, b = 6.1, and c = 3.5, evaluate the expression a bc2 . 87. Given a = 4.5, b = 6.9, and c = 4.6, evaluate the expression ab c2 . 88. Given a = 8.3, b = 8.2, and c = 5.4, evaluate the expression ab c2 .

Version: Fall 2010

5.3. MULTIPLYING DECIMALS 89. A circle has a diameter of 8.56 inches. Using 3.14, nd the circumference of the circle, correct to the nearest tenth of an inch. 90. A circle has a diameter of 14.23 inches. Using 3.14, nd the circumference of the circle, correct to the nearest tenth of an inch. 91. A circle has a diameter of 12.04 inches. Using 3.14, nd the circumference of the circle, correct to the nearest tenth of an inch. 92. A circle has a diameter of 14.11 inches. Using 3.14, nd the circumference of the circle, correct to the nearest tenth of an inch.

383 93. A circle has a diameter of 10.75 inches. Using 3.14, nd the area of the circle, correct to the nearest hundredth of a square inch. 94. A circle has a diameter of 15.49 inches. Using 3.14, nd the area of the circle, correct to the nearest hundredth of a square inch. 95. A circle has a diameter of 13.96 inches. Using 3.14, nd the area of the circle, correct to the nearest hundredth of a square inch. 96. A circle has a diameter of 15.95 inches. Using 3.14, nd the area of the circle, correct to the nearest hundredth of a square inch.

97. Sue has decided to build a circular sh pond near her patio. She wants it to be 15 feet in diameter and 1.5 feet deep. What is the volume of water it will hold? Use 3.14. Hint: The volume of a cylinder is given by the formula V = r2 h, which is the area of the circular base times the height of the cylinder. 98. John has a decision to make regarding his employment. He currently has a job at Taco Loco in Fortuna. After taxes, he makes about $9.20 per hour and works about 168 hours a month. He currently pays $400 per month for rent. He has an opportunity to move to Santa Rosa and take a job at Mi Ultimo Refugio which would pay $10.30 per hour after taxes for 168 hours a month, but his rent would cost $570 per month. a) After paying for housing in Fortuna, how much does he have left over each month for other expenditures? b) After paying for housing in Santa Rosa, how much would he have left over each month for other expenditures? c) For which job would he have more money left after paying rent and how much would it be? 99. John decided to move to Santa Rosa and take the job at Mi Ultimo Refugio (see Exercise 98). He was able to increase his income because he could work 4 Sundays a month at time-and-a half. So now he worked 32 hours a month at time-and-a-half and 136 hours at the regular rate of $10.30 (all after taxes were removed). Note: He previously had worked 168 hours per month at $10.30 per hour. a) What was his new monthly income? b) How much did his monthly income increase? Version: Fall 2010

384

CHAPTER 5. DECIMALS

100. Electric Bill. On a recent bill, PGE charged $0.11531 per Kwh for the rst 333 Kwh of electrical power used. If a household used 312 Kwh of power, what was their electrical bill? 101. Cabernet. In Napa Valley, one acre of good land can produce about 3.5 tons of quality grapes. At an average price of $3,414 per ton for premium cabernet, how much money could you generate on one acre of cabernet farming? Associated Press-Times-Standard 03/11/10 Grape moth threatens Napa Valley growing method. 102. Fertilizer. Using the 2008 Ohio Farm Custom Rates, the average cost for spreading dry bulk fertilizer is about $4.50 per acre. What is the cost to fertilize 50 acres? 103. Agribusiness. Huge corporate agribarns house 1000 pigs each. a) If each pig weighs approximately 100 pounds, how many pounds of pig is in each warehouse? b) At an average $1.29 per pound, what is the total cash value for a corporate agribarn? Associated Press-Times-Standard 12/29/09 Pressure rises to stop antibiotics in agriculture. 104. Shipwrecks. A dozen centuries-old shipwrecks were found in the Baltic Sea by a gas company building an underwater pipeline between Russia and Germany. The 12 wrecks were found in a 30-mile-long and 1.2-mile-wide corridor at a depth of 430 feet. Model the corridor with a rectangle and nd the approximate area of the region where the ships were found. Associated Press-Times-Standard 03/10/10 Centuries-old shipwrecks found in Baltic Sea. 105. Radio dish. The diameter of the workhorse eet of radio telescopes, like the one in Goldstone, California, is 230 feet. What is the circumference of the radio telescope dish to the nearest tenth? Associated Press-Times-Standard 03/09/2010 NASA will repair deep space antenna in California desert.

l l l
1. 0.201 3. 170.51 5. 18.86 7. 30.12 9. 6.282 11. 190.491 13. 42.57 15. 66.33 Version: Fall 2010

Answers

l l l

17. 154.05 19. 2.07 21. 134.493 23. 0.0696 25. 20.234 27. 30.876 29. 0.6587 31. 6.66

5.3. MULTIPLYING DECIMALS 33. 30.34 35. 88.452 37. 16.64 39. 31.44 41. 1.262 43. 281.48 45. 3899.75 47. 149.28 49. 0.175 51. 0.204 53. 42.955 55. 79.211 57. 242.64 59. 538670 61. 5096 63. 37968 65. 6130.3 67. 74896 69. 5.176 71. 20.488 87. 9.89 89. 26.9 in. 91. 37.8 in. 93. 90.72 square inches 95. 152.98 square inches 97. 264.9375 cubic feet 99. a) $1895.20 b) $164.80 101. $11, 949 103. a) 100, 000 pounds b) $129,000 105. 722.2 feet 73. 9.828 75. 356.958 77. 7.004 79. 79.38 81. 26.976 83. 16.12 85. 244.325

385

Version: Fall 2010

386

CHAPTER 5. DECIMALS

5.4

Dividing Decimals

In this and following sections we make use of the terms divisor, dividend, quotient, and remainder. Divisor, Dividend, Quotient, and Remainder. This schematic reminds readers of the position of these terms in the division process. quotient divisor) dividend ... remainder

Now that these terms are dened, we begin the discussion of division of decimal numbers. Suppose that we wish to divide 637 by 100. We could do this in fraction form, change the result to a mixed fraction, then the mixed fraction to decimal form. 37 637 =6 = 6.37 100 100 We can also arrange the division much as we would the division of two whole numbers. 6.37 100)637.00 600 37 0 30 0 7 00 7 00 0 Note that adding two zeros after the decimal point in the dividend does not change the value of 637. Further, note that we proceed as if we are dividing two whole numbers, placing the decimal point in the quotient directly above the decimal point in the dividend. These observations lead to the following algorithm. Dividing a Decimal by a Whole Number. To divide a decimal number by a whole number, proceed as follows: 1. Set up the long division as you would the division of two whole numbers. 2. Perform the division as if the numbers were both whole numbers, adding zeros to the right of the decimal point in the dividend as necessary to complete the division. 3. Place the decimal point in the quotient immediately above the decimal Version: Fall 2010 point in the dividend.

5.4. DIVIDING DECIMALS

387

You Try It! EXAMPLE 1. Divide 23 by 20. Solution. Arrange as if using long division to divide whole numbers, adding enough zeros to the right of the decimal point in the dividend to complete the division. 1.15 20)23.00 20 30 20 1 00 1 00 0 Hence, 23 divided by 20 is 1.15.

Adding Zeros to the Right of the Decimal Point. Usually, one does not immediately see how many zeros to the right of the decimal point in the dividend are needed. These zeros are usually added at each step of the division, until the division is completed or the user is willing to terminate the process and accept only an estimate of the quotient.

You Try It! EXAMPLE 2. Divide: 155.2 25. Solution. Arrange as if using long division to divide whole numbers, and begin. 6.2 25)155.2 150 52 50 2 We still have a nonzero remainder. Adding another zero does no good. 6.20 25)155.20 150 52 50 20 Version: Fall 2010 Divide: 42.55 23

388

CHAPTER 5. DECIMALS

However, if we add one more additional zero, the division completes with a zero remainder. 6.208 25)155.200 150 52 50 200 200 0 Answer: 1.85 Thus, 155.2 divided by 25 is 6.208.

Decimal Divisors
When the divisor contains a decimal point, we have a little work to do before we begin the division process. Suppose that we wish to divide 1.25 by 2.5. In fraction form, we could start with 1.25 , 2.5 then clear the decimals from the denominator by multiplying both numerator and denominator by 10. Note: Recall that multiplying by 10 moves the decimal point one place to the right. 1.25 10 1.25 = 2.5 2.5 10 12.5 = 25 Thus, dividing 1.25 by 2.5 is equivalent to dividing 12.5 by 25. This we know how to do. 0.5 25)12.5 12 5 0 Thus, 1.25 divided by 2.5 is 0.5. Writing Mathematics. Never write .5. Always add the leading zero in the ones place and write 0.5. Instead of working in fraction form, we can take care of positioning the decimal point in the long division framework. Start with: Version: Fall 2010

5.4. DIVIDING DECIMALS

389

2.5)1.25 Move the decimal point in the divisor to the end of the divisor, then move the decimal point in the dividend an equal number of places. 2.5 )1.2 5 Thus, the division becomes 25)12.5 and we proceed as above to nd the quotient. This discussion motivates the following algorithm. Dividing by a Decimal Divisor. If the divisor contains a decimal, proceed as follows: 1. Move the decimal to the end of the divisor. 2. Move the decimal in the dividend an equal number of places.

We can then complete the division using the rules for dividing a decimal by a whole number. You Try It! EXAMPLE 3. Divide: 0.36)4.392 Solution. Move the decimal in the divisor to the end of the divisor. Move the decimal in the dividend an equal number of places (two places) to the right. 0.36 )4.39 2 Now we can follow the algorithm for dividing a decimal number by a whole number. 12.2 36)439.2 36 79 72 72 72 0 Divide: 0.45)36.99

Version: Fall 2010

390 Answer: 82.2 Thus, 4.392 divided by 0.36 is 12.2.

CHAPTER 5. DECIMALS

Dividing Signed Decimal Numbers


The rules governing division of signed decimal numbers are identical to the rules governing division of integers. Like Signs. The quotient of two decimal numbers with like signs is positive. That is: () (+) =+ and =+ (+) () Unlike Signs. The quotient of two decimal numbers with unlike signs is negative. That is: (+) () = and = () (+)

You Try It! Divide: 0.0113 0.05 EXAMPLE 4. Divide: 0.03 0.024. Solution. First, divide the magnitudes. Move the decimal in the divisor to the end of the divisor. Move the decimal in the dividend an equal number of places (three places) to the right. Note that this requires an extra trailing zero in the dividend. 0.024 )0.030 Our problem then becomes: 24)30 We can now follow the algorithm for dividing a decimal number by a whole number. Note that we have to add two trailing zeros in the dividend to complete the division with a zero remainder. 1.25 24)30.00 24 60 48 1 20 1 20 0 Version: Fall 2010

5.4. DIVIDING DECIMALS

391

Finally, because the quotient of unlike signs is negative, 0.03 divided by 0.024 is 1.25. That is, 0.03 = 1.25. 0.024 Answer: 0.226

Rounding
Sometimes an exact decimal representation of a fraction is not needed and is an approximation is more than adequate. You Try It! EXAMPLE 5. Convert 4/7 to a decimal. Round your answer to the nearest hundredth. Solution. We need to carry the division one place beyond the hundredths place. 0.571 7)4.000 35 50 49 10 7 3 To round to the nearest hundredth, rst identify the rounding and test digits. Test digit 0.5 7 1 Rounding digit Because the test digit is less 5, leave the rounding digit alone and truncate. Therefore, correct to the nearest hundredth, 4/7 0.57. Convert 5/7 to a decimal. Round your answer to the nearest hundredth.

Answer: 0.71

Version: Fall 2010

392

CHAPTER 5. DECIMALS

Dividing by Powers of Ten


Recall: 101 = 10 102 = 10 10 = 100 103 = 10 10 10 = 1000 Powers of Ten. In the expression 10n , the exponent matches the number of zeros in the answer. Hence, 10n will be a 1 followed by n zeros. Thus, 104 = 10, 000, 105 = 100, 000, etc. The exponent tells us how many zeros will follow the 1. Lets divide 123456.7 by 1000. 123.4567 1000)123456.7000 1000 2345 2000 3456 3000 456 7 400 0 56 70 50 00 6 700 6 000 7000 7000 0 Note the result: 123456.7 divided by 1000 is 123.4567. Dividing by 1000 moves the decimal point 3 places to the left! 123456.7 1000 = 123 456.7 = 123.4567 This discussion leads to the following result. Dividing a Decimal by a Power of Ten. Dividing a decimal number by 10n will move the decimal point n places to the left.

Version: Fall 2010

5.4. DIVIDING DECIMALS

393

You Try It! EXAMPLE 6. Simplify: 123456.7 104 Simplify: 123456.7 102

Solution.Dividing by 104 (or equivalently, 10,000) moves the decimal point four places to the left. Thus, 123456.7 104 = 12.34567.

Answer: 1234.567

Order of Operations
We remind readers of the Rules Guiding Order of Operations. Rules Guiding Order of Operations. When evaluating expressions, proceed in the following order. 1. Evaluate expressions contained in grouping symbols rst. If grouping symbols are nested, evaluate the expression in the innermost pair of grouping symbols rst. 2. Evaluate all exponents that appear in the expression. 3. Perform all multiplications and divisions in the order that they appear in the expression, moving left to right. 4. Perform all additions and subtractions in the order that they appear in the expression, moving left to right.

In addition, when fractions are present: Fractional Expressions. If a fractional expression is present, simplify the numerator and denominator separately, then divide.

You Try It! EXAMPLE 7. Evaluate ab/(c + d), given that a = 2.1, b = 3.4, c = 1.3, and d = 1.1. Solution. Recall that it is good practice to prepare an expression for substitution by using parentheses. ab/(c + d) = / + Version: Fall 2010 If a = 2.1, b = 1.7, c = 4, and d = 0.05, evaluate: a+b cd

394

CHAPTER 5. DECIMALS

Substitute the given values for a, b, c, and d, then use the Rules Guiding Order of Operations to simplify the resulting expression. ab/(c + d) = (2.1)(3.4)/((1.3) + (1.1)) 2.1, 3.4, 1.3, 1.1 for a, b, c, d. = (2.1)(3.4)/(0.2) Parens: (1.3) + (1.1) = 0.2. = 7.14/(0.2) = 35.7 Answer: 2 Multiply: (2.1)(3.4) = 7.14. Divide: 7.14/(0.2) = 35.7.

You Try It! If a = 0.5 and b = 0.125, evaluate: 2a b a + 2b EXAMPLE 8. Given a = 0.1 and b = 0.3, evaluate the expression a + 2b . 2a + b Solution. Substitute the given values, then use the Rules Guiding Order of Operations to simplify the resulting expression. a + 2b (0.1) + 2(0.3) = 2a + b 2(0.1) + (0.3) 0.1 for a, 0.3 for b.

Simplify the numerator, simplify the denominator, then divide. = = 0.1 + (0.6) 0.2 + (0.3) 0.5 0.1 Numerator: 2(0.3) = 0.6. Denominator: 2(0.1) = 0.2. Numerator: 0.1 + (0.6) = 0.5. Denominator: 0.2 + (0.3) = 0.1. Divide: 0.5/(0.1) = 5.

=5 Answer: 4.5

Version: Fall 2010

5.4. DIVIDING DECIMALS

395

l l l
In Exercises 1-16, divide the numbers. 1. 2. 3. 4. 5. 6. 7. 8. 39 52 16 25 755.3 83 410.4 76 333 74 117 65 32.12 73 12.32 44

Exercises

l l l

9. 10. 11. 12. 13. 14. 15. 16.

37.63 71 20.46 31 138 92 110 25 17 25 18 75 229.5 51 525.6 72

In Exercises 17-40, divide the decimals. 17. 18. 19. 20. 21. 22. 23. 24. 0.3478 0.47 0.4559 0.97 1.694 2.2 1.008 1.8 43.61 4.9 22.78 3.4 1.107 0.41 2.465 0.29 25. 26. 27. 28. 29. 30. 31. 32. 2.958 0.51 5.141 0.53 71.76 7.8 14.08 8.8 0.8649 0.93 0.3901 0.83 0.6958 0.71 0.1829 0.31 Version: Fall 2010

396 1.248 0.52 6.375 34. 0.85 62.56 35. 9.2 28.08 36. 7.8 33.

CHAPTER 5. DECIMALS 37. 6.278 8.6 3.185 38. 4.9 2.698 39. 7.1 4.959 40. 8.7

In Exercises 41-64, divide the decimals. 11.04 1.6 31.27 5.3 3.024 5.6 3.498 5.3 0.1056 0.22 0.2952 0.72 0.3204 0.89 0.3306 0.38 1.419 0.43 1.625 0.25 16.72 2.2 66.24 9.2 2.088 0.87 2.025 0.75 1.634 8.6 3.094 3.4 0.119 0.85 0.5766 0.62 3.591 6.3 3.016 5.8 36.96 4.4 78.26 8.6 2.156 0.98 6.072 0.66

41. 42. 43. 44. 45. 46. 47. 48. 49. 50. 51. 52.

53. 54. 55. 56. 57. 58. 59. 60. 61. 62. 63. 64.

In Exercises 65-76, divide the decimal by the given power of 10. Version: Fall 2010 65. 524.35 100 66. 849.39 100

5.4. DIVIDING DECIMALS 67. 68. 69. 70. 71. 563.94 103 884.15 103 116.81 102 578.01 103 694.55 10 72. 73. 74. 75. 76. 578.68 100 341.16 103 46.63 104 113.02 1000 520.77 1000

397

77. Compute the quotient 52/83, and round your answer to the nearest tenth. 78. Compute the quotient 43/82, and round your answer to the nearest tenth. 79. Compute the quotient 51/59, and round your answer to the nearest tenth. 80. Compute the quotient 17/69, and round your answer to the nearest tenth. 81. Compute the quotient 5/74, and round your answer to the nearest hundredth. 82. Compute the quotient 3/41, and round your answer to the nearest hundredth.

83. Compute the quotient 5/94, and round your answer to the nearest hundredth. 84. Compute the quotient 3/75, and round your answer to the nearest hundredth. 85. Compute the quotient 7/72, and round your answer to the nearest hundredth. 86. Compute the quotient 4/57, and round your answer to the nearest hundredth. 87. Compute the quotient 16/86, and round your answer to the nearest tenth. 88. Compute the quotient 21/38, and round your answer to the nearest tenth.

In Exercises 89-100, simplify the given expression. 89. 90. 91. 7.5 7.1 19.5 0.54 1.5(8.8) (18.6) 1.8 95. 12.9 (10.98) 0.52 5.1 (16.5) 96. (1.5)2 97. 98. 9.5 1.6 3.7 3.6

17.76 (11.7) 0.52 14.8 2.1 92. 2.62 18.22 6.7 93. 14.75 7.75 1.4 13.25 6.84 (2.1)

94.

6.5(1.6) 3.35 2.75 14.98 9.6 99. 17.99 19.99 5.6 7.5 100. 5.05 1.5 Version: Fall 2010

398

CHAPTER 5. DECIMALS

101. Given a = 2.21, c = 3.3, and d = 0.5, evaluate and simplify the following expression. ac d2 102. Given a = 2.8, c = 14.68, and d = 0.5, evaluate and simplify the following expression. ac d2 103. Given a = 5.8, b = 10.37, c = 4.8, and d = 5.64, evaluate and simplify the following expression: ab cd 104. Given a = 10.79, b = 3.94, c = 3.2, and d = 8.11, evaluate and simplify the

following expression: ab cd 105. Given a = 1.5, b = 4.7, c = 18.8, and d = 11.75, evaluate and simplify the following expression. ab c d 106. Given a = 9.3, b = 6.6, c = 14.27, and d = 0.2, evaluate and simplify the following expression. ab c d

107. Biodiesel plants. There are about 180 biodiesel plants operating in about 40 states. Of the states that have them, what is the average number of biodiesel plants per state? Associated Press-Times-Standard 01/02/10 Fledgling biofuel industry ends year on a dour note. 108. Bat fungus. A fungus called white-nose syndrome has killed an estimated 500,000 bats throughout the country. This means about 2,400,000 pounds of bugs arent eaten over the year, says Forest Service biologist Becky Ewing. How many pounds of insects does an average bat eat annually? Associated Press-Times-Standard 5/2/09 109. Patent backlog. In the U.S. Patent and Trademark Oce, 6000 examiners have a backlog of 770,000 new, unexamined applications for patents. How many applications is that for each examiner to catch up on? Round your answer to the nearest tenth. Associated Press-TimesStandard 5/5/09 110. Doing well. The large health insurer Wellpoint, Inc., owner of Anthem Blue Cross, earned $536 million in the last three months of 2009. What was the average earnings per month for the insurer over that period? Round to the nearest million. Associated Press-Times-Standard 02/09/10 HHS secretary asks Anthem Blue Cross to justify rate hike. 111. Cyber attacks. The Pentagon has spent $100 million over a six-month period responding to and repairing damage from cyber-attacks and other computer network problems. Whats the average amount of money spent per month over that time? Round your answer to the nearest hundredth of a million. Associated Press-Times-Standard 4/19/09

Version: Fall 2010

5.4. DIVIDING DECIMALS

399

112. Daily milk. The average California cow can produce 2,305 gallons of milk annually. How much milk can a cow produce each day? Round your answer to the nearest hundredth of a gallon. http://www.moomilk.com/faq.htm 113. Media mail. To promote her business, Theresa mails several packages via Media Mail. One package weighing 2 lbs. costs $2.77, another package weighing 3 lbs. costs $3.16, and the third package weighing 5 lbs. costs $3.94 to mail. What was the average cost per pound to mail the packages? Round your result to the nearest penny. http://www.usps.com/prices/media-mailprices.htm

l l l
1. 0.75 3. 9.1 5. 4.5 7. 0.44 9. 0.53 11. 1.5 13. 0.68 15. 4.5 17. 0.74 19. 0.77 21. 8.9 23. 2.7 25. 5.8 27. 9.2 29. 0.93 31. 0.98

Answers
33. 2.4 35. 6.8

l l l

37. 0.73 39. 0.38 41. 6.9 43. 0.54 45. 0.48 47. 0.36 49. 3.3 51. 7.6 53. 2.4 55. 0.19 57. 0.14 59. 0.57 61. 8.4 63. 2.2 65. 5.2435 67. 0.56394 Version: Fall 2010

400 69. 1.1681 71. 69.455 73. 0.34116 75. 0.11302 77. 0.6 79. 0.9

CHAPTER 5. DECIMALS 93. 3.56 95. 7.68 97. 5.25 99. 12.29 101. 22.04 103. 19.25

81. 0.07 105. 2.2 83. 0.05 85. 0.10 87. 0.2 89. 62.5 91. 117.84 107. 4.5 biodiesel plants 109. 128.3 111. $16.67 million 113. $0.99 per pound

Version: Fall 2010

5.5. FRACTIONS AND DECIMALS

401

5.5

Fractions and Decimals

When converting a fraction to a decimal, only one of two things can happen. Either the process will terminate or the decimal representation will begin to repeat a pattern of digits. In each case, the procedure for changing a fraction to a decimal is the same. Changing a Fraction to a Decimal. To change a fraction to a decimal, divide the numerator by the denominator. Hint: If you rst reduce the fraction to lowest terms, the numbers will be smaller and the division will be a bit easier as a result.

Terminating Decimals
Terminating Decimals. First reduce the fraction to lowest terms. If the denominator of the resulting fraction has a prime factorization consisting of strictly twos and/or ves, then the decimal representation will terminate.

You Try It! EXAMPLE 1. Change 15/48 to a decimal. Solution. First, reduce the fraction to lowest terms. 15 35 = 48 3 16 5 = 16 Next, note that the denominator of 5/16 has prime factorization 16 = 2 2 2 2. It consists only of twos. Hence, the decimal representation of 5/16 should terminate. 0.3125 16)5.0000 48 20 16 40 32 80 80 0 The zero remainder terminates the process. Hence, 5/16 = 0.3125. Version: Fall 2010 Answer: 0.625 Change 10/16 to a decimal.

402

CHAPTER 5. DECIMALS

You Try It!


11 Change 7 20 to a decimal. 7 EXAMPLE 2. Change 3 20 to a decimal.

Solution. Note that 7/20 is reduced to lowest terms and its denominator has prime factorization 20 = 2 2 5. It consists only of twos and ves. Hence, the decimal representation of 7/20 should terminate. 0.35 20)7.00 60 1 00 1 00 0 The zero remainder terminates the process. Hence, 7/20 = 0.35. Therefore, 7 3 20 = 3.35.

Answer: 7.55

Repeating Decimals

Repeating Decimals. First reduce the fraction to lowest terms. If the prime factorization of the resulting denominator does not consist strictly of twos and ves, then the division process will never have a remainder of zero. However, repeated patterns of digits must eventually reveal themselves.

You Try It! Change 5/12 to a decimal. EXAMPLE 3. Change 1/12 to a decimal. Solution. Note that 1/12 is reduced to lowest terms and the denominator has a prime factorization 12 = 2 2 3 that does not consist strictly of twos and ves. Hence, the decimal representation of 1/12 will not terminate. We need to carry out the division until a remainder reappears for a second time. This will indicate repetition is beginning. .083 12)1.000 96 40 36 4 Version: Fall 2010

5.5. FRACTIONS AND DECIMALS

403

Note the second appearance of 4 as a remainder in the division above. This is an indication that repetition is beginning. However, to be sure, lets carry the division out for a couple more places. .08333 12)1.00000 96 40 36 40 36 40 36 4 Note how the remainder 4 repeats over and over. In the quotient, note how the digit 3 repeats over and over. It is pretty evident that if we were to carry out the division a few more places, we would get 1 = 0.833333 . . . 12 The ellipsis is a symbolic way of saying that the threes will repeat forever. It is the mathematical equivalent of the word etcetera. Answer: 0.41666 . . .

There is an alternative notation to the ellipsis, namely 1 = 0.083. 12 The bar over the 3 (called a repeating bar) indicates that the 3 will repeat indenitely. That is, 0.083 = 0.083333 . . . .

Using the Repeating Bar. To use the repeating bar notation, take whatever block of digits are under the repeating bar and duplicate that block of digits innitely to the right. Thus, for example: 5.345 = 5.3454545 . . . . 0.142857 = 0.142857142857142857 . . ..

Version: Fall 2010

404

CHAPTER 5. DECIMALS

Important Observation. Although 0.833 will also produce 0.8333333 . . ., as a rule we should use as few digits as possible under the repeating bar. Thus, 0.83 is preferred over 0.833.

You Try It! Change 5/33 to a decimal. EXAMPLE 4. Change 23/111 to a decimal. Solution. The denominator of 23/111 has prime factorization 111 = 3 37 and does not consist strictly of twos and ves. Hence, the decimal representation will not terminate. We need to perform the division until we spot a repeated remainder. 0.207 111)23.000 22 2 800 777 23 Note the return of 23 as a remainder. Thus, the digit pattern in the quotient should start anew, but lets add a few places more to our division to be sure. 0.207207 111)23.000000 22 2 800 777 230 222 800 777 23 Aha! Again a remainder of 23. Repetition! At this point, we are condent that 23 = 0.207207 . . . . 111 Using a repeating bar, this result can be written 23 = 0.207. 111 Answer: 0.151515 . . .

Version: Fall 2010

5.5. FRACTIONS AND DECIMALS

405

Expressions Containing Both Decimals and Fractions


At this point we can convert fractions to decimals, and vice-versa, we can convert decimals to fractions. Therefore, we should be able to evaluate expressions that contain a mix of fraction and decimal numbers. You Try It! EXAMPLE 5. Simplify: 3 1.25. 8 Solution. Lets change 1.25 to an improper fraction. 1.25 = 125 100 5 = 4 Two decimal places = two zeros. Reduce to lowest terms. Simplify: 7 6.5 8

In the original problem, replace 1.25 with 5/4, make equivalent fractions with a common denominator, then subtract. 3 3 1.25 = 8 8 3 = 8 3 = 8 3 = + 8 13 = 8 5 4 52 42 10 8 10 8 Replace 1.25 with 5/4. Equivalent fractions, LCD = 8. Simplify numerator and denominator. Add the opposite. Add.

Thus, 3/8 1.25 = 13/8. Alternate Solution. Because 3/8 is reduced to lowest terms and 8 = 2 2 2 consists only of twos, the decimal representation of 3/8 will terminate. 0.375 8)3.000 24 60 56 40 40 Hence, 3/8 = 0.375. Now, replace 3/8 in the original problem with 0.375, then simplify. 3 1.25 = 0.375 1.25 8 = 0.375 + (1.25) = 1.625 Replace 3/8 with 0.375. Add the opposite. Add. Version: Fall 2010

406 Thus, 3/8 1.25 = 1.625.

CHAPTER 5. DECIMALS

Are They the Same? The rst method produced 13/8 as an answer; the second method produced 1.625. Are these the same results? One way to nd out is to change 1.625 to an improper fraction. 1.625 = 1625 1000 5 5 5 5 13 = 222555 13 = 222 13 = 8 Three places = three zeros. Prime factor. Cancel common factors. Simplify.

3 Answer: 7 8 or 7.375

Thus, the two answers are the same.

Lets look at another example. You Try It! Simplify: 4 + 0.25 9 2 + 0.35. 3 Solution. Lets attack this expression by rst changing 0.35 to a fraction. EXAMPLE 6. Simplify: 2 2 35 + 0.35 = + 3 3 100 2 7 = + 3 20 Change 0.35 to a fraction. Reduce 35/100 to lowest terms.

Find an LCD, make equivalent fractions, then add. 2 20 73 + 3 20 20 3 40 21 = + 60 60 19 = 60 2 19 Thus, + 0.35 = . 3 60 = Equivalent fractions with LCD = 60. Simplify numerators and denominators. Add.

Answer: 7/36

In Example 6, we run into trouble if we try to change 2/3 to a decimal. The decimal representation for 2/3 is a repeating decimal (the denominator is not made up of only twos and ves). Indeed, 2/3 = 0.6. To add 0.6 and 0.35, we have to align the decimal points, then begin adding at the right end. But 0.6 has no right end! This observation leads to the following piece of advice. Version: Fall 2010

5.5. FRACTIONS AND DECIMALS

407

Important Observation. When presented with a problem containing both decimals and fractions, if the decimal representation of any fraction repeats, its best to rst change all numbers to fractions, then simplify.

Version: Fall 2010

408

CHAPTER 5. DECIMALS

l l l

Exercises

l l l

In Exercises 1-20, convert the given fraction to a terminating decimal. 1. 2. 3. 4. 5. 6. 7. 8. 9. 10. 59 16 19 5 35 4 21 4 1 16 14 5 6 8 7 175 3 2 15 16 11. 12. 13. 14. 15. 16. 17. 18. 19. 20. 119 175 4 8 9 8 5 2 78 240 150 96 25 10 2 4 9 24 216 150

In Exercises 21-44, convert the given fraction to a repeating decimal. Use the repeating bar notation. 21. 22. 23. 24. 25. 26. 256 180 268 180 364 12 292 36 81 110 82 99 Version: Fall 2010 27. 28. 29. 30. 31. 32. 76 15 23 9 50 99 53 99 61 15 37 18

5.5. FRACTIONS AND DECIMALS 33. 34. 35. 36. 37. 38. 98 66 305 330 190 495 102 396 13 15 65 36 39. 40. 41. 42. 43. 44. 532 21 44 60 26 198 686 231 47 66 41 198

409

In Exercises 45-52, simplify the given expression by rst converting the fraction into a terminating decimal. 7 7.4 4 3 46. 2.73 2 7 47. + 5.31 5 7 48. + 3.3 4 45. 49. 9 8.61 10 3 50. + 3.7 4 6 51. 7.65 5 3 52. + 8.1 10

In Exercises 53-60, simplify the given expression by rst converting the decimal into a fraction. 7 2.9 6 11 54. + 1.12 6 4 55. 0.32 3 11 0.375 56. 6 53. 2 57. + 0.9 3 2 58. 0.1 3 4 59. 2.6 3 5 60. + 2.3 6

Version: Fall 2010

410 In Exercises 61-64, simplify the given expression. 61. 5 + 2.375 6 5 62. + 0.55 3 11 63. + 8.2 8 13 64. + 8.4 8 65.

CHAPTER 5. DECIMALS

7 + 1.2 10 7 66. 3.34 5 11 67. + 0.375 6 5 68. 1.1 3

l l l
1. 3.6875 3. 8.75 5. 0.0625 7. 0.75 9. 1.5 11. 0.68 13. 1.125 15. 0.325 17. 2.5 19. 0.375 21. 1.42 23. 30.3 25. 0.736 27. 5.06 29. 0.50 31. 4.06 33. 1.48 35. 0.38 Version: Fall 2010

Answers

l l l

37. 0.86 39. 25.3 41. 0.13 43. 0.712 45. 5.65 47. 6.71 49. 7.71 51. 6.45 53. 55. 57. 26 15 124 75

7 30 19 15

59. 61.

77 24

63. 9.575 65. 0.5 67. 35 24

5.6. EQUATIONS WITH DECIMALS

411

5.6

Equations With Decimals

We can add or subtract the same decimal number from both sides of an equation without aecting the solution. You Try It! EXAMPLE 1. Solve for x: x 1.35 = 2.6 x 1.35 + 1.35 = 2.6 + 1.35 x = 1.25 x 1.35 = 2.6. Solve for x: x + 1.25 = 0.6

Solution. To undo subtracting 1.35, add 1.35 to both sides of the equation. Original equation. Add 1.35 to both sides. Simplify: 2.6 + 1.35 = 1.25.

Answer: 0.65

We can still multiply both sides of an equation by the same decimal number without aecting the solution. You Try It! EXAMPLE 2. Solve for x: x = 4.2. 0.35 Solution. To undo dividing by 0.35, multiply both sides of the equation by 0.35. x = 4.2 Original equation. 0.35 x 0.35 = 0.35 (4.2) Multiply both sides by 0.35. 0.35 x = 1.470 Simplify: 0.35(4.2) = 1.470. Solve for y: y = 1.52 0.37

Answer: 0.5624

We can still divide both sides of an equation by the same decimal number without aecting the solution. You Try It! EXAMPLE 3. Solve for x: 1.2x = 4.08. Solve for z: 2.5z = 1.4

Solution. To undo multiplying by 1.2, divide both sides of the equation by 1.2. 1.2x = 4.08 4.08 1.2x = 1.2 1.2 x = 3.4 Original equation. Divide both sides by 1.2. Simplify: 4.08/(1.2) = 3.4. Version: Fall 2010

Answer: 0.56

412

CHAPTER 5. DECIMALS

Combining Operations
We sometimes need to combine operations. You Try It! Solve for u: 0.02u 3.2 = 1.75 EXAMPLE 4. Solve for x: 3.8x 1.7 = 17.28. Original equation. Add 1.7 to both sides Simplify: 17.28 + 1.7 = 15.58.

Solution. To undo subtracting 1.7, add 1.7 to both sides of the equation. 3.8x 1.7 = 17.28 3.8x 1.7 + 1.7 = 17.28 + 1.7 3.8x = 15.58

Next, to undo multiplying by 3.8, divide both sides of the equation by 3.8. 3.8x 15.58 = 3.8 3.8 x = 4.1 Answer: 72.5 Divide both sides by 3.8. Simplify: 15.58/(3.8) = 4.1.

Combining Like Terms


Combining like terms with decimal coecients is done in the same manner as combining like terms with integer coecients. You Try It! Simplify: 1.185t + 3.2t EXAMPLE 5. Simplify the expression: 3.2x + 1.16x. Solution. To combine these like terms we must add the coecients. To add coecients with unlike signs, rst subtract the coecient with the smaller magnitude from the coecient with the larger magnitude. 3.20 1.16 2.04 We can now combine like terms as follows: 3.2x + 1.16x = 2.04x Answer: 2.015t Prex the sign of the decimal number having the larger magnitude. Hence: 3.2 + 1.16 = 2.04.

Version: Fall 2010

5.6. EQUATIONS WITH DECIMALS When solving equations, we sometimes need to combine like terms.

413

You Try It! EXAMPLE 6. Solve the equation for x: 4.2 3.1x + 2x = 7.02. Solution. Combine like terms on the left-hand side of the equation. 4.2 3.1x + 2x = 7.02 4.2 1.1x = 7.02 4.2 1.1x 4.2 = 7.02 4.2 1.1x = 11.02 11.22 1.1x = 1.1 1.1 x = 10.2 Original equation. Combine like terms: 3.1x + 2x = 1.1x. Subtract 4.2 from both sides. Subtract: 7.02 4.2 = 11.22. Divide both sides by 1.1. Divide: 11.22/(1.1) = 10.2. Solve for r: 4.2 + 3.6r 4.1r = 1.86

Thus, the solution of the equation is 10.2. Check. Like all equations, we can check our solution by substituting our answer in the original equation. 4.2 3.1x + 2x = 7.02 Original equation. 4.2 3.1(10.2) + 2(10.2) = 7.02 Substitute 10.2 for x. 4.2 31.62 + 20.4 = 7.02 Multiply: 3.1(10.2) = 31.62, 2(10.2) = 20.4. 27.42 + 20.4 = 7.02 Order of Ops: Add, left to right. 4.2 31.62 = 27.42. 7.02 = 7.02 Add: 27.42 + 20.4 = 7.02. Because the last line is a true statement, the solution x = 10.2 checks. Answer: 12.12

Using the Distributive Property


Sometimes we will need to employ the distributive property when solving equations. Distributive Property. Let a, b, and c be any numbers. Then, a(b + c) = ab + ac.

Version: Fall 2010

414

CHAPTER 5. DECIMALS

You Try It! Solve for x: 2.5x 0.1(x 2.3) = 8.03 EXAMPLE 7. Solve the equation for x: 6.3x 0.4(x 1.2) = 0.86. Solution. We rst distribute the 0.4 times each term in the parentheses, then combine like terms. 6.3x 0.4(x 1.2) = 0.86 6.3x 0.4x + 0.48 = 0.86 6.7x + 0.48 = 0.86 Original equation. Distribute. Note that 0.4(1.2) = 0.48. Combine like terms.

Next, subtract 0.48 from both sides, then divide both sides of the resulting equation by 6.7. 6.7x + 0.48 0.48 = 0.86 0.48 Subtract 0.48 from both sides. 6.7x = 1.34 6.7x 1.34 = 6.7 6.7 x = 0.2 Answer: 3 Simplify: 0.86 0.48 = 1.34. Divide both sides by 6.7. Simplify: 1.34/(6.7) = 0.2.

Rounding Solutions
Sometimes an approximate solution is adequate. You Try It! Solve for x: 4.2x 1.25 = 3.4 + 0.71x EXAMPLE 8. Solve the equation answer to the nearest tenth. 3.1x + 4.6 = 2.5 2.2x for x. Round the

Solution. We need to isolate the terms containing x on one side of the equation. We begin by adding 2.2x to both sides of the equation. 3.1x + 4.6 = 2.5 2.2x Original equation. 3.1x + 4.6 + 2.2x = 2.5 2.2x + 2.2x Add 2.2x to both sides. 5.3x + 4.6 = 2.5 Combine terms: 3.1x + 2.2x = 5.3x.

To undo adding 4.6, subtract 4.6 from both sides of the equation. 5.3x + 4.6 4.6 = 2.5 4.6 5.3x = 2.1 Version: Fall 2010 Subtract 4.6 from both sides. Simplify: 2.5 4.6 = 2.1.

5.6. EQUATIONS WITH DECIMALS

415

To undo the eect of multiplying by 5.3, divide both sides of the equation by 5.3. 5.3x 2.1 = 5.3 5.3 x 0.4 To round the answer to the nearest tenth, we must carry the division out one additional place. 0.39 53)21.00 15 9 5 10 4 77 33 Thus, 2.1/5.3 0.39. Divide both sides by 5.3. Round solution to nearest tenth.

Because the test digit is greater than or equal to 5, add 1 to the rounding digit and truncate. Test digit 0. 3 9 Rounding digit Thus, 0.39 0.4. Answer: 1.33

Applications
Lets look at some applications that involve equations containing decimals. For convenience, we repeat the Requirements for Word Problem Solutions.

Requirements for Word Problem Solutions. 1. Set up a Variable Dictionary. You must let your readers know what each variable in your problem represents. This can be accomplished in a number of ways: Statements such as Let P represent the perimeter of the rectangle. Labeling unknown values with variables in a table. Labeling unknown quantities in a sketch or diagram. 2. Set up an Equation. Every solution to a word problem must include a carefully crafted equation that accurately describes the constraints in the problem statement. 3. Solve the Equation. You must always solve the equation set up in the previous step.

Version: Fall 2010

416

CHAPTER 5. DECIMALS

4. Answer the Question. This step is easily overlooked. For example, the problem might ask for Janes age, but your equations solution gives the age of Janes sister Liz. Make sure you answer the original question asked in the problem. Your solution should be written in a sentence with appropriate units. 5. Look Back. It is important to note that this step does not imply that you should simply check your solution in your equation. After all, its possible that your equation incorrectly models the problems situation, so you could have a valid solution to an incorrect equation. The important question is: Does your answer make sense based on the words in the original problem statement.

Lets start with a rectangular garden problem. You Try It! Etas dog run is in the shape of a rectangle with area 500 square feet. If the length of the run is 28 feet, nd the width of the run, correct to the nearest tenth of a foot. EXAMPLE 9. Molly needs to create a rectangular garden plot covering 200 square meters (200 m2 ). If the width of the plot is 8.9 meters, nd the length of the plot correct to the nearest tenth of a meter. Solution. We will follow the Requirements for Word Problem Solutions. 1. Set up a Variable Dictionary. We will use a sketch to dene our variables.

A = 200 m2

8.9 m

L Note that L represents the length of the rectangle. 2. Set Up an Equation. The area A of a rectangle is given by the formula A = LW, where L and W represent the length and width of the rectangle, respectively. Substitute 200 for A and 8.9 for W in the formula to obtain 200 = L(8.9), or equivalently, 200 = 8.9L. Version: Fall 2010

5.6. EQUATIONS WITH DECIMALS

417

3. Solve the Equation. Divide both sides of the last equation by 8.9, then round your answer to the nearest tenth. 200 8.9L = 8.9 8.9 22.5 L To round the answer to the nearest tenth, we must carry the division out one additional place. 22.47 89)2000.00 178 220 178 42 0 35 6 6 40 6 23 0 Divide both sides by 8.9. Round to nearest tenth. Because the test digit is greater than or equal to 5, add 1 to the rounding digit and truncate. Test digit 22. 4 7 Rounding digit Thus, 200/8.9 22.5.

4. Answer the Question. To the nearest tenth of a meter, the length of the rectangular plot is L 22.5 meters. 5. Look Back. We have L 22.5 meters and W = 8.9 meters. Multiply length and width to nd the area. Area (22.5 m)(8.9 m) 200.25 m2 . Note that this is very nearly the exact area of 200 square meters. The discrepancy is due to the fact that we found the length rounded to the nearest tenth of a meter. Answer: 17.9 feet

You Try It! EXAMPLE 10. Childrens tickets to the circus go on sale for $6.75. The Boys and Girls club of Eureka has $1,000 set aside to purchase these tickets. Approximately how many tickets can the Girls and Boys club purchase? Solution. We will follow the Requirements for Word Problem Solutions. 1. Set up a Variable Dictionary. Let N represent the number of tickets purchased by the Boys and Girls club of Eureka. Version: Fall 2010 Adult tickets to the circus cost $12.25 apiece. If the club has $1,200 set aside for adult ticket purchase, how many adult tickets can they purchase?

418 2. Set Up an Equation. Note that Price per Ticket 6.75 Hence, our equation is 6.75N = 1000. Number of Tickets N

CHAPTER 5. DECIMALS

times

is =

Full Purchase Price 1,000

3. Solve the Equation. Divide both sides of the equation by 6.75. 1000 6.75N = 6.75 6.75 N 148 Push the decimal point to the right-end of the divisor and the decimal point in the dividend an equal number of places. 6.75 )1000.00 Divide both sides by 6.75. Truncate to nearest unit. Well stop the division at the units position. 148 675)100000 675 3250 2700 5500 5400 100

4. Answer the Question. The Boys and Girls club can purchase 148 tickets. 5. Look Back. Lets calculate the cost of 148 tickets at $6.75 apiece. 1 48 6.75 7 40 103 6 888 999.00 Thus, at $6.75 apiece, 148 tickets will cost $999. Because the Boys and Girls club of Eureka has $1,000 to work with, note that the club doesnt have enough money left for another ticket. Answer: 97

Version: Fall 2010

5.6. EQUATIONS WITH DECIMALS

419

You Try It! EXAMPLE 11. Marta has 20 feet of decorative fencing which she will use for the border of a small circular garden. Find the diameter of the circular garden, correct to the nearest hundredth of a foot. Use 3.14. Solution. The formula governing the relation between the circumference and diameter of a circle is C = d. The 20 feet of decorative fencing will be the circumference of the circular garden. Substitute 20 for C and 3.14 for . 20 = 3.14d Divide both sides of the equation by 3.14. 3.14d 20 = 3.14 3.14 20 =d 3.14 Dylan has a circular dog pen with circumference 100 feet. Find the radius of the pen, correct to the nearest tenth of a foot. Use 3.14

Move the decimal point to the end of the divisor, then move the decimal point in the dividend an equal number of places (two places) to the right. Note that we must add two trailing zeros in the dividend. 3.14 )20.00 Thus, the problem becomes: 314)2000 We need to round to the nearest hundredth. This requires that we carry the division one additional place to the right of the hundredths place (i.e., to the thousandths place). 6.369 314)2000.000 1884 116 0 94 2 21 80 18 84 2 960 2 826 134 Version: Fall 2010

420

CHAPTER 5. DECIMALS

For the nal step, we must round 6.369 to the nearest hundredth. In the schematic that follows, weve boxed the hundredths digit (the rounding digit) and the test digit that follows the rounding digit. Test digit 6.3 6 9 Rounding digit Because the test digit is greater than or equal to 5, we add 1 to the rounding digit, then truncate. Therefore, to the nearest hundredth of a foot, the diameter of the circle is approximately d 6.37 ft. Answer: 15.9 feet

Version: Fall 2010

5.6. EQUATIONS WITH DECIMALS

421

l l l
In Exercises 1-16, solve the equation. 1. 5.57x 2.45x = 5.46 2. 0.3x 6.5x = 3.4 3. 5.8x + 0.32 + 0.2x = 6.96 4. 2.2x 0.8 7.8x = 3.3 5. 4.9x + 88.2 = 24.5 6. 0.2x 32.71 = 57.61 7. 0.35x 63.58 = 55.14 8. 0.2x 67.3 = 93.5

Exercises

l l l

9. 10.3x + 82.4 = 0 10. 1.33x 45.22 = 0 11. 12.5x + 13.5 = 0 12. 44.15x 8.83 = 0 13. 7.3x 8.9 8.34x = 2.8 14. 0.9x + 4.5 0.5x = 3.5 15. 0.2x + 2.2x = 6.8 16. 7.9x + 2.9x = 8.6

In Exercises 17-34, solve the equation. 17. 6.24x 5.2 = 5.2x 18. 0.6x + 6.3 = 1.5x 19. 0.7x 2.4 = 3.7x 8.91 20. 3.4x 4.89 = 2.9x + 3.6 21. 4.9x = 5.4x + 8.4 22. 2.5x = 4.5x + 5.8 23. 2.8x = 2.3x 6.5 24. 1.2x = 0.35x 1.36 25. 2.97x 2.6 = 3.47x + 7.47 26. 8.6x 2.62 = 7.1x + 8.54 27. 1.7x = 0.2x 0.6 28. 3.89x = 5.11x + 5.4 29. 1.02x + 7.08 = 2.79x 30. 1.5x 2.4 = 0.3x 31. 4.75x 6.77 = 7.45x + 3.49 32. 1.2x 2.8 = 0.7x 5.6 33. 4.06x 7.38 = 4.94x 34. 4.22x + 7.8 = 6.3x

In Exercises 35-52, solve the equation. 35. 2.3 + 0.1(x + 2.9) = 6.9 36. 6.37 + 6.3(x + 4.9) = 1.33 37. 0.5(1.5x 6.58) = 6.88 38. 0.5(2.5x 4.7) = 16.9 39. 6.3x 0.4(x 1.8) = 16.03 40. 2.8x + 5.08(x 4.84) = 19.85 41. 2.4(0.3x + 3.2) = 11.4 42. 0.7(0.2x + 5.48) = 16.45 43. 0.8(0.3x + 0.4) = 11.3 44. 7.5(4.4x + 7.88) = 17.19 Version: Fall 2010

422 45. 7.57 2.42(x + 5.54) = 6.95 46. 5.9 0.5(x + 5.8) = 12.15 47. 1.7 5.56(x + 6.1) = 12.2 48. 7.93 + 0.01(x + 7.9) = 14.2

CHAPTER 5. DECIMALS 49. 4.3x 0.7(x + 2.1) = 8.61 50. 1.5x 4.5(x + 4.92) = 15.6 51. 4.8x + 3.3(x 0.4) = 7.05 52. 1.1x + 1.3(x + 1.3) = 19.88

In Exercises 53-58, solve the equation. 53. 0.9(6.2x 5.9) = 3.4(3.7x + 4.3) 1.8 54. 0.4(4.6x+ 4.7) = 1.6(2.2x+ 6.9) 4.5 55. 1.8(1.6x + 1.7) = 1.8(3.6x 4.1) 56. 3.3(6.3x + 4.2) 5.3 = 1.7(6.2x + 3.2) 57. 0.9(0.4x + 2.5) 2.5 = 1.9(0.8x + 3.1) 58. 5.5(6.7x + 7.3) = 5.5(4.2x + 2.2)

59. Stacy runs a business out of her home making bird houses. Each month she has xed costs of $200. In addition, for each bird house she makes, she incurs an additional cost of $3.00. If her total costs for the month were $296.00, how many bird houses did she make? 60. Stella runs a business out of her home making curtains. Each month she has xed costs of $175. In addition, for each curtain she makes, she incurs an additional cost of $2.75. If her total costs for the month were $274.00, how many curtains did she make? 61. A stationary store has staplers on sale for $1.50 apiece. A business purchases an unknown number of these and the total cost of their purchase is $36.00. How many were purchased? 62. A stationary store has CD packs on sale for $2.50 apiece. A business purchases an unknown number of these and the total cost of their purchase is $40.00. How many were purchased? 63. Julie runs a business out of her home making table cloths. Each month she has xed costs of $100. In addition, for each taVersion: Fall 2010

ble cloth she makes, she incurs an additional cost of $2.75. If her total costs for the month were $221.00, how many table cloths did she make? 64. Stella runs a business out of her home making quilts. Each month she has xed costs of $200. In addition, for each quilt she makes, she incurs an additional cost of $1.75. If her total costs for the month were $280.50, how many quilts did she make? 65. Marta has 60 feet of decorative fencing which she will use for the border of a small circular garden. Find the diameter of the circular garden, correct to the nearest hundredth of a foot. Use 3.14. 66. Trinity has 44 feet of decorative fencing which she will use for the border of a small circular garden. Find the diameter of the circular garden, correct to the nearest hundredth of a foot. Use 3.14. 67. Childrens tickets to the ice capades go on sale for $4.25. The YMCA of Sacramento has $1,000 set aside to purchase these tickets. Approximately how many tickets can the YMCA of Sacramento purchase?

5.6. EQUATIONS WITH DECIMALS 68. Childrens tickets to the ice capades go on sale for $5. The Knights of Columbus has $1,200 set aside to purchase these tickets. Approximately how many tickets can the Knights of Columbus purchase? 69. A stationary store has mechanical pencils on sale for $2.25 apiece. A business purchases an unknown number of these and the total cost of their purchase is $65.25. How many were purchased? 70. A stationary store has engineering templates on sale for $2.50 apiece. A business purchases an unknown number of these and the total cost of their purchase is $60.00. How many were purchased? 71. Marta has 61 feet of decorative fencing which she will use for the border of a small circular garden. Find the diameter of the circular garden, correct to the nearest hundredth of a foot. Use 3.14. 72. Kathy has 86 feet of decorative fencing which she will use for the border of a small circular garden. Find the diameter of the circular garden, correct to the nearest hundredth of a foot. Use 3.14. 73. Kathy needs to create a rectangular garden plot covering 100 square meters (100 m2 ). If the width of the plot is 7.5 meters,

423 nd the length of the plot correct to the nearest tenth of a meter. 74. Marianne needs to create a rectangular garden plot covering 223 square meters (223 m2 ). If the width of the plot is 8.3 meters, nd the length of the plot correct to the nearest tenth of a meter. 75. Childrens tickets to the stock car races go on sale for $4.5. The Boys and Girls club of Eureka has $1,300 set aside to purchase these tickets. Approximately how many tickets can the Boys and Girls club of Eureka purchase? 76. Childrens tickets to the movies go on sale for $4.75. The Lions club of Alameda has $800 set aside to purchase these tickets. Approximately how many tickets can the Lions club of Alameda purchase? 77. Ashley needs to create a rectangular garden plot covering 115 square meters (115 m2 ). If the width of the plot is 6.8 meters, nd the length of the plot correct to the nearest tenth of a meter. 78. Molly needs to create a rectangular garden plot covering 268 square meters (268 m2 ). If the width of the plot is 6.1 meters, nd the length of the plot correct to the nearest tenth of a meter.

79. Crude Inventory. US commercial crude oil inventories decreased by 3.8 million barrels in the week ending June 19. If there were 353.9 million barrels the following week, what were crude oil inventories before the decline? rttnews.com 06/24/09 80. Undocumented. In 2008, California had 2.7 million undocumented residents. This is double the number in 1990. How many undocumented residents were in California in 1990? Associated Press Times-Standard 4/15/09 81. Diamonds Shining. The index of refraction n indicates the number of times slower that a light wave travels in a particular medium than it travels in a vacuum. A diamond has an index of refraction of 2.4. This is about one and one-quarter times greater than the index of refraction of a zircon. What is the index of refraction of a zircon? Round your result to the nearest tenth.

Version: Fall 2010

424

CHAPTER 5. DECIMALS

l l l
1. 1.75 3. 1.3 5. 13 7. 339.2 9. 8 11. 1.08 13. 11.25 15. 3.4 17. 5

Answers

l l l

43. 45.75 45. 11.54 47. 8.6 49. 2.8 51. 3.82 53. 2.59 55. 2.9 57. 3 59. 32

19. 2.17 61. 24 21. 16.8 63. 44 23. 13 65. 19.11 feet 25. 20.14 27. 0.4 29. 4 31. 3.8 33. 0.82 35. 43.1 37. 13.56 39. 2.5 41. 26.5 67. 235 tickets 69. 29 71. 19.43 feet 73. 13.3 meters 75. 288 tickets 77. 16.9 meters 79. 357.7 million barrels 81. 1.9

Version: Fall 2010

5.7. INTRODUCTION TO SQUARE ROOTS

425 List of Squares x x2 0 0 1 1 2 4 3 9 4 16 5 25 6 36 7 49 8 64 9 81 10 100 11 121 12 144 13 169 14 196 15 225 16 256 17 289 18 324 19 361 20 400 21 441 22 484 23 529 24 576 25 625

5.7

Introduction to Square Roots


x2 = x x.

Recall that

The Square of a Number. The number x2 is called the square of the number x. Thus, for example: 92 = 9 9 = 81. Therefore, the number 81 is the square of the number 9. (4)2 = (4)(4) = 16. Therefore, the number 16 is the square of the number 4. In the margin, weve placed a List of Squares of the whole numbers ranging from 0 through 25, inclusive.

Square Roots
Once youve mastered the process of squaring a whole number, then you are ready for the inverse of the squaring process, taking the square root of a whole number. Above, we saw that 92 = 81. We called the number 81 the square of the number 9. Conversely, we call the number 9 a square root of the number 81. Above, we saw that (4)2 = 16. We called the number 16 the square of the number 4. Conversely, we call the number 4 a square root of the number 16. Square Root. If a2 = b, then a is called a square root of the number b.

You Try It! EXAMPLE 1. Find the square roots of the number 49. Solution. To nd a square root of 49, we must think of a number a such that a2 = 49. Two numbers come to mind. (7)2 = 49. Therefore, 7 is a square root of 49. 72 = 49. Therefore, 7 is a square root of 49. Note that 49 has two square roots, one of which is positive and the other one is negative. Version: Fall 2010 Answer: 16, 16 Find the square roots of 256.

426

CHAPTER 5. DECIMALS

You Try It! Find the square roots of 625. EXAMPLE 2. Find the square roots of the number 196. Solution. To nd a square root of 196, we must think of a number a such that a2 = 196. With help from the List of Squares, two numbers come to mind. (14)2 = 196. Therefore, 14 is a square root of 196. 142 = 196. Therefore, 14 is a square root of 196. Answer: 25, 25 Note that 196 has two square roots, one of which is positive and the other one is negative. Square Roots x x 0 0 1 1 4 2 9 3 16 4 25 5 36 6 49 7 64 8 81 9 100 10 121 11 144 12 169 13 196 14 225 15 256 16 289 17 324 18 361 19 400 20 441 21 484 22 529 23 576 24 625 25

You Try It! Find the square roots of 9. EXAMPLE 3. Find the square roots of the number 0. Solution. To nd a square root of 0, we must think of a number a such that a2 = 0. There is only one such number, namely zero. Hence, 0 is the square root of 0.

Answer: 3, 3

You Try It! Find the square roots of 81. EXAMPLE 4. Find the square roots of the number 25. Solution. To nd a square root of 25, we must think of a number a such that a2 = 25. This is impossible because no square of a real number (whole number, integer, fraction, or decimal) can be negative. Positive times positive is positive and negative times negative is also positive. You cannot square and get a negative answer. Therefore, 25 has no square roots2 .

Answer: There are none.

Radical Notation
Because (3)2 = 9 and 32 = 9, both 3 and 3 are square roots of 9. Special notation, called radical notation, is used to request these square roots.
2 At least not in Prealgebra. In later courses, you will be introduced to the set of complex numbers, where 25 will have two square roots

Version: Fall 2010

5.7. INTRODUCTION TO SQUARE ROOTS

427

The radical notation 9, pronounced the nonnegative square root of 9, calls for the nonnegative3 square root of 9. Hence, 9 = 3. The radical notation 9, pronounced the negative square root of 9, calls for the negative square root of 9. Hence, 9 = 3. Radical Notation. In the expression 9, the symbol is called a radical and the number within the radical, in this case the number 9, is called the radicand. List of Squares x x2 0 0 1 1 2 4 3 9 4 16 5 25 6 36 7 49 8 64 9 81 10 100 11 121 12 144 13 169 14 196 15 225 16 256 17 289 18 324 19 361 20 400 21 441 22 484 23 529 24 576 25 625 For example,

529, the number 529 is the radicand. In the expression a2 + b2 , the expression a2 + b2 is the radicand. In the expression

Radical Notation and Square Root. If b is a positive number, then 1. b calls for the nonnegative square root of b. 2. b calls for the negative square root of b. Note: Nonnegative is equivalent to saying not negative; i.e., positive or zero.

You Try It! EXAMPLE 5. Simplify: Solution. (a) Referring to the list of squares, we note that 112 = 121 and (11)2 = 121. Therefore, both 11 and 11 are square roots of 121. However, 121 calls for the nonnegative square root of 121. Thus, 121 = 11. (b) Referring to the list of squares, we note that 252 = 625 and (25)2 = 625. Therefore, both 25 and 25 are square roots of 625. However, 625 calls for the negative square root of 625. Thus, 625 = 25.
3 Nonnegative

(a) 121, (b) 625, and (c) 0.

Simplify: 144 a) b) 324

is equivalent to saying not negative; i.e., positive or zero.

Version: Fall 2010

428

CHAPTER 5. DECIMALS

(c) There is only one square root of zero. Therefore, 0 = 0. Answer: (a) 12 (b) 18

You Try It! Simplify: a) 36 36 b) EXAMPLE 6. Simplify: Solution. = (a) Because 52 = 25 and (5)2 25, both 5 and 5 are square roots of 25. However, the notation 25 calls for the negative square root of 25. Thus, 25 = 5. (b) It is not possible to square a real number (whole number, integer, fraction, or decimal) and get 25. Therefore, there is no real square root of 25. That is, 25 Answer: (a) 6 (b) undened is not a real number. It is undened4 . (a) 25, and (b) 25. Square Roots x x 0 0 1 1 4 2 9 3 16 4 25 5 36 6 49 7 64 8 81 9 100 10 121 11 144 12 169 13 196 14 225 15 256 16 289 17 324 18 361 19 400 20 441 21 484 22 529 23 576 24 625 25

Order of Operations
With the addition of radical notation, the Rules Guiding Order of Operations change slightly. Rules Guiding Order of Operations. When evaluating expressions, proceed in the following order. 1. Evaluate expressions contained in grouping symbols rst. If grouping symbols are nested, evaluate the expression in the innermost pair of grouping symbols rst. 2. Evaluate all exponents and radicals that appear in the expression. 3. Perform all multiplications and divisions in the order that they appear in the expression, moving left to right. 4. Perform all additions and subtractions in the order that they appear in the expression, moving left to right.

The only change in the rules is in item #2, which says: Evaluate all exponents and radicals that appear in the expression, putting radicals on the same level Version: Fall 2010 as exponents.
4 At least in Prealgebra. In later courses you will be introduced to the set of complex numbers, where 25 will take on a new meaning.

5.7. INTRODUCTION TO SQUARE ROOTS

429

You Try It! EXAMPLE 7. Simplify: 3 9 + 12 4. Simplify: 2 43 9

Solution. According to the Rules Guiding Order of Operations, we must evaluate the radicals in this expression rst. 3 9 + 12 4 = 3(3) + 12(2) Evaluate radicals rst: 9 = 3 and 4 = 2. = 9 + 24 Multiply: 3(3) = 9 and 12(2) = 24. = 15 Add: 9 + 24 = 15.

Answer: 5 List of Squares x x2 0 0 1 1 2 4 3 9 4 16 5 25 6 36 7 49 8 64 9 81 10 100 11 121 12 144 13 169 14 196 15 225 16 256 17 289 18 324 19 361 20 400 21 441 22 484 23 529 24 576 25 625

You Try It! EXAMPLE 8. Simplify: 2 3 36. Simplify: 5 8 169

Solution. According to the Rules Guiding Order of Operations, we must evaluate the radicals in this expression rst, moving left to right. 2 3 36 = 2 3(6) Evaluate radicals rst: 36 = 6 = 2 18 Multiply: 3(6) = 18. = 20 Subtract: 2 18 = 2 + (18) = 20.

Answer: 99

You Try It! EXAMPLE 9. Simplify: (a) 9 + 16 and (b) 9 + 16. Simplify: 25 + 144 a) 25 + 144 b)

Solution. Apply the Rules Guiding Order of Operations. a) In this case, the radical acts like grouping symbols, so we must evaluate what is inside the radical rst. 9 + 16 = 25 Add: 9 + 16 = 25. =5 Take nonnegative square root: 25 = 5. b) In this example, we must evaluate the square roots rst. 9 + 16 = 3 + 4 Square root: 9 = 3 and 16 = 4. =7 Add: 3 + 4 = 7.

Answer: (a) 13 (b) 17 Version: Fall 2010

430

CHAPTER 5. DECIMALS

Fractions and Decimals


We can also nd square roots of fractions and decimals. You Try It! Simplify: a)
25 49

EXAMPLE 10. Simplify: Solution. (a) Because 2 3


2

(a)

4 , and (b) 0.49. 9

b) 0.36

2 3

2 3

4 , then 9 2 4 = . 9 3

(b) Because (0.7)2 = (0.7)(0.7) = 0.49 and (0.7)2 = (0.7)(0.7) = 0.49, both 0.7 and 0.7 are square roots of 0.49. However, 0.49 calls for the negative square root of 0.49. Hence, 0.49 = 0.7. Answer: (a) 5/7 (b) 0.6

Estimating Square Roots


The squares in the List of Squares are called perfect squares. Each is the square of a whole number. Not all numbers are perfect squares. For example, in the case of 24, there is no whole number whose square is equal to 24. However, this does not prevent 24 from being a perfectly good number. We can use the List of Squares to nd decimal approximations when the radicand is not a perfect square. You Try It! Estimate: 83 EXAMPLE 11. Estimate 24 by guessing. Use a calculator to nd a more accurate result and compare this result with your guess. Solution. From the List of Squares, note that 24 lies betwen 16 and 25, so 24 will lie between 4 and 5, with 24 much closer to 5 than it is to 4. 16 4 Version: Fall 2010 24 4.8 25 5

Square Roots x x 0 0 1 1 4 2 9 3 16 4 25 5 36 6 49 7 64 8 81 9 100 10 121 11 144 12 169 13 196 14 225 15 256 16 289 17 324 18 361 19 400 20 441 21 484 22 529 23 576 24 625 25

5.7. INTRODUCTION TO SQUARE ROOTS Lets guess As a check, lets square 4.8. (4.8)2 = (4.8)(4.8) = 23.04 24 4.8.

431

Not quite 24! Clearly, 24 must be a little bit bigger than 4.8. Lets use a scientic calculator to get a better approximation. From our calculator, using the square root button, we nd 24 4.89897948557. Even though this is better than our estimate of 4.8, it is still only an approximation. Our calculator was only capable providing 11 decimal places. of However, the exact decimal representation of 24 is an innite decimal that never terminates and never establishes a pattern of repetition. Just for fun, here is a decimal approximation of 24 that is accurate to 1000 places, courtesy of http://www.wolframalpha.com/.
4.8989794855663561963945681494117827839318949613133402568653851 3450192075491463005307971886620928046963718920245322837824971773 09196755146832515679024745571056578254950553531424952602105418235 40446962621357973381707264886705091208067617617878749171135693149 44872260828854054043234840367660016317961567602617940145738798726 16743161888016008874773750983290293078782900240894528962666325870 21889483627026570990088932343453262850995296636249008023132090729 18018687172335863967331332533818263813071727532210516312358732472 35822058934417670915102576710597966482011173804100128309322482347 06798820862115985796934679065105574720836593103436607820735600767 24633259464660565809954782094852720141025275395093777354012819859 11851434656929005776183028851492605205905926474151050068455119830 90852562596006129344159884850604575685241068135895720093193879959 87119508123342717309306912496416512553772738561882612744867017729 60314496926744648947590909762887695867274018394820295570465751182 126319692156620734019070649453

If you were to multiply this number by itself (square the number), you would get a number that is extremely close to 24, but it would not be exactly 24. There would still be a little discrepancy.

Answer: 9.1

Important Observation. A calculator can only produce a nite number of decimal places. If the decimal representation of your number does not terminate within this limited number of places, then the number in your calculator window is only an approximation.

Version: Fall 2010

432

CHAPTER 5. DECIMALS

The decimal representation of 1/8 will terminate within three places, so most calculators will report the exact answer, 0.125. For contrast, 2/3 does not terminate. A calculator capable of reporting 11 places of accuracy produces the number 0.666666666667. However, the exact decimal representation of 2/3 is 0.6. Note that the calculator has rounded in the last place and only provides an approximation of 2/3. If your instructor asks for an exact answer on an exam or quiz then 0.666666666667, being an approximation, is not acceptable. You must give the exact answer 2/3.

Version: Fall 2010

5.7. INTRODUCTION TO SQUARE ROOTS

433

l l l

Exercises

l l l

In Exercises 1-16, list all square roots of the given number. If the number has no square roots, write none. 1. 256 2. 361 3. 289 4. 400 5. 441 6. 36 7. 324 8. 0 9. 144 10. 100 11. 144 12. 100 13. 121 14. 196 15. 529 16. 400

In Exercises 17-32, compute the exact square root. If the square root is undened, write undened. 17. 18. 19. 20. 21. 22. 23. 24. 9 196 576 289 529 256 25 225 25. 484 26. 36 27. 196 28. 289 29. 441 30. 324 31. 4 32. 100

In Exercises 33-52, compute the exact square root. 33. 34. 35. 36. 0.81 5.29 3.61 0.09 37. 225 16 100 81 3.24 Version: Fall 2010

38. 39.

434 40. 41. 42. 43. 44. 45. 5.76 121 49 625 324 529 121 4 121 48.

CHAPTER 5. DECIMALS 144 25 49 36 256 361 529 16

47.

49.

50. 51.

2.89 46. 4.41

0.49 52. 4.84

In Exercises 53-70, compute the exact value of the given expression. 53. 6 54. 55. 56. 57. 58. 59. 60. 61. 576 2 7 576 82 + 152 72 + 242 6 16 9 49 3 441 + 6 484 52 + 122 152 + 202 32 + 42 62. 63. 64. 65. 66. 67. 68. 69. 70. 62 + 82 2 324 6 361 6 576 8 121 4 3 529 1 + 625 9 484 + 7 81 625 5 576 2 16 8 6 400

In Exercises 71-76, complete the following tasks to estimate the given square root. a) Determine the two integers that the square root lies between. b) Draw a number line, and locate the approximate location of the square root between the two integers found in part (a). c) Without using a calculator, estimate the square root to the nearest tenth. 71. 58 72. 27 73. 79 74. 12 75. 44 76. 88

Version: Fall 2010

5.7. INTRODUCTION TO SQUARE ROOTS

435

In Exercises 77-82, use a calculator to approximate the square root to the nearest tenth. 469 78. 73 79. 615 77. 80. 81. 82. 162 444 223

l l l
1. 16, 16 3. none 5. 21, 21 7. 18, 18 9. 12, 12 11. none 13. 11, 11

Answers
35. 1.9 37. 15 4

l l l

39. 1.8 41. 43. 11 7 23 11

45. 1.7 47. 15. 23, 23 17. undened 19. 24 21. undened 23. 5 25. 22 27. 14 29. 21 31. 2 33. 0.9 49. 12 5 16 19

51. 0.7 53. 18 55. 17 57. 39 59. 13 61. 5 63. 150 65. 73 67. 135 Version: Fall 2010

436 69. 2 71. 7.6 73. 8.9 75. 6.6

CHAPTER 5. DECIMALS 77. 21.7 79. 24.8 81. 21.1

Version: Fall 2010

5.8. THE PYTHAGOREAN THEOREM

437

5.8

The Pythagorean Theorem

Pythagoras was a Greek mathematician and philosopher, born on the island of Samos (ca. 582 BC). He founded a number of schools, one in particular in a town in southern Italy called Crotone, whose members eventually became known as the Pythagoreans. The inner circle at the school, the Mathematikoi, lived at the school, rid themselves of all personal possessions, were vegetarians, and observed a strict vow of silence. They studied mathematics, philosophy, and music, and held the belief that numbers constitute the true nature of things, giving numbers a mystical or even spiritual quality. Today, nothing is known of Pythagorass writings, perhaps due to the secrecy and silence of the Pythagorean society. However, one of the most famous theorems in all of mathematics does bear his name, the Pythagorean Theorem. Prior to revealing the contents of the Pythagorean Theorem, we pause to provide the denition of a right triangle and its constituent parts.

Right Triangle. A triangle with one right angle (90 ) is called a right triangle. In the gure below, the right angle is marked with a little square.

hypotenuse leg

leg The side of the triangle that is directly opposite the right angle is called the hypotenuse. The sides of the triangle that include the right angle are called the legs of the right triangle.

Now we can state one of the most ancient theorems of mathematics, the Pythagorean Theorem.

Pythagorean Theorem. Let c represent the length of the hypotenuse of a right triangle, and let a and b represent the lengths of its legs, as pictured in the image that follows.

Version: Fall 2010

438

CHAPTER 5. DECIMALS

b The relationship involving the legs and hypotenuse of the right triangle, given by a2 + b 2 = c2 , is called the Pythagorean Theorem.

Here are two important observations.

Observations Regarding the Hypotenuse. Two important facts regarding the hypotenuse of the right triangle are: 1. The hypotenuse is the longest side of the triangle and lies directly opposite the right angle. 2. In the Pythagorean equation a2 + b2 = c2 , the hypotenuse lies by itself on one side of the equation.

The Pythagorean Theorem can only be applied to right triangles. Lets look at a simple application of the Pythagorean Theorem. You Try It! The legs of a right triangle measure 5 and 12 feet, respectively. Find the length of the hypotenuse. EXAMPLE 1. The legs of a right triangle measure 3 and 4 meters, respectively. Find the length of the hypotenuse. Solution. Lets follow the Requirements for Word Problem Solutions.

1. Set up a Variable Dictionary. Let c represent the length of the hypotenuse, as pictured in the following sketch. Version: Fall 2010

5.8. THE PYTHAGOREAN THEOREM

439

3m

4m 2. Set up an Equation. The Pythagorean Theorem says that a2 + b 2 = c2 . In this example, the legs are known. Substitute 4 for a and 3 for b (3 for a and 4 for b works equally well) into the Pythagorean equation. 4 2 + 3 2 = c2 3. Solve the Equation. 4 2 + 3 2 = c2 16 + 9 = c 5=c
2

The Pythagorean equation. Exponents rst: 42 = 16 and 32 = 9. Add: 16 + 9 = 25. Take the nonnegative square root.

25 = c2

Technically, there are two answers to c2 = 25, i.e., c = 5 or c = 5. However, c represents the hypotenuse of the right triangle and must be nonnegative. Hence, we must choose c = 5. 4. Answer the Question. The hypotenuse has length 5 meters. 5. Look Back. Do the numbers satisfy the Pythagorean Theorem? The sum of the squares of the legs should equal the square of the hypotenuse. Lets check. 42 + 32 = 52 16 + 9 = 25 25 = 25 All is well! Answer: 13 feet

You Try It! EXAMPLE 2. Given the following right triangle, nd the length of the missing side. Version: Fall 2010 The hypotenuse of a right triangle measures 25 centimeters. One leg of the right triangle measures 24 centimeters. Find the length of the remaining leg.

440

CHAPTER 5. DECIMALS

13

x Solution. Note that the hypotenuse (across from the right angle) has length 13. This quantity should lie on one side of the Pythagorean equation all by itself. The sum of the squares of the legs go on the other side. Hence, 52 + x2 = 132 Solve the equation for x. 25 + x2 = 169 25 + x2 25 = 169 25 x = 144 x = 12 Answer: 7 centimeters
2

Exponents rst: 52 = 25 and 132 = 169. Subtract 25 from both sides. Simplify both sides. Take the nonnegative square root of 144.

Perfect squares are nice, but not required. You Try It! The hypotenuse and one leg of a right triangle measure 9 and 7 inches, respectively. Find the length of the remaining leg. EXAMPLE 3. Given the following right triangle, nd the exact length of the missing side. 7

12

Solution. Note that the hypotenuse (across from the right angle) has length 12. This quantity should lie on one side of the Pythagorean equation all by itself. The sum of the squares of the legs go on the other side. Hence, x2 + 72 = 122 Version: Fall 2010

5.8. THE PYTHAGOREAN THEOREM Solve the equation for x. x2 + 49 = 144 x2 + 49 49 = 144 49 x = 95 x = 95
2

441

Exponents rst: 72 = 49 and 122 = 144. Subtract 49 from both sides. Simplify both sides. 32 inches

Take the nonnegative square root of 95. Hence, the exact length of the missing side is 95.

Answer:

Important Observation. Any attempt to use your calculator to approximate 95 in Example 3 would be an error as the instructions asked for an exact answer. Sometimes an approximate answer is desired, particularly in applications. You Try It! EXAMPLE 4. Ginny want to create a vegetable garden in the corner of her yard in the shape of a right triangle. She cuts two boards of length 8 feet which will form the legs of her garden. Find the length of board she should cut to form the hypotenuse of her garden, correct to the nearest tenth of a foot. Solution. We follow the Requirements for Word Problem Solutions. 1. Set up a Variable Dictionary. We begin with a labeled sketch. Let x represent the length of the unknown hypotenuse. 8 ft A 15 foot ladder leans against the wall of a building. The base of the ladder lies 5 feet from the base of the wall. How high up the wall does the top of the ladder reach? Round your answer to the nearest tenth of a foot.

8 ft

2. Set Up an Equation. The hypotenuse is isolated on one side of the Pythagorean equation. x2 = 82 + 82 3. Solve the Equation. x2 = 82 + 82 x = 64 + 64 x2 = 128 x = 128
2

The Pythagorean equation. Exponents rst: 82 = 64 and 82 = 64. Add: 64 + 64 = 128. Take the nonnegative square root. Version: Fall 2010

442

CHAPTER 5. DECIMALS

4. Answer the Question. The exact length of the hypotenuse is 128 feet, but were asked to nd the hypotenuse to the nearest tenth of a foot. Using a calculator, we nd an approximation for 128. 128 11.313708499 To round to the nearest tenth, rst identify the rounding and test digits. Test digit 11. 3 1 3708499 Rounding digit The test digit is less than ve. So we leave the rounding digit alone and truncate. Therefore, correct to the nearest tenth of a foot, the length of the hypotenuse is approximately 11.3 feet. 5. Look Back. The sum of the squares of the legs is 82 + 82 = 64 + 64 = 128. The square of the hypotenuse is (11.3)2 = 127.69 These are almost the same, the discrepancy due to the fact that we rounded to nd an approximation for the hypotenuse. Answer: 14.1 feet

Version: Fall 2010

5.8. THE PYTHAGOREAN THEOREM

443

l l l

Exercises

l l l

In Exercises 1-16, your solutions should include a well-labeled sketch. 1. The length of one leg of a right triangle is 15 meters, and the length of the hypotenuse is 25 meters. Find the exact length of the other leg. 2. The length of one leg of a right triangle is 7 meters, and the length of the hypotenuse is 25 meters. Find the exact length of the other leg. 3. The lengths of two legs of a right triangle are 12 meters and 16 meters. Find the exact length of the hypotenuse. 4. The lengths of two legs of a right triangle are 9 meters and 12 meters. Find the exact length of the hypotenuse. 5. The length of one leg of a right triangle is 13 meters, and the length of the hypotenuse is 22 meters. Find the exact length of the other leg. 6. The length of one leg of a right triangle is 6 meters, and the length of the hypotenuse is 15 meters. Find the exact length of the other leg. 7. The lengths of two legs of a right triangle are 2 meters and 21 meters. Find the exact length of the hypotenuse. 8. The lengths of two legs of a right triangle are 7 meters and 8 meters. Find the exact length of the hypotenuse. 9. The length of one leg of a right triangle is 12 meters, and the length of the hypotenuse is 19 meters. Find the exact length of the other leg. 10. The length of one leg of a right triangle is 5 meters, and the length of the hypotenuse is 10 meters. Find the exact length of the other leg. 11. The lengths of two legs of a right triangle are 6 meters and 8 meters. Find the exact length of the hypotenuse. 12. The lengths of two legs of a right triangle are 5 meters and 12 meters. Find the exact length of the hypotenuse. 13. The length of one leg of a right triangle is 6 meters, and the length of the hypotenuse is 10 meters. Find the exact length of the other leg. 14. The length of one leg of a right triangle is 9 meters, and the length of the hypotenuse is 15 meters. Find the exact length of the other leg. 15. The lengths of two legs of a right triangle are 6 meters and 22 meters. Find the exact length of the hypotenuse. 16. The lengths of two legs of a right triangle are 9 meters and 19 meters. Find the exact length of the hypotenuse.

In Exercises 17-24, your solutions should include a well-labeled sketch. 17. The lengths of two legs of a right triangle are 3 meters and 18 meters. Find the length of the hypotenuse. Round your answer to the nearest hundredth. 18. The lengths of two legs of a right triangle are 10 feet and 16 feet. Find the length of the hypotenuse. Round your answer to the nearest tenth. Version: Fall 2010

444 19. The length of one leg of a right triangle is 2 meters, and the length of the hypotenuse is 17 meters. Find the length of the other leg. Round your answer to the nearest tenth. 20. The length of one leg of a right triangle is 4 meters, and the length of the hypotenuse is 12 meters. Find the length of the other leg. Round your answer to the nearest hundredth. 21. The lengths of two legs of a right triangle are 15 feet and 18 feet. Find the length of the hypotenuse. Round your answer to the nearest hundredth.

CHAPTER 5. DECIMALS 22. The lengths of two legs of a right triangle are 6 feet and 13 feet. Find the length of the hypotenuse. Round your answer to the nearest tenth. 23. The length of one leg of a right triangle is 4 meters, and the length of the hypotenuse is 8 meters. Find the length of the other leg. Round your answer to the nearest hundredth. 24. The length of one leg of a right triangle is 3 meters, and the length of the hypotenuse is 15 meters. Find the length of the other leg. Round your answer to the nearest tenth.

25. Greta and Fritz are planting a 13-meter by 18-meter rectangular garden, and are laying it out using string. They would like to know the length of a diagonal to make sure that right angles are formed. Find the length of a diagonal. Round your answer to the nearest hundredth. Your solution should include a well-labeled sketch. 26. Markos and Angelina are planting an 11meter by 19-meter rectangular garden, and are laying it out using string. They would like to know the length of a diagonal to make sure that right angles are formed. Find the length of a diagonal. Round your answer to the nearest tenth. Your solution should include a well-labeled sketch.

27. The base of a 24-meter long guy wire is located 10 meters from the base of the telephone pole that it is anchoring. How high up the pole does the guy wire reach? Round your answer to the nearest hundredth. Your solution should include a well-labeled sketch. 28. The base of a 30-foot long guy wire is located 9 feet from the base of the telephone pole that it is anchoring. How high up the pole does the guy wire reach? Round your answer to the nearest hundredth. Your solution should include a well-labeled sketch.

29. Hiking Trail. A hiking trail runs due south for 8 kilometers, then turns west for about 15 kilometers, and then heads northeast on a direct path to the starting point. How long is the entire trail? 30. Animal Trail. An animal trail runs due east from a watering hole for 12 kilometers, then goes north for 5 kilometers. Then the trail turns southwest on a direct path back to the watering hole. How long is the entire trail? Version: Fall 2010

5.8. THE PYTHAGOREAN THEOREM

445

31. Upper Window. A 10-foot ladder leans against the wall of a house. How close to the wall must the bottom of the ladder be in order to reach a window 8 feet above the ground? 32. How high? A 10-foot ladder leans against the wall of a house. How high will the ladder be if the bottom of the ladder is 4 feet from the wall? Round your answer to the nearest tenth.

l l l
1. 20 meters 3. 20 meters 5. 7. 9. 315 meters 445 meters 217 meters

Answers

l l l

17. 18.25 meters 19. 16.9 meters 21. 23.43 feet 23. 6.93 meters 25. 22.20 meters 27. 21.82 meters 29. 40 kilometers 31. 6 ft.

11. 10 meters 13. 8 meters 15. 520 meters

Version: Fall 2010

Chapter

Ratio and Proportion

From the beginnings of the human race, weve long compared one quantity with another, a comparison that is called a ratio in mathematics. Their tribe has twice as many cattle as ours or Two baskets of wheat cost 12 ducats are examples of ratios that ring from distant times. Indeed, the concept of a ratio cannot be assigned to any one individual or class of individual. In his History of Mathematics, D. E. Smith writes: It is rather protless to speculate as to the domain in which the concept of ratio rst appeared. The idea that one tribe is twice as large as another and the idea that one leather strap is only half as long as another both involve the notion of ratio; both are such as would develop early in the history of the race, and yet one has to do with ratio of numbers and the other with the ratio of geometric magnitudes. Indeed, when we come to the Greek writers we nd Nicomachus including ratio in his arithmetic, Eudoxus in his geometry, and Theon of Smyrna in his chapter on music. Examples and applications of ratios are limitless: speed is a ratio that compares changes in distance with respect to time, acceleration is a ratio that compares changes in speed with respect to time, and percentages compare the part with the whole. Weve already studied one classic ratio, the ratio of the circumference of a circle to its diameter, which gives us the denition of . One of the most famous ratios in history involves the division of a line segment AB into two segments AC and CB by selecting a point C on the segment AB.

The idea is to select a point C on the segment AB so that AB AC = . AC CB 447

448

CHAPTER 6. RATIO AND PROPORTION

This ratio has a special name, the Golden Ratio, and has an exact value equal to (1+ 5)/2. The Golden Ratio has been known since the time of Euclid. Ancient and modern architects have long held that the most pleasing rectangular shape is the one whose ratio of length to width is equal to the Golden Ratio. The comparison of two ratios, such as AB/AC = AC/CB, is called a proportion. Proportions are used in a number of practical ways. For example, if 5 cans of tomato sauce cost 2 dollars, we can nd the number of cans that can be purchased with 10 dollars by comparing two ratios in a proportion: x cans of tomato sauce 5 cans of tomato sauce = 2 dollars 10 dollars Any discussion of ratio involves comparing two quantities, so the units of each quantity become extremely important. Two dierent systems of units are used when measuring length, capacity, and time: the American system of units and the metric system of units. In this chapter we will discuss both systems and explain how to convert quantities measured in one system to quantities measured in the other system. Lets begin the journey.

Version: Fall 2010

6.1. INTRODUCTION TO RATIOS AND RATES

449

6.1

Introduction to Ratios and Rates

We use ratios to compare two numeric quantities or quantities with the same units.

Ratio. A ratio is the quotient of two numerical quantities or two quantities with the same physical units.

For example, ancient Greek geometers believed that the most pleasing rectangle to the eye had length and width such that the ratio of length to width was a specic number, called the Golden Ratio, approximately equal to 1.6180339887.. . . Architects to this day use this ratio in their designs. There are a number of equivalent ways of expressing ratios, three of which we will use in this text: fraction notation, to notation, and colon notation. 3/4 is a ratio, read as the ratio of 3 to 4. 3 to 4 is a ratio, read as the ratio of 3 to 4. 3:4 is a ratio, read as the ratio of 3 to 4.

You Try It! EXAMPLE 1. Express each of the following ratios as a fraction reduced to lowest terms: (a) 36 to 24, and (b) 0.12 : 0.18. Solution Express 0.12 : 0.3 as a fraction reduced to lowest terms.

(a) To express the ratio 36 to 24 as a fraction, place 36 over 24 and reduce. 3 12 36 = 24 2 12 & 3& 12 = & 2& 12 3 = 2 Thus, the ratio 36 to 24 equals 3/2. Version: Fall 2010 Factor. Cancel common factor.

450

CHAPTER 6. RATIO AND PROPORTION

(b) To express the ratio 0.12:0.18 as a fraction, place 0.12 over 0.18 and reduce. (0.12)(100) 0.12 = 0.18 (0.18)(100) 12 = 18 26 = 36 26 = 3 6 2 = 3 Answer: 2/5 Multiply numerator and denominator by 100. Move each decimal 2 places right. Factor. Cancel.

Thus, the ratio 0.12:0.18 equals 2/3.

You Try It!


1 A rectangle has length 8 4 1 inches and width 3 2 inches. Express the ratio of length to width as a fraction reduced to lowest terms.

EXAMPLE 2. For the rectangle that follows, express the ratio of length to width as a fraction reduced to lowest terms.

2 1 ft 2 3 1 ft 4 Solution. The ratio length to width can be expressed as a fraction and reduced as follows. 3 1 ft length = 4 width 2 1 ft 2 31 4 ft 21 2 ft 13 4 = 5 2 = Version: Fall 2010 Length to width as a fraction. Cancel common units.

Mixed to improper fractions.

6.1. INTRODUCTION TO RATIOS AND RATES Invert and multiply, factor, and cancel common factors. = = = = = 13 2 4 5 26 20 13 2 10 2 13 2 10 2 13 10 Invert and multiply. Multiply numerators and denominators. Factor numerator and denominator. Cancel common factors.

451

Hence, the ratio length to width is 13/10.

Answer: 33/14

Rates
We now introduce the concept of rate, a special type of ratio. Rate. A rate is a quotient of two measurements with dierent units. The physical interpretation of a rate in terms of its units is an important skill. You Try It! EXAMPLE 3. An automobile travels 224 miles on 12 gallons of gasoline. Express the ratio distance traveled to gas consumption as a fraction reduced to lowest terms. Write a short sentence explaining the physical signicance of your solution. Include units in your description. Solution. Place miles traveled over gallons of gasoline consumed and reduce. 56 4 mi 224 mi = 12 gal 3 4 gal 56 mi 4 = 3 gal 4 56 mi = 3 gal Factor Cancel common factor. Lanny travels 180 kilometers on 14 liters of gasoline. Express the ratio distance traveled to gas consumption as a fraction reduced to lowest terms.

Thus, the rate is 56 miles to 3 gallons of gasoline. In plain-speak, this means that the automobile travels 56 miles on 3 gallons of gasoline.

Answer: 90/7 kilometers per litre.

Version: Fall 2010

452

CHAPTER 6. RATIO AND PROPORTION

Unit Rates
When making comparisons, it is helpful to have a rate in a form where the denominator is 1. Such rates are given a special name. Unit Rate. A unit rate is a rate whose denominator is 1.

You Try It! Jacob drives 120 kilometers in 3 hours. Find his average rate of speed. EXAMPLE 4. Herman drives 120 miles in 4 hours. Find his average rate of speed. Solution. Place the distance traveled over the time it takes to drive that distance. 120 miles 30 miles = 4 hours 1 hour = 30 miles/hour Answer: 40 kilometers per hour. Divide: 120/4 = 30.

Hence, Hermans average rate of speed is 30 miles per hour.

You Try It! Frannie works 5.5 hours and receives $120 for her eorts. What is her hourly salary rate? Round your answer to the nearest penny. EXAMPLE 5. Aditya works 8.5 hours and receives $95 for his eorts. What is his hourly salary rate? Solution. Lets place money earned over hours worked to get the following rate: 95 dollars 8.5 hours We will get a much better idea of Adityas salary rate if we express the rate with a denominator of 1. To do so, divide. Push the decimal in the divisor to the far right, then move the decimal an equal number of places in the dividend. As we are dealing with dollars and cents, we will round our answer to the nearest hundredth. 11.176 85)950.000 85 100 Test digit 85 15 0 11.1 7 6 85 6 50 Rounding digit 5 95 550 510 Version: Fall 2010 40

6.1. INTRODUCTION TO RATIOS AND RATES

453

Because the test digit is greater than or equal to 5, we add 1 to the rounding digit and truncate; i.e., 95/8.5 11.18. Hence, 95 dollars 11.18 dollars = 8.5 hours 1 hour = 11.18 dollars/hour. Divide: 95/8.5 11.18.

That is, his salary rate is 11.18 dollars per hour.

Answer: $21.82 per hour

You Try It! EXAMPLE 6. One automobile travels 422 miles on 15 gallons of gasoline. A second automobile travels 354 miles on 13 gallons of gasoline. Which automobile gets the better gas mileage? Solution. Decimal division (rounded to the nearest tenth) reveals the better gas mileage. In the case of the rst autombile, we get the following rate: 422 mi 15 gal Divide. 28.13 15)422.00 30 122 120 20 15 50 45 5 To the nearest tenth, 28.1. Divide. 27.23 13)354.00 26 94 91 30 26 40 39 1 To the nearest tenth, 27.2. In the case of the second autombile, we get the following rate: 354 mi 13 gal Alicia works 8 hours and makes $100. Connie works 10 hours and makes $122. Which woman works at the larger hourly rate?

In the case of the rst automobile, the mileage rate is 28.1 mi/1 gal, which can be read 28.1 miles per gallon. In the case of the second automobile, the mileage rate is 27.2 mi/1 gal, which can be read 27.2 miles per gallon. Therefore, the rst automobile gets the better gas mileage.

Answer: Alicia

Version: Fall 2010

454

CHAPTER 6. RATIO AND PROPORTION

l l l

Exercises

l l l

In Exercises 1-24, express the given ratio as a fraction reduced to lowest terms. 1. 0.14 : 0.44 2. 0.74 : 0.2 3. 0.05 : 0.75 4. 0.78 : 0.4 5. 0.1 : 0.95 6. 0.93 : 0.39
1 7. 2 2 : 1 3 9 8 13. 2 9 : 2 2 3 2 14. 1 3 : 3 8 9 8 15. 3 9 : 2 1 3 5 16. 1 9 : 1 1 3 5 17. 2 8 : 1 3 4 4 18. 2 9 : 1 1 3

19. 10 : 35 20. 132 : 84 21. 9 : 33 22. 35 : 10 23. 27 : 99 24. 12 : 28

8.

32 3

4 29

9. 0.36 : 0.6 10. 0.58 : 0.42 11. 15 : 21 12. 77 : 121

25. One automobile travels 271.8 miles on 10.1 gallons of gasoline. A second automobile travels 257.9 miles on 11.1 gallons of gasoline. Which automobile gets the better gas mileage? 26. One automobile travels 202.9 miles on 13.9 gallons of gasoline. A second automobile travels 221.6 miles on 11.8 gallons of gasoline. Which automobile gets the better gas mileage? 27. Todd is paid 183 dollars for 8.25 hours work. What is his hourly salary rate, rounded to the nearest penny? 28. David is paid 105 dollars for 8.5 hours work. What is his hourly salary rate, rounded to the nearest penny? 29. An automobile travels 140 miles in 4 hours. Find the average rate of speed. Version: Fall 2010

30. An automobile travels 120 miles in 5 hours. Find the average rate of speed. 31. Judah is paid 187 dollars for 8 hours work. What is his hourly salary rate, rounded to the nearest penny? 32. Judah is paid 181 dollars for 8.75 hours work. What is his hourly salary rate, rounded to the nearest penny? 33. One automobile travels 234.2 miles on 10.8 gallons of gasoline. A second automobile travels 270.5 miles on 10.8 gallons of gasoline. Which automobile gets the better gas mileage? 34. One automobile travels 297.6 miles on 10.7 gallons of gasoline. A second automobile travels 298.1 miles on 12.6 gallons of gasoline. Which automobile gets the better gas mileage?

6.1. INTRODUCTION TO RATIOS AND RATES 35. An automobile travels 180 miles in 5 hours. Find the average rate of speed.

455

36. An automobile travels 220 miles in 5 hours. Find the average rate of speed.

37. Antarctic trek. Seven women on a 562-mile Antarctic ski trek reached the South Pole 38 days after they began their adventure. What was the ladies average rate of speed per day? Round your result to the nearest tenth of a mile. Associated Press-Times-Standard 12/31/09 After 562-mile ski trek, seven women reach the South Pole.

l l l
1. 3. 5. 7. 9. 11. 13. 7 22 1 15 2 19 5 3 3 5 5 7 13 12

Answers
19. 2 7 3 11 3 11

l l l

21.

23.

25. The rst automobile has the better mileage per gallon. 27. 22.18 dollars/hr 29. 35 mi/hr 31. 23.38 dollars/hr 33. The second automobile has the better mileage per gallon. 35. 36 mi/hr 37. 14.8 miles per day

5 15. 3 17. 3 2

Version: Fall 2010

456

CHAPTER 6. RATIO AND PROPORTION

6.2

Introduction to Proportion

In Section 6.1, we introduced the concepts or ratio and rate. In this section, we equate these ratios in a construct called a proportion. Proportions. A proportion is a statement that equates two ratios or rates. For example, each of the equations 1 2 = , 3 6 15 miles 30 miles = , 2 hours 4 hours and a c = b d

compare two ratios or rates and is a proportion. The proportion 2 1 = 3 6 is read one is to three as two is to six. The four numbers that make up this proportion are called the terms of the proportion and are ordered in a natural manner. 2 Third term First term 1 = 6 Fourth term Second term 3 Extremes and Means. The rst and fourth terms are called the extremes of the proportion. The second and third terms are called the means of the proportion. In the proportion

c a = , b d the terms a and d are the extremes; the terms b and c are the means. a c extremes = b d a b c d means

If we multiply both sides of the proportion by the common denominator, bd then cancel, b d we get the following result. ad = bc This leads to the following observation. Version: Fall 2010 a = bd b a = bd b c d c d

6.2. INTRODUCTION TO PROPORTION

457

Product of Extremes and Means. In the proportion a c = b d the product of the means equals the product of the extremes. That is, ad = bc.

We can get an equivalent result using a technique called cross multiplication. a c = b d Product of means = bc Product of extremes = ad You Try It! 7 2 , or EXAMPLE 1. Which of the following is a valid proportion: (a) = 3 12 12 4 . (b) = 9 27 Solution. (a) Cross multiply 2 7 = 3 12 to get 24 = 21. Hence, the product of the extremes does not equal the product of the means, so 2/3 = 7/12 is not a valid proportion. (b) Cross multiply 12 4 = 9 27 to get 108 = 108. Hence, the product of the extremes equals the product of the means, so 4/9 = 12/27 is a valid proportion. Answer: No Product of means Product of extremes Product of means Product of extremes Is the following a valid proportion? 16 4 = 3 11

Version: Fall 2010

458

CHAPTER 6. RATIO AND PROPORTION

Solving Proportions
We already have all the tools needed to solve proportions. Lets begin with the rst example. You Try It! Solve the proportion for n: 2 n = 3 9 x 3 = . 4 12 Solution. Cross multiply, then solve the resulting equation. EXAMPLE 2. Solve the proportion for x: x 3 = 4 12 4 x = 3 12 4x = 36 4x 36 = 4 4 x=9 Original proportion. Products of means and extremes are equal. Simplify. Divide both sides by 4. Simplify.

Check. Substitute 9 for x into the original proportion and check. x 3 = 4 12 3 9 = 4 12 Cross multiply. 3 9 = 4 12 Answer: 6 Thus, the solution 9 checks. Product of means = 36 Product of extremes = 36 Original proportion. Substitute 9 for x.

You Try It! Solve the proportion for m: 9 m = 6 4 24 3 = . 2 n Solution. Cross multiply, then solve the resulting equation. EXAMPLE 3. Solve the proportion for n: 24 3 = 2 n 3 n = 2 24 3n = 48 48 3n = 3 3 n = 16 Version: Fall 2010 Original proportion. Products of means and extremes are equal. Simplify. Divide both sides by 3. Simplify.

6.2. INTRODUCTION TO PROPORTION Check. Substitute 16 for n into the original proportion and check. 3 24 = 2 n 3 24 = 2 16 Cross multiply. Product of means = 48 Product of extremes = 48 Original proportion. Substitute 16 for n.

459

3 24 = 2 16 Thus, the solution 16 checks.

Answer: 6

You Try It! EXAMPLE 4. Solve the proportion for x: 1 2x + 1 = . 15 3 Solve the proportion for y: 6 + 2y 8 = 18 9

Solution. Cross multiply, then solve the resulting equation. 2x + 1 1 = 15 3 3(2x + 1) = 15(1) 6x + 3 = 15 6x + 3 3 = 15 3 6x = 12 6x 12 = 6 6 x=2 Original proportion. Products of means and extremes are equal. On the left, distribute. On the right, multiply. Subtract 3 from both sides. Simplify. Divide both sides by 6. Simplify both sides.

Check. Well leave it to our readers to check this solution.

Answer: 5

Applications
A number of practical applications involve solving a proportion. You Try It! EXAMPLE 5. If 5 oranges cost $1.15, what will be the cost for 15 oranges (assuming an equal rate)? Version: Fall 2010 If 7 apples cost $3.15, how much will 10 apples cost (assuming an equal rate)?

460

CHAPTER 6. RATIO AND PROPORTION

Solution. Let x represent the cost for 15 oranges. Assuming the rate for 5 oranges at $1.15 equals the rate for 15 oranges at an unknown cost x, we set up the following proportion. 15 5 = 1.15 x Product of means = (1.15)(15) Cross multiply 5 15 = 1.15 x Product of extremes = 5x to get 5x = 17.25. Solve for x. 17.25 5x = 5 5 x = 3.45 Answer: $4.50 Thus, 15 oranges cost $3.45.

Checking Units is Extremely Important. When setting up a proportion, check to make sure that both numerators have the same units and both denominators have the same units. For example, in Example 5, both numerators have oranges as units and both denominators have dollars as units. Oranges Dollars 5 15 = 1.15 x Oranges Dollars

This proportion is set up correctly, because both numerators have the same units and both denominators have the same units. On the other hand, if we had set the proportion up incorrectly as follows, Oranges Dollars 5 x = 1.15 15 Dollars Oranges

a quick check of the units reveals the error; i.e., the numerators have dierent units and the denominators have dierent units. Checking units helps us avoid errors! You Try It! Eloise and Susannah are planning a trip is Sequoia National Park. On their map, 3 inches represents 50 miles. How long is their trip if the route measures 4 1 2 inches on the map? EXAMPLE 6. Dylan and David are planning a backpacking trip in Yosemite National Park. On their map, the legend indicates that 1.2 centimeters represents 2 miles. How long is their trip if the route measures 10.6 centimeters on the map? Round your answer to the nearest tenth of a mile. Version: Fall 2010

6.2. INTRODUCTION TO PROPORTION Solution. Lets set up the proportion with units.

461

1.2 cm 10.6 cm = 2 mi x mi Note how including the units aids in the setup of the proportion. Now, lets drop the units and solve for x. 10.6 1.2 = 2 x 1.2x = (2)(10.6) 1.2x = 21.2 1.2x 21.2 = 1.2 1.2 x 17.66 Original proportion. Cross multiply. Simplify right-hand side. Divide both sides by 1.2. On the right: Divide.

We carried the division on the right one decimal place past the tenths place. The rounding digit is a 6 and the following test digit is a 6. Add 1 to the rounding digit and truncate. To the nearest tenth of a mile, the backpacking route is approximately 17.7 miles.

Answer: 75 miles

You Try It! EXAMPLE 7. A recipe making 2 dozen cookies requires 1 3 cups of our, 4 among other ingredients. If the baker wishes to make twice that number of cookies, how much our is required? Solution. Twice 2 dozen is 4 dozen cookies. Let x represent the amount of our needed for 4 dozen cookies. Assuming an equal rate for 2 dozen cookies 3 (2 dozen requires 1 4 cups of our), we set up the following proportion. Again, using units helps us craft the correct proportion. 2 dozen 4 dozen = 3 x cups 1 4 cups Note how including the units aids in the setup of the proportion. Now, lets drop the units and solve for x. 2 4 3 = x 14 3 2x = 1 4 4 7 2x = 4 4 2x = 7 Original proportion. Cross multiply. Change to improper fraction. Multiply. Version: Fall 2010 Dough for 3 pizzas requires 8 1 cups of our. If the baker 2 wishes to make 9 pizzas, how many cups of our are required?

462

CHAPTER 6. RATIO AND PROPORTION

Divide both sides of the equation by 2 and nish. 2x 7 = 2 2 7 x= 2 Answer:


1 25 2

Divide both sides by 2.

cups

1 Change the improper fraction to a mixed fraction. Thus, it will take 3 2 cups of our to make 4 dozen cookies.

Version: Fall 2010

6.2. INTRODUCTION TO PROPORTION

463

l l l

Exercises

l l l

In Exercises 1-12, which of the following is a true proportion? 1. 2. 3. 4. 5. 6. 9 7 6 7 7 6 7 6 6 5 9 8 = = = = = = 27 , 21 18 , 21 28 , 24 2 , 8 24 , 20 4 , 3 4 9 7 8 = , = , 3 7 2 9 2 8 4 3 = , = , 3 6 3 2 5 5 9 7 = , = , 6 4 5 3 4 5 3 15 = , = , 5 7 4 20 7 2 2 2 = , = , 3 4 4 6 5 10 8 5 = , = , 7 14 6 4 4 8 8 9 9 2 8 4 5 2 8 5 = = = = = = 9 6 3 8 8 9 8 7 2 8 2 6 7. 8. 9. 10. 11. 12. 3 5 7 3 5 4 7 6 9 7 4 3 = = = = = = 2 , 8 7 , 6 25 , 20 6 , 9 4 , 3 8 , 7 3 6 = , 7 14 4 8 = , 7 14 9 9 = , 3 6 7 2 = , 3 5 9 7 = , 4 9 2 5 = , 6 8 5 2 7 5 = , = 6 4 4 9 5 7 5 6 = , = 3 8 7 9 7 3 3 9 = , = 4 6 5 4 6 30 4 2 = , = 7 35 7 8 3 6 3 9 = , = 5 10 9 5 7 3 9 36 = , = 2 6 4 16

In Exercises 13-36, solve the given proportion. 13. 14. 15. 16. 17. 18. 19. 20. 21. 22. x 17 = 3 18 x 16 = 5 20 11 6x + 10 = 6 3 5 4x + 8 = 12 3 x 17 = 9 18 8 x = 9 18 x 11 = 2 8 x 11 = 4 8 10 7x + 15 = 15 3 5 7x + 3 = 8 4 23. 24. 25. 26. 27. 28. 29. 30. 31. 32. 33. x 11 = 2 10 x 19 = 6 18 2 5x + 8 = 12 3 3 3x + 12 = 6 2 24 2 = 15 x 14 7 = 8 x 6 3 = 16 x 12 4 = 21 x 20 5 = 22 x 21 3 = 22 x 14 2x + 10 = 6 3 Version: Fall 2010

464 2x + 9 13 = 9 3 21 7 35. = 2 x

CHAPTER 6. RATIO AND PROPORTION 36. 18 2 = 15 x

34.

37. If 13 dog bones cost $1.97, what will be the cost for 7 dog bones (assuming an equal rate)? Round your answer to the nearest penny. 38. If 2 watermelons cost $3.89, what will be the cost for 11 watermelons (assuming an equal rate)? Round your answer to the nearest penny. 39. If 7 bananas cost $2.55, what will be the cost for 14 bananas (assuming an equal rate)? Round your answer to the nearest penny. 40. If 2 apples cost $2.05, what will be the cost for 8 apples (assuming an equal rate)? Round your answer to the nearest penny. 41. If 13 oranges cost $3.61, what will be the cost for 11 oranges (assuming an equal rate)? Round your answer to the nearest penny. 42. If 3 watermelons cost $1.05, what will be the cost for 9 watermelons (assuming an equal rate)? Round your answer to the nearest penny.

43. If 3 dog bones cost $1.23, what will be the cost for 13 dog bones (assuming an equal rate)? Round your answer to the nearest penny. 44. If 3 watermelons cost $4.41, what will be the cost for 7 watermelons (assuming an equal rate)? Round your answer to the nearest penny. 45. If 3 apples cost $3.24, what will be the cost for 13 apples (assuming an equal rate)? Round your answer to the nearest penny. 46. If 6 apples cost $3.43, what will be the cost for 7 apples (assuming an equal rate)? Round your answer to the nearest penny. 47. If 4 dog bones cost $1.03, what will be the cost for 8 dog bones (assuming an equal rate)? Round your answer to the nearest penny. 48. If 4 oranges cost $4.28, what will be the cost for 3 oranges (assuming an equal rate)? Round your answer to the nearest penny.

49. Two rolls. In Haiti, two at rolls cost 5 gourdes, about 12 cents. How many cents would 20 rolls cost? Associated Press-Times-Standard 02/18/10 Haitis earthquake camps turning into shanty towns. 50. Turbines. As proposed, the Shell Wind Energy project consists of 25 ridge-top turbines that can generate up to 50 megawatts, or enough to supply electricity to about 1, 000 homes. Estimate the number of ridge-top turbines that would be needed to supply electricity to 70, 000 homes, the approximate number of properties in Humboldt County, CA. John Driscoll Times-Standard 12/24/09 Wind power project goes under analysis.

Version: Fall 2010

6.2. INTRODUCTION TO PROPORTION

465

51. Dumptrucks. U.S. Highway 199 had a landslide where as much as 3, 000 cubic yards of material fell on the road, reportedly requiring about 200 large dumptrucks to remove. Only a week earlier, 40, 000 cubic yards of material fell on Highway 96. Estimate the number of dumptrucks needed for that slide rounded to the nearest whole number. Associated Press-Times-Standard 03/09/10 Another highway closed by slide. 52. Timber sales. Alaskas 26, 000 square mile Tongass National Forest plan allows for timber sales of up to 267 million board-feet per year enough for nearly 27, 000 two-bedroom homes, but demand for timber is far short of that. Less than 25 million board-feet was logged in the forest in 2009. Forest Service ocials have said they hope to increase logging in the Tongass to about 100 million board-feet per year. Associated Press-Times-Standard 02/18/10 Industry loses lawsuit over logging in Alaska. i) Estimate the number of 2-bedroom homes that 25 million board-feet of timber would build. ii) How many 2-bedroom homes would 100 million board-feet of timber build? 53. Costly spill. In Australia, penalties on ships causing oil spills are approximately $1.75 million Australian dollars, equivalent to about $1.64 million US dollars. After an oil tanker was grounded onto a coral reef, Australian ocials are considering raising the ne to $10 million Australian dollars. What will the new ne be in US dollars? Round your answer to the nearest hundredth of a million dollars. Associated Press-Times-Standard 04/13/10 Ship that leaked oil on Great Barrier Reef removed.

l l l
1. 27 9 = is a proportion 7 21

Answers
17. 34 19. 44 21. 5 23. 55 25. 0

l l l

7 28 3. = is a proportion 6 24 5. 7. 9. 24 6 = is a proportion 5 20 6 3 = is a proportion 7 14 25 5 = is a proportion 4 20

27. 180 29. 32 31. 88 33. 9 35. 6 37. $1.06 Version: Fall 2010

6 3 is a proportion 11. = 5 10 13. 102 15. 2

466 39. $5.10 41. $3.05 43. $5.33 45. $14.04

CHAPTER 6. RATIO AND PROPORTION 47. $2.06 49. $0.48 51. 2, 667 loads 53. $9.37 million US dollars

Version: Fall 2010

6.3. UNIT CONVERSION: AMERICAN SYSTEM

467

6.3

Unit Conversion: American System

In this section we will develop a technique for converting units used in the American system. We begin with a discussion of common measurements of length in the United States.

Units of Length
The most common units of length are the inch, foot, yard, and mile. Our focus will be on the technique used to convert from one unit of length to another.

American Units of Length. Facts relating common units of length. 1 foot (ft) = 12 inches (in) 1 mile (mi) = 5280 feet (ft) 1 yard (yd) = 3 feet (ft)

Take for example, the fact that there are 3 feet in 1 yard, which can be stated as an equation, using the common abbreviations for feet (ft) and yards (yd). 3 ft = 1 yd If we divide both sides of the equation by 3 ft, 3 ft 1 yd = , 3 ft 3 ft or equivalently, 1= 1 yd . 3 ft

The key observation is the fact that the ratio 1 yd/3 ft equals the number 1. Consequently, multiplying by the conversion factor 1 yd/3 ft is equivalent to multiplying by 1. This can be used to change a measurement in feet to yards. You Try It! EXAMPLE 1. Change 36 feet to yards. Version: Fall 2010 Change 81 feet to yards.

468

CHAPTER 6. RATIO AND PROPORTION

Solution. Multiply by the conversion factor 1 yd/3 ft. 36 ft = 36 ft 1 1 yd = 36 ft 3 ft 1 yd = 36 ft 3 ft 36 1 yd = 3 36 = yd 3 = 12 yd Answer: 27 yards Hence, 36 feet equals 12 yards. Multiplicative Identity Property. Replace 1 with 1 yd/3 ft. Cancel common unit. Multiply fractions. Simplify. Divide.

On the other hand, we can start again with 3 ft = 1 yd and divide both sides of the equation by 1 yd. 1 yd 3 ft = 1 yd 1 yd This gives the conversion factor 3 ft = 1. 1 yd The key observation is the fact that the ratio 3 ft/1 yd equals the number 1. Consequently, multiplying by the conversion factor 3 ft/1 yd is equivalent to multiplying by 1. This can be used to change a measurement in yards to feet. You Try It! Change 15 yards to feet. EXAMPLE 2. Change 18 yards to feet. Solution. Multiply by the conversion factor 3 ft/1 yd. 18 yd = 18 yd 1 3 ft = 18 yd 1 yd 3 ft = 18 yd 1 yd 18 3 = ft 1 = 54 ft Multiplicative Identity Property. Replace 1 with 3 ft/1 yd. Cancel common unit. Multiply fractions. Simplify.

Version: Fall 2010

6.3. UNIT CONVERSION: AMERICAN SYSTEM Hence, 18 yards equals 54 feet.

469 Answer: 45 feet

Another common comparison is the fact that there are 12 inches in 1 foot. This can be represented as an equation using the common abbreviation for inches (in) and feet (ft). 12 in = 1 ft Dividing both sides by 12 in 1 ft 12 in = , 12 in 12 in yields the conversion factor 1 ft . 12 in The key observation is the fact that the ratio 1 ft/12 in equals the number 1. Consequently, multiplying by the conversion factor 1 ft/12 in is equivalent to multiplying by 1. This can be used to change a measurement in inches to feet. 1= You Try It! EXAMPLE 3. Change 24 inches to feet. Solution. Multiply by the conversion factor 1 ft/12 in. 24 in = 24 in 1 1 ft = 24 in 12 in 1 ft = 24 in 12 in 24 1 ft = 12 = 2 ft Hence, 24 inches equals 2 feet. Multiplicative Identity Property. Replace 1 with 1 ft/12 in. Cancel common unit. Multiply fractions. Simplify. Answer: 4 feet Change 48 inches to feet.

We provide a summary of conversion factors for units of length in Table 6.1. Convert feet to inches yards to feet miles to feet Conversion Factor 12 in/1 ft 3 ft/1 yd 5280 ft/1 mi Convert inches to feet feet to yards feet to miles Conversion Factor 1 ft/12 in 1 yd/3 ft 1 mi/5280 ft

Table 6.1: Conversion factors for units of length. Some conversions require more than one application of a conversion factor. Version: Fall 2010

470

CHAPTER 6. RATIO AND PROPORTION

You Try It! Change 8 yards to inches. EXAMPLE 4. Change 4 yards to inches. Solution. We multiply by a chain of conversion factors, the rst to change yards to feet, the second to change feet to inches. 4 yd = 4 yd 3 ft 12 in 1 yd 1 ft 3 12 in ft = 4 yd ft 1 1 yd 4 3 12 in = 11 = 144 in Multiply by conversion factors. Cancel common units. Multiply fractions. Simplify.

Answer: 288 inches

Hence, 4 yards equals 144 inches.

You Try It! Change 5 miles to yards. EXAMPLE 5. Change 2 miles to yards. Solution. We multiply by a chain of conversion factors, the rst to change miles to feet, the second to change feet to yards. 2 mi = 2 mi 5280 ft 1 yd 1 mi 3 ft 5280 1 yd ft & = 2& mi & 1& mi 3 ft 2 5280 1 yd = 13 = 3520 yd Multiply by conversion factors. Cancel common units. Multiply fractions. Simplify.

Answer: 8,800 yards

Hence, 2 miles equals 3,520 yards.

Units of Weight
The most common units of weight are the ounce, pound, and ton. Our focus will remain on how to convert from one unit to another. American Units of Weight. Facts relating common units of weight. 1 pound (lb) = 16 ounces (oz) Version: Fall 2010 1 ton = 2000 pounds (lb)

6.3. UNIT CONVERSION: AMERICAN SYSTEM The above facts lead to the conversion factors in Table 6.2. Convert pounds to ounces tons to pounds Conversion Factor 16 oz/1 lb 2000 lb/1 ton Convert ounces to pounds pounds to tons

471

Conversion Factor 1 lb/16 oz 1 ton/2000 lb

Table 6.2: Conversion factors for units of weight.

You Try It! EXAMPLE 6. Change 2 1 pounds to ounces. 2 Solution. Multiply by the appropriate conversion factor. 1 1 16 oz 2 lb = 2 lb 2 2 1 lb 1 16 oz =2 lb 2 1 lb 1 = 2 16 oz 2 5 = 16 oz 2 80 = oz 2 = 40 oz Multiply by conversion factor. Cancel common units. Multiply fractions. Mixed to improper fraction. Multiply. Divide.
1 Change 6 4 pounds to ounces.

1 Hence, 2 2 pounds equals 40 ounces.

Answer: 100 ounces

You Try It! EXAMPLE 7. Change 3.2 tons to ounces. Solution. This exercise requires multiplying by a chain of conversion factors. 3.2 ton = 3.2 ton 2000 lb 16 oz 1 ton 1 lb lb 2000 16 oz = 3.2 ton 1 ton 1 lb 3.2 2000 16 oz = 11 = 102, 400 oz Multiply by conversion factors. Cancel common units. Multiply fractions. Simplify. Answer: 128,000 ounces Change 4.1 tons to ounces.

Hence, 3.2 tons equals 102,400 ounces.

Version: Fall 2010

Units of Volume
The most common units of volume are uid ounces, cups, pints, quarts, and gallons. We will focus on converting from one unit to another.

472

CHAPTER 6. RATIO AND PROPORTION

American Units of Volume. Facts relating common units of volume. 1 cup (c) = 8 uid ounces ( oz) 1 quart (qt) = 2 pints (pt) 1 pint (pt) = 2 cups (c) 1 gallon (gal) = 4 quarts (qt)

These facts lead to the conversion factors listed in Table 6.3. Convert cups to ounces pints to cups quarts to pints gallons to quarts Conversion Factor 8 oz/1 c 2 c/1 pt 2 pt/1 qt 4 qt/1 gal Convert ounces to cups cups to pints pints to quarts quarts to gallons Conversion Factor 1 c/8 oz 1 pt/2 c 1 qt/2 pt 1 gal/4 qt

Table 6.3: Conversion factors for units of volume.

You Try It! Change 3.2 gallons to pints. EXAMPLE 8. Change 5.6 gallons to pints. Solution. This exercise requires multiplying by a chain of conversion factors. 5.6 gal = 5.6 gal 4 qt 2 pt 1 gal 1 qt 4 qt 2 pt & gal = 5.6 & & 1& 1 qt gal 5.6 4 2 = pt 11 = 44.8 pt Multiply by conversion factors. Cancel common units. Multiply fractions. Simiplify.

Answer: 25.6 pints

Hence, 5.6 gallons equals 44.8 pints.

Units of Time
The most common units of time are seconds, minutes, hours, days, and years. American Units of Time. Facts relating common units of time. 1 minute (min) = 60 seconds (s) 1 day (day) = 24 hours (hr) 1 hour (hr) = 60 minutes (min) 1 year (yr) = 365 days (day)

Version: Fall 2010

6.3. UNIT CONVERSION: AMERICAN SYSTEM These facts lead to the conversion factors in Table 6.4. Convert minutes to seconds hours to minutes days to hours years to days Conversion Factor 60 s/1 min 60 min/1 hr 24 hr/1 day 365 day/1 yr Convert seconds to minutes minutes to hours hours to days days to years

473

Conversion Factor 1 min/60 s 1 hr/60 min 1 day/24 hr 1 yr/365 day

Table 6.4: Conversion factors for units of time.

You Try It! EXAMPLE 9. How many seconds in a year? Solution. A chain of conversion factors is needed. 365 day 24 hr 60 min 60 s 1 yr = 1 yr 1 yr 1 day 1 hr 1 min & 365 day 24 & 60 60 s hr min = 1& yr & 1 & 1 & 1& yr 1 day hr min & 1 365 24 60 60 = s 1111 = 31, 536, 000 s Thus, 1 year equals 31,536,000 seconds. How many seconds in a day?

Conversion factors. Cancel common units. Multiply fractions. Simplify. Answer: 86,400 seconds

Converting Units of Speed


Ever wonder how fast a baseball is moving? You Try It! EXAMPLE 10. A professional pitcher can throw a baseball at 95 miles per hour. How fast is this in feet per second? Round your answer to the nearest foot per second. Solution. There are 5280 feet in a mile, 60 minutes in an hour, and 60 seconds in a minute. mi 5280 ft 1h 1 min mi 95 Conversion factors. 95 h h 1 mi 60 min 60 s & mi 1h 1 min & 5280 ft 95 Cancel common units. 60 s & 60 min 1& mi h 95 5280 1 1 ft Multiply fractions. 1 60 60 s ft 139.3 Multiply and divide. s Version: Fall 2010 A womens softball pitcher can throw her fastball at 60 miles per hour. How fast is this in feet per second? Round your answer to the nearest foot per second.

474

CHAPTER 6. RATIO AND PROPORTION

To round to the nearest foot per second, identify the rounding and test digits. Test digit 13 9 . 3 Rounding digit Because the test digit is less than 5, leave the rounding digit alone and truncate. Thus, to the nearest foot per second, the speed is approximately 139 feet per second. Whew! Since the batter stands at home plate, which is about 60 feet from where the pitch is delivered, the batter has less than 1/2 a second to react to the pitch!

Answer: 88 feet per second

Version: Fall 2010

6.3. UNIT CONVERSION: AMERICAN SYSTEM

475

l l l
1. Change 8 yards to feet. 2. Change 60 yards to feet. 3. Change 261 feet to yards. 4. Change 126 feet to yards.

Exercises

l l l

20. Change 19,257 feet to miles. Round your answer to the nearest tenth of a mile. 21. Change 5 yards to feet. 22. Change 20 yards to feet. 23. Change 169,312 inches to miles, correct to the nearest tenth of a mile. 24. Change 162,211 inches to miles, correct to the nearest tenth of a mile. 25. Change 1.5 yards to inches. Round your answer to the nearest inch. 26. Change 2.1 yards to inches. Round your answer to the nearest inch. 27. Change 360 inches to feet. 28. Change 768 inches to feet. 29. Change 48 inches to feet. 30. Change 528 inches to feet. 31. Change 15,363 feet to miles. Round your answer to the nearest tenth of a mile. 32. Change 8,540 feet to miles. Round your answer to the nearest tenth of a mile. 33. Change 1.7 miles to inches. 34. Change 4.7 miles to inches. 35. Change 3.1 miles to inches. 36. Change 1.8 miles to inches. 37. Change 3.6 miles to feet. 38. Change 3.1 miles to feet. 39. Change 18 feet to inches. 40. Change 33 feet to inches.

5. Change 235 inches to yards. Round your answer to the nearest tenth of a yard. 6. Change 244 inches to yards. Round your answer to the nearest tenth of a yard. 7. Change 141 feet to yards. 8. Change 78 feet to yards. 9. Change 2.8 miles to feet. 10. Change 4.9 miles to feet. 11. Change 104 inches to yards. Round your answer to the nearest tenth of a yard. 12. Change 101 inches to yards. Round your answer to the nearest tenth of a yard. 13. Change 168,372 inches to miles, correct to the nearest tenth of a mile. 14. Change 198,550 inches to miles, correct to the nearest tenth of a mile. 15. Change 82 feet to inches. 16. Change 80 feet to inches. 17. Change 2.9 yards to inches. Round your answer to the nearest inch. 18. Change 4.5 yards to inches. Round your answer to the nearest inch. 19. Change 25,756 feet to miles. Round your answer to the nearest tenth of a mile.

Version: Fall 2010

476 41. Change 5 1 pounds to ounces. 8 42. Change 3


1 16

CHAPTER 6. RATIO AND PROPORTION 51. Change 9,560 pounds to tons. Round your answer to the nearest tenth of a ton. 52. Change 9,499 pounds to tons. Round your answer to the nearest tenth of a ton. 53. Change 2 1 pounds to ounces. 2
1 54. Change 7 16 pounds to ounces.

pounds to ounces.

43. Change 2.4 tons to ounces. 44. Change 3.4 tons to ounces. 45. Change 34 ounces to pounds. Express your answer as a fraction reduced to lowest terms. 46. Change 78 ounces to pounds. Express your answer as a fraction reduced to lowest terms. 47. Change 2.2 tons to pounds. 48. Change 4.8 tons to pounds. 49. Change 70 ounces to pounds. Express your answer as a fraction reduced to lowest terms. 50. Change 20 ounces to pounds. Express your answer as a fraction reduced to lowest terms.

55. Change 5.9 tons to pounds. 56. Change 2.1 tons to pounds. 57. Change 2.5 tons to ounces. 58. Change 5.3 tons to ounces. 59. Change 8,111 pounds to tons. Round your answer to the nearest tenth of a ton. 60. Change 8,273 pounds to tons. Round your answer to the nearest tenth of a ton.

61. Change 4.5625 pints to uid ounces. 62. Change 2.9375 pints to uid ounces. 63. Change 32 uid ounces to pints. 64. Change 160 uid ounces to pints. 65. Change 3.7 gallons to pints. 66. Change 2.4 gallons to pints. 67. Change 216 pints to gallons. 68. Change 96 pints to gallons.

69. Change 544 uid ounces to pints. 70. Change 432 uid ounces to pints. 71. Change 112 pints to gallons. 72. Change 200 pints to gallons. 73. Change 7.7 gallons to pints. 74. Change 5.7 gallons to pints. 75. Change 3.875 pints to uid ounces. 76. Change 3 pints to uid ounces.

77. Change 7.8 years to hours. 78. Change 4.7 years to hours. 79. Change 7.6 years to hours. 80. Change 6.6 years to hours. 81. Change 4,025,005 seconds to days. Round your answer to the nearest tenth of a day. Version: Fall 2010

82. Change 4,672,133 seconds to days. Round your answer to the nearest tenth of a day. 83. Change 37,668 hours to years. 84. Change 40,296 hours to years. 85. Change 22,776 hours to years. 86. Change 29,784 hours to years.

6.3. UNIT CONVERSION: AMERICAN SYSTEM 87. Change 96 days to seconds. 88. Change 50 days to seconds. 89. Change 40 days to seconds. 90. Change 10 days to seconds.

477

91. Change 3,750,580 seconds to days. Round your answer to the nearest tenth of a day. 92. Change 4,493,469 seconds to days. Round your answer to the nearest tenth of a day.

93. Change 367 feet per second to miles per hour. Round your answer to the nearest mile per hour. 94. Change 354 feet per second to miles per hour. Round your answer to the nearest mile per hour. 95. Change 442 feet per second to miles per hour. Round your answer to the nearest mile per hour. 96. Change 388 feet per second to miles per hour. Round your answer to the nearest mile per hour.

97. Change 30 miles per hour to feet per second. Round your answer to the nearest foot per second. 98. Change 99 miles per hour to feet per second. Round your answer to the nearest foot per second. 99. Change 106 miles per hour to feet per second. Round your answer to the nearest foot per second. 100. Change 119 miles per hour to feet per second. Round your answer to the nearest foot per second.

101. Strong man. Famed strongman Joe Rollino, who was still bending quarters with his ngers at age 104, once lifted 3, 200 pounds at Coney Island Amusement Park. How many tons did Joe lift that day? Associated Press-Times-Standard 01/12/10 NYC amusement park strongman, 104, killed by van. 102. Earth day. The amount of time it takes the Earth to rotate once around its axis is one day. How many seconds is that? 103. Water break. The average age of Washington, DCs water pipes is 76 years, and they are not alone. Every two minutes, somewhere in the country, a pipe breaks. How many pipes break each year in the US? New York Times 03/14/10 Saving U.S. water and sewer systems could be costly.

l l l
1. 24 feet 3. 87 yards 5. 6.5 yards

Answers

l l l

7. 47 yards 9. 14,784 feet 11. 2.9 yards Version: Fall 2010

478 13. 2.7 miles 15. 984 inches 17. 104 inches 19. 4.9 miles 21. 15 feet 23. 2.7 miles 25. 54 inches 27. 30 feet 29. 4 feet 31. 2.9 miles 33. 107,712 inches 35. 196,416 inches 37. 19,008 feet 39. 216 inches 41. 82 ounces 43. 76,800 ounces 45. 2
1 8

CHAPTER 6. RATIO AND PROPORTION 59. 4.1 tons 61. 73 uid ounces 63. 2 pints 65. 29.6 pints 67. 27 gallons 69. 34 pints 71. 14 gallons 73. 61.6 pints 75. 62 uid ounces 77. 68,328 hours 79. 66,576 hours 81. 46.6 days 83. 4.3 years 85. 2.6 years 87. 8,294,400 seconds 89. 3,456,000 seconds 91. 43.4 days 93. 250 mi/hr 95. 301 mi/hr 97. 44 ft/s 99. 155 ft/s 101. 1.6 tons 103. 262, 800

pounds

47. 4,400 pounds 49. 4


3 8

pounds

51. 4.8 tons 53. 40 ounces 55. 11,800 pounds 57. 80,000 ounces

Version: Fall 2010

6.4. UNIT CONVERSION: METRIC SYSTEM

479

6.4

Unit Conversion: Metric System

The metric system of units is the standard system of units preferred by scientists. It is based on the base ten number system and its decimal format is more friendly to users of this system. There is a common set of prexes adopted by the metric system to indicate a power of ten to apply to the base unit. Metric System Prexes. This is a list of standard prexes for the metric system and their meanings. deka = 10 hecto = 100 kilo = 1000 deci = 1/10 centi = 1/100 milli = 1/1000

Thus, for example, a decameter is 10 meters, a hectoliter is 100 liters, and a kilogram is 1000 grams. Similarly, a decimeter is 1/10 of a meter, a centiliter is 1/100 of a liter, and a milligram is 1/1000 of a gram.

Units of Length
The standard measure of length in the metric system is the meter. Historically, the meter was dened by the French Academy of Sciences as the length between two marks on a platinum-iridium bar, which was designed to represent 110,000,000 of the distance from the equator to the north pole through Paris. In 1983, it was redened by the International Bureau of Weights and Measures (BIPM) as the distance travelled by light in free space in 1299,792,458 of a second. (Wikipedia) We can apply the standard prexes to get the following result. Metric Units of Length. These units of length are used in the metric system. Unit Length 1 kilometer = 1000 meters 1 hectometer = 100 meters 1 dekameter = 10 meters 1 meter 1 decimeter = 1 centimeter = 1 millimeter =
1 10

Unit Abbreviation km hm dam m meter meter meter dm cm mm Version: Fall 2010


1 100 1 1000

480

CHAPTER 6. RATIO AND PROPORTION

We can use these facts to build conversion factors as we did in Section 6.3. For example, because 1 km = 1000 m, we can divide both sides by 1000 m to produce the conversion factor 1= 1 km . 1000 m

This conversion factor can help change meters into kilometers. Before using this conversion factor in an example, we repeat here the rules for multiplying and dividing by powers of ten. We will be making heavy use of these rules in this section. Multiplying and Dividing by Powers of Ten. Multiplying a decimal number by 10n will move the decimal point n places to the right. For example, 3.2567 102 = 3.2567 100 = 325.67. Dividing a decimal number by 10n will move the decimal point n places to the left. For example, 3.2567/102 = 3.2567/100 = 0.032567.

And now the example. You Try It! Change 1,156 meters to kilometers. EXAMPLE 1. Change 2,326 meters to kilometers. Solution. Multiply by the conversion factor 1 km/1000 m. 2326 m = 2326 m 1 km 1000 m 1 km = 2326 & m 1000 & m 2326 1 = km 1000 = 2.326 km

Apply conversion factor. Cancel common units. Multiply fractions. Simplify.

In the last step, note that dividing by 1000 moves the decimal point three places to the left. Thus, 2326 meters is equal to 2.326 kilometers. Alternate Solution. A second solution depends upon the fact that multiplying or dividing by a power of ten will move the decimal point right or left a number of places equal to the number of zeros present in the multiplier or divisor. Thus, as we saw above, dividing by 1000 moved the decimal point 3 places to the left. Suppose that we arrange the metric units of length in order, from largest to smallest, as shown below. Version: Fall 2010

6.4. UNIT CONVERSION: METRIC SYSTEM km hm dam m dm cm mm

481

Note that we must move 3 places left to move from the meters (m) abbreviation to the kilometers (km) abbreviation. In like manner, if we write 2,326 meters as 2,326.0 meters, then we can convert to kilometers by moving the decimal 3 places to the left. 2, 326.0 m = 2.236 0 km = 2.326 km Answer: 1.156 kilometers

You Try It! EXAMPLE 2. Change 537 centimeters to meters. Solution. We know that 1 m, 100 or multiplying both sides of this result by 100, 1 cm = 100 cm = 1 m. Dividing both sides of this last result by 100 cm, we obtain the conversion factor 1 m/100 cm. 537 cm = 537 cm 1m 100 cm 1m = 537 cm 100 cm 537 1 = m 100 = 5.37 m Apply conversion factor. Cancel common units. Multiply fractions. Change 276 centimeters to meters.

In the last step, note that dividing by 100 moves the decimal point two places to the left. Alternately, we can set up our ordered list of units. km hm dam m dm cm mm

Note that we must move 2 places left to move from the centimeters (cm) abbreviation to the meters (m) abbreviation. In like manner, if we write 537 centimeters as 537.0 centimeters, then we can convert to meters by moving the decimal 2 places to the left. 537.0 cm = 5.37 0 m = 5.37 m Answer: 2.76 meters

Version: Fall 2010

482

CHAPTER 6. RATIO AND PROPORTION

Sometimes more than one conversion factor is needed. You Try It! Change 13.5 dekameters to centimeters. EXAMPLE 3. Change 10.2 dekameters to centimeters. Solution. We have two facts: 1 dam=10 m, which yields the conversion factor 10 m/1 dam. 1 cm=(1/100) m or 100 cm=1 m, which yields the conversion factor 100 cm/1 m. 10.2 dam = 10.2 dam 10 m 100 cm 1 dam 1m m $ 10 & 100 cm = 10.2 $ dam 1 $$ 1& m dam 10.2 10 100 = cm 11 = 10, 200 cm

Apply conversion factors. Cancel common units. Multiply fractions.

In the last step, note that multiplying by 10, then by 100, moves the decimal point three places to the right. Alternately, we can set up our ordered list of units. km hm dam m dm cm mm

Note that we must move 3 places right to move from the dekameters (dam) abbreviation to the centimeters (cm) abbreviation. In like manner, we can convert 10.2 dekameters to centimeters by moving the decimal 3 places to the right. 10.2 dam = 10 200 cm = 10, 200 cm Answer: 13,500 centimeters

Units of Mass
The fundamental unit of mass in the metric system is called a gram. Originally, it was dened to be equal to one cubic centimeter of water measured at the temperature of melting ice. Now it is simply dened as 1/1000 of a kilogram, which is dened by a physical prototype preserved by the International Bureau of Weights and Measures (Wikipedia). The mass of an object is not the same as an objects weight, but rather a resistance to motion when an external force is applied. The same metric system prexes apply. Version: Fall 2010

6.4. UNIT CONVERSION: METRIC SYSTEM

483

Metric Units of Mass. These units of mass are used in the metric system. Unit of Mass 1 kilogram = 1000 grams 1 hectogram = 100 grams 1 dekagram = 10 grams 1 gram 1 decigram = 1 centigram = 1 milligram =
1 10

Unit Abbreviation kg hg dag g gram gram gram dg cg mg

1 100 1 1000

You Try It! EXAMPLE 4. Convert 0.025 dekagrams to milligrams. Solution. Well use two conversion factors: 1 dag=10 g, which yields the conversion factor 10 g/1 dag. 1 mg=(1/1000) g, which yields the conversion factor 1000 mg/1 g. 0.025 dag = 0.025 dag 10 g 1000 mg 1 dag 1g 10 g 1000 mg = 0.025 dag 1 dag 1g 0.025 10 1000 = mg 11 = 250 mg Apply conversion factors. Cancel common units. Multiply fractions. Convert 0.05 dekagrams to milligrams.

Alternately, we can set up our ordered list of units. kg hg dag g dg cg mg

Note that we must move 4 places right to move from the dekagrams (dag) abbreviation to the milligrams (mg) abbreviation. In like manner, we can convert 0.025 dekagrams to milligrams by moving the decimal 4 places to the right. 0.0250 dag = 0 0250. mg = 250 mg Answer: 500 milligrams

Version: Fall 2010

484

CHAPTER 6. RATIO AND PROPORTION

Units of Volume
The fundamental unit of volume in the metric system is called a litre. Originally, one litre was dened as the volume of one kilogram of water measured at 4 C at 760 millimeters of mercury (Wikipedia). Currently, 1 litre is dened as 1 cubic decimeter (imagine a cube with each edge 1/10 of a meter). The same metric system prexes apply. Metric Units of Volume. These units of volume are used in the metric system. Unit of Volume 1 kilolitre = 1000 litres 1 hectolitre = 100 litres 1 dekalitre = 10 litres 1 litre 1 decilitre = 1 centilitre = 1 millilitre =
1 10

Unit Abbreviation kL hL daL L

litre litre litre

dL cL mL

1 100 1 1000

You Try It! Convert 5,763 millilitres to dekalitres. EXAMPLE 5. Convert 11,725 millilitres to dekalitres. Solution. Well use two conversion factors: 1 daL=10 L, which yields the conversion factor 1 daL/10 L. 1 mL=(1/1000) L, which yields the conversion factor 1 L/1000 mL. 11, 725 mL = 11, 725 mL 1 daL 1L 1000 mL 10 L 1 L 1 daL = 11, 725 mL 10  1000 mL L 11, 725 1 1 daL = 1000 10 = 1.1725 daL Apply conversion factors. Cancel common units. Multiply fractions.

Alternately, we can set up our ordered list of units. kL hL daL L dL cL mL

Version: Fall 2010

6.4. UNIT CONVERSION: METRIC SYSTEM

485

Note that we must move 4 places left to move from the millitres (mL) abbreviation to the dekalitres (daL) abbreviation. In like manner, we can convert 11,725 millilitres to dekalitres by moving the decimal 4 places to the left. 11, 725.0 mL = 1.1725 0 daL = 1.1725 daL Answer: 0.5763 dekalitres

Version: Fall 2010

486

CHAPTER 6. RATIO AND PROPORTION

l l l

Exercises

l l l

1. What is the meaning of the metric system prex centi? 2. What is the meaning of the metric system prex deka? 3. What is the meaning of the metric system prex hecto?

4. What is the meaning of the metric system prex kilo? 5. What is the meaning of the metric system prex deci? 6. What is the meaning of the metric system prex milli?

7. What is the meaning of the metric system abbreviation mg? 8. What is the meaning of the metric system abbreviation g? 9. What is the meaning of the metric system abbreviation m? 10. What is the meaning of the metric system abbreviation km? 11. What is the meaning of the metric system abbreviation kL? 12. What is the meaning of the metric system abbreviation daL? 13. What is the meaning of the metric system abbreviation hm? 14. What is the meaning of the metric system abbreviation dm? 15. What is the meaning of the metric system abbreviation dam?

16. What is the meaning of the metric system abbreviation cm? 17. What is the meaning of the metric system abbreviation dL? 18. What is the meaning of the metric system abbreviation L? 19. What is the meaning of the metric system abbreviation hg? 20. What is the meaning of the metric system abbreviation kg? 21. What is the meaning of the metric system abbreviation dg? 22. What is the meaning of the metric system abbreviation dag? 23. What is the meaning of the metric system abbreviation hL? 24. What is the meaning of the metric system abbreviation cL?

25. Change 5,490 millimeters to meters. 26. Change 8,528 millimeters to meters. 27. Change 64 meters to millimeters. 28. Change 65 meters to millimeters. 29. Change 4,571 millimeters to meters. Version: Fall 2010

30. Change 8,209 millimeters to meters. 31. Change 15 meters to centimeters. 32. Change 12 meters to centimeters. 33. Change 569 centimeters to meters. 34. Change 380 centimeters to meters.

6.4. UNIT CONVERSION: METRIC SYSTEM 35. Change 79 meters to centimeters. 36. Change 60 meters to centimeters. 37. Change 7.6 kilometers to meters. 38. Change 4.9 kilometers to meters. 39. Change 861 centimeters to meters. 40. Change 427 centimeters to meters. 41. Change 4,826 meters to kilometers.

487

42. Change 1,929 meters to kilometers. 43. Change 4,724 meters to kilometers. 44. Change 1,629 meters to kilometers. 45. Change 6.5 kilometers to meters. 46. Change 7.9 kilometers to meters. 47. Change 17 meters to millimeters. 48. Change 53 meters to millimeters.

49. Change 512 milligrams to centigrams. 50. Change 516 milligrams to centigrams. 51. Change 541 milligrams to centigrams. 52. Change 223 milligrams to centigrams. 53. Change 70 grams to centigrams. 54. Change 76 grams to centigrams. 55. Change 53 centigrams to milligrams. 56. Change 30 centigrams to milligrams. 57. Change 83 kilograms to grams. 58. Change 70 kilograms to grams. 59. Change 8,196 grams to kilograms. 60. Change 6,693 grams to kilograms.

61. Change 564 centigrams to grams. 62. Change 884 centigrams to grams. 63. Change 38 grams to centigrams. 64. Change 88 grams to centigrams. 65. Change 77 centigrams to milligrams. 66. Change 61 centigrams to milligrams. 67. Change 5,337 grams to kilograms. 68. Change 4,002 grams to kilograms. 69. Change 15 kilograms to grams. 70. Change 45 kilograms to grams. 71. Change 833 centigrams to grams. 72. Change 247 centigrams to grams.

73. Change 619,560 centilitres to kilolitres. 74. Change 678,962 centilitres to kilolitres. 75. Change 15.2 litres to millilitres. 76. Change 9.7 litres to millilitres. 77. Change 10,850 centilitres to litres. 78. Change 15,198 centilitres to litres. 79. Change 10.7 litres to millilitres. 80. Change 17.3 litres to millilitres. 81. Change 15,665 millilitres to litres.

82. Change 12,157 millilitres to litres. 83. Change 6.3 kilolitres to centilitres. 84. Change 8.3 kilolitres to centilitres. 85. Change 4.5 kilolitres to centilitres. 86. Change 6.2 kilolitres to centilitres. 87. Change 10.6 litres to centilitres. 88. Change 16.6 litres to centilitres. 89. Change 14,383 centilitres to litres. 90. Change 11,557 centilitres to litres. Version: Fall 2010

488 91. Change 9.9 litres to centilitres. 92. Change 19.5 litres to centilitres. 93. Change 407,331 centilitres to kilolitres.

CHAPTER 6. RATIO AND PROPORTION 94. Change 827,348 centilitres to kilolitres. 95. Change 14,968 millilitres to litres. 96. Change 18,439 millilitres to litres.

l l l
1. 1/100 3. 100 5. 1/10 7. milligram 9. meter 11. kilolitre or kiloliter 13. hectometer 15. dekameter 17. decilitre or deciliter

Answers

l l l

39. 8.61 meters 41. 4.826 kilometers 43. 4.724 kilometers 45. 6,500 meters 47. 17,000 millimeters 49. 51.2 centigrams 51. 54.1 centigrams 53. 7,000 centigrams 55. 530 milligrams 57. 83,000 grams

19. hectogram 59. 8.196 kilograms 21. decigram 61. 5.64 grams 23. hectolitre or hectoliter 25. 5.49 meters 27. 64,000 millimeters 29. 4.571 meters 31. 1,500 centimeters 33. 5.69 meters 35. 7,900 centimeters 37. 7,600 meters Version: Fall 2010 63. 3,800 centigrams 65. 770 milligrams 67. 5.337 kilograms 69. 15,000 grams 71. 8.33 grams 73. 6.1956 kilolitres 75. 15,200 millilitres 77. 108.5 litres

6.4. UNIT CONVERSION: METRIC SYSTEM 79. 10,700 millilitres 81. 15.665 litres 83. 630,000 centilitres 93. 4.07331 kilolitres 85. 450,000 centilitres 87. 1,060 centilitres 95. 14.968 litres 89. 143.83 litres 91. 990 centilitres

489

Version: Fall 2010

490

CHAPTER 6. RATIO AND PROPORTION

6.5

American Units to Metric Units and Vice-Versa

We often need to convert from the American system of units to the metric system of units or vice-versa (imagine traveling to a European country using the metric system). That will be our focus in this section.

Converting Units of Length


One meter is slightly longer than one yard. Indeed, 1 m 1.0936 yd. If we divide both sides of this equation by 1.0936, then 1m 1.0936 yd 1.0936 1.0936 0.9144 m 1 yd Further conversions can be made. For example, to change meters to feet, we make the following conversions. 1m 1m 3 ft 1 yd 0.9144m 1 yd 3 ft 1 yd 1& m 0.9144& 1 m yd 113 ft 0.9144 1 3.2808 ft Apply conversion factors. Cancel common units. Multiply fractions.

Table 6.5 shows some of the most common conversions between American units of length and metric units of length. American to Metric 1 in 2.54 cm 1 ft 0.3048 m 1 yd 0.9144 m 1 mi 1.6093 km Metric to American 1 cm 0.3937 in 1 m 3.2808 ft 1 m 1.0936 yd 1 km 0.6214 mi

Table 6.5: Length conversions: American Metric.

You Try It! Change 227 miles to kilometers. Round to the nearest tenth of a kilometer. EXAMPLE 1. A cars speedometer shows that a family has currently traveled 154 miles in route to their vacation destination. Convert this distance to kilometers. Version: Fall 2010

6.5. AMERICAN UNITS TO METRIC UNITS AND VICE-VERSA Solution. Choose 1 mi = 1.6093 km from Table 6.5. 154 mi 154 mi 1.6093 km 1 mi 1.6093 km & 154 & mi & 1& mi 154 1.6093 km 1 247.8 km Apply conversion factor. Cancel common units. Multiply fractions.

491

Hence, 154 miles is approximately 247.8 kilometers. Alternate Solution. Note that this would work equally well if we chose 1 km 0.6214 mi from Table 6.5. 154 mi 154 mi 1 km 0.6214 mi 1 km & 154 & mi & 0.6214 & mi 154 1 km 0.6214 247.8 km Apply conversion factor. Cancel common units. Multiply fractions.

Answer: 365.3 kilometers

Converting Units of Weight and Mass


It is known that 1 kg 2.2 lb. Dividing both sides of this equation by 2.2, 2.2 lb 1 kg 2.2 2.2 0.454 kg 1 lb A summary of the more common conversion factors regarding mass and weight are given in Table 6.6. American to Metric 1 oz 28.35 g 1 lb 0.454 kg Metric to American 1 g 0.035 oz 1 kg 2.2 lb

Table 6.6: MassWeight conversions: American Metric.

Version: Fall 2010

492

CHAPTER 6. RATIO AND PROPORTION

You Try It! Change 5.7 kilograms to ounces. Round to the nearest ounce. EXAMPLE 2. Change 2.3 kilograms to ounces. Solution. One kilogram weighs 2.2 pounds and there are 16 ounces in a pound. 2.3 kg 2.3 kg 2.2 lb 16 oz 1 kg 1 lb 2.2 16 oz lb 2.3 kg kg 1 lb 1 2.3 2.2 16 oz 11 80.96 oz Apply conversion factors. Cancel common units. Multiply fractions.

Hence, 2.3 kilograms weighs 80.96 ounces. Alternate Solution. Another approach uses the facts that 1 kilogram equals 1000 grams and 1 ounce equals 28.35 grams. 2.3 kg 2.3 kg 1000 g 1 oz 1 kg 28.35 g 1000 g 1 oz 2.3 kg 1 kg 28.35 g 2.3 1000 1 oz 1 28.35 81.13 oz Apply conversion factors. Cancel common units. Multiply fractions.

Roundo Error. Why the discrepancy in answers? This dierence in approximations is due to something called round-o error. Indeed, in the rst calculation, we used a conversion factor that is rounded to the nearest tenth; i.e., 1 kg 2.2 lb. If we use a more accurate conversion factor to change kilograms to pounds, namely 1 kg 2.2046 lb, we get the following result. 2.3 kg 2.3 kg 2.2046 lb 16 oz 1 kg 1 lb 2.2046 16 oz lb 2.3 kg 1 kg 1 lb 2.3 2.2046 16 oz 11 81.13 oz Apply conversion factors. Cancel common units. Multiply fractions.

Answer: 201 ounces

Note that this result is in better agreement with the second result above.

Version: Fall 2010

6.5. AMERICAN UNITS TO METRIC UNITS AND VICE-VERSA

493

Important Observation. To obtain better approximations, you need to use a conversion factor that is more accurate. Any time you feel you need more accuracy, you might try an online conversion utility, such as the one at: http://www.france-property-and-information.com/metric_ conversion_table.htm

Converting Units of Volume


It is known that 1 qt 0.946 L. Dividing both sides of this equation by 0.946, 1 L 1.06 qt. Again, these conversion factors for volume have been well worked out by scientists. The more common conversion factors for volume are shown in Table 6.7. American to Metric 1 oz 0.030 L 1 pt 0.473 L 1 qt 0.946 L 1 gal 3.785 L Metric to American 1 L 33.8 oz 1 L 2.1 pt 1 L 1.06 qt 1 L 0.264 gal

Table 6.7: Volume conversions: American Metric.

You Try It! EXAMPLE 3. Change 2.5 dekalitres to gallons. Solution. Recall that 1 daL = 10 L and 1 L = 0.264 gal. 2.5 daL 2.5 daL 10 L 0.264 gal 1 daL 1L 10  0.264 gal L 2.5 daL 1 daL 1 L 2.5 10 0.264 gal 11 6.6 gal Apply conversion factors. Cancel common units. Multiply fractions. Change 3.2 dekalitres to gallons.Round your answer to the nearest tenth of a gallon.

Hence, 2.5 dekalitres is approximately equal to 6.6 gallons.

Answer: 0.8 gallons

Version: Fall 2010

494

CHAPTER 6. RATIO AND PROPORTION

Converting Units of Speed


Modern speedometers often show a cars speed in both miles per hour and kilometers per hour. You Try It! A cars speedometer registers 45 kilometers per hour. Change this speed to miles per hour. Round your answer to the nearest mile per hour. EXAMPLE 4. A cars speedometer is showing it speeding along at 60 kilometers per hour. How fast is it traveling in miles per hour? Round your answer to the nearest mile per hour. Solution. From Table 6.5, 1 km 0.6214 mi. 60 km 0.6214 mi km 60 h h 1 km km 0.6214 mi 60 h 1 km 60 0.6214 mi 1 h mi 37.284 h Apply conversion factor. Cancel common units. Multiply fractions.

To round to the nearest mile per hour, identify the rounding and test digits. Test digit 3 7 . 2 84 Rounding digit Because the test digit is less than 5, leave the rounding digit alone and truncate. Thus, to the nearest mile per hour, the speed is approximately 37 miles per hour.

Answer: 28 miles per hour

Version: Fall 2010

6.5. AMERICAN UNITS TO METRIC UNITS AND VICE-VERSA

495

l l l

Exercises

l l l

1. Use the conversion 1 in = 2.54 cm to convert 68 inches to centimeters, rounded to the nearest tenth of a centimeter. 2. Use the conversion 1 in = 2.54 cm to convert 42 inches to centimeters, rounded to the nearest tenth of a centimeter. 3. Use the conversion 1 in = 2.54 cm to convert 44 centimeters to inches, rounded to the nearest tenth of a inch. 4. Use the conversion 1 in = 2.54 cm to convert 22 centimeters to inches, rounded to the nearest tenth of a inch. 5. Use the conversion 1 mi = 1.6093 km to convert 79 miles to kilometers, rounded to the nearest tenth of a kilometer. 6. Use the conversion 1 mi = 1.6093 km to convert 39 miles to kilometers, rounded to the nearest tenth of a kilometer. 7. Use the conversion 1 yd = 0.9144 m to convert 1489 centimeters to yards, rounded to the nearest tenth of a yard. 8. Use the conversion 1 yd = 0.9144 m to convert 1522 centimeters to yards, rounded to the nearest tenth of a yard. 9. Use the conversion 1 yd = 0.9144 m to convert 28 yards to centimeters, rounded to the nearest tenth of a centimeter. 10. Use the conversion 1 yd = 0.9144 m to convert 34 yards to centimeters, rounded to the nearest tenth of a centimeter. 11. Use the conversion 1 m = 3.2808 ft to convert 8.6 meters to inches, rounded to the nearest tenth of a inch. 12. Use the conversion 1 m = 3.2808 ft to convert 8.3 meters to inches, rounded to the nearest tenth of a inch.

13. Use the conversion 1 in = 2.54 cm to convert 60 inches to centimeters, rounded to the nearest tenth of a centimeter. 14. Use the conversion 1 in = 2.54 cm to convert 75 inches to centimeters, rounded to the nearest tenth of a centimeter. 15. Use the conversion 1 m = 3.2808 ft to convert 208 inches to meters, rounded to the nearest tenth of a inch. 16. Use the conversion 1 m = 3.2808 ft to convert 228 inches to meters, rounded to the nearest tenth of a inch. 17. Use the conversion 1 m = 1.0936 yd to convert 20 yards to meters, rounded to the nearest tenth of a meter. 18. Use the conversion 1 m = 1.0936 yd to convert 44 yards to meters, rounded to the nearest tenth of a meter. 19. Use the conversion 1 mi = 1.6093 km to convert 29 miles to kilometers, rounded to the nearest tenth of a kilometer. 20. Use the conversion 1 mi = 1.6093 km to convert 15 miles to kilometers, rounded to the nearest tenth of a kilometer. 21. Use the conversion 1 m = 1.0936 yd to convert 8.2 meters to yards, rounded to the nearest tenth of a yard. 22. Use the conversion 1 m = 1.0936 yd to convert 6.9 meters to yards, rounded to the nearest tenth of a yard. 23. Use the conversion 1 mi = 1.6093 km to convert 4.9 kilometers to miles, rounded to the nearest tenth of a mile. 24. Use the conversion 1 mi = 1.6093 km to convert 4.2 kilometers to miles, rounded to the nearest tenth of a mile. Version: Fall 2010

496

CHAPTER 6. RATIO AND PROPORTION 33. Use the conversion 1 m = 3.2808 ft to convert 3.7 meters to inches, rounded to the nearest tenth of a inch. 34. Use the conversion 1 m = 3.2808 ft to convert 7.9 meters to inches, rounded to the nearest tenth of a inch. 35. Use the conversion 1 yd = 0.9144 m to convert 1323 centimeters to yards, rounded to the nearest tenth of a yard. 36. Use the conversion 1 yd = 0.9144 m to convert 1715 centimeters to yards, rounded to the nearest tenth of a yard. 37. Use the conversion 1 m = 1.0936 yd to convert 8.4 meters to yards, rounded to the nearest tenth of a yard. 38. Use the conversion 1 m = 1.0936 yd to convert 7.3 meters to yards, rounded to the nearest tenth of a yard. 39. Use the conversion 1 m = 3.2808 ft to convert 289 inches to meters, rounded to the nearest tenth of a inch. 40. Use the conversion 1 m = 3.2808 ft to convert 251 inches to meters, rounded to the nearest tenth of a inch.

25. Use the conversion 1 m = 1.0936 yd to convert 25 yards to meters, rounded to the nearest tenth of a meter. 26. Use the conversion 1 m = 1.0936 yd to convert 2 yards to meters, rounded to the nearest tenth of a meter. 27. Use the conversion 1 in = 2.54 cm to convert 47 centimeters to inches, rounded to the nearest tenth of a inch. 28. Use the conversion 1 in = 2.54 cm to convert 19 centimeters to inches, rounded to the nearest tenth of a inch. 29. Use the conversion 1 mi = 1.6093 km to convert 8.3 kilometers to miles, rounded to the nearest tenth of a mile. 30. Use the conversion 1 mi = 1.6093 km to convert 4.8 kilometers to miles, rounded to the nearest tenth of a mile. 31. Use the conversion 1 yd = 0.9144 m to convert 41 yards to centimeters, rounded to the nearest tenth of a centimeter. 32. Use the conversion 1 yd = 0.9144 m to convert 20 yards to centimeters, rounded to the nearest tenth of a centimeter.

41. Use the conversion 1 kg = 2.2 lb to convert 15.8 kilograms to pounds, rounded to the nearest tenth of a pound. 42. Use the conversion 1 kg = 2.2 lb to convert 19.4 kilograms to pounds, rounded to the nearest tenth of a pound. 43. Use the conversion 1 oz = 28.35 g to convert 35 ounces to grams, rounded to the nearest tenth of a gram. 44. Use the conversion 1 oz = 28.35 g to convert 33 ounces to grams, rounded to the nearest tenth of a gram.

45. Use the conversion 1 kg = 2.2 lb to convert 2.48 kilograms to ounces, rounded to the nearest tenth of a ounce. 46. Use the conversion 1 kg = 2.2 lb to convert 3.74 kilograms to ounces, rounded to the nearest tenth of a ounce. 47. Use the conversion 1 kg = 2.2 lb to convert 2.35 kilograms to ounces, rounded to the nearest tenth of a ounce. 48. Use the conversion 1 kg = 2.2 lb to convert 3.57 kilograms to ounces, rounded to the nearest tenth of a ounce.

Version: Fall 2010

6.5. AMERICAN UNITS TO METRIC UNITS AND VICE-VERSA 49. Use the conversion 1 kg = 2.2 lb to convert 15 pounds to kilograms, rounded to the nearest tenth of a kilogram. 50. Use the conversion 1 kg = 2.2 lb to convert 27 pounds to kilograms, rounded to the nearest tenth of a kilogram. 51. Use the conversion 1 kg = 2.2 lb to convert 10.4 kilograms to pounds, rounded to the nearest tenth of a pound. 52. Use the conversion 1 kg = 2.2 lb to convert 17.7 kilograms to pounds, rounded to the nearest tenth of a pound. 53. Use the conversion 1 kg = 2.2 lb to convert 352 ounces to kilograms, rounded to the nearest tenth of a kilogram. 54. Use the conversion 1 kg = 2.2 lb to convert 326 ounces to kilograms, rounded to the nearest tenth of a kilogram. 55. Use the conversion 1 oz = 28.35 g to convert 96 grams to ounces, rounded to the nearest tenth of a ounce. 56. Use the conversion 1 oz = 28.35 g to convert 100 grams to ounces, rounded to the nearest tenth of a ounce.

497

57. Use the conversion 1 oz = 28.35 g to convert 14 ounces to grams, rounded to the nearest tenth of a gram. 58. Use the conversion 1 oz = 28.35 g to convert 29 ounces to grams, rounded to the nearest tenth of a gram. 59. Use the conversion 1 kg = 2.2 lb to convert 54 pounds to kilograms, rounded to the nearest tenth of a kilogram. 60. Use the conversion 1 kg = 2.2 lb to convert 82 pounds to kilograms, rounded to the nearest tenth of a kilogram. 61. Use the conversion 1 oz = 28.35 g to convert 92 grams to ounces, rounded to the nearest tenth of a ounce. 62. Use the conversion 1 oz = 28.35 g to convert 103 grams to ounces, rounded to the nearest tenth of a ounce. 63. Use the conversion 1 kg = 2.2 lb to convert 388 ounces to kilograms, rounded to the nearest tenth of a kilogram. 64. Use the conversion 1 kg = 2.2 lb to convert 395 ounces to kilograms, rounded to the nearest tenth of a kilogram.

65. Use the conversion 1 qt = 0.946 L to convert 55.1 litres to quarts, rounded to the nearest tenth of a quart. 66. Use the conversion 1 qt = 0.946 L to convert 50.3 litres to quarts, rounded to the nearest tenth of a quart. 67. Use the conversion 1 L = 33.8 oz to convert 72073 uid ounces to kilolitres, rounded to the nearest tenth of a kilolitre. 68. Use the conversion 1 L = 33.8 oz to convert 56279 uid ounces to kilolitres, rounded to the nearest tenth of a kilolitre. 69. Use the conversion 1 L = 33.8 oz to convert 2.5 kilolitres to uid ounces.

70. Use the conversion 1 L = 33.8 oz to convert 4.5 kilolitres to uid ounces. 71. Use the conversion 1 qt = 0.946 L to convert 24 quarts to litres, rounded to the nearest tenth of a litre. 72. Use the conversion 1 qt = 0.946 L to convert 21 quarts to litres, rounded to the nearest tenth of a litre. 73. Use the conversion 1 qt = 0.946 L to convert 30 quarts to litres, rounded to the nearest tenth of a litre. 74. Use the conversion 1 qt = 0.946 L to convert 22 quarts to litres, rounded to the nearest tenth of a litre. Version: Fall 2010

498

CHAPTER 6. RATIO AND PROPORTION 82. Use the conversion 1 qt = 0.946 L to convert 55.4 litres to quarts, rounded to the nearest tenth of a quart. 83. Use the conversion 1 gal = 3.785 L to convert 17.2 gallons to litres, rounded to the nearest tenth of a litre. 84. Use the conversion 1 gal = 3.785 L to convert 19.6 gallons to litres, rounded to the nearest tenth of a litre. 85. Use the conversion 1 L = 33.8 oz to convert 51274 uid ounces to kilolitres, rounded to the nearest tenth of a kilolitre. 86. Use the conversion 1 L = 33.8 oz to convert 82164 uid ounces to kilolitres, rounded to the nearest tenth of a kilolitre. 87. Use the conversion 1 gal = 3.785 L to convert 55.6 litres to gallons, rounded to the nearest tenth of a gallon. 88. Use the conversion 1 gal = 3.785 L to convert 59.2 litres to gallons, rounded to the nearest tenth of a gallon.

75. Use the conversion 1 gal = 3.785 L to convert 11.8 gallons to litres, rounded to the nearest tenth of a litre. 76. Use the conversion 1 gal = 3.785 L to convert 13.5 gallons to litres, rounded to the nearest tenth of a litre. 77. Use the conversion 1 gal = 3.785 L to convert 50.5 litres to gallons, rounded to the nearest tenth of a gallon. 78. Use the conversion 1 gal = 3.785 L to convert 55.9 litres to gallons, rounded to the nearest tenth of a gallon. 79. Use the conversion 1 L = 33.8 oz to convert 8.3 kilolitres to uid ounces. 80. Use the conversion 1 L = 33.8 oz to convert 5.3 kilolitres to uid ounces. 81. Use the conversion 1 qt = 0.946 L to convert 42.4 litres to quarts, rounded to the nearest tenth of a quart.

89. Change 60 miles per hour to kilometers per hour. Round your answer to the nearest kilometer per hour. 90. Change 56 miles per hour to kilometers per hour. Round your answer to the nearest kilometer per hour. 91. Change 77 miles per hour to kilometers per hour. Round your answer to the nearest kilometer per hour. 92. Change 57 miles per hour to kilometers per hour. Round your answer to the nearest kilometer per hour.

93. Change 42 kilometers per hour to miles per hour. Round your answer to the nearest mile per hour. 94. Change 56 kilometers per hour to miles per hour. Round your answer to the nearest mile per hour. 95. Change 62 kilometers per hour to miles per hour. Round your answer to the nearest mile per hour. 96. Change 63 kilometers per hour to miles per hour. Round your answer to the nearest mile per hour.

97. Tallest tower. The worlds tallest tower in Dubai has 160 oors at a height of 2, 717 feet. Convert the height of the tower to the nearest tenth of a meter. Associated Press-Times-Standard 02/09/10 Worlds tallest tower closed a month after opening. Version: Fall 2010

6.5. AMERICAN UNITS TO METRIC UNITS AND VICE-VERSA

499

98. High peaks. For the rst time, foreigners will be allowed to climb nearly 100 high-altitude Himalayan peaks on the Indian side of Kashmir, peaks ranging from 9, 840 feet to nearly 26, 246 feet. Convert the highest of the peaks to the nearest tenth of a meter. Associated Press-TimesStandard 04/11/10 India opens Himalayan peaks to foreigners. 99. Ancient nd. In the southern Egyptian town of Luxor, a 3, 400-year-old 4-meter statue of Thoth, the ancient Egyptian god of Wisdom and Magic, was unearthed. Convert the height of the statue to the nearest tenth of a foot. Associated Press-Times-Standard 03/17/10 3,400-year-old statues unearthed in Egypt. 100. Arctic wind. Blizzard condition winds in the Arctic blew 80 miles per hour. Find the wind speed to the nearest kilometer per hour. Associated Press-Times-Standard 12/31/09 After 562-mile ski trek, seven women reach the South Pole. 101. Solar plane. The Solar Impulse lifted o from a military airport at a speed no faster than 28 miles per hour. Convert the speed of the solar-powered plane to the nearest kilometer per hour. Associated Press-Times-Standard 04/09/10 Solar-powered plane makes successful maiden ight.

l l l
1. 172.7 centimeters 3. 17.3 inches 5. 127.1 kilometers 7. 16.3 yards

Answers

l l l

25. 22.9 meters 27. 18.5 inches 29. 5.2 miles 31. 3749.0 centimeters 33. 145.7 inches

9. 2560.3 centimeters 11. 338.6 inches 13. 152.4 centimeters 15. 5.3 meters 17. 18.3 meters

35. 14.5 yards 37. 9.2 yards 39. 7.3 meters 41. 34.8 pounds 43. 992.3 grams 45. 87.3 ounces

19. 46.7 kilometers 21. 9.0 yards 23. 3.0 miles

47. 82.7 ounces 49. 6.8 kilograms 51. 22.9 pounds Version: Fall 2010

500 53. 10.0 kilograms 55. 3.4 ounces 57. 396.9 grams 59. 24.5 kilograms 61. 3.2 ounces 63. 11.0 kilograms

CHAPTER 6. RATIO AND PROPORTION 79. 280540 ounces 81. 44.8 quarts 83. 65.1 litres 85. 1.5 kilolitres 87. 14.7 gallons 89. 97 km/hr

65. 58.2 quarts 91. 124 km/hr 67. 2.1 kilolitres 69. 84500 ounces 71. 22.7 litres 73. 28.4 litres 75. 44.7 litres 77. 13.3 gallons 93. 26 mi/hr 95. 39 mi/hr 97. 828.2 meters 99. 13.1 feet 101. 45 km/hr

Version: Fall 2010

Chapter

Percent
When one hears the word percent, other words come immediately to mind, words such as century, cents, or centimeters. A century equals 100 years. There are one hundred cents in a dollar and there are 100 centimeters in a meter. Thus, it should come as no surprise that percent means parts per hundred. In the world we live in we are constantly bombarded with phrases that contain the word percent. The sales tax in California is 8.25%. An employee is asking his boss for a 5% raise. A union has seen a 6.25% increase in union dues. The population of a town is increasing at a rate of 2.25% per year. In this chapter we introduce the concept of percent, rst addressing how to facilitate writing percents in fraction or decimal form and also performing the reverse operations, changing fractions and decimals to percents. Next we use our expertise in solving equations to solve the more common forms that involve percents, then we apply this ability to solving common applications from the real world that use percents. Well tackle applications of commission and sales tax, discount and marked price, percent increase or decrease, and simple interest. Lets begin the journey.

501

502

CHAPTER 7. PERCENT

7.1

Percent, Decimals, Fractions

In the square shown in Figure 7.1, a large square has been partitioned into ten rows of ten little squares in each row. I Figure 7.1, weve shaded 20 of 100 possible little squares, or 20% of the total number of little squares.

Figure 7.1: Shading 20 of 100 little squares, or 20% of the total number of little squares.

The Meaning of Percent. Percent means parts per hundred. In Figure 7.1, 80 out of a possible 100 squares are left unshaded. Thus, 80% of the little squares are unshaded. If instead we shaded 35 out of the 100 squares, then 35% of the little squares would be shaded. If we shaded all of the little squares, then 100% of the little squares would be shaded (100 out of 100). So, when you hear the word percent, think parts per hundred.

Changing a Percent to a Fraction


Based on the discussion above, it is fairly straightforward to change a percent to a fraction. Percent to Fraction. To change a percent to a fraction, drop the percent sign and put the number over 100.

Version: Fall 2010

7.1. PERCENT, DECIMALS, FRACTIONS

503

You Try It! EXAMPLE 1. Change 24% to a fraction. Solution. Drop the percent symbol and put 24 over 100. 24% = 24 100 6 = 25 Percent: Parts per hundred. Reduce. Answer: 9/25 Change 36% to a fraction reduced to lowest terms.

Hence, 24% = 6/25.

You Try It!


2 EXAMPLE 2. Change 14 7 % to a fraction. 2 Solution. Drop the percent symbol and put 14 7 over 100. 2 14 7 2 14 % = 7 100 100 7 = 100 100 1 = 7 100 1 100 = 7 100 1 = 7 2 Hence, 14 7 % = 1/7. 1 Change 11 9 % to a fraction reduced to lowest terms.

Percent: Parts per hundred.

Mixed to improper fraction. Invert and multiply. Cancel.

Answer: 1/9

You Try It! EXAMPLE 3. Change 28.4% to a fraction. Version: Fall 2010 Change 87.5% to a fraction reduced to lowest terms.

504

CHAPTER 7. PERCENT

Solution. Drop the percent symbol and put 28.4 over 100. 28.4% = = = = = Answer: 7/8 28.4 100 28.4 10 100 10 284 1000 71 4 250 4 71 250 Percent: Parts per hundred. Multiply numerator and denominator by 10. Multiplying by 10 moves decimal point one place right. Factor. Cancel common factor.

Changing a Percent to a Decimal


To change a percent to a decimal, we need only remember that percent means parts per hundred. You Try It! Change 2.4% to a decimal. EXAMPLE 4. Change 23.25% to a decimal. Solution. Drop the percent symbol and put 23.25 over 100. 23.25% = 23.25 100 = 0.23 25 Percent: Parts per hundred. Dividing by 100 moves decimal point 2 places left.

Answer: 0.024

Therefore, 23.25% = 0.2325.

This last example motivates the following simple rule. Changing a Percent to a Decimal. To change a percent to a decimal, drop the percent symbol and move the decimal point two places to the left.

You Try It!


3 Change 6 4 % to a decimal.

EXAMPLE 5. Change 5 1 % to a decimal. 2 Version: Fall 2010

7.1. PERCENT, DECIMALS, FRACTIONS

505

Solution. Note that 1/2 = 0.5, then move the decimal 2 places to the left. 1 5 % = 5.5% 2 = 0.05 5 = 0.055
1 Thus, 5 2 % = 0.055.

1/2 = 0.5. Drop % symbol. Move decimal point 2 places left.

Answer: 0.0675

Changing a Decimal to a Percent


Changing a decimal to a percent is the exact opposite of changing a percent to a decimal. In the latter case, we drop the percent symbol and move the decimal point 2 places to the left. The following rule does just the opposite. Changing a Decimal to a Percent. To change a decimal to a percent, move the decimal point two places to the right and add a percent symbol.

You Try It! EXAMPLE 6. Change 0.0725 to a percent. Solution. Move the decimal point two places to the right and add a percent symbol. 0.0725 = 0 07.25% = 7.25% Answer: 3.75% Change to 0.0375 to a percent.

You Try It! EXAMPLE 7. Change 1.025 to a percent. Solution. Move the decimal point two places to the right and add a percent symbol. 1.025 = 1 02.5% = 102.5% Answer: 52.5% Change 0.525 to a percent.

Version: Fall 2010

506

CHAPTER 7. PERCENT

Changing a Fraction to a Percent


One way to proceed is to rst change the fraction to a decimal, then change the resulting decimal to a percent. Fractions to Percents: Technique #1. To change a fraction to a percent, follow these steps: 1. Divide numerator by the denominator to change the fraction to a decimal. 2. Move the decimal point in the result two places to the right and append a percent symbol.

You Try It! Change 5/16 to a percent. EXAMPLE 8. Use Technique #1 to change 5/8 to a percent. Solution. Change 5/8 to a decimal, then change the decimal to a percent. To change 5/8 to a decimal, divide 5 by 8. Since the denominator is a product of twos, the decimal should terminate. 0.625 8)5.000 48 20 16 40 40 0 To change 0.625 to a percent, move the decimal point 2 places to the right and append a percent symbol. 0.625 = 0 62.5% = 62.5%

Answer: 31.35%

A second technique is to create an equivalent fraction with a denominator of 100. Fractions to Percents: Technique #2. To change a fraction to a percent, create an equivalent fraction with a denominator of 100.

You Try It! Change 4/9 to a percent. EXAMPLE 9. Use Technique #2 to change 5/8 to a percent. Solution. Create an equivalent fraction for 5/8 with a denominator of 100. Version: Fall 2010 5 x = 8 100

7.1. PERCENT, DECIMALS, FRACTIONS Solve this proportion for x. 8x = 500 8x 500 = 8 8 125 x= 2 x = 62.5 Thus, 5 62.5 = = 62.5%. 8 100 Cross multiply. Divide both sides by 8. Reduce: Divide numerator and denominator by 4. Divide.

507

Alternate Ending. We could also change 125/2 to a mixed fraction; i.e., 1 125/2 = 62 2 . Then, 1 62 2 5 1 = = 62 %. 8 100 2 Same answer.

4 Answer: 44 9 %

Sometimes we will be content with an approximation. You Try It! EXAMPLE 10. Change 4/13 to a percent. Round your answer to the nearest tenth of a percent. Solution. We will use Technique #1. To change 4/13 to a decimal, divide 4 by 13. Since the denominator has factors other than 2s and 5s, the decimal will repeat. However, we intend to round to the nearest tenth of a percent, so we will carry the division to four decimal places only. (Four places are necessary because we will be moving the decimal point two places to the right.) 0.3076 13)4.0000 39 100 91 90 78 12 To change the decimal to a percent, move the decimal point two places to the right. 0.3076 0 30.76% 30.76% To round to the nearest tenth of a percent, identify the rounding and test digits. Test digit 30. 7 6 % Rounding digit Because the test digit is greater than or equal to 5, add 1 to the rounding digit and truncate. Thus, 0.03076 30.8%. Version: Fall 2010 Answer: 23.5% Change 4/17 to a percent. Round your answer to the nearest tenth of a percent.

508

CHAPTER 7. PERCENT

l l l

Exercises

l l l

In Exercises 1-18, convert the given percent to a fraction, and simplify the result.
7 1. 4 10 % 5 10. 8 8 %

2. 7 1 % 4 3. 7 2 % 9 4.
9 4 10 %

11. 192% 12. 5% 13. 86% 14. 177% 15. 130% 16. 80% 17. 4.07% 18. 6.5%

5. 11.76% 6. 15.2% 7. 13.99% 8. 18.66% 9. 4 1 % 2

In Exercises 19-34, convert the given percent to a decimal. 19. 124% 20. 4% 21. 0.6379% 22. 0.21% 23. 28% 24. 5.4% 25. 0.83% 26. 0.3344% 27. 8% 28. 3% 29. 59.84% 30. 0.17% 31. 155% 32. 7% 33. 36.5% 34. 39.7%

In Exercises 35-50, convert the given decimal to a percent. 35. 8.888 36. 5.1 37. 0.85 38. 0.08 39. 1.681 Version: Fall 2010 40. 3.372 41. 0.14 42. 4.89 43. 8.7 44. 8.78

7.1. PERCENT, DECIMALS, FRACTIONS 45. 0.38 46. 1.67 47. 0.02 48. 0.07 49. 0.044 50. 0.29

509

In Exercises 51-68, convert the given fraction to a percent. 51. 52. 53. 54. 55. 56. 57. 58. 59. 1 2 29 8 5 2 4 5 8 5 7 20 14 5 3 2 9 2 60. 61. 62. 63. 64. 65. 66. 67. 68. 18 25 9 4 7 8 7 5 4 25 6 5 23 8 12 5 13 2

69. Convert 24/29 to a percent, and round your answer to the nearest hundredth of a percent. 70. Convert 5/3 to a percent, and round your answer to the nearest hundredth of a percent. 71. Convert 15/7 to a percent, and round your answer to the nearest tenth of a percent. 72. Convert 10/7 to a percent, and round your answer to the nearest tenth of a percent. 73. Convert 7/24 to a percent, and round your answer to the nearest hundredth of a percent.

74. Convert 5/6 to a percent, and round your answer to the nearest hundredth of a percent. 75. Convert 8/3 to a percent, and round your answer to the nearest tenth of a percent. 76. Convert 22/21 to a percent, and round your answer to the nearest tenth of a percent. 77. Convert 9/23 to a percent, and round your answer to the nearest tenth of a percent. 78. Convert 11/9 to a percent, and round your answer to the nearest tenth of a percent. Version: Fall 2010

510 79. Convert 17/27 to a percent, and round your answer to the nearest hundredth of a percent.

CHAPTER 7. PERCENT 80. Convert 22/27 to a percent, and round your answer to the nearest hundredth of a percent.

81. Crime rates. Preliminary crime rates for the rst six months of 2009 compared to the same period in 2008 are shown below. Associated Press-Times-Standard 12/22/09 Despite recession, the national crime rates keep falling. Murder 10.0% Forcible rape 3.3% Robbery 6.5% Aggravated assault 3.2% Burglary 2.5% Larceny-theft 5.3% Motor vehicle theft 18.75% Arson 8.2% Source: Federal Bureau of Investigation i) What do the negative signs indicate? ii) Which type of crime decreased the most? iii) Which type of crime decreased the least? 82. Major Hurricanes. 5 of the 8 hurricanes in 2008 were categorized as major. Write the fractional number of major hurricanes in 2008 as a percent. NOAA Associated Press 5/22/09 83. Chance of ood. These excerpts are from the story Corps: Dam work lessens Seattle-area ood chance published in the Times-Standard on Nov. 6, 2009. Write all four of the odds of ooding as a percent chance. Round to the nearest tenth of a percent if necessary. i) Col. Anthony Wright, from the U.S. Army Corps of Engineers, speaking of the repairs to the Green River Dam, reported there was now a 1-in-25 chance that a storm would force the corps to release enough water from the dams reservoir to cause a ood downstream in the Green River Valley. ii) The odds of widespread ooding in the valley improve to 1-in-32 when all the sandbagging and ood-protection eorts are factored in. iii) Previously, the Corps of Engineers said the chance of widespread ooding was 1-in-4. iv) When the dam operates at capacity, there is a 1-in-140 chance of ooding.

l l l
1. 47 1000

Answers
3.

l l l
13 180

Version: Fall 2010

7.1. PERCENT, DECIMALS, FRACTIONS 5. 7. 147 1250 1399 10000 49. 4.4% 51. 50% 53. 250% 55. 160% 57. 280% 59. 450% 61. 225% 63. 140% 65. 120% 67. 240% 69. 82.76% 71. 214.3% 73. 29.17% 75. 266.7% 77. 39.1% 79. 62.96% 81.

511

9 9. 200 11. 13. 15. 17. 48 25 43 50 13 10 407 10000

19. 1.24 21. 0.006379 23. 0.28 25. 0.0083 27. 0.08 29. 0.5984 31. 1.55 33. 0.365 35. 888.8% 37. 85% 39. 168.1% 41. 14% 43. 870% 45. 38% 47. 2%

i) The negative signs indicate the crime rate has decreased from previous measures. ii) Motor vehicle theft decreased the most with an 18.75% decrease. iii) Burglary decreased the least with a 2.5% decrease.

83.

i) 4% chance of ood ii) 3.1% chance of ood iii) 25% chance of ood iv) 0.7% chance of ood Version: Fall 2010

512

CHAPTER 7. PERCENT

7.2

Solving Basic Percent Problems

There are three basic types of percent problems: 1. Find a given percent of a given number. For example, nd 25% of 640. 2. Find a percent given two numbers. For example, 15 is what percent of 50? 3. Find a number that is a given percent of another number. For example, 10% of what number is 12? Lets begin with the rst of these types.

Find a Given Percent of a Given Number


Lets begin with our rst example. You Try It! What number is 36% of 120? EXAMPLE 1. What number is 25% of 640? Solution. Let x represent the unknown number. Translate the words into an equation. What number x Now, solve the equation for x. x = 25% 640 x = 0.25 640 x = 160 Thus, 25% of 640 is 160. Alternate Solution. We could also change 25% to a fraction. x = 25% 640 1 x = 640 4 640 x= 4 x = 160 Answer: 43.2 Same answer. Original equation. Change 25% to a fraction: 25% = 25/100 = 1/4. Multiply numerators and denominators. Divide: 640/4 = 160. Original equation. Change 25% to a decimal: 25% = 0.25. Multiply: 0.25 640 = 160. is = 25% 25% of 640 640

Version: Fall 2010

7.2. SOLVING BASIC PERCENT PROBLEMS

513

You Try It! EXAMPLE 2. What number is 8 1 % of 120? 3 Solution. Let x represent the unknown number. Translate the words into an equation. What number x is = 81% 3 81% 3 of 120 120 What number is 4 1 % of 6 1,200?

Now, solve the equation for x. Because 1 8 % = 8.3% = 0.083, 3 working with decimals requires that we work with a repeating decimal. To do so, we would have to truncate the decimal representation of the percent at some place and satisfy ourselves with an approximate answer. Instead, lets change the percent to a fraction and seek an exact answer. 81 1 8 %= 3 3 100 =
25 3

Percent: Parts per hundred. Mixed to improper fraction. Invert and multiply. Multiply numerators and denominators. Reduce: Divide numerator and denominator by 25.

100 25 1 = 3 100 25 = 300 1 = 12

Now we can solve our equation for x. 1 x = 8 % 120 3 1 x= 120 12 120 x= 12 x = 10


1 Thus, 8 3 % of 120 is 10.

Original equation. 1 8 % = 1/12. 3 Multiply numerators and denominators. Divide: 120/12=10. Answer: 50

You Try It!


1 EXAMPLE 3. What number is 105 4 % of 18.2? 3 What number is 105 4 % of 222?

Version: Fall 2010

514

CHAPTER 7. PERCENT

Solution. Let x represent the unknown number. Translate the words into an equation. What number x is =
1 105 4 % 1 105 4 %

of

18.2 18.2

In this case, the fraction terminates as 1/4 = 0.25, so 1 105 % = 105.25% = 1.0525. 4 Now we can solve our equation for x. 1 x = 105 % 18.2 4 x = 1.0525 18.2 x = 19.1555 Answer: 234.765
1 Thus, 105 4 % of 18.2 is 19.1555.

Original equation. 1 5 % = 1.0525. 4 Multiply.

Find a Percent Given Two Numbers


Now well address our second item on the list at the beginning of the section. You Try It! 14 is what percent of 25? EXAMPLE 4. 15 is what percent of 50? Solution. Let x represent the unknown percent. Translate the words into an equation. 15 15 is = what percent x of 50 50

The commutative property of multiplication allows us to change the order of multiplication on the right-hand side of this equation. 15 = 50x. Now we can solve our equation for x. 15 = 50x 50x 15 = 50 50 15 =x 50 x = 0.30 Version: Fall 2010 Original equation. Divide both sides by 50. Simplify right-hand side. Divide: 15/50 = 0.30.

7.2. SOLVING BASIC PERCENT PROBLEMS

515

But we must express our answer as a percent. To do this, move the decimal two places to the right and append a percent symbol. 0.30 = 0 30.% = 30% Thus, 15 is 30% of 50. Alternative Conversion. At the third step of the equation solution, we had x= 15 . 50

We can convert this to an equivalent fraction with a denominator of 100. x= Thus, 15/50 = 30/100 = 30%. 30 15 2 = 50 2 100 Answer: 56%

You Try It! EXAMPLE 5. 10 is what percent of 80? Solution. Let x represent the unknown percent. Translate the words into an equation. 10 10 is = what percent x of 80 80 10 is what percent of 200?

The commutative property of multiplication allows us to write the right-hand side as 10 = 80x. Now we can solve our equation for x. 10 = 80x 80x 10 = 80 80 1 =x 8 0.125 = x Original equation. Divide both sides by 80. Reduce: 10/80 = 1/8. Divide: 1/8 = 0.125.

But we must express our answer as a percent. To do this, move the decimal two places to the right and append a percent symbol. 0.125 = 0 12.5% = 12.5% Thus, 10 is 12.5% of 80. Version: Fall 2010

516

CHAPTER 7. PERCENT

Alternative Conversion. At the third step of the equation solution, we had x= 1 . 8

We can convert this to an equivalent fraction with a denominator of 100 by setting up the proportion 1 n = 8 100 Cross multiply and solve for n. 8n = 100 8n 100 = 8 8 25 n= 2 1 n = 12 2 Hence, Cross multiply. Divide both sides by 8. Reduce: Divide numerator and denominator by 4. Change 25/2 to mixed fraction.
1 12 2 1 1 = = 12 %. 8 100 2

Answer: 5%

Same answer.

Find a Number that is a Given Percent of Another Number


Lets address the third item on the list at the beginning of the section. You Try It! 20% of what number is 45? EXAMPLE 6. 10% of what number is 12? Solution. Let x represent the unknown number. Translate the words into an equation. 10% 10% of what number x is = 12 12

Change 10% to a fraction: 10% = 10/100 = 1/10. 1 x = 12 10 Now we can solve our equation for x. 10 1 x 10 = 10(12) Multiply both sides by 10. Simplify.

x = 120 Version: Fall 2010 Thus, 10% of 120 is 12.

7.2. SOLVING BASIC PERCENT PROBLEMS

517

Alternative Solution. We can also change 10% to a decimal: 10% = 0.10. Then our equation becomes 0.10x = 12 Now we can divide both sides of the equation by 0.10. 0.10x 12 = 0.10 0.10 x = 120 Same answer. Divide both sides by 0.10. Divide: 12/0.10 = 120. Answer: 225

You Try It!


1 EXAMPLE 7. 11 9 % of what number is 20? 2 12 3 % of what number is 760?

Solution. Let x represent the unknown number. Translate the words into an equation.
1 11 9 % 1 11 9 % 1 Change 11 9 % to a fraction. 1 11 9 1 11 % = 9 100

of

what number x

is =

20 20

Percent: Parts per hundred. Mixed to improper: 11 Invert and multiply. Cancel. Simplify. 1 = 100/9. 9

100 100 1 = 9 100 1 100 = 9 100 1 = 9

100 9

1 Replace 11 9 % with 1/9 in the equation and solve for x.

1 x = 20 9 9 1 x 9 = 9(20) x = 180
1 Thus, 11 9 % of 180 is 20.

1 11 % = 1/9. 9 Multiply both sides by 9.

Answer: 6,000

Version: Fall 2010

518

CHAPTER 7. PERCENT

l l l
1. What number is 22.4% of 125? 2. What number is 159.2% of 125? 3. 60% of what number is 90? 4. 25% of what number is 40? 5. 200% of what number is 132? 6. 200% of what number is 208? 7. 162.5% of what number is 195? 8. 187.5% of what number is 90? 9. 126.4% of what number is 158? 10. 132.5% of what number is 159? 11. 27 is what percent of 45? 12. 9 is what percent of 50? 13. 37.5% of what number is 57? 14. 162.5% of what number is 286? 15. What number is 85% of 100? 16. What number is 10% of 70? 17. What number is 200% of 15? 18. What number is 50% of 84? 19. 50% of what number is 58? 20. 132% of what number is 198? 21. 5.6 is what percent of 40? 22. 7.7 is what percent of 35? 23. What number is 18.4% of 125? 24. What number is 11.2% of 125? 25. 30.8 is what percent of 40? 26. 6.3 is what percent of 15? 27. 7.2 is what percent of 16? 28. 55.8 is what percent of 60? 29. What number is 89.6% of 125? 30. What number is 86.4% of 125? Version: Fall 2010

Exercises

l l l

31. 60 is what percent of 80? 32. 16 is what percent of 8? 33. What number is 200% of 11? 34. What number is 150% of 66? 35. 27 is what percent of 18? 36. 9 is what percent of 15?
1 37. 133 3 % of what number is 80? 2 38. 121 3 % of what number is 73? 1 39. What number is 54 3 % of 6? 2 40. What number is 82 5 % of 5? 1 41. What number is 62 2 % of 32? 3 42. What number is 118 4 % of 32? 1 43. 77 7 % of what number is 27? 2 44. 82 3 % of what number is 62? 6 45. What number is 142 7 % of 77? 2 46. What number is 116 3 % of 84? 1 47. 143 2 % of what number is 5.74? 1 48. 77 2 % of what number is 6.2? 2 49. 141 3 % of what number is 68? 1 50. 108 3 % of what number is 78? 2 51. What number is 66 3 % of 96? 1 52. What number is 79 6 % of 48? 1 53. 59 2 % of what number is 2.38? 1 54. 140 5 % of what number is 35.05? 1 55. 78 2 % of what number is 7.85? 1 56. 73 2 % of what number is 4.41? 2 57. What number is 56 3 % of 51? 1 58. What number is 64 2 % of 4? 1 59. What number is 87 2 % of 70? 1 60. What number is 146 4 % of 4?

7.2. SOLVING BASIC PERCENT PROBLEMS

519

61. It was reported that 80% of the retail price of milk was for packaging and distribution. The remaining 20% was paid to the dairy farmer. If a gallon of milk cost $3.80, how much of the retail price did the farmer receive? 62. At $1.689 per gallon of gas the cost is distributed as follows: Crude oil supplies $0.95 Oil Companies $0.23 State and City taxes $0.23 Federal tax $0.19 Service Station $0.10 Data is from Money, March 2009 p. 22, based on U. S. averages in December 2008. Answer the following questions rounded to the nearest whole percent. a) What % of the cost is paid for crude oil supplies? b) What % of the cost is paid to the service station?

l l l
1. 28 3. 150 5. 66 7. 120 9. 125 11. 60

Answers
25. 77 27. 45

l l l

29. 112 31. 75 33. 22 35. 150 37. 60

13. 152 39. 3.26 15. 85 17. 30 19. 116 21. 14 23. 23 41. 20 43. 35 45. 110 47. 4 49. 48 Version: Fall 2010

520 51. 64 53. 4 55. 10

CHAPTER 7. PERCENT 57. 28.9 59. 61.25 61. $0.76

Version: Fall 2010

7.3. GENERAL APPLICATIONS OF PERCENT

521

7.3

General Applications of Percent

In this section we will look at an assortment of practical problems involving percent. You Try It! EXAMPLE 1. Myrna notes that 20% of her class is absent.If the class has 45 students, how many students are absent? Solution. Let n represent the number of students that are absent. Then we can translate the problem statement into words and symbols. Number absent n Because 20% = 0.20, n = 0.20 45 n=9 Therefore, 9 students are absent. 20% = 0.20 Multiply: 0.20 45 = 9. Answer: 12 is = 20% 20% of total number of students in the class 45 Aaron notes that 15% of his class is absent. If the class has 80 students, how many students are absent?

You Try It! EXAMPLE 2. Misty answered 90% of the questions on her mathematics examination correctly. If Misty had 27 correct answers, how many questions were on the exam? Solution. Let N represent the number of questions on the examination. Number of correct answers 27 is = 90% 90% of total number of questions N Erin asnwered 85% of the questions on her english examination correctly. If she had 34 correct answers, how many questions were on her exam?

Because 90% = 0.90, this last equation can be written as 27 = 0.90N. Solve for N . 0.90N 27 = 0.90 0.90 30 = N Divide both sides by 0.90. Divide: 27/0.90 = 30. Answer: 40 Version: Fall 2010

Hence, there were 30 questions on the examination.

522

CHAPTER 7. PERCENT

You Try It! Alphonso answered 19 of 25 questions on his biology test correctly. What percent of the questions did he mark correctly? EXAMPLE 3. Misty answered 30 of 40 possible questions on her sociology examination correctly. What percent of the total number of questions did Misty mark correctly? Solution. Let p represent the percent of the total number of questions marked correctly. Then we can translate the problem statement into words and symbols. Number of correct answers 30 is = what percent p of total number of questions 40

Because multiplication is commutative, we can write the last equation in the form 30 = 40p. Solve for p. 40p 30 = 40 40 3 =p 4 Divide both sides by 40. Reduce: 30/40 = 3/4.

We need to change p = 3/4 to a percent. There are two ways to do this: We can divide 3 by 4 to get p= 3 4 = 0.75 = 75%

Divide: 3/4 = 0.75. Move decimal point 2 places right.

We can create an equivalent fraction with a denominator of 100; i.e., p= 3 4 3 25 = 4 25 75 = 100 = 75%.

Multiply numerator and denominator by 25. Simplify numerator and denominator. Percent means parts per hundred.

Answer: 76%

Either way, Misty got 75% of the questions on her sociology examination correct.

Version: Fall 2010

7.3. GENERAL APPLICATIONS OF PERCENT

523

You Try It! EXAMPLE 4. 35 millilitres of a 60 millilitre solution is hydrochloric acid. What percent of the solution is hydrochloric acid? Solution. Let p represent the percent of the percent of the solution that is hydrochloric acid. Then we can translate the problem statement into words and symbols. Amount of hydrochloric acid 35 is = what percent p of the total amount of solution 60 25 millilitres of a 40 millilitre solution is sulfuric acid. What percent of the solution is sulfuric acid?

Because multiplication is commutative, we can write the right-hand side of the last equation as follows. 35 = 60p Now we can solve for p. 35 60p = 60 60 7 =p 12 Divide both sides by 60. Reduce: Divide numerator and denominator by 5.

Now we must change p to a percent. We can do this exactly by creating an equivalent fraction with a denominator of 100. n 7 = 12 100 Solve for n. 12n = 700 700 12n = 12 12 175 n= 3 1 n = 58 3 Hence, Cross multiply. Divide both sides by 12. Reduce: Divide numerator and denominator by 4. Change improper to mixed fraction.

1 58 3 7 1 = = 58 %. 12 100 3 1 Thus, 58 3 % of the solution is hydrochloric acid.

p=

Approximate Solution. If all that is needed is an approximate answer, say correct to the nearest tenth of a percent, then we would take a dierent approach starting with the line from above that has 35 = p. 60 Version: Fall 2010

524 We would divide 35 by 60 to get p 0.5833.

CHAPTER 7. PERCENT

Move the decimal two places to the right and append a percent symbol. p 0.5833 0 58.33% 58.33%. Round to the nearest tenth of a percent. Test digit p 58. 3 3 % Rounding digit Because the test digit is less than 5, leave the rounding digit alone and truncate. Thus, correct to the nearest tenth of a percent, p 58.3%. Answer: 62.5%
1 Note that p 58.3% is approximate, but p = 58 3 % is exact.

Version: Fall 2010

7.3. GENERAL APPLICATIONS OF PERCENT

525

l l l

Exercises

l l l

1. 31 millilitres of a 250 millilitre solution is sulphuric acid. What percent of the solution is sulphuric acid? Round your answer to the nearest tenth of a percent. 2. 34 millilitres of a 211 millilitre solution is phosphoric acid. What percent of the solution is phosphoric acid? Round your answer to the nearest tenth of a percent. 3. A family has completed 186 miles of a planned 346 mile trip. Find the percentage of the planned trip already traveled. Round your answer to the nearest percent. 4. A family has completed 153 miles of a planned 431 mile trip. Find the percentage of the planned trip already traveled. Round your answer to the nearest percent. 5. Erin takes roll in her fth grade class and nds that 19 out of 34 total students on her roster are present. Find the percentage of the class that is present, correct to the nearest percent. 6. Barbara takes roll in her fth grade class and nds that 15 out of 38 total students on her roster are present. Find the percentage of the class that is present, correct to the nearest percent. 7. Raven answered 135 of 150 possible questions on the meteorology examination correctly. What percent of the total number of questions did Raven mark correctly? 8. Liz answered 30 of 50 possible questions on the algebra examination correctly. What percent of the total number of questions did Liz mark correctly?

9. A family has traveled 114 miles of a planned trip. This is 37% of the total distance they must travel on the trip. Find, correct to the nearest mile, the total distance they will travel on their trip. 10. A family has traveled 102 miles of a planned trip. This is 23% of the total distance they must travel on the trip. Find, correct to the nearest mile, the total distance they will travel on their trip. 11. Trudy takes roll in her class at the university and nds that 65 students are present. If this is 50% of the total class enrollment, how many students are in the class? 12. Sandra takes roll in her class at the university and nds that 104 students are present. If this is 80% of the total class enrollment, how many students are in the class? 13. Bill earns a commission on all sales he makes. He sells a bed for $591 and earns a commission of $43. Find the percent commission, rounded to the nearest tenth of a percent. 14. Ira earns a commission on all sales he makes. He sells a sofa for $408 and earns a commission of $39. Find the percent commission, rounded to the nearest tenth of a percent. 15. Tami answered 70% of the questions on the physics examination correctly. If Tami had 98 correct answers, how many questions were on the exam? 16. Trinity answered 90% of the questions on the chemistry examination correctly. If Trinity had 99 correct answers, how many questions were on the exam? Version: Fall 2010

526 17. A state charges 8% sales tax on all sales. If the sales tax on a computer is $20, nd the sales price of the computer, correct to the nearest dollar. 18. A state charges 6.5% sales tax on all sales. If the sales tax on a bed is $33, nd the sales price of the bed, correct to the nearest dollar. 19. Kenon earns 6% commission all his sales. If the sale of a computer earns him a $37 commission, nd the sales price of the computer, correct to the nearest dollar. 20. Donald earns 4.5% commission all his sales. If the sale of a dryer earns him a $24 commission, nd the sales price of the dryer, correct to the nearest dollar.

CHAPTER 7. PERCENT 21. A 23% nitric acid solution contains 59 millilitres of nitric acid. How many total millilitres of solution are present? Round your answer to the nearest millilitre. 22. A 27% sulphuric acid solution contains 67 millilitres of sulphuric acid. How many total millilitres of solution are present? Round your answer to the nearest millilitre. 23. In a state, a television sold for $428 is assessed a sales tax of $45. Find the sales tax rate, rounded to the nearest tenth of a percent. 24. In a state, a refrigerator sold for $503 is assessed a sales tax of $44. Find the sales tax rate, rounded to the nearest tenth of a percent.

25. Mars gravity. The force of gravity on Mars is only 38% of the force of gravity on earth. If you weigh 150 pounds on earth, how much will you weigh on Mars? 26. Wiretaps. In 2008, there were a total of 1,891 applications to federal and state judges to authorize the interception of wire, oral, or electronic communications. If 94% of all wiretap applications were for a portable device such as a cell phone or pager, how many applications were made to tap mobile devices? Round-o to the nearest application. Associated Press Times-Standard 4/28/09 27. Seniors. 13% of Humboldt Countys population is age 65 and older, about 2% more than the states average. If the population of Humboldt County is approximately 130,000, how many people in Humboldt County are age 65 and older? Times-Standard 6/10/2009 28. Antibiotics. The U.S. used about 35 million pounds of antibiotics last year. 70 percent of the drugs went to pigs, chickens, and cows. How many million pounds of antibiotics went to the pigs, chickens, and cows? Associated Press-Times-Standard 12/29/09 Pressure rises to stop antibiotics in agriculture. 29. Grow faster. Approximately 28 million pounds of antibiotics were fed to farm animals in the US during 2008. Thirteen percent of that was fed to healthy animals to make them grow faster. How many pounds of antibiotics were fed to healthy animals? Associated Press-Times-Standard 12/29/09 Pressure rises to stop antibiotics in agriculture. 30. CO2 emissions. The accord agreed to by the US at the Copenhagen climate talks had greenhouse gas emissions held to 3.5% of 1990 levels. If 1990 levels were 5022 MMT (millions of metric tons), how many millions of metric tons might greenhouse emissions be held to? Round the result to the nearest MMT. Associated Press-Times-Standard 12/19/09 Elements of new Copenhagen accord. Version: Fall 2010

7.3. GENERAL APPLICATIONS OF PERCENT

527

31. Water supply. A new water desalination plant, the largest in the Western hemisphere, could come online by 2012 in Carlsbad, California, providing 50 million gallons of drinking water per day, or 10% of the supply for San Diego County. What is the total amount of drinking water supplied to San Diego County daily? Associated Press-Times-Standard 32. Earthquake damage. After the recent earthquake in Chile, an estimated 33 million gallons of Chilean wine, or 13% of annual production, was lost. Estimate the total annual production of Chilean wine rounded to the nearest millions of gallons. Associated Press-Times Standard 03/24/10 Hemorrhaging cabernet: Earthquake hits winemakers in Chile. 33. Snowpack. At a meadow near Echo Summit in the northern Sierra Nevada, water ocials measured the snow at 65.7 inches. The water content was 25.9 inches, which is 92% of the average for this time of year. Determine the average water content for this time of year rounded to the nearest tenth of an inch. Associated Press-Times Standard 04/02/10 Californias Sierra snowpack slightly above normal. 34. Storefronts. According to the Times-Standard, as of April 2008 the Bayshore Mall had 55 occupied storefronts and 17 vacant storefronts. What percent of total storefronts are vacant? Round your answer to the nearest whole number. Times-Standard 4/19/09 35. Recovered. In Humboldt County, California, 427 of the 499 vehicles stolen between August 2008 and August 2009 were recovered. What percent of the stolen vehicles were recovered? Round your result to the nearest tenth of a percent. Times-Standard CHP oers tips on avoiding vehicle theft. 36. Freshman admissions. Stanford University sent acceptance letters to 2, 300 of 32, 022 freshman applicants. What percent of freshman applicants got acceptance letters, rounded to the nearest percent? Associated Press-Times-Standard 03/30/10 Stanford U. reports record-low admission rate. 37. Reduce. Each year, Americans throw out an average of about 1, 600 pounds of waste per person. Arcata, CA resident Michael Winkler only uses one trash bag every year totaling at most 40 pounds. Find the percent of average annual waste per person Mr. Winkler throws out to a tenth of a percent. Times-Standard Allison White 12/26/09 Waste not... 38. Population decrease. The table below shows the population of Detroit, Michigan. Associated Press-Times-Standard 03/09/10 Detroit wants to save itself by shrinking. Year Population 1950 1,849,568 1990 1,027,974 2005 890,963 What is the population of Detroit in 2005 as a percent of the population in 1950? Round your result to the nearest percent.

Version: Fall 2010

528

CHAPTER 7. PERCENT

l l l
1. 12.4 3. 54 5. 56 7. 90 9. 308 mi 11. 130 students 13. 7.3 15. 140 17. $250 19. $617

Answers

l l l

21. 257 ml 23. 10.5 25. 57 pounds 27. 16,900 29. 3.84 million pounds 31. 500 million gallons 33. The average water content is 28.2 inches. 35. 85.6% of the stolen vehicles were recovered. 37. Mr. Winkler throws out 2.5% of the average Americans waste.

Version: Fall 2010

7.4. PERCENT INCREASE OR DECREASE

529

7.4

Percent Increase or Decrease

A persons salary can increase by a percentage. A towns population can decrease by a percentage. A clothing rm can discount its apparel. These are the types of applications we will investigate in this section.

Percent Increase
You Try It! EXAMPLE 1. A salesperson is granted a 5% salary increase. If the salespersons current salary is $4,000 per month, what will be his new salary? Solution. Let x represent the salespersons salary increase. Then we can translate the problem into words and symbols. Salary increase x Solve for x. x = 0.05 4000 x = 200 5% = 0.05. Multiply: 0.05 4000 = 200. is = 5% 5% of original salary 4000 A computer technician is granted a 4% salary increase. If the salespersons current salary is $2,800 per month, what will be his new salary?

Therefore, the salary increase is $200. To compute the new salary N , we must add this increase to the original salary. New salary N is = original salary 4000 plus + increase 200

Thus, the new salary is N = $4, 200 per month. Alternative Solution. If the salesperson is to receive a 5% increase in his salary, then his new salary will be 105% of his original salary. Let N represent his new monthly salary. Then, New salary N Solve for N . N = 1.05 4000 N = 4200 Same answer. Version: Fall 2010 105% = 1.05. Multiply: 1.05 4000 = 4200. Answer: $2,912 is = 105% 105% of original salary 4000

530

CHAPTER 7. PERCENT

You Try It! A statistician making a salary of $3,200 per month has his salary increased to $3,368 per month. What is the percent increase? EXAMPLE 2. A salesperson making a salary of $4,500 per month has his salary increased to $5,000 per month. What is the percent increase? Solution. To nd the increase in salary, rst subtract the original salary from the new salary. Salary increase = new salary original salary = 5000 4500 = 500 Hence, the salesperson sees an increase in salary of $500. Next, let p represent the salespersons percent salary increase. Then we can translate the problem into words and symbols. Salary increase 500 is = what percent p of original salary 4500

The commutative property of multiplication allows us to change the order of multiplication on the right-hand side of this last equation. 500 = 4500p Solve for p. 500 4500p = 4500 4500 1 =p 9 Divide both sides by 4500. Reduce by dividing numerator and denominator of 500/4500 by 500.

We need to change p = 1/9 to a percent. We can nd an exact answer by creating an equivalent fraction with a denominator of 100. 1 n = 9 100 9n = 100 9n 100 = 9 9 1 n = 11 9 Hence, the percent increase is p= Make equivalent fraction. Cross multiply. Divide both sides by 9. Convert 100/9 to mixed fraction.

1 11 9 1 1 = = 11 %. 9 9 100

Alternative Solution. An alternative approach is to ask what percent of the original salary equals the new salary. In this approach, let p represent the percent of the original salary that equals the new salary. Version: Fall 2010

7.4. PERCENT INCREASE OR DECREASE

531

New salary 5000 Solve for p. 5000 = 4500p 5000 4500p = 4500 4500 10 =p 9

is =

what percent p

of

original salary 4500

Change the order of multiplication. Divide both sides by 4500. Reduce: Divide numerator and denominator of 5000/4500 by 500.

We need to change 10/9 to a percent. Again, create an equivalent fraction with a denominator of 100. n 10 = 9 100 9n = 1000 1000 9n = 9 9 1 n = 111 9 Thus, Make equivalent fraction. Cross multiply. Divide both sides by 9. Convert 1000/9 to a mixed fraction.

1 111 9 10 1 = = 111 %. 9 9 100 1 Hence, the new salary is 111 9 % of the original salary. To nd the percent increase, subtract 100% from 111 1 %. 9

p=

1 1 111 % 100% = 11 % 9 9 This represents an 11 1 % increase in salary, which is the same answer garnered 9 by the rst solution technique.
1 Answer: 5 4 %

Percent Decrease
You Try It! EXAMPLE 3. Due to a mill closure, the population of Silvertown decreases by 8.5%. If the original population was 10,200 hardy souls, what is the new population? Solution. Let x represent the population decrease. Then we can translate the problem into words and symbols. Version: Fall 2010 Several retail stores close and the population of Athens decreases by 7.2% as a result. If the original population was 12,500, what is the new population?

532

CHAPTER 7. PERCENT

Population decrease x Solve for x. x = 0.085 10200 x = 867

is =

8.5% 8.5%

of

original population 10200

8.5% = 0.085. Multiply: 0.085 10200 = 867.

Therefore, the population decrease is 867. To compute the new population P , we must subtract this decrease from the original population.

New population P

is =

original population 10200

minus

population decrease 867

Thus, the new population is P = 9, 333 hardy souls. Alternative Solution. Subtract 8.5% from 100% to obtain 100% 8.5% = 91.5%. Thus, if 8.5% of the population leaves town, then 91.5% of the population stays. Thus, the new population P is calculated from the original as follows: New population P Solve for P . P = 0.915 10200 P = 9333 Answer: 11,600 Same answer. 91.5% = 0.915. Multiply: 0.915 10200 = 9333. is = 91.5% 91.5% of original population 10200

You Try It! A textile mill closure results in the population of the adjacent town decreasing from 8,956 to 7,800. What is the percent decrease in the population, rounded to the nearest tenth of a percent? EXAMPLE 4. Millertown falls on hard times and its population decreases from 11,256 to 10,923 in the space of one year. What is the percent decrease, rounded to the nearest hundredth of a percent? Solution. To nd the decrease in population, rst subtract the current population from the original population. Population decrease = original population current population = 11256 10923 = 333 Version: Fall 2010

7.4. PERCENT INCREASE OR DECREASE

533

Hence, the population has decreased by 333 people. Next, let p represent the percent population decrease. Then we can translate the problem into words and symbols.

Population decrease 333 Solve for p.

is =

what percent p

of

original population 11256

11256p 333 = 11256 11256 0.02958 p

Divide both sides by 11256. Divide: 333/11256 0.02958.

To change p to a percent, move the decimal point two places to the right and append a percent symbol. p 0.02958 0 02.958% 2.958%. We are asked to round to the nearest hundredth of a percent. Test digit p 2.9 5 8 % Rounding digit Because the test digit is greater than or equal to 5, add 1 to the rounding digit and truncate. That is, p 2.96%. Thus, the population of Millertown decreases approximately 2.96%. Alternative Solution. An alternative approach is to ask what percent of the original population equals the new population. New population 10923 Solve for p. 10923 = 11256p 11256p 10923 = 11256 11256 0.97041 p Change the order of multiplication. Divide both sides by 11256. Divide: 10923/11256 0.97041. is = what percent p of original population 11256

To change p to a percent, move the decimal two places to the right and append a percent symbol. p 0.97041 0 97.041% 97.041%. We are asked to round to the nearest hundredth of a percent. Version: Fall 2010

534

CHAPTER 7. PERCENT Test digit p 97.0 4 1 % Rounding digit

Because the test digit is less than 5, leave the rounding digit alone and truncate. That is, p 97.04%. Thus, 97.04% of the Millertown population remains. To nd the percent decrease (the percent who left), subtract 97.04% from 100%. 100% 97.04% = 2.96% Answer: 12.9% Hence, the population of Millertown decreases by 2.96%. Same answer.

Discount
Another important application of percent is the discounting of goods. You Try It! A pair of hiking boots is marked at $200. During a sale, the boots are discounted by 8%. What is the new price of the boots? EXAMPLE 5. A pair of skis is marked at $310. However, a sign in the shop indicates that skis are being discounted at 15%. What will be the new selling price of the skis? Solution. Let D represent the discount (in dollars) given for our pair of skis. Then, in words and symbols: Discount D Solve for D. D = 0.15 310 D = 46.50 15% = 0.15. Multiply: 0.15 310 = 46.50. is = 15% 15% of original marked price 310

Hence, the discount is $46.50. To nd the new selling price, subtract this discount from the original selling price. New selling price = original selling price discount = 310 46.50 = 263.50 Version: Fall 2010

7.4. PERCENT INCREASE OR DECREASE Hence, the new selling price is $263.50. Alternate Solution. Subtract 15% from 100% to obtain 100% 15% = 85%.

535

That is, if an item is discounted 15%, then its new selling price S is 85% of its original marked price. New selling price S Solve for S. S = 0.85 310 S = 263.50 85% = 0.85. Multiply: 0.85 310 = 263.50. Answer: $184 is = 85% 85% of original marked price 310

Thus, the new selling price is $263.50. Same answer.

You Try It! EXAMPLE 6. A pair of ski boots marked at $210 is sold for $180. Find the percent discount, correct to the nearest tenth of a percent. Solution. We can nd the discount (in dollars) by subtracting the sale price from the original marked price. Discount = original marked price sale price = 210 180 = 30 Hence, the boots are discounted $30. Let p represent the percent discount. Then, in words and symbols: Discount 30 Solve for p. 30 = 210p 210p 30 = 210 210 1 =p 7 p 0.1428 Change order of multiplication. Divide both sides by 210. Reduce: Divide numerator and denominator of 30/210 by 30. Divide: 1/7 0.1428. Version: Fall 2010 is = percent discount p of original marked price 210 A computer marked at $2,000 is sold at a discount for $1,850. Find the percent discount, correct to the nearest tenth of a percent.

536

CHAPTER 7. PERCENT

To change p to a percent, move the decimal point two places to the right and append a percent symbol. p 0.1428 0 14.28% 14.28%. Round to the nearest tenth of a percent. Test digit p 14. 2 8 % Rounding digit Because the test digit is greater than or equal to 5, add 1 to the rounding digit and truncate. Thus, correct to the nearest tenth of a percent, the percent discount is p 14.3%. Alternate Solution. An alternative approach is to ask what percent p of the original marked price equals the selling price. New selling price 180 Solve for p. 180 = 210p 180 210p = 210 210 6 =p 7 p 0.8571 Change the order of multiplication. Divide both sides by 210. Reduce: Divide numerator and denominator of 180/210 by 30. Divide: 6/7 0.8571. is = what percent p of original marked price 210

To change p to a percent, move the decimal point two places to the right and append a percent symbol. p 0.8571 0 85.71% 85.71%. Round to the nearest tenth of a percent. Test digit p 85. 7 1 % Rounding digit Version: Fall 2010

7.4. PERCENT INCREASE OR DECREASE

537

Because the test digit is less than 5, do nothing to the rounding digit and truncate. Thus, correct to the nearest tenth of a percent, p 85.7%. Thus, the new selling price is 85.7% of the original marked price. Subtract 85.7% from 100%. 100% 85.7% = 14.3%. That is, if the new selling price is 85.7% of the original price, then the percent discount is 14.3%. This is the same answer found with the rst method. Answer: 7.5%

Version: Fall 2010

538

CHAPTER 7. PERCENT

l l l

Exercises

l l l

1. A television set is marked at $447. However, a sign in the shop indicates that the television set is being discounted at 20.5%. What will be the new selling price of the television set? Round your answer to the nearest penny. 2. A stereo set is marked at $380. However, a sign in the shop indicates that the stereo set is being discounted at 7.5%. What will be the new selling price of the stereo set? Round your answer to the nearest penny. 3. Due to a ball bearing plant closure, Anselm falls on hard times and its population decreases from 10,794 to 8,925 in the space of one year. What is the percent decrease, rounded to the nearest hundredth of a percent? 4. Due to a logging mill closure, Carlytown falls on hard times and its population decreases from 12,113 to 10,833 in the space of one year. What is the percent decrease, rounded to the nearest hundredth of a percent? 5. A car rack is marked at $500. However, a sign in the shop indicates that the car rack is being discounted at 3.5%. What will be the new selling price of the car rack? Round your answer to the nearest penny. 6. A car rack is marked at $405. However, a sign in the shop indicates that the car rack is being discounted at 17.5%. What will be the new selling price of the car rack? Round your answer to the nearest penny. 7. Due to a textile mill closure, the population of Silvertown decreases by 4.1%. If the original population was 14,678 hardy souls, what is the new population, correct to the nearest person? Version: Fall 2010

8. Due to a department store closure, the population of Petroria decreases by 5.3%. If the original population was 14,034 hardy souls, what is the new population, correct to the nearest person? 9. A bartender is granted a 4.6% salary increase. If the bartenders current salary is $2,500 per month, nd the bartenders new monthly salary, rounded to the nearest dollar. 10. A bartender is granted a 5.5% salary increase. If the bartenders current salary is $2,900 per month, nd the bartenders new monthly salary, rounded to the nearest dollar. 11. A car rack marked at $358 is sold for $292. Find the percent discount, correct to the nearest tenth of a percent. 12. A bicycle marked at $328 is sold for $264. Find the percent discount, correct to the nearest tenth of a percent. 13. Due to a auto manufacturing plant closure, Carlytown falls on hard times and its population decreases from 14,393 to 12,623 in the space of one year. What is the percent decrease, rounded to the nearest hundredth of a percent? 14. Due to a ball bearing plant closure, Mayville falls on hard times and its population decreases from 8,494 to 6,609 in the space of one year. What is the percent decrease, rounded to the nearest hundredth of a percent? 15. Due to a auto manufacturing plant closure, the population of Silvertown decreases by 2.4%. If the original population was 8,780 hardy souls, what is the new population, correct to the nearest person?

7.4. PERCENT INCREASE OR DECREASE 16. Due to a textile mill closure, the population of Ghosttown decreases by 6.1%. If the original population was 14,320 hardy souls, what is the new population, correct to the nearest person? 17. A clerk making a salary of $2,600 per month has her salary increased to $2,950 per month. Find the percent increase correct to the nearest tenth of a percent. 18. A clerk making a salary of $3,600 per month has her salary increased to $4,100 per month. Find the percent increase correct to the nearest tenth of a percent. 19. A bartender making a salary of $4,200 per month has her salary increased to $4,300 per month. Find the percent increase correct to the nearest tenth of a percent.

539

20. A bartender making a salary of $3,200 per month has her salary increased to $3,550 per month. Find the percent increase correct to the nearest tenth of a percent. 21. A gardener is granted a 5.1% salary increase. If the gardeners current salary is $3,200 per month, nd the gardeners new monthly salary, rounded to the nearest dollar. 22. A secretary is granted a 2.8% salary increase. If the secretarys current salary is $3,600 per month, nd the secretarys new monthly salary, rounded to the nearest dollar. 23. A television set marked at $437 is sold for $347. Find the percent discount, correct to the nearest tenth of a percent. 24. A camera marked at $390 is sold for $328. Find the percent discount, correct to the nearest tenth of a percent.

25. Suppose that the price of an 8-ounce can of tomato sauce increased from $0.20 to $0.28. a) What was the amount of increase? b) What was the percent increase? 26. The following table summarizes summertime gasoline prices in San Francisco, CA. The price is the number of dollars required to purchase one gallon of unleaded gasoline. Data from gasbuddy.com. Year Price per gallon 2003 1.80 2004 2.28 2.57 2005 2006 3.20 2007 3.28 4.61 2008 2009 3.01 What is the percent increase or decrease from 2003 to 2005? Round your answer to the nearest whole percent. 27. Refer to the table of gas prices in Exercise 26. What is the percent increase or decrease from 2005 to 2007? Round your answer to the nearest whole percent. Version: Fall 2010

540

CHAPTER 7. PERCENT

28. Refer to the table of gas prices in Exercise 26. What is the percent increase or decrease from 2007 to 2009? Round your answer to the nearest whole percent. 29. Rate hike. A family paying $858 monthly for health coverage is faced with a 39% hike in rates. What will be their new monthly cost after the increase? Associated Press-Times-Standard 02/09/10 HHS secretary asks Anthem Blue Cross to justify rate hike. 30. Parking ne. San Franciscos Metropolitan Transportation Agency was expected to consider raising nes for the use of fake, stolen, or borrowed disabled parking tags from $100 to $825. What is the percent increase for this ne? Associated Press-Times-Standard 01/06/10 Fines for fake disabled parking tags may go up in San Francisco. 31. Industrial move. Regulations in California have caused factory farmers to move out of state. Idahos industrial dairy went from 180, 000 cows in 1990 to 530, 000 cows in 2009. What is the percent increase for Idaho industrial dairy cows, rounded to the nearest percent? Associated Press-Times-Standard 02/09/10 Idaho, others prepare for California egg farm exodus. 32. Whooping crane. Whooping crane populations made a remarkable comeback from just 15 birds in 1941 to about 400 birds worldwide in 2010. What is the percent increase for the whooping crane population over the past seventy years, rounded to the nearest percent? Associated PressTimes-Standard 03/17/10 Plucky whooping crane gives wildlife experts hope. 33. Underwater. The loss of Arctic sea ice will allow for an underwater ber optic cable that will cut the time it takes to send a message from London to Tokyo from a current 140 milliseconds down to 88 milliseconds. Associated Press-Times-Standard 01/22/10 Global warming opens up Northwest Passage for underwater ber optic cable. a) What is the estimated percent decrease in communication time? b) At a cost of $1.2 billion, what is the cost per millisecond of saving (rounded to the nearest dollar)? 34. Chinook salmon. During the 2009 season in the Sacramento river basin, a record low 39, 500 chinook salmon were recorded, way down from the more than 750, 000 counted in 2002. What is the percent decrease in the salmon count from 2002 to 2009, rounded to the nearest percent? Associated Press-Times-Standard 02/25/10 Feds predict better year for California salmon. 35. Daylight hours. In middle latitudes, summer days can have as many as 14 hours of daylight, while winter days can have a few as 10 hours of daylight. What percent more daylight is there in summer than in winter? 36. Cyber-experts. Defense Secretary Robert Gates said the Pentagon will increase the number of cyber-experts it can train each year from 80 to 250 by 2011. What percent increase is this? Round your answer to the nearest percent. Associated Press Times-Standard 4/19/09

Version: Fall 2010

7.4. PERCENT INCREASE OR DECREASE

541

37. Home prices. Real estate data for Humboldt County, California, is given below. Associated Press-Times-Standard 01/17/10 How is our local real estate market compared to other regions? Year Number of homes sold Average home price 2000 1,358 $152,257 2005 1,432 $344,500 2009 833 $285,000 a) What percent change in average home price occurred between 2000 and 2009? b) What percent change in homes sold occurred between 2000 and 2009?

l l l
1. $355.36 3. 17.32% 5. $482.50 7. 14,076 people 9. $2,615 11. 18.4% 13. 12.30% 15. 8,569 17. 13.5% 19. 2.4% 21. $3,363

Answers

l l l

23. 20.6% 25. a) $0.08 b) 40% 27. 28% increase 29. $1,193 31. 194% increase 33. a) 37% b) About $23,076,923 per millisecond 35. 40% more daylight 37. a) 87% increase in average home price b) 39% decrease in home sales

Version: Fall 2010

542

CHAPTER 7. PERCENT

7.5

Interest

One way of awarding interest is called simple interest. Before we provide the formula used in calculating simple interest, lets rst dene some basic terms. Balance. The balance is the current amount in an account or the current amount owed on a loan. Principal. The principal is the initial amount invested or borrowed. Rate. This is the interest rate, usually given as a percent per year. Time. This is the time duration of the loan or investment. If the interest rate is per year, then the time must be measured in years. To calculate the simple interest on an account or loan, use the following formula. Simple Interest. Simple interest is calculated with the formula I = P rt, where I is the interest, P is the principal, r is the interest rate, and t is the time.

You Try It! How much simple interest is earned if $2,500 is invested at 5% per year for 8 years? EXAMPLE 1. How much simple interest is earned if $1,200 is invested at 4% per year for 5 years? Solution. Set up the formula for simple interest. I = P rt The principal is P = $1200, the interest rate is r = 4% = 0.04 per year, and the time or duration of the investment is t = 5 years. Substitute each of these numbers into the simple interest formula I = P rt. I = (1200)(0.04)(5) = 240 Answer: $1,000 Substitute 1200 for P , 0.04 for r, and 5 for t. Multiply.

Hence, the interest earned in 5 years is $240.

Version: Fall 2010

7.5. INTEREST To nd the balance, we must add the interest to the principal.

543

Calculating the Balance. To nd the balance, add the interest to the principal. That is, Balance = Principal + Interest.

You Try It! EXAMPLE 2. A contractor borrows $5,000 at 4.5% per year. The interest accrued is simple interest. The duration of the loan is 6 months. How much will the contractor have to pay back at the end of the 6-month loan period? Solution. Set up the formula for simple interest. I = P rt The principal is P = $5000, the interest rate is r = 4.5% = 0.045 per year, and the time or duration of the loan is t = 6 months. Because the interest rate is per year, the time must be changed to years. That is, 6 months = 6 months 1 yr 12 months 1 yr $ $$ = 6$ months $ 12 $$$ months 6 = yr 12 = 1 yr 2 Apply conversion factor. Cancel common unit. Multiply numerators; multiply denominators. Reduce. An accountant borrows $8,000 at 5.5% per year. The interest accrued is simple interest. The duration of the loan is 3 months. How much will the aacountant have to pay back at the end of the 3-month loan period?

Substitute these numbers into the simple interest formula I = P rt. I = (5000)(0.045) = 112.50 1 2 Substitute 5000 for P , 0.045 for r, and 1/2 for t. Multiply.

Hence, the interest accrued in 6 months is $112.50. Therefore, Amount owed = Principal + Interest = 5000 + 112.50 = 5112.50 That is, the amount owed at the end of the 6-month loan period is $5,112.50. Answer: $8,110

Version: Fall 2010

544

CHAPTER 7. PERCENT

You Try It! The owner of Alioto Motors takes out a 8-month loan at 4% per year simple interest. At the end of the 8-month loan period, the interest owed is $80. What was the principal amount borrowed? EXAMPLE 3. A business owner takes out a 4-month loan at 5.4% per year simple interest. At the end of the 4-month loan period, the interest owed is $90. What was the principal amount borrowed? Solution. Set up the formula for simple interest. I = P rt The interest owed is I = $90, the interest rate is r = 5.4% = 0.054 per year, and the time or duration of the loan is t = 4 months. Because the interest rate is per year, the time must be changed to years. That is, 4 months = 4 months 1 yr 12 months 1 yr $ = 4 $$$ months $ 12 $$$ months 4 = yr 12 = 1 yr 3 Apply conversion factor. Cancel common unit. Multiply numerators; multiply denominators. Reduce.

Substitute these numbers into the simple interest formula I = P rt. 90 = P (0.054) 90 = 0.054 P 3 90 = 0.018P 1 3 Substitute 90 for I, 0.054 for r, and 1/3 for t. Rearrange order of multiplication. Divide: 0.054/3 = 0.018.

Solve the equation for P . 90 0.018P = 0.018 0.018 5000 = P Answer: $3,000 Divide both sides by 0.018. Divide: 90/0.018 = 5000.

Thus, the principal amount borrowed was $5,000.

You Try It! A manufacturer borrows $10,000 for 4 years. At the end of the 4-year loan period, the interest owed is $3,200. What was the simple interest rate? EXAMPLE 4. A pet shop owner borrows $8,000 for 6 months. At the end of the 6-month loan period, the interest owed is $200. What was the simple interest rate? Solution. Set up the formula for simple interest. I = P rt Version: Fall 2010

7.5. INTEREST

545

The principal is P = $8, 000, the interest owed is I = $200, and the duration of the loan is t = 6 months. As we saw in Example 2, 6 months equals 1/2 year. Substitute these numbers into the simple interest formula I = P rt. 200 = (8000)(r) 200 = 8000 r 2 200 = 4000r 1 2 Substitute 8000 for P , 200 for I, and 1/2 for t. Rearrange order of multiplication. Divide: 8000/2 = 4000.

Solve this last equation for r. 200 4000r = 4000 4000 1 =r 20 Divide both sides by 4000. Reduce: Divide numerator and denominator by 200.

We need to change r to a percent. This is easily accomplished by creating an equivalent fraction with a denominator of 100. 15 1 = 20 20 5 5 = 100 = 5% Thus, the simple interest rate is 5%. Answer: 8%

Extending the Simple Interest Formula


In Example 2, we had to add the interest to the principal to discover the balance owed at the end of the loan. That is, Balance = Principal + Interest, or in symbols, A = P + I, where A is the balance, P is the principal, and I is the simple interest. Because I = P rt, we substitute P rt for I in the last equation to get A = P + P rt. Use the distributive property to factor P from each term on the right. A = P 1 + P rt A = P (1 + rt).

Version: Fall 2010

546

CHAPTER 7. PERCENT

Balance Formula Using Simple Interest. If simple interest is applied, then the balance is given by the formula A = P (1 + rt), where A is the balance, P is the principal, r is the simple interest rate, and t is the duration of the loan or investment.

You Try It! If $8,000 is invested at 4.25% simple interest, what will be the balance after 4 years? EXAMPLE 5. If $4,000 is invested at 6.25% simple interest, what will be the balance after 2 years? Solution. Start with the balance formula for simple interest. A = P (1 + rt) The principal is P = $4, 000, the rate is r = 6.25% = 0.0625 per year, and the time is t = 2 years. Substitute these numbers in the balance formula A = P (1 + rt). A = 4000 1 + (0.0625)(2) A = 4000(1 + 0.125) A = 4000(1.125) A = 4500 $9,360 Substitute 4000 for P , 0.0625 for r, and 2 for t. Order of Ops: 0.0625 2 = 0.125. Order of Ops: 1 + 0.125 = 1.125. Multiply: 4000 1.125 = 4500.

Hence, the balance at the end of two years is A = $4, 500.

You Try It! The balance due on a 4-year loan is $6,300. If the simple interest rate is 10%, what was the principal borrowed? EXAMPLE 6. The balance due on a 2-year loan is $3,360. If the simple interest rate is 6%, what was the principal borrowed? Solution. Start with the balance formula for simple interest. A = P (1 + rt) The balance is A = $3360, the rate is r = 6% = 0.06 per year, and the time is t = 2 years. Substitute these numbers in the balance formula A = P (1 + rt). 3360 = P 1 + (0.06)(2) 3360 = P (1 + 0.12) 3360 = P (1.12) 3360 = 1.12P Version: Fall 2010 Substitute 3360 for A, 0.06 for r, and 2 for t. Order of Ops: 0.06 2 = 0.12. Order of Ops: 1 + 0.12 = 1.12. Change order of multiplication.

7.5. INTEREST Solve this last equation for P . 1.12P 3360 = 1.12 1.12 3000 = P Divide both sides by 1.12. Divide: 3360/1.12 = 3000.

547

Hence, the principal borrowed was P = $3, 000.

Answer: $4,500

You Try It! EXAMPLE 7. The balance due on a 2-year loan is $2,200. If the principal borrowed was $2,000, what was the rate of simple interest? Solution. Start with the balance formula for simple interest. A = P (1 + rt) The balance is A = $2, 200, the principal is P = $2, 000, and the time is t = 2 years. Substitute these numbers in the balance formula A = P (1 + rt). 2200 = 2000(1 + (r)(2)) 2200 = 2000(1 + 2r) Solve this last equation for r. 2200 = 2000 + 4000r 2200 2000 = 2000 + 4000r 2000 200 = 4000r 200 4000r = 4000 4000 1 =r 20 Distribute 2000. Subtract 2000 from both sides. Simplify both sides. Divide both sides by 4000. Reduce: 200/4000 = 1/20. Substitute 2200 for A, 2000 for P , and t = 2. Change the order of muliplication. The balance due on a 2-year loan is $4,640. If the principal borrowed was $4,000, what was the rate of simple interest?

Of course, r must be changed to a percent. In Example 4, we encountered this same fraction. 5 1 = = 5% r= 20 100 Hence, the rate of simple interest is r = 5%. Answer: 8%

Version: Fall 2010

548

CHAPTER 7. PERCENT

l l l

Exercises

l l l

1. How much simple interest is earned if $7,600 is invested at 8% per year for 7 years? 2. How much simple interest is earned if $2,500 is invested at 5% per year for 6 years? 3. How much simple interest is earned if $5,800 is invested at 3.25% per year for 4 years? 4. How much simple interest is earned if $2,000 is invested at 8.5% per year for 6 years?

5. How much simple interest is earned if $2,400 is invested at 8.25% per year for 5 years? 6. How much simple interest is earned if $4,000 is invested at 6.5% per year for 6 years? 7. How much simple interest is earned if $4,000 is invested at 7.25% per year for 6 years? 8. How much simple interest is earned if $8,200 is invested at 8% per year for 6 years?

9. A business owner borrows $3,600 for 2 months at a 4.5% per year simple interest rate. At the end of the 2-month loan period, how much interest is owed? 10. A business owner borrows $3,200 for 4 months at a 9% per year simple interest rate. At the end of the 4-month loan period, how much interest is owed? 11. A business owner borrows $2,400 for 6 months at a 2% per year simple interest rate. At the end of the 6-month loan period, how much interest is owed? 12. A business owner borrows $2,200 for 4 months at a 3% per year simple interest rate. At the end of the 4-month loan period, how much interest is owed? 13. A business owner takes out a 6-month loan at a 8% per year simple interest rate. At the end of the 6-month loan period, the interest owed is $68. What was the principal amount borrowed? Version: Fall 2010

14. A business owner takes out a 4-month loan at a 6% per year simple interest rate. At the end of the 4-month loan period, the interest owed is $194. What was the principal amount borrowed? 15. A business owner borrows $3,600 for 3 months at a 8% per year simple interest rate. At the end of the 3-month loan period, how much interest is owed? 16. A business owner borrows $2,400 for 4 months at a 8.25% per year simple interest rate. At the end of the 4-month loan period, how much interest is owed? 17. A business owner takes out a 2-month loan at a 8.5% per year simple interest rate. At the end of the 2-month loan period, the interest owed is $85. What was the principal amount borrowed?

7.5. INTEREST 18. A business owner takes out a 3-month loan at a 2% per year simple interest rate. At the end of the 3-month loan period, the interest owed is $45. What was the principal amount borrowed? 19. A business owner borrows $4,000 for 3 months. At the end of the 3-month loan period, the interest owed is $35. What was the simple yearly interest rate (as a percent)? 20. A business owner borrows $4,200 for 4 months. At the end of the 4-month loan period, the interest owed is $63. What was the simple yearly interest rate (as a percent)? 21. A business owner takes out a 6-month loan at a 7% per year simple interest rate. At the end of the 6-month loan period, the interest owed is $287. What was the principal amount borrowed? 22. A business owner takes out a 6-month loan at a 2% per year simple interest rate. At the end of the 6-month loan period, the interest owed is $40. What was the principal amount borrowed?

549 23. A business owner borrows $7,300 for 2 months. At the end of the 2-month loan period, the interest owed is $73. What was the simple yearly interest rate (as a percent)? 24. A business owner borrows $5,600 for 6 months. At the end of the 6-month loan period, the interest owed is $182. What was the simple yearly interest rate (as a percent)? 25. A business owner borrows $3,200 for 6 months. At the end of the 6-month loan period, the interest owed is $96. What was the simple yearly interest rate (as a percent)? 26. A business owner borrows $5,700 for 4 months. At the end of the 4-month loan period, the interest owed is $133. What was the simple yearly interest rate (as a percent)?

27. Suppose that $6,700 is invested at 9% simple interest per year. What will the balance be after 4 years? 28. Suppose that $5,200 is invested at 3.5% simple interest per year. What will the balance be after 2 years? 29. Suppose that $1,600 is invested at 2% simple interest per year. What will the balance be after 3 years? 30. Suppose that $8,100 is invested at 8.25% simple interest per year. What will the balance be after 4 years?

31. Suppose that $8,900 is invested at 2.5% simple interest per year. What will the balance be after 2 years? 32. Suppose that $9,800 is invested at 2.75% simple interest per year. What will the balance be after 6 years? 33. Suppose that $5,400 is invested at 4.25% simple interest per year. What will the balance be after 2 years? 34. Suppose that $8,400 is invested at 4.5% simple interest per year. What will the balance be after 4 years?

Version: Fall 2010

550 35. The balance on a 6-year loan is $10,222. If the principal borrowed was $7,600, what was the simple interest rate (as a percent)? 36. The balance on a 8-year loan is $12,264. If the principal borrowed was $8,400, what was the simple interest rate (as a percent)? 37. The balance on a 5-year loan is $4,640. If the simple interest rate is 9% per year, what was the principal borrowed? 38. The balance on a 6-year loan is $6,838. If the simple interest rate is 5.25% per year, what was the principal borrowed? 39. The balance on a 9-year loan is $9,593. If the simple interest rate is 9% per year, what was the principal borrowed? 40. The balance on a 8-year loan is $10,032. If the simple interest rate is 4% per year, what was the principal borrowed? 41. The balance on a 3-year loan is $5,941. If the principal borrowed was $5,200, what was the simple interest rate (as a percent)? 42. The balance on a 2-year loan is $9,589. If the principal borrowed was $8,600, what was the simple interest rate (as a percent)?

CHAPTER 7. PERCENT 43. The balance on a 5-year loan is $5,400. If the principal borrowed was $4,000, what was the simple interest rate (as a percent)? 44. The balance on a 6-year loan is $12,635. If the principal borrowed was $9,500, what was the simple interest rate (as a percent)? 45. The balance on a 5-year loan is $11,550. If the simple interest rate is 7.5% per year, what was the principal borrowed? 46. The balance on a 8-year loan is $3,160. If the simple interest rate is 7.25% per year, what was the principal borrowed? 47. The balance on a 4-year loan is $5,720. If the principal borrowed was $4,400, what was the simple interest rate (as a percent)? 48. The balance on a 8-year loan is $4,422. If the principal borrowed was $3,300, what was the simple interest rate (as a percent)? 49. The balance on a 8-year loan is $9,768. If the simple interest rate is 4% per year, what was the principal borrowed? 50. The balance on a 2-year loan is $8,322. If the simple interest rate is 7% per year, what was the principal borrowed?

l l l
1. $4, 256 3. $754 5. $990 7. $1, 740 9. $27 Version: Fall 2010

Answers

l l l

11. $24 13. $1, 700 15. $72 17. $6, 000 19. 3.5%

7.5. INTEREST 21. $8, 200 23. 6% 25. 6% 27. $9, 112 29. $1, 696 45. $8, 400 31. $9, 345 47. 7.5% 33. $5, 859 35. 5.75% 49. $7, 400 37. $3, 200 39. $5, 300 41. 4.75% 43. 7%

551

Version: Fall 2010

552

CHAPTER 7. PERCENT

7.6

Pie Charts

In this section we turn our attention to pie charts, but before we do, we need to establish some fundamentals regarding measurement of angles. If you take a circle and divide it into 360 equal increments, then each increment is called one degree (1 ). See Figure 7.2.

1
01 40 15

30

110 20

100 90 80 70

60

50 40
30

190 180 170 160

20

10
0 350 3 40

200 10

3 30 32

20

0 31

Figure 7.2: There are 360 degrees (360 ) in a cricle. A ray is a line that starts at a point and then extends indenitely in one direction. The starting point of the ray is called its vertex. V Figure 7.3: A ray with vertex V extends indenitely in one direction. If two rays have a common vertex, they form what is called an angle. In Figure 7.4 weve labeled the rst ray as the Initial Side of the angle, and the second as the Terminal Side of the angle. Version: Fall 2010

30

80 270 260 25 90 2 02 02

40 23

7.6. PIE CHARTS

553

Terminal Side

Initial Side

Figure 7.4: Two rays with a common vertex V form an angle.

We can nd the degree measure of the angle by using a device called a protractor. Align the notch in the center of the base of the protractor with the vertex of the angle, then align the base of the protractor with the initial side of the angle. The terminal side of the angle will intersect the protractor edge where we can read the degree measure of the angle (see Figure 7.5). In Figure 7.5, note that the terminal side of the angle passes through the tick mark at the number 30, indicating that the degree measure of this angle is 30 .

1 0
0

30

110 20

100 90 80 70

60

50 40
30

Terminal Side

170 1 60 15

14

20

10
Initial Side

V Figure 7.5: The degree measure is 30 .

Pie Charts
Now that we can measure angles, we can turn our attention to constructing pie charts.

Version: Fall 2010

554

CHAPTER 7. PERCENT

Pie Chart. A pie chart is a circular chart that is divided into sectors, each sector representing a particular quantity. The area of each sector is a percentage of the area of the whole circle and is determined by calculating the quantity it represents as a percentage of the whole.

You Try It! In a recent Gallup poll, 50% of the American public say it is the federal governments responsibility to make sure all Americans have health care, 47% disagree, and 3% were undecided. Create a pie chart that displays these percentages. EXAMPLE 1. In a recent Gallup poll, 66% of those polled approved of the Presidents job performance, 28% disapproved, and 6% were undecided. Create a pie chart that displays these percentages. Solution. Lets begin with 6% undecided. We want to create a sector that is 6% of the area of the whole circle. There are 360 degrees in a full circle, so 6% of this number is 6% 360 = 0.06 360 = 21.6 . Start with a circle, set the baseline notch of the protractor on the center of the circle, then mark an angle of 21.6, as seen in Figure 7.6(a). Shade the resulting region as shown in Figure 7.6(b), called a sector, which represents 6% of the total area of the circle.
110
90

0 13

70

50
30

170 15 0

(a) Mark an angle of 21.6 .

Figure 7.6: Sector with central angle 21.6 represents the 6% of the polling sample that were undecided about the presidents performance. Next, 28% disapproved of the Presidents job performance. Thus, 28% 360 = 0.28 360 = 100.8. Therefore, a sector with a central angle of 100.8 will represent 28% job disapproval. Version: Fall 2010

10

(b) Shaded sector is 6% of total circular area.

7.6. PIE CHARTS

555

Place the notch on the baseline of your protractor on the center of the circle, then align the baseline of the protractor with the terminal side of the rst angle, as shown in Figure 7.7(a). Mark a central angle of 100.8, as shown in Figure 7.7(a). Shade the resulting second sector with a darker shade of gray, as shown in Figure 7.7(b). This sector contains 28% of the total area of the circle and represents the portion of the polling sample that disapproved of the presidents job performance.
70
50

0 11

90

30

10

170 150 13 0

(a) Mark the next angle at 100.8 .

(b) Second shaded sector is 28% of total circular area.

Figure 7.7: Sector with central angle 100.8 represents the 28% of the polling sample that disapproved of the presidents performance. Finally, as weve shaded the sectors representing 6% and 28% of the polling data in Figure 7.7(b), the remaining sector in Figure 7.7(b), shaded in white, represents the 66% of the polling sample who approved of the presidents job performance (and 66% of the area of the whole circle). Once you have computed and plotted the correct central angles for each of the sectors, you will want to label your pie chart. One possible annotation method is shown in Figure 7.8. Disapprove (28%)

Answer:

Agree (50%)

Undecided (6%)

Undecided (3%) Disagree (47%)

Approve (66%) Figure 7.8: Annotating the nal pie chart.

Version: Fall 2010

556

CHAPTER 7. PERCENT

You Try It! Two hundred people were asked whether they vote Yes or No on Proposition 8. There were 150 Yes votes and 50 No votes. Create a pie chart showing the distribution of these responses. EXAMPLE 2. One thousand people were polled with the question Where does your dog sleep during the night? The responses are shown in the following table. Location Outside Another Room On the Bedroom Floor On the Bed Totals Number 30 220 330 420 1000

Create a pie chart showing the distribution of these responses. Solution. The rst step is to express the number in each location as a percentage of the totals. For example, Outside 30 Solving for p, 1000p 30 = 1000 1000 p = 0.03 Divide both sides by 1000. Divide: 30/1000 = 0.03. is = what percent p of total 1000

Thus, p = 3%. In similar fashion, divide the number in each location by 1000 to nd the following percents. Location Outside Another Room On the Bedroom Floor On the Bed Totals Number 30 220 330 420 1000 Percent 3% 22% 33% 42% 100%

Note that the individual percents must total 100%. Lets begin with the fact that 3% of the dog owners have their dogs sleep outside. To nd the portion of the full circle that represents 3%, we take 3% of 360 . 3% 360 = 0.03 360 = 10.8 Start with a circle, set the baseline notch of the protractor on the center of the circle, then mark an angle of 10.8, as seen in Figure 7.9(a). Shade the Version: Fall 2010

7.6. PIE CHARTS

557

0 13

110

90

70

50
30

170 15 0

(a) Mark an angle of 10.8 .

Figure 7.9: Sector with central angle 10.8 represents the 3% of the polling sample that have their dogs sleep outside.

resulting sector as shown in Figure 7.9(b), which represents 3% of the total area of the circle. Next, 22% have their dog sleep in another room. 22% 360 = 0.22 360 = 79.2 . Therefore, a sector with a central angle of 79.2 will represent the fact that 22% of the dog owners have their dog sleep in another room. Place the notch on the baseline of your protractor on the center of the circle, then align the baseline of the protractor with the terminal side of the rst angle, as shown in Figure 7.10(a). Mark a central angle of 79.2, as shown in Figure 7.7(a). Shade the resulting second sector with a darker shade of gray, as shown in Figure 7.10(b). This sector contains 22% of the total area of the circle and represents the portion of the polling sample whose dog sleeps in another room. Next, 33% allow their dog sleep on the bedroom oor. 33% 360 = 0.33 360 = 118.8 . Therefore, a sector with a central angle of 118.8 will represent the fact that 33% of the dog owners allow their dog to sleep on the bedroom oor. Place the notch on the baseline of your protractor on the center of the circle, then align the baseline of the protractor with the terminal side of the second sector, as shown in Figure 7.11(a). Mark a central angle of 118.8, as shown in Figure 7.11(a). Shade the resulting second sector with a darker shade of gray, as shown in Figure 7.11(b). This sector contains 33% of the total area of the circle and represents the portion of the polling sample whose dog sleeps on the bedroom oor. Version: Fall 2010

10

(b) Shaded sector is 3% of total circular area.

558

CHAPTER 7. PERCENT

0 11
13 0

90

70

50
30

170 150

10

(a) Mark the next angle at 79.2 .

(b) Second shaded sector is 22% of total circular area.

Figure 7.10: Sector with central angle 79.2 represents the 22% of the polling sample whose dogs sleep in another room.

30

10

70

50

0 13

110

90

(a) Mark the next angle at 118.8 .

Figure 7.11: Sector with central angle 118.8 represents the 33% of the polling sample whose dogs sleep on the bedroom oor.

Because the rst three sectors, shaded in various levels of gray, represent 3%, 22%, and 33% of the total circular area, respectively, the remaining sector (shaded in white) automatically represents 100% (3% + 22% + 33%) = 42% of the total circular area. This region represents the percent of dog owners who allow their dogs to sleep on the bed. The nal result, with annotations, is shown in Figure 7.12.

Version: Fall 2010

170 15 0

(b) Third shaded sector is 33% of total circular area.

7.6. PIE CHARTS

559 Answer:

Another room (22%) In bedroom (33%) Outside (3%)

Yes (75%)

No (25%)

On bed (42%) Figure 7.12: Annotating the nal pie chart.

Version: Fall 2010

560

CHAPTER 7. PERCENT

l l l

Exercises

l l l

1. In an election for class president, Raven received 21% of the votes, Anita received 27%, Jamal received 24% of the votes, and 28% of the votes were received by other candidates. If a pie chart is created for this data, what will be the degree measure of the central angle of the sector representing Ravens share of the vote? Round your answer to the nearest degree. 2. In an election for class president, Fernando received 26% of the votes, Luisa received 26%, Ali received 26% of the votes, and 22% of the votes were received by other candidates. If a pie chart is created for this data, what will be the degree measure of the central angle of the sector representing Fernandos share of the vote? Round your answer to the nearest degree. 3. In an election for class president, Akbar received 23% of the votes, Ali received 27%, Juanita received 30% of the votes, and 20% of the votes were received by other candidates. If a pie chart is created for this data, what will be the degree measure of the central angle of the sector representing Akbars share of the vote? Round your answer to the nearest degree.

4. In an election for class president, Kamili received 21% of the votes, Bernardo received 22%, Fernando received 30% of the votes, and 27% of the votes were received by other candidates. If a pie chart is created for this data, what will be the degree measure of the central angle of the sector representing Kamilis share of the vote? Round your answer to the nearest degree. 5. In an election for class president, Jamal received 30% of the votes, Luisa received 20%, Kamili received 28% of the votes, and 22% of the votes were received by other candidates. If a pie chart is created for this data, what will be the degree measure of the central angle of the sector representing Jamals share of the vote? Round your answer to the nearest degree. 6. In an election for class president, Juanita received 30% of the votes, Ali received 24%, Estevan received 24% of the votes, and 22% of the votes were received by other candidates. If a pie chart is created for this data, what will be the degree measure of the central angle of the sector representing Juanitas share of the vote? Round your answer to the nearest degree.

7. In an election for class president, Chin received 5 votes, Mabel received 13 votes, and Juanita received the remaining 32 votes cast. If a pie chart is created for this voting data, what will be the degree measure of the central angle of the sector representing Chins share of the vote? Round your answer to the nearest degree.

8. In an election for class president, Anita received 11 votes, Jose received 9 votes, and Bernardo received the remaining 30 votes cast. If a pie chart is created for this voting data, what will be the degree measure of the central angle of the sector representing Anitas share of the vote? Round your answer to the nearest degree.

Version: Fall 2010

7.6. PIE CHARTS 9. In an election for class president, Kamili received 14 votes, Jamal received 9 votes, and Jose received the remaining 27 votes cast. If a pie chart is created for this voting data, what will be the degree measure of the central angle of the sector representing Kamilis share of the vote? Round your answer to the nearest degree. 10. In an election for class president, Jun received 13 votes, Abdul received 15 votes, and Raven received the remaining 22 votes cast. If a pie chart is created for this voting data, what will be the degree measure of the central angle of the sector representing Juns share of the vote? Round your answer to the nearest degree.

561 11. In an election for class president, Hue received 13 votes, Ali received 6 votes, and Henry received the remaining 31 votes cast. If a pie chart is created for this voting data, what will be the degree measure of the central angle of the sector representing Hues share of the vote? Round your answer to the nearest degree. 12. In an election for class president, Mercy received 9 votes, Bernardo received 7 votes, and Hans received the remaining 34 votes cast. If a pie chart is created for this voting data, what will be the degree measure of the central angle of the sector representing Mercys share of the vote? Round your answer to the nearest degree.

13. In an election for class president, the vote distribution among three candidates is shown in the following pie chart. Raven (31%)

Jamila (35%)

Jamal (34%) If there were a total of 95 votes cast in the election, nd the number of votes that Raven received, correct to the nearest vote.

Version: Fall 2010

562

CHAPTER 7. PERCENT

14. In an election for class president, the vote distribution among three candidates is shown in the following pie chart. Henry (38%)

Abdul (36%)

Estevan (26%) If there were a total of 79 votes cast in the election, nd the number of votes that Henry received, correct to the nearest vote. 15. In an election for class president, the vote distribution among three candidates is shown in the following pie chart. Ali (39%)

Jun (37%) Mabel (24%) If there were a total of 58 votes cast in the election, nd the number of votes that Ali received, correct to the nearest vote.

Version: Fall 2010

7.6. PIE CHARTS

563

16. In an election for class president, the vote distribution among three candidates is shown in the following pie chart. Mercy (34%)

Lisa (38%)

Rosa (28%) If there were a total of 65 votes cast in the election, nd the number of votes that Mercy received, correct to the nearest vote. 17. In an election for class president, the vote distribution among three candidates is shown in the following pie chart. Hue (34%)

Mercy (40%)

Abdul (26%) If there were a total of 95 votes cast in the election, nd the number of votes that Hue received, correct to the nearest vote.

Version: Fall 2010

564

CHAPTER 7. PERCENT

18. In an election for class president, the vote distribution among three candidates is shown in the following pie chart. Anita (40%)

Estevan (35%) Jun (25%) If there were a total of 75 votes cast in the election, nd the number of votes that Anita received, correct to the nearest vote.

19. In an election for class president, the vote distribution among three candidates is shown in the following table. Candidate Votes Ali 45 Jamal 34 Jun 52 Use a protractor to help create a pie chart showing the distribution of votes. 20. In an election for class president, the vote distribution among three candidates is shown in the following table. Candidate Votes Aisha 39 Akbar 31 36 Fernando Use a protractor to help create a pie chart showing the distribution of votes.

21. In an election for class president, the vote distribution among three candidates is shown in the following table. Candidate Votes Bernardo 44 Rosa 40 Abdul 58 Use a protractor to help create a pie chart showing the distribution of votes. 22. In an election for class president, the vote distribution among three candidates is shown in the following table. Candidate Votes Estevan 46 Ali 58 49 Henry Use a protractor to help create a pie chart showing the distribution of votes.

Version: Fall 2010

7.6. PIE CHARTS 23. In an election for class president, the vote distribution among three candidates is shown in the following table. Candidate Votes Mercy 56 Hans 53 Lisa 41 Use a protractor to help create a pie chart showing the distribution of votes. 24. In an election for class president, the vote distribution among three candidates is shown in the following table. Candidate Votes Estevan 60 Hue 33 Aisha 31 Use a protractor to help create a pie chart showing the distribution of votes. 25. In an election for class president, the vote distribution among three candidates is shown in the following table. Candidate Votes Raven 43 Mabel 40 52 Bernardo Use a protractor to help create a pie chart showing the distribution of votes. 26. In an election for class president, the vote distribution among three candidates is shown in the following table. Candidate Votes Hue 48 48 Lisa Akbar 47 Use a protractor to help create a pie chart showing the distribution of votes.

565 27. In an election for class president, the vote distribution among three candidates is shown in the following table. Candidate Votes Jun 57 Lisa 30 Aisha 58 Use a protractor to help create a pie chart showing the distribution of votes. 28. In an election for class president, the vote distribution among three candidates is shown in the following table. Candidate Votes Bernardo 54 Mabel 38 Henry 49 Use a protractor to help create a pie chart showing the distribution of votes. 29. In an election for class president, the vote distribution among three candidates is shown in the following table. Candidate Votes Henry 35 Bernardo 32 47 Estevan Use a protractor to help create a pie chart showing the distribution of votes. 30. In an election for class president, the vote distribution among three candidates is shown in the following table. Candidate Votes Bernardo 38 49 Fernando Aisha 44 Use a protractor to help create a pie chart showing the distribution of votes.

Version: Fall 2010

566

CHAPTER 7. PERCENT

31. Guard deployment. The table shows the number of guard troop services since Sept. 11, 2001 (as of Dec. 2008; some troops have activated multiple times). Associated Press-Times-Standard 02/18/10 Guard troops wait for promised pay. Mission Troops Operation Iraqi Freedom 193,598 Operation Enduring Freedom (Afghanistan) 29,212 35,849 Other missions Use a protractor to help create a pie chart showing the distribution of National Guard troops.

l l l
1. 76 3. 83 5. 108 7. 36

Answers
21.

l l l

Candidate Bernardo Rosa Abdul

Votes 44 40 58

Percent 31.0% 28.2% 40.8%

Degrees 111.6 101.52 146.88 Bernardo (31.0%)

9. 101 11. 94 13. 29 votes 15. 23 votes Abdul (40.8%) 17. 32 votes 23. 19. Candidate Ali Jamal Jun Votes 45 34 52 Percent 34.4% 26.0% 39.7% Degrees 123.84 93.6 142.92 Candidate Mercy Hans Lisa Votes 56 53 41 Percent 37.3% 35.3% 27.3% Degrees 134.28 127.08 98.28 Rosa (28.2%)

Mercy (37.3%) Ali (34.4%) Jamal (26.0%) Hans (35.3%)

Lisa (27.3%) Jun (39.7%) Version: Fall 2010

7.6. PIE CHARTS 25. Candidate Raven Mabel Bernardo Votes 43 40 52 Percent 31.9% 29.6% 38.5% Degrees 114.84 106.56 138.6 29. Candidate Henry Bernardo Estevan Votes 35 32 47

567

Percent 30.7% 28.1% 41.2%

Degrees 110.52 101.16 148.32 Henry (30.7%)

Raven (31.9%) Mabel (29.6%) Bernardo (28.1%)

Bernardo (38.5%) 27. Candidate Jun Lisa Aisha Votes 57 30 58 Percent 39.3% 20.7% 40.0% Degrees 141.48 74.52 144 31.

Estevan (41.2%) Mission Operation Iraqi Freedom Operation Enduring Freedom Other missions Operation Iraqi Freedom (74.8%) Troops 193,598 29,212 35,849 Percent 74.8% 11.3% 13.9% Degrees 269.28 40.68 50.04

Jun (39.3%)

Lisa (20.7%) Other missions (13.9%) Aisha (40.0%) Operation Enduring Freedom (11.3%)

Version: Fall 2010

Chapter

Graphing

Ren Descartes (1596-1650) was a French philosopher and mathematician. As a e philosopher, he is famous for the saying Cogito ergo sum (I think, therefore I am), and his writings led many to consider him the Father of Modern Philosophy. Even today, a number of his writings are standard faire in university philosophy departments. However, it is Descartes work in mathematics that form the basis for this chapter, particularly his invention of the Cartesian Coordinate System which bears his name. Descartes invention of the coordinate system created an entirely new branch of mathematics called analytic geometry, which established a permanent link between the plane and solid geometry of the ancient Greeks and the algebra and analysis of modern mathematics. As a result of his work, mathematicians were able to describe curves with equations, unheard of before Descartes invention of the coordinate system. Rather than describing a circle as the locus of all points equidistant from a given point, mathematicians were now able to refer to a circle centered at the point (0, 0) with radius r as the graph of the equation x2 + y 2 = r2 . The bridge created between geometry and analysis as a result of Descartes methods laid the groundwork for the discovery of the calculus by Newton and Leibniz. For his eorts, mathematicians often refer to Descartes as the Father of Analytic Geometry. In this chapter we will introduce readers to the Cartesian coordinate system and explain the correspondence between points in the plane and ordered pairs of numbers. Once an understanding of the coordinate system is suciently developed, we will develop the concept of the graph of an equation. In particular, we will address the graphs of a class of equations called linear equations.

569

570

CHAPTER 8. GRAPHING

8.1

The Cartesian Coordinate System

Lets begin with the concept of an ordered pair of whole numbers. Ordered Pairs of Whole Numbers. The construct (x, y), where x and y are whole numbers, is called an ordered pair of whole numbers.

Examples of ordered pairs of whole numbers are (0, 0), (2, 3), (5, 1), and (4, 9). Order Matters. Pay particular attention to the phrase ordered pairs. Order matters. Consequently, the ordered pair (x, y) is not the same as the ordered pair (y, x), because the numbers are presented in a dierent order.

Weve seen how to plot whole numbers on a number line. For example, in Figure 8.1, weve plotted the whole numbers 2, 5, and 7 as shaded dots on the number line.

0 1 2 3 4 5 6 7 8 Figure 8.1: Plotting the whole numbers 2, 5, and 7 on a number line. To plot ordered pairs, we need two number lines, called the horizontal and vertical axes, that intersect at the zero location of each line and are at right angles to one another, as shown in Figure 8.2(a). The point where the zero locations touch is called the origin of the coordinate system and has coordinates (0, 0). In Figure 8.2(b), weve added a grid. The resulting construct is an example of a Cartesian Coordinate System. 8 7 6 5 4 3 2 1 0 8 7 6 5 4 3 2 1 0

(0, 0): The origin. 0 1 2 3 4 5 6 7 8


(a) Horizontal and vertical axes.

0 1 2 3 4 5 6 7 8
(b) Adding a grid.

Figure 8.2: A Cartesian coordinate system. Version: Fall 2010

8.1. THE CARTESIAN COORDINATE SYSTEM

571

Now, consider the ordered pair of whole numbers (5, 6). To plot this point on the coordinate system in Figure 8.3(a), start at the origin (0, 0), then move 5 units in the horizontal direction, then 6 units in the vertical direction, then plot a point. The result is shown in Figure 8.3(a). Adding a grid of horizontal and vertical lines at each whole number makes plotting the point (5, 6) much clearer, as shown in Figure 8.3(b). 8 7 6 5 4 3 2 1 0 8 7 6 5 4 3 2 1 0

(5, 6)

(5, 6)

6 units 5 units 0 1 2 3 4 5 6 7 8
(a) 5 right and 6 up.

0 1 2 3 4 5 6 7 8
(b) Plotting on a grid.

Figure 8.3: Plotting the Point (5, 6) in a Cartesian Coordinate System. The numbers in the ordered pair (5, 6) are called the coordinates of the plotted point in Figure 8.3(b). The rst number of the ordered pair is called the abscissa and measures the horizontal distance to the plotted point. The second number is called the ordinate and measures the vertical distance to the plotted point. The combination of axes and grid in Figure 8.3(b) is called a coordinate system. The grid in Figure 8.3(b) is a visualization that greatly eases the plotting of ordered pairs. However, you dont have to draw these gridlines yourself. Instead, you should work on graph paper. Graph Paper Requirement. All plotting should be done on graph paper.

You Try It! EXAMPLE 1. Plot the following ordered pairs of whole numbers: (3, 2), (8, 6), and (2, 7). Solution. Create a Cartesian coordinate system on graph paper, then: To plot the ordered pair (3, 2), start at the origin, then move 3 units to the right and 2 units up. Version: Fall 2010 Plot the following ordered pairs of whole numbers: (2, 2), (5, 5), and (7, 4).

572

CHAPTER 8. GRAPHING

To plot the ordered pair (8, 6), start at the origin, then move 8 units to the right and 6 units up. To plot the ordered pair (2, 7), start at the origin, then move 2 units to the right and 7 units up. Answer: 8 7 6 5 4 3 2 1 0 The results are shown on the following Cartesian coordinate system. 8 7 6 5 4 3 2 1 0

(5, 5) (7, 4) (2, 2) 012345678

(2, 7) (8, 6)

(3, 2)

0 1 2 3 4 5 6 7 8

Allowing for Negative Numbers


Again, weve seen how to plot both positive and negative numbers on a number line. For example, in Figure 8.4, weve plotted the numbers 4, 3/2, 2.2 and 4. 3/2 4 2.2 4 4 3 2 1 0 1 2 3 4

Figure 8.4: Plotting the numbers 4, 3/2, 2.2, and 4. Note that the positive direction is to the right, the negative to the left. That is, to plot the number 2.2, we move 2.2 units to the right on the line, but to plot the number 3/2, we move 3/2 units to the left. To plot ordered pairs having both positive and negative numbers, we need two such number lines that intersect at the zero location of each line and are at right angles to one another, as shown in Figure 8.5(a). As before, adding a grid of horizontal and vertical lines at each integer will be extremely helpful when plotting points (see Figure 8.5(b)). The system of axes and grid in Figure 8.5(b) is called the Cartesian Coordinate System, named after its inventor, Ren` Descartes. e

Version: Fall 2010

8.1. THE CARTESIAN COORDINATE SYSTEM

573

Origin 5 5 -5 5

5
(a) Horizontal and vertical axes.

-5

(b) Adding a grid.

Figure 8.5: The Cartesian coordinate system.

Plotting Points in the Cartesian Coordinate System. On the horizontal axis, the positive direction is to the right, negative is to the left. On the vertical axis, the positive direction is up, negative is down. The point (0, 0) is called the origin of the coordinate system, and is the starting point for all point plotting.

You Try It! EXAMPLE 2. Sketch the points (4, 3), (3, 2), (2, 4), and (3, 3) on a Cartesian coordinate system. Solution. Set up a Cartesian coordinate system on graph paper. To plot the point (4, 3), start at the origin, move 4 units to the right, then 3 units up. To plot the point (3, 2), start at the origin, move 3 units to the left, then 2 units up. To plot the point (2, 4), start at the origin, move 2 units to the left, then 4 units down. To plot the point (3, 3), start at the origin, move 3 units to the right, then 3 units down. These points are plotted and shown in Figure 8.6. Sketch the points (3, 4), (4, 3), (3, 4), and (4, 3) on a Cartesian coordinate system.

Version: Fall 2010

574 Answer: (4, 3) 5 (3, 4) 5

CHAPTER 8. GRAPHING

(4, 3) (3, 2) -5 (3, 4) -5 5 (4, 3) -5 (2, 4) -5 Figure 8.6: Plotting points in the Cartesian coordinate system. Origin 5 (3, 3)

You Try It! EXAMPLE 3. What are the coordinates of the points P , Q, R, and S in the Cartesian coordinate system that follows? 5 Q P Origin -5 R -5 S 5

Solution. Make all measurements from the origin. To obtain the coordinates of point P , start at the origin, move 3 units to the right, then 3 units up. Hence, the coordinates of the point P are (3, 3). Version: Fall 2010

8.1. THE CARTESIAN COORDINATE SYSTEM

575

To obtain the coordinates of point Q, start at the origin, move 4 units to the left, then 3 units up. Hence, the coordinates of the point Q are (4, 3). To obtain the coordinates of point R, start at the origin, move 3 units to the left, then 4 units down. Hence, the coordinates of the point R are (3, 4). To obtain the coordinates of point S, start at the origin, move 4 units to the right, then 3 units down. Hence, the coordinates of the point S are (4, 3). These results are shown on the following Cartesian coordinate system. 5 Q (4, 3) P (3, 3)

-5 R (3, 4) -5

5 S (4, 3)

Version: Fall 2010

576

CHAPTER 8. GRAPHING

l l l
1. Identify the coordinates of the point P . y 5 P

Exercises

l l l

3. Identify the coordinates of the point P . y 5

x 0 5

x 0 5

2. Identify the coordinates of the point P . y 5

4. Identify the coordinates of the point P . y 5

x 0 5

x 0 5

Version: Fall 2010

8.1. THE CARTESIAN COORDINATE SYSTEM 5. Identify the coordinates of the point P . y 5

577

7. Identify the coordinates of the point P . y 5

P 5 P x 5 5 x 5

6. Identify the coordinates of the point P . y 5

8. Identify the coordinates of the point P . y 5

5 P 5

x 5

x 5

9. The points A(1, 1), B(1, 1), C(1, 2), and D(1, 2) are the vertices of a rectangle. Plot these points, draw the rectangle ABCD, then compute the area of rectangle ABCD.

10.

The points A(3, 4), B(4, 4), C(4, 1), and D(3, 1) are the vertices of a rectangle. Plot these points, draw the rectangle ABCD, then compute the area of rectangle ABCD.

Version: Fall 2010

578 11. The points A(2, 1), B(3, 1), C(3, 3), and D(2, 3) are the vertices of a rectangle. Plot these points, draw the rectangle ABCD, then compute the area of rectangle ABCD.

CHAPTER 8. GRAPHING 12. The points A(3, 1), B(2, 1), C(2, 2), and D(3, 2) are the vertices of a rectangle. Plot these points, draw the rectangle ABCD, then compute the area of rectangle ABCD.

13. The points A(4, 2), B(1, 2), C(1, 1), and D(4, 1) are the vertices of a rectangle. Plot these points, draw the rectangle ABCD, then compute the perimeter of rectangle ABCD. 14. The points A(4, 4), B(1, 4), C(1, 3), and D(4, 3) are the vertices of a rectangle. Plot these points, draw the rectangle ABCD, then compute the perimeter of rectangle ABCD.

15. The points A(1, 2), B(3, 2), C(3, 3), and D(1, 3) are the vertices of a rectangle. Plot these points, draw the rectangle ABCD, then compute the perimeter of rectangle ABCD. 16. The points A(4, 2), B(3, 2), C(3, 4), and D(4, 4) are the vertices of a rectangle. Plot these points, draw the rectangle ABCD, then compute the perimeter of rectangle ABCD.

17. The points A(3, 1), B(1, 1), and C(3, 0) are the vertices of a triangle. Plot these points, draw the triangle ABC, then compute the area of the triangle ABC. 18. The points A(3, 2), B(1, 2), and C(3, 2) are the vertices of a triangle. Plot these points, draw the triangle ABC, then compute the area of the triangle ABC.

19. The points A(1, 2), B(0, 2), and C(1, 0) are the vertices of a triangle. Plot these points, draw the triangle ABC, then compute the area of the triangle ABC. 20. The points A(2, 3), B(1, 3), and C(2, 1) are the vertices of a triangle. Plot these points, draw the triangle ABC, then compute the area of the triangle ABC.

21. Plot the points A(3, 3) and B(0, 0) and nd the straight-line distance between the two points. Hint: Create a right triangle, then use the Pythagorean Theorem. 22. Plot the points A(2, 3) and B(1, 2) and nd the straight-line distance between the two points. Hint: Create a right triangle, then use the Pythagorean Theorem.

23. Plot the points A(2, 3) and B(0, 0) and nd the straight-line distance between the two points. Hint: Create a right triangle, then use the Pythagorean Theorem. 24. Plot the points A(3, 2) and B(2, 2) and nd the straight-line distance between the two points. Hint: Create a right triangle, then use the Pythagorean Theorem.

Version: Fall 2010

8.1. THE CARTESIAN COORDINATE SYSTEM

579

25. Find the area of the shaded triangle.

27. Find the area of the shaded triangle.

4 1

4 3 2

26. Find the area of the shaded triangle.

28. Find the area of the shaded triangle.

2 3 4 4

2 1 5 5

Version: Fall 2010

580

CHAPTER 8. GRAPHING

Hint: In Exercises 29-32, surround the triangle with a rectangle, like those shown in Exercises 25-28. 29. Find the area of the triangle with vertices at A(4, 1), B(4, 2), and C(1, 3). 30. Find the area of the triangle with vertices at A(4, 2), B(3, 0), and C(0, 4). 31. Find the area of the triangle with vertices at A(3, 1), B(3, 3), and C(1, 4). 32. Find the area of the triangle with vertices at A(1, 2), B(3, 0), and C(2, 4).

l l l
1. (1, 4) 3. (4, 1) 5. (3, 1) 7. (1, 1) 9. 2 square units 11. 20 square units 13. 16 units 15. 10 units

Answers

l l l

17. 2 square units 19. 1 square units 21. 23. 18 13

25. 6 27. 7 29. 37 2

31. 17

Version: Fall 2010

8.2. GRAPHING LINEAR EQUATIONS

581

8.2

Graphing Linear Equations

Consider y = x + 1 an equation in two variables. If we substitute the ordered pair (x, y) = (1, 2) into the equation y = x + 1, that is, if we replace x with 1 and y with 2, we get a true statement. y =x+1 2=1+1 2=2 Original equation. Substitute: 1 for x and 2 for y. Simplify.

We say that the ordered pair (1, 2) is a solution of the equation y = x + 1. Solution of an Equation in Two Variables. If substituting the ordered pair (x, y) = (a, b) into an equation (replace x with a and y with b) produces a true statement, then the ordered pair (a, b) is called a solution of the equation and is said to satisfy the equation.

You Try It! EXAMPLE 1. Which of the ordered pairs are solutions of the equation y = 2x + 5: (a) (3, 2), or (b) (5, 15)? Solution. Substitute the points into the equation to determine which are solutions. a) To determine if (3, 2) is a solution of y = 2x + 5, substitute 3 for x and 2 for y in the equation y = 2x + 5. y = 2x + 5 2 = 2(3) + 5 2 = 6 + 5 2 = 1 Original equation. Substitute: 3 for x and 2 for y. Multiply rst: 2(3) = 6 Add: 6 + 5 = 1. Which of the ordered pairs (1, 7) and (2, 9) are solution of the equation y = 3x + 4?

Because the resulting statement is false, the ordered pair (3, 2) does not satisfy the equation. The ordered pair (3, 2) is not a solution of y = 2x + 5. a) To determine if (5, 15) is a solution of y = 2x + 5, substitute 5 for x and 15 for y in the equation y = 2x + 5. y = 2x + 5 15 = 2(5) + 5 15 = 10 + 5 15 = 15 Original equation. Substitute: 5 for x and 15 for y. Multiply rst: 2(5) = 10 Add: 10 + 5 = 15. Answer: (1, 7)

The resulting statement is true. The ordered pair (5, 15) does satisfy the equation. Hence, (5, 15) is a solution of y = 2x + 5. Version: Fall 2010

582

CHAPTER 8. GRAPHING

The Graph of an Equation


We turn our attention to the graph of an equation. The Graph of an Equation. The graph of an equation is the set of all ordered pairs that are solutions of the equation.

In the equation y = 2x + 5, the variable y depends on the value of the variable x. For this reason, we call y the dependent variable and x the independent variable. Were free to make choices for x, but the value of y will depend upon our choice for x. We will also assign the horizontal axis to the independent variable x and the vertical axis to the dependent variable y (see Figure 8.7). The graph of y = 2x+5 consists of all ordered pairs that are solutions of the equation y = 2x+ 5. So, our rst task is to nd ordered pairs that are solutions of y = 2x + 5. This is easily accomplished by selecting an arbitrary number of values, substituting them for x in the equation y = 2x + 5, then calculating the resulting values of y. With this thought in mind, we pick arbitrary integers 7, 6, . . . , 2, substitute them for x in the equation y = 2x + 5, calculate the resulting value of y, and store the results in a table. y = 2x + 5 y (x, y) 9 (7, 9) 7 (6, 7) 5 (5, 5) 3 (4, 3) 1 (3, 1) 1 (2, 1) 3 (1, 3) 5 (0, 5) 7 (1, 7) 9 (2, 9)

y y y y y y y y y y

= 2(7) + 5 = 9 = 2(6) + 5 = 7 = 2(5) + 5 = 5 = 2(4) + 5 = 3 = 2(3) + 5 = 1 = 2(2) + 5 = 1 = 2(1) + 5 = 3 = 2(0) + 5 = 5 = 2(1) + 5 = 7 = 2(2) + 5 = 9

x 7 6 5 4 3 2 1 0 1 2

The result is 10 ordered pairs that satisfy the equation y = 2x + 5. Therefore, we have 10 ordered pairs that belong to the graph of y = 2x + 5. They are plotted in Figure 8.7(a). However, were not nished, because the graph of the equation y = 2x + 5 is the set of all points that satisfy the equation and weve only plotted 10 such points. Lets plot some more points. Select some more x-values, compute the corresponding y-value, and record the results in a table. Version: Fall 2010

8.2. GRAPHING LINEAR EQUATIONS y = 2x + 5 y (x, y) 10 (7.5, 10) 8 (6.5, 8) 6 (5.5, 6) 4 (4.5, 4) 2 (3.5, 2) 0 (2.5, 0) 2 (1.5, 2) 4 (0.5, 4) 6 (0.5, 6) 8 (1.5, 8) 10 (2.5, 10)

583

y y y y y y y y y y y

= 2(7.5) + 5 = 10 = 2(6.5) + 5 = 8 = 2(5.5) + 5 = 6 = 2(4.5) + 5 = 4 = 2(3.5) + 5 = 2 = 2(2.5) + 5 = 0 = 2(1.5) + 5 = 2 = 2(0.5) + 5 = 4 = 2(0.5) + 5 = 6 = 2(1.5) + 5 = 8 = 2(2.5) + 5 = 10

x 7.5 6.5 5.5 4.5 3.5 2.5 1.5 0.5 0.5 1.5 2.5

Thats 11 additional points that we add to the graph in Figure 8.7(b). y 10 10 y

10

x 10

10

x 10

10
(a) Ten points that satisfy the equation y = 2x + 5.

10
(b) Eleven additional points that satisfy the equation y = 2x + 5.

Figure 8.7: Plotting points that satisfy the equation y = 2x + 5.

Note that we can continue indenitely in this manner, adding points to the table and plotting them. However, sooner or later, we have to make a leap of faith, and imagine what the nal graph will look like when all of the points that satisfy the equation y = 2x + 5 are plotted. We do so in Figure 8.8, where the nal graph takes on the appearance of a line. Version: Fall 2010

584

CHAPTER 8. GRAPHING

Ruler Use. All lines must be drawn with a ruler. This includes the x- and y-axes.

Important Observation. When we use a ruler to draw a line through the plotted points in Figure 8.7(b), arriving at the nal result in Figure 8.8, we must understand that this is a shortcut technique for plotting all of the remaining ordered pairs that satisfy the equation. Were not really drawing a line through the plotted points. Rather, were shading all of the ordered pairs that satisfy the equation y = 2x + 5. y 10

10

x 10

10 Figure 8.8: The graph of the equation y = 2x + 5.

The Result. The graph of the equation y = 2x + 5, pictured in Figure 8.8, is a line. Actually, the graph is an innite collection of points satisfying the equation y = 2x + 5 that takes the shape of a line, but its all right to say the graph of y = 2x + 5 is a line. Ordered Pairs and the Graph. Because the graph of an equation is the collection of all ordered pairs that satisfy the equation, we have two important results: 1. If an ordered pair satises an equation, then the point in the Cartesian plane represented by the ordered pair is on the graph of the equation. 2. If a point is on the graph of an equation, then the ordered pair representation of that point satises the equation.

Version: Fall 2010

8.2. GRAPHING LINEAR EQUATIONS

585

You Try It! EXAMPLE 2. Find the value of k so that the point (2, k) is on the graph of the equation y = 3x 2. Solution. If the point (2, k) is on the graph of y = 3x 2, then it must satisfy the equation y = 3x 2. y = 3x 2 k = 3(2) 2 k =62 k=4 Thus, k = 4. Original equation. The point (2, k) is on the graph. Substitute 2 for x and k for y in y = 3x 2. Multiply: 3(2) = 6. Subtract: 6 2 = 4. Answer: k = 5/4 Find the value of k so that the point (k, 3) is on the graph of the equation y = 4x + 2.

Linear Equations
Lets plot the graph of another equation. You Try It! EXAMPLE 3. Sketch the graph of y = 2x + 1. Solution. Select arbitrary values of x: 4, 3, . . . , 5. Substitute these values into the equation y = 2x + 1, calculate the resulting value of y, then arrange your results in a table. y = 2x + 1 x y (x, y) 4 9 (4, 9) 3 7 (3, 7) 2 5 (2, 5) 1 3 (1, 3) 0 1 (0, 1) 1 1 (1, 1) 2 3 (2, 3) 3 5 (3, 5) 4 7 (4, 7) 5 9 (5, 9) Sketch the graph of y = 2x 2.

y y y y y y y y y y

= 2(4) + 1 = 9 = 2(3) + 1 = 7 = 2(2) + 1 = 5 = 2(1) + 1 = 3 = 2(0) + 1 = 1 = 2(1) + 1 = 1 = 2(2) + 1 = 3 = 2(3) + 1 = 5 = 2(4) + 1 = 7 = 2(5) + 1 = 9

Weve plotted the points in the table in Figure 8.9(a). There is enough evidence in Figure 8.9(a) to imagine that if we plotted all of the points that satised the equation y = 2x + 1, the result would be the line shown in Figure 8.9(b). Version: Fall 2010

586 Answer: 10

CHAPTER 8. GRAPHING

y 10

y 10

10 10

x 10 10 x 10 10 x 10

10
(a) Ten points that satisfy the equation y = 2x + 1.

10
(b) Plotting all points that satisfy the equation y = 2x + 1.

Figure 8.9: The graph of the equation y = 2x + 1 is a line.

The graph of y = 2x + 5 in Figure 8.8 is a line. The graph of y = 2x + 1 in Figure 8.9(b) is also a line. This would lead one to suspect that the graph of the equation y = mx + b, where m and b are constants, will always be a line. Indeed, this is always the case. Linear Equations. The graph of y = mx + b, where m and b are constants, will always be a line. For this reason, the equation y = mx + b is called a linear equation.

You Try It! Which of the following equations is a linear equation? a) y = 2x3 + 5 b) y = 3x 5 EXAMPLE 4. Which of the following equations is a linear equation? (a) y = 3x + 4, (b) y = 2 x + 3, and (c) y = 2x2 + 4. 3 Solution. Compare each equation with the general form of a linear equation, y = mx + b. a) Note that y = 3x + 4 has the form y = mx + b, where m = 3 and b = 4. Hence, y = 3x + 4 is a linear equation. Its graph is a line. b) Note that y = 2 x + 3 has the form y = mx + b, where m = 2/3 and b = 3. 3 Hence, y = 2 x + 3 is a linear equation. Its graph is a line. 3 Version: Fall 2010

8.2. GRAPHING LINEAR EQUATIONS

587

c) The equation y = 2x2 + 4 does not have the form y = mx + b. The exponent of 2 on the x prevents this equation from being linear. This is a nonlinear equation. Its graph is not a line.

Answer: y = 3x 5

The fact that y = mx + b is a linear equation enables us to quickly sketch its graph. You Try It!
3 EXAMPLE 5. Sketch the graph of y = 2 x + 4. 3 Solution. The equation y = 2 x + 4 has the form y = mx + b. Therefore, the equation is linear and the graph will be a line. Because two points determine 3 a line, we need only nd two points that satisfy the equation y = 2 x + 4, plot them, then draw a line through them with a ruler. We choose x = 2 and x = 2, calculate y, and record the results in a table. 3 y = 2x + 4 x y (x, y) 2 7 (2, 7) 2 1 (2, 1)

Sketch the graph of 1 y = 2 x + 2.

y = 3 (2) + 4 = 3 + 4 = 7 2 y = 3 (2) + 4 = 3 + 4 = 1 2

Plot the points (2, 7) and (2, 1) and draw a line through them. The result is shown in Figure 8.10. y 10 (2, 7)

Answer: 10

10 (2, 1) 10 x 10 10

x 10

10
3 Figure 8.10: The graph of y = 2 x + 4 is a line.

Version: Fall 2010

588

CHAPTER 8. GRAPHING

You may have noted in Example 5 that are choices of 2 and 2 for x eased the calculation of the corresponding y-values because of the resulting cancellation. Choosing Strategic Values. When plotting a linear equation, it is a good strategy to choose values of x that simplify the calculation of the corresponding y-values.

You Try It! Sketch the graph of y = 2 x + 1. 3 EXAMPLE 6. Sketch the graph of y = 1 x + 3. 3 Solution. The equation y = 1 x + 3 has the form y = mx + b. Therefore, the 3 equation is linear and the graph will be a line. Because two points determine a line, we need only nd two points that satisfy the equation y = 1 x + 3, plot 3 them, then draw a line through them with a ruler. We choose x = 6 and x = 6, calculate y, and record the results in a table. y x 6 6 = 1x + 3 3 y (x, y) 1 (6, 1) 5 (6, 5)

y = 1 (6) + 3 = 2 + 3 = 1 3 y = 1 (6) + 3 = 2 + 3 = 5 3

Answer: 10

Plot the points (6, 1) and (6, 5) and draw a line through them. The result is shown in Figure 8.11. y 10 (6, 5) (6, 1)

10 10

x 10 10

x 10

10 Figure 8.11: The graph of y = 1 x + 3 is a line. 3

Version: Fall 2010

8.2. GRAPHING LINEAR EQUATIONS

589

l l l

Exercises

l l l

1. Which of the points (2, 14), (1, 6), (8, 11), and (3, 13) is a solution of the equation y = 2x 8? 2. Which of the points (1, 2), (8, 23), (3, 23), and (8, 24) is a solution of the equation y = 4x 9? 3. Which of the points (1, 1), (2, 20), (4, 31), and (9, 64) is a solution of the equation y = 6x + 7? 4. Which of the points (8, 61), (4, 42), (3, 18), and (6, 46) is a solution of the equation y = 9x + 8?

5. Which of the points (2, 15), (8, 74), (2, 18), and (5, 40) is a solution of the equation y = 9x 3? 6. Which of the points (9, 52), (8, 44), (7, 37), and (8, 35) is a solution of the equation y = 5x 5? 7. Which of the points (2, 12), (1, 12), (3, 10), and (2, 14) is a solution of the equation y = 5x + 4? 8. Which of the points (6, 25), (8, 14), (8, 33), and (7, 9) is a solution of the equation y = 3x + 9?

9. Determine k so that the point (9, k) is a solution of y = 6x + 1. 10. Determine k so that the point (9, k) is a solution of y = 2x + 3. 11. Determine k so that the point (k, 7) is a solution of y = 4x + 1. 12. Determine k so that the point (k, 4) is a solution of y = 8x + 3.

13. Determine k so that the point (k, 1) is a solution of y = 4x + 8. 14. Determine k so that the point (k, 7) is a solution of y = 7x + 5. 15. Determine k so that the point (1, k) is a solution of y = 5x + 3. 16. Determine k so that the point (3, k) is a solution of y = 3x + 3.

In Exercises 17-24, which of the given equations is a linear equation? x 17. y = 6x2 + 4, y = 2 + 6x + 4, y = 6x + 4, y = 6x + 4 18. y = 2x + 1, y = x2 2x + 1, y = 2x + 1, y = 2x2 + 1 19. y = x + 7, y = x + 7, y = x2 + 7, y = x2 + x + 7 20. y = x2 + 5x + 1, y = 5x2 + 1, y = 5x + 1, y = 5x + 1 21. y = x2 2x 2, y = 2x2 2, y = 2x 2, y = 2x 2 22. y = x2 + 5x 8, y = 5x2 8, y = 5x 8, y = 5x 8 23. y = x2 + 7x 3, = 7x2 3, y y = 7x 3, y = 7x 3 24. y = 4x 3, y = x2 4x 3, y = 4x 3, y = 4x2 3

Version: Fall 2010

590

CHAPTER 8. GRAPHING

In Exercises 25-28, which of the given equations has the given graph?
3 25. y = 2 x + 2, y = 3 x 3, 2 y = 3x + 1, y = 2x + 1 y 10

27. y = 5 x 2, y = 3x + 3, 2 y = 3 x + 1, y = 1 x + 1 2 2 y 10

10

x 10

10

x 10

10 26. y = 3x 2, y = 3 x + 1, 2 y = 2x 1, y = 5 x + 2 2 y 10

10 28. y = 3x + 1, y = 5 x 1, 2 y = 5 x 3, y = 3 x 2 2 2 y 10

10

x 10

10

x 10

10

10

In Exercises 29-44, on graph paper, sketch the graph of the given equation. 29. y = 3x 2 30. y = 5 x + 1 2 31. y = 2x 1 32. y = 5 x + 2 2 33. y = 2x + 2 34. y =
5 2x

36. y = 5 x + 1 2 37. y = 2x 2 38. y = 5 x 1 2 39. y = 3 x + 1 2 40. y = 2x + 2 41. y = 2x 3 42. y = 5 x 1 2

35. y = 2x 2 Version: Fall 2010

8.2. GRAPHING LINEAR EQUATIONS 43. y = 3 x + 3 2 44. y = 3x + 1

591

45. Sketch the lines y = 1 x 1 and y = 5 x 2 2 2 on graph paper. As you sweep your eyes from left to right, which line rises more quickly? 46. Sketch the lines y = 5 x + 1 and y = 3x + 1 2 on graph paper. As you sweep your eyes from left to right, which line rises more quickly?
1 47. Sketch the line y = 2 x + 1 and y = 3x+ 3. As you sweep your eyes from left to right, which line falls more quickly?

49. Sketch the line y = 3x 1 and y = 1 2 x2. As you sweep your eyes from left to right, which line falls more quickly? 50. Sketch the line y = 3x 1 and y = 1 2 x+1. As you sweep your eyes from left to right, which line falls more quickly? 51. Sketch the lines y = 3 x 2 and y = 3x + 1 2 on graph paper. As you sweep your eyes from left to right, which line rises more quickly? 52. Sketch the lines y = 1 x+ 3 and y = 5 x+ 1 2 2 on graph paper. As you sweep your eyes from left to right, which line rises more quickly?

48. Sketch the line y = 3x 1 and y = 5 x2. As you sweep your eyes from left 2 to right, which line falls more quickly?

l l l
1. (1, 6) 3. (4, 31) 5. (2, 15) 7. (2, 14) 9. k = 53 11. k = 3 2

Answers

l l l

15. k = 8 17. y = 6x + 4 19. y = x + 7 21. y = 2x 2 23. y = 7x 3 25. y = 3x + 1 27. y = 3 x + 1 2

7 13. k = 4

Version: Fall 2010

592 29. y = 3x 2 y 10

CHAPTER 8. GRAPHING 35. y = 2x 2 y 10

10

x 10

10

x 10

10

10

31. y = 2x 1 y 10

37. y = 2x 2 y 10

10

x 10

10

x 10

10

10

33. y = 2x + 2 y 10

39. y = 3 x + 1 2 y 10

10

x 10

10

x 10

10

10

Version: Fall 2010

8.2. GRAPHING LINEAR EQUATIONS 41. y = 2x 3 10

593 45. The graph of y = 5 x2 rises more quickly. 2 47. The graph of y = 3x + 3 falls more quickly. 49. The graph of y = 3x 1 falls more quickly. 51. The graph of y = 3x+1 rises more quickly.

10

x 10

10 43. y = 3 x + 3 2 10

10

x 10

10

Version: Fall 2010

Index
<, 99 >, 99 , 377, 378 abscissa, 571 absolute value, 102, 151 as magnitude, 114 evaluating, 180 addition decimals, 359 signed, 362 fractions, 275 integers, 113 like signs, 115 unlike signs, 117 whole numbers, 15 additive identity property, 17, 120 additive inverse property, 120 algebraic expressions evaluating, 179 simplify, 201 simplifying, 187 angle, 552 degree, 552, 554, 556 protractor, 552, 554, 556 area parallelogram, 255 rectangle, 43 trapezoid, 318 triangle, 257, 334 arrays counting, 41 595 associative property addition, 17, 119 multiplication, 35, 137, 187 simplifying algebraic expressions, 187 axes, 570, 571 bar chart, 23, 104, 131 borrowing mixed fractions, 306 cancellation, 236, 253 Cartesian coordinate system, 570 change as a dierence, 130 cicle pie chart, 553 circle, 376 area, 379, 379 chord, 377 circumference, 377, 378, 419 degree, 552 diameter, 377 radius, 376 circumference, 419 clearing fractions complex fractions, 317 equations, 329 coecient, 197, 200 combining like terms, 197, 200, 208 decimals, 412 common divisor, 234 commutative property addition, 16, 119

596 multiplication, 34, 137, 187 comparing integers, 99 comparing fractions, 284 complex fractions, 315 clearing fractions, 317 composite number, 53 consecutive integers, 216 odd, 218 coordinate system, 570, 571, 573, 574 axes, 570, 571 grid, 571 ordered pair, 571 origin, 570 point, 571 coordinates, 571 plotting, 571, 573 reading from a graph, 574 counting arrays, 41 decimal converting from percent, 504 converting to percent, 505 decimals, 342 as divisors, 388 comparing, 350 equations, 411 notation, 342 parts, 343 pronouncing, 344 reading, 345 repeating, 402, 403 rounding, 348, 419 square roots of, 430 terminating, 401 to fractions, 346 with fractions, 405 degree angle, 552, 554, 556 protractor, 552, 554, 556 denominator, 231 digit, 4, 342 rounding, 6 test, 6 displayed mathematical notation, 238 Version: Fall 2010

INDEX distributing minus 1, 193 distributing minus one, 208 distributive property, 139, 189, 190, 199 extended, 192 factoring, 199 dividend, 37 divisibility tests by eight, 52 by ve, 52 by four, 52 by nine, 52 by six, 52 by three, 52 by two, 52 divisible, 51 division by zero, 144 decimals, 386, 389 by powers of ten, 392 signed, 390 fractions, 265, 267 grouping, 38 integers, 137 like signs, 143 repeated subtraction, 38 symbols, 37 unlike signs, 143 whole numbers, 33, 37 division by zero, 40 divisor, 37, 51, 234, 235 decimal, 388 greatest common, 234 ellipsis, 99 equation dividing by same amount, 158 equivalent, 156 graph, 582, 584586 multiplying by same amount, 158 solution, 156, 581 equations, 156 adding same amount, 156 addition, 75 checking solution, 208, 326

INDEX clearing fractions, 329 combining like terms, 208 denition, 75 dierent from expressions, 75 distributive property, 212 division, 88 equivalent, 158 equivalent equations, 76, 88 form x = c, 210 isolate terms with solution variable, 209 multiplication, 88 multiplication and division as inverse operations, 88 operations producing equivalent equations, 77 solutions of an equation, 75 solving, 326 subtracting same amount, 156 subtraction, 75 using reciprocals, 327 variables on both sides, 209 whole numbers, 75 with decimals, 411 word problems, 81 writing mathematics, 78 equivalent equations, 156, 158 equivalent fractions, 231, 239 creating, 233 evaluating algebraic expressions, 179 evaluating expressions, 313 absolute value, 180 fractions, 180 evaluating fractions, 150 even exponents, 142 even powers, 312 exponent, 55 base, 55 exponents even and odd, 142 expressions evaluating, 313 extremes, 456 factor, 33, 51, 235

597 factor trees, 54, 236 fraction converting from percent, 502 converting to percent, 506 denominator, 231 equivalent fraction, 506 numerator, 231 fractions adding mixed fractions, 301, 302 adding with common denominator, 275 adding with dierent denominators, 278 addition, 275 comparing, 284 complex, 315 division, 265, 267 equivalent, 231, 239 evaluating, 150, 180 improper, 291 improper to mixed, 294 mixed, 291 mixed to improper, 292 multiplication, 249, 251 order of operations, 315 proper, 291 reducing to lowest terms, 235 signs, 240, 241 square roots of, 430 subtracting mixed fractions, 304, 305 subtracting with common denominator, 275 subtracting with dierent denominators, 278 to decimals, 401 with decimals, 405 GCD, 234 geometry area of rectangle, 43 circle, 419 parallelogram, 255 perimeter, 21, 203, 217 polygon, 21 rectangle, 203, 218 Version: Fall 2010

598 square, 22, 203 trapezoid, 318 triangle, 217, 257 graph line, 584, 586588 solutions of an equation, 582, 584586 graph of an equation, 582, 584586 graphing on the number line, 2 tables, 8 graphs bar chart, 23, 104, 131 line graph, 24, 105, 132 greater than, 99 >, 99 greatest common divisor, 234 by lists, 234 grid, 571 grouping for eciency, 121 grouping symbols, 16 braces, 16 brackets, 16 parentheses, 16 identity addition, 17 multiplication, 34, 137, 265 improper fraction, 291 improper fractions change to mixed fraction, 294 inline mathematical notation, 238 integer absolute value, 102 negative, 102 positive, 102 integers, 98 addition, 113 like signs, 115 unlike signs, 117 as vectors, 113, 114 comparing, 99 consecutive, 216 consecutive odd, 218 denition, 99 Version: Fall 2010

INDEX division, 137 introduction, 98 multiplication, 137 order, 99 order of operations, 129, 148 subtraction, 128 interest, 542 balance, 542, 543, 546 principal, 542, 544 rate, 542, 544, 547 LCD, 277 LCM, 280 least common denominator, 277 least common multiple, 280 by lists, 281 prime factorization, 281 less than, 99 <, 99 like signs division, 143 multiplication, 141 like terms, 198 combining, 200 equations, 208 line graph, 584, 586588 line graph, 24, 105, 132 linear equations, 586588 look back, 163, 216 magnitude, 113 as absolute value, 114 mathematical notation displayed, 238 inline, 238 means, 456 Metric System, 479 centimeters to inches, 490 conversion, 480 converting units of speed, 494 feet to meters, 490 uid ounces to liters, 493 gallons to liters, 493 grams to ounces, 491 inches to centimeters, 490

INDEX kilograms to pounds, 491 kilometers to miles, 490 length, 480 liters to uid ounces, 493 liters to gallons, 493 liters to pints, 493 liters to quarts, 493 mass, 482 meters to feet, 490 meters to yards, 490 miles to kilometers, 490 ounces to grams, 491 pints to liters, 493 pounds to kilograms, 491 prexes, 479 quarts to liters, 493 volumes, 484 yards to meters, 490 mixed fractions, 291 adding, 301, 302 borrowing, 306 change to improper fractions, 292 division, 296 multiplication, 295 subtracting, 304, 305 vertical format, 303, 306 multiple, 280 multiplication by minus one, 140, 193 by one, 137 by zero, 139 decimals, 370, 371 by powers of ten, 376 signed, 373 fractions, 249, 251 integers, 137 like signs, 141 repeated addition, 33, 137 symbols, 33 unlike signs, 141 whole numbers, 33 multiplication by zero, 35 multiplicative identity property, 34, 265 multiplication, 137

599 multiplicative inverse property, 266 negating a sum, 194, 208 negative, 98, 102 additive inverse, 120 of negative, 101 notation disiplayed, 238 expanded, 4 inline, 238 using correct, 122 number composite, 53 number line, 2 absolute value, 103 addition, 113 addition of whole numbers, 15 division, 38 multiplication, 33 negative number, 98 order, 99 subtraction, 19, 128 vectors, 113, 128 numbers comparing, 4 counting, 2 natural, 2 prime, 53 whole, 2 numerator, 231 odd exponents, 142 odd powers, 312 one multiplicative identity, 34, 137, 265 opposite, 98 additve inverse, 120 of opposite, 101 order of operations, 20, 129, 148, 311, 393, 428 fraction bars, 67 fractions, 315 grouping symbols, 64 rules, 64 rules guiding, 148, 312, 374, 393 Version: Fall 2010

600 the distributive property, 68 whole numbers, 64 ordered pair, 570 abscissa, 571 coordinates, 571 ordinate, 571 plotting, 571, 573 point, 571, 573, 574 reading from a graph, 574 solution of an equation, 581, 582, 584586 ordinate, 571 origin, 570 parallelogram altitude, 255 area, 255 base, 255 parentheses use in substitution, 313 percent, 502 applications, 521523, 529532, 534, 535, 542 calculating an unknown percent, 514 calculating with a given percent, 512, 516 converting from a decimal, 505 converting from a fraction, 506 converting to a decimal, 504 converting to a fraction, 502 discount, 534, 535 percent decrease, 531, 532, 534, 535 percent increase, 529, 530 pie chart, 554, 556 rounding, 507 simple interest, 542 perimeter, 21 polygon, 21 rectangle, 22, 203, 218 square, 22, 203 triangle, 217 pi, 377, 378 pie chart, 553 percent, 554, 556 Version: Fall 2010

INDEX point ordered pair, 571, 573 solution of an equation, 581, 582, 584586 polygon, 21 positive, 102 powers even, 312 odd, 312 prime factorization, 51 LCM, 281 prime factorization theorem, 55 prime number, 53 product, 33 proper fraction, 291 Proportions applications, 459 denition, 456 extremes, 456 means, 456 product of extremes and means, 457 solving, 458 units in, 460 protractor angle, 552, 554, 556 degree, 552, 554, 556 pythagorean theorem, 437 quotient, 37 radicals, 426 notation, 426 Ratio denition, 449 rates, 451 unit rates, 452 reciprocal, 266 reciprocals solving equations, 327 rectangle, 22 area, 43 perimeter, 22, 203, 218 reducing to lowest terms, 235 requirements for word problems, 163, 216, 332

INDEX right triangles hypotenuse, 438 pythagorean theorem, 437 rounding, 6 digit, 6 percent, 507 rounding digit, 349 test digit, 349 Rules Guiding Order of Operations, 312, 374, 393 set, 2 simplify denition, 201 simplifying algebraic expressions, 187 solution, 156 solution of an equation ordered pair, 581, 582, 584586 point, 581, 582, 584586 solving equations, 156, 208, 326 by clearing fractions, 329 combining like terms, 208 using reciprocals, 327 square, 22 of a number, 425 perimeter, 22, 203 square root, 425, 425 decimals, 430 estimating, 430 fractions, 430 radical notation, 427 square unit, 42 subscripts, 318 substitution using parentheses, 313 subtraction adding the opposite, 128 decimals, 361 signed, 362 integers, 128 whole numbers, 15 symbols multiplication, 33 tables, 8

601 graphing, 8 term, 197 tips for evaluating algebraic expressions, 179 trapezoid altitude, 318 area, 318 base, 318 triangle altitude, 257 area, 257, 334 base, 257 perimeter, 217 vertex, 257 undo, 89, 160 Units american system, 467 american units to metric units, 490 length, 467 length conversion, 469 metric system, 479 metric units to american units, 490 time, 472 volume conversion, 472 volumes, 471 weight, 470 weight conversion, 471 unlike signs division, 143 multiplication, 141 unlike terms, 198 unwrap, 160 variable, 156 dictionary, 163, 216 whole numbers, 75 variable part, 197200 vectors, 113 as negative integers, 114 as positive integers, 114 direction, 113 magnitude, 113, 114 whole number Version: Fall 2010

602 negative, 98 opposite, 98 whole numbers addition, 15 dividing, 33 multiplication, 33 solving equations, 75 subtraction, 15 variable, 75 word problems requirements, 163, 216, 332 using tables, 219 writing mathematics, 191 zero additive identity, 17, 120 dividing by, 40, 144 multiplication, 35, 139

INDEX

Version: Fall 2010

You might also like